CURRENT DIAGNOSIS AND TREATMENT PEDIATRICS 24TH LATEST Chapter 1 The Newborn Infant 1. Which milestone is developmentally appropriate for a 2-month-old infant when the nurse pulls the infant to a sitting position? a. Head lag is present when the infants trunk is lifted. b. The infant is able to support the head when the trunk is lifted. c. The infant is briefly able to hold the head erect. d. The infant is fully able to support and hold the head in a straight line. ANS: C A 2-month-old infant is able to hold the head erect only briefly and continues to have some head lag. It is not until 4 months of age that the infant can keep his or her head in a straight line when pulled to a sitting position. 2. Approximately what should a newborn weigh at 1 year of age if the newborns birth weight was 7 pounds 6 ounces? a. 14 3/4 pounds b. 22 1/8 pounds c. 29 1/2 pounds d. Unable to estimate weight at 1 year ANS: B An infant triples the birth weight by 1 year of age. An infant doubles the birth weight by 6 months of age. An infant quadruples the birth weight by 2 years of age. Weight at 6 months, 1 year, and 2 years of age can be estimated from the birth weight.
CURRENT DIAGNOSIS AND TREATMENT PEDIATRICS 24TH LATEST 3. Which statement made by a parent would be consistent with a developmental delay? a. I have noticed that my 9-month-old infant responds consistently to the sound of his name. b. I have noticed that my 12-month-old child does not get herself to a sitting position or pull to stand. c. I am so happy when my 1 1/2-month-old infant smiles at me. d. My 5-month-old infant is not rolling over in both directions yet. ANS: B Critical developmental milestones for gross motor development in a 12 month old include standing briefly without support, getting to a sitting position, and pulling to stand. If a 12-monthold child does not perform these activities, it may be indicative of a developmental delay. An infant who responds to his name at 9 months of age is demonstrating abilities to both hear and interpret sound. A social smile is present by 2 months of age. Rolling over in both directions is not a critical milestone for gross motor development until the child reaches 6 months of age. 4. At a healthy 2-month-old infants well-child clinic visit, the nurse should give which immunizations? a. DTaP, IPV, HepB, Hib, PCV, rotavirus b. MMR, DTaP, PVC, and IPV c. Hib, DTaP, rotavirus, and OPV d. Hib and MMR, IPV, and rotavirus ANS: A DTaP, IPV, HepB, Hib, PCV, and rotavirus are the appropriate sequence of immunizations for a healthy 2-month-old infant. MMR is given at or after 12 months of age. Oral polio vaccine (OPV) is no longer administered in the U.S. 5. The nurse advises the mother of a 3-month-old infant, exclusively breast-fed, to: a. start giving the infant a vitamin D supplement.
CURRENT DIAGNOSIS AND TREATMENT PEDIATRICS 24TH LATEST b. start using an infant feeder and add rice cereal to the formula. c. start feeding the infant rice cereal with a spoon at the evening feeding. d. continue breast-feeding without any supplements. ANS: A Breast milk does not provide an adequate amount of dietary vitamin D. Infants who are exclusively breast-fed need vitamin D supplements to prevent rickets. An infant feeder is an inappropriate method of providing the infant with caloric intake. Solid foods and rice cereal are not recommended for a 3-month-old infant. Solid feedings do not typically begin before 4 to 6 months of age. Because breast milk is not an adequate source of fluoride, infants need to be given a fluoride supplement. 6. At
months of age, an infant should first be expected to locate an object hidden from
view. a. 4 b. 6 c. 9 d. 20 ANS: C By 9 months of age an infant will actively search for an object that is out of sight. Four-monthold infants are not cognitively capable of searching out objects hidden from their view. Infants at this developmental level do not pursue hidden objects. Six-month-old infants have not developed the ability to perceive objects as permanent and do not search out objects hidden from their view. Twenty-month-old infants actively pursue objects not in their view and are capable of recalling the location of an object not in their view. They first look for hidden objects around the age of 9 months. 7. The parents of a newborn infant state, We will probably not have our baby immunized because we are concerned about the risk of our child being injured. Which is the best response for the nurse to make?
CURRENT DIAGNOSIS AND TREATMENT PEDIATRICS 24TH LATEST a. It is your decision. b. Have you talked with your parents about this? They can probably help you think about this decision. c. The risks of not immunizing your baby are greater than the risks from the immunizations. d. You are making a mistake. ANS: C Although immunizations have been documented to have a negative effect in a small number of cases, an unimmunized infant is at greater risk for development of complications from childhood diseases than from the vaccines. It is the parents decision not to immunize the child; however, the nurse has a responsibility to inform parents about the risks to infants who are not immunized. Grandparents can be supportive but are not the primary decision makers for the infant. Telling parents that they are making a mistake is an inappropriate response. 8. The mother of a 9-month-old infant is concerned because the infant cries when she leaves him. Which is the best response for the nurse to make to the mother? a. You could consider leaving the infant more often so he can adjust. b. You might consider taking him to the doctor because he may be ill. c. Have you noticed whether the baby is teething? d. This can be a healthy sign of attachment. ANS: D Healthy attachment is manifested by stranger anxiety in late infancy. An infant who manifests stranger anxiety can be supported by the mother leaving the infant for only short periods of time. Assessing developmental needs is appropriate before taking an infant to a physician. Pain from teething expressed by the infants cries would not occur only when the mother left the room. 9. Which statement concerning physiological factors of infancy is true? a. The infant has a slower metabolic rate than an adult.
CURRENT DIAGNOSIS AND TREATMENT PEDIATRICS 24TH LATEST b. An infant is not able to digest protein and lactase. c. Infants have a slower circulatory response than adults. d. The kidneys of an infant are less efficient in concentrating urine than an adults kidneys. ANS: D The infants kidneys are not as effective at concentrating urine compared with an adults kidneys because of immaturity of the renal system and a slower glomerular filtration rates. The infants metabolic rate is faster, not slower, than an adults. Although the newborn infants gastrointestinal system is immature, it is capable of digesting protein and lactase, but the ability to digest and absorb fat does not reach adult levels until approximately 6 to 9 months of age. 10. Which is a priority in counseling parents of a 6-month-old infant? a. Increased appetite from secondary growth spurt b. Allowing the infant to self-feed c. Securing a developmentally safe environment for the infant d. Strategies to teach infants to sit up ANS: C Safety is a primary concern as an infant becomes increasingly mobile. The infants appetite and growth velocity decrease in the second half of infancy. Fine motor development, which is refined in the second half of infancy, is necessary before the infant can self-feed. Unless the infant has a neuromuscular deficit, strategies for teaching a normally developing infant to sit up are not necessary. 11. A mother of a 2-month-old infant tells the nurse, My child doesnt sleep as much as his older brother did at the same age. What is the best response for the nurse? a. Have you tried to feed the baby more often? b. Infant sleep patterns vary widely, with some infants sleeping only 2 to 3 hours at a time.
CURRENT DIAGNOSIS AND TREATMENT PEDIATRICS 24TH LATEST c. It is helpful to keep a record of your babys eating, waking, sleeping, and elimination patterns and to come back in a week to discuss them. d. This infant is difficult. It is important for you to identify what is bothering the baby. ANS: B Newborn infants may sleep as much as 17 to 20 hours per day. Sleep patterns vary widely, with some infants sleeping only 2 to 3 hours at a time. Infants typically do not need more caloric intake to improve sleep behaviors. Keeping intake, output, waking, and sleeping data is not typically helpful in discussing differences among infants behaviors. Identifying an infant as difficult without identifying helpful actions is not a therapeutic response for a parent concerned about sleep. 12. The mother of a 10-month-old infant tells the nurse that her infant really likes cows milk. Which is the best response to this mother? a. Milk is good for him. b. It is best to wait until he is a year old before giving him cows milk. c. Limit cows milk to his bedtime bottle. d. Mix his cereal with cows milk and give him formula in a bottle. ANS: B It is best to wait until the infant is at least a year old before giving him cows milk because of the risk of allergies and intestinal problems. Cows milk protein intolerance is the most common food allergy during infancy. Although milk is a good source of calcium and protein for children after the first year of life, it is not the best source of nutrients for children younger than 1 year old. Bedtime bottles of formula or milk are contraindicated because of their high sugar content, which leads to dental decay in primary teeth. Cereal can be mixed with formula. 13. The mother of a 10-month-old infant asks the nurse about beginning to wean her child from the bottle. Which statement by the mother suggests that the child is not ready to be weaned? a. My son is frequently throwing his bottle down.
CURRENT DIAGNOSIS AND TREATMENT PEDIATRICS 24TH LATEST b. The baby takes a few ounces of formula from the bottle. c. He is constantly chewing on the nipple. It concerns me. d. He consistently is sucking. ANS: D Consistent sucking is a sign that the child is not ready to be weaned. A decreased interest in the bottle starts between 6 and 12 months. Throwing the bottle down is a sign of a decreased interest in the bottle. When the child is taking more fluids from a cup and decreasing amounts from the bottle, the child is demonstrating a readiness for weaning. Chewing on the nipple is another sign that the infant is ready to be weaned. 14. Which is appropriate play for a 6-month-old infant? a. Pat-a-cake, peek-a-boo b. Ball rolling, hide and seek game c. Bright rattles and tactile toys d. Push and pull toys ANS: A Six-month-old children enjoy playing pat-a-cake and peek-a-boo. Nine-month-old infants enjoy rolling a ball and playing hide and seek games. Four-month-old infants enjoy bright rattles and tactile toys. Twelve-month-old infants enjoy playing with push and pull toys. 15. Which statement by a mother indicates that her 5-month-old infant is ready for solid food? a. When I give my baby solid foods, she has difficulty getting it to the back of her throat to swallow. b. She has just started to sit up without any support. c. I am surprised that she only weighs 11 pounds. I expected her to have gained some weight. d. I find that she really has to be encouraged to eat. ANS: B
CURRENT DIAGNOSIS AND TREATMENT PEDIATRICS 24TH LATEST Sitting is a sign that the child is ready to begin with solid foods. Children who are ready to manage solid foods are able to move food to the back of their throats to swallow. Infants who weigh less than 13 pounds and demonstrate a lack of interest in eating are not ready to be started on solid foods. Infants who are difficult feeders and do not demonstrate an interest in solid foods are not ready to be started on them. 16. A mother asks the nurse, When should I begin to clean my babys teeth? What is the best response for the nurse? a. You can begin when all her baby teeth are in. b. You can easily begin now. Just put some toothpaste on a gauze pad to clean the teeth. c. I dont think you have to worry about that until she can handle a toothbrush. d. You can begin as soon as your child has a tooth. The easiest way is to take cotton swabs or a face cloth and just wipe the teeth. Toothpaste is not necessary. ANS: D An infants teeth need to be cleaned as soon as they erupt. Cleaning the teeth with cotton swabs or a face cloth is appropriate. Waiting until all the baby teeth are in is inappropriate and prolongs cleaning until 2 years of age. Because toothpaste contains fluoride and infants will swallow the toothpaste, parents should avoid its use. Even when a child has the ability to hold a toothbrush, the parent should continue cleaning the childs teeth. 1. An infants length was 20 inches at birth. What should the nurse expect the infants length to be at 6 months (in inches)? ANS: 26 During the first 6 months, infants increase their birth length by approximately 1 inch (2.54 cm) per month, slowing to 1/2 inch (1.27 cm) per month over the next 6 months.
CURRENT DIAGNOSIS AND TREATMENT PEDIATRICS 24TH LATEST
Chapter 2. Child Development and Behavior 1. A nurse is reviewing developmental concepts for infants and children. Which statement best describes development in infants and children? a. Development, a predictable and orderly process, occurs at varying rates within normal limits. b. Development is primarily related to the growth in the number and size of cells. c. Development occurs in a proximodistal direction with fine muscle development occurring first. d. Development is more easily and accurately measured than growth. ANS: A Development, a continuous orderly process, provides the basis for increases in the childs function and complexity of behavior. The increases in rate of function and complexity can vary normally within limits for each child. An increase in the number and size of cells is a definition for growth. Development proceeds in a proximodistal direction with fine muscle organization occurring as a result of large muscle organization. Development is a more complex process that is affected by many factors; therefore, it is less easily and accurately measured. Growth is a predictable process with standard measurement methods. 2. Frequent developmental assessments are important for which reason? a. Stable developmental periods during infancy provide an opportunity to identify any delays or deficits. b. Infants need stimulation specific to the stage of development. c. Critical periods of development occur during childhood. d. Child development is unpredictable and needs monitoring. ANS: C Critical periods are blocks of time during which children are ready to master specific developmental tasks. Children can master these tasks more easily during particular periods of
CURRENT DIAGNOSIS AND TREATMENT PEDIATRICS 24TH LATEST time in their growth and developmental process. Infancy is a dynamic time of development that requires frequent evaluations to assess appropriate developmental progress. Infants in a nurturing environment will develop appropriately and will not necessarily need stimulation specific to their developmental stage. Normal growth and development are orderly and proceed in a predictable pattern on the basis of each individuals abilities and potentials. 3. Which factor has the greatest influence on child growth and development? a. Culture b. Environment c. Genetics d. Nutrition ANS: C Genetic factors (heredity) determine each individuals growth and developmental rate. Although factors such as environment, culture, nutrition, and family can influence genetic traits, they do not eliminate the effect of the genetic endowment, which is permanent. Culture is a significant factor that influences how children grow toward adulthood. Culture influences both growth and development but does not eliminate inborn genetic influences. Environment has a significant role in determining growth and development both before and after birth. The environment can influence how and to which extent genetic traits are manifested, but environmental factors cannot eliminate the effect of genetics. Nutrition is critical for growth and plays a significant role throughout childhood. 4. A nurse is planning a teaching session with a child. According to Piagetian theory, the period of cognitive development in which the child is able to distinguish fact from fantasy is the period of cognitive development. a. sensorimotor b. formal operations c. concrete operations d. preoperational
CURRENT DIAGNOSIS AND TREATMENT PEDIATRICS 24TH LATEST ANS: C Concrete operations is the period of cognitive development in which childrens thinking is shifted from egocentric to being able to see anothers point of view. They develop the ability to distinguish fact from fantasy. The sensorimotor stage occurs in infancy and is a period of reflexive behavior. During this period, the infants world becomes more permanent and organized. The stage ends with the infant demonstrating some evidence of reasoning. Formal operations is a period in development in which new ideas are created through previous thoughts. Analytic reason and abstract thought emerge in this period. The preoperational stage is a period of egocentrism in which the childs judgments are illogical and dominated by magical thinking and animism. 5. The theorist who viewed developmental progression as a lifelong series of conflicts that need resolution is: a. Erikson. b. Freud. c. Kohlberg. d. Piaget. ANS: A Erik Erikson viewed development as a series of conflicts affected by social and cultural factors. Each conflict must be resolved for the child to progress emotionally, with unsuccessful resolution leaving the child emotionally disabled. Sigmund Freud proposed a psychosexual theory of development in which certain parts of the body assume psychological significance as foci of sexual energy. The foci shift as the individual moves through the different stages (oral, anal, phallic, latency, and genital) of development. Lawrence Kohlberg described moral development as having three levels (preconventional, conventional, and postconventional). His theory closely parallels Piagets. Jean Piagets cognitive theory interprets how children learn and think and how this thinking progresses and differs from adult thinking. Stages of his theory include sensorimotor, preoperations, concrete operations, and formal operations. 6. What does the nurse need to know when observing chronically ill children at play?
CURRENT DIAGNOSIS AND TREATMENT PEDIATRICS 24TH LATEST a. Play is not important to hospitalized children. b. Children need to have structured play periods. c. Childrens play is an indication of a childs response to treatment. d. Play is to be discouraged because it tires hospitalized children. ANS: C Play for all children is an activity woven with meaning and purpose and is a mechanism for mastering their environment. For chronically ill children, play can indicate their state of wellness and response to treatment. Play is important to all children in all environments. Although childrens play activities appear unorganized and at times chaotic, play has purpose and meaning. Imposing structure on play interferes with the tasks being worked on. Children who have fewer energy reserves still require play. For these children, less-active play activities will be important. 7. Which child is most likely to be frightened by hospitalization? a. A 4-month-old infant admitted with a diagnosis of bronchiolitis b. A 2-year-old toddler admitted for cystic fibrosis c. A 9-year-old child hospitalized with a fractured femur d. A 15-year-old adolescent admitted for abdominal pain ANS: B Toddlers are most likely to be frightened by hospitalization because their thought processes are egocentric, magical, and illogical. They feel very threatened by unfamiliar people and strange environments. Young infants are not as likely to be frightened as toddlers by hospitalization because they are not as aware of the environment. The 9-year-old childs cognitive ability is sufficient enough for the child to understand the reason for the hospitalization. The 15-year-old adolescent has the cognitive ability to interpret the reason for the hospitalization. 8. Which statement made by a 15-year-old adolescent with a diagnosis of neurofibromatosis (an autosomal dominant genetic disorder) best demonstrates an understanding of the mechanism of inheritance for the disease?
CURRENT DIAGNOSIS AND TREATMENT PEDIATRICS 24TH LATEST a. My babies will probably not have neurofibromatosis. b. My babies have a 50% chance of having neurofibromatosis. c. Whether my babies have problems depends on the father. d. My babies have a 25% chance of having neurofibromatosis. ANS: B Neurofibromatosis is an autosomal dominant genetic disorder that occurs when the abnormal gene is carried on the affected chromosome with a normal gene. Because the abnormal gene is dominant, an individual with the defective gene has a 50% chance of transmitting the defect to an infant with each pregnancy. Neurofibromatosis is not a sex-linked genetic disease; therefore, either the father or the mother genetically transfers it to the infant. A parent with the defective gene will genetically transfer either a normal or abnormal gene to an infant. Because the defective gene is dominant, there is a 50% probability of the child inheriting the disease. 9. During a routine healthcare visit, a parent asks the nurse why her 9-month-old infant is not walking as her older child did at the same age. Which response by the nurse best demonstrates an understanding of child development? a. Shes a little slow. b. If she is pulling up, you can help her by holding her hand. c. Babies progress at different rates. Your infants development is within normal limits. d. Maybe she needs to see a behavioral specialist. ANS: C Ninety percent of infants walk by 14 months of age. The infant is within normal developmental limits. It is inappropriate for the nurse to state that the infant is a little slow. Infants will walk when they are developmentally ready. Hurrying an infant does not result in the developmental task being achieved at an earlier time period. Consulting a behavioral specialist for diagnostic evaluation is indicated when a child demonstrates developmental delays. The child has no evidence of a delay.
CURRENT DIAGNOSIS AND TREATMENT PEDIATRICS 24TH LATEST 10. Which expected outcome would be developmentally appropriate for a hospitalized 4-year-old child? a. The child will be dressed and fed by the parents. b. The child will independently ask for play materials or other personal needs. c. The child will be able to verbalize an understanding of the reason for the hospitalization. d. The child will have a parent stay in the room at all times. ANS: B Erikson identifies initiative as a developmental task for the preschool child. Initiating play activities and asking for play materials or assistance with personal needs demonstrate developmental appropriateness. Parents need to foster appropriate developmental behavior in the 4-year-old child. Dressing and feeding the child do not encourage independent behavior. A 4year-old child cannot be expected to cognitively understand the reason for his or her hospitalization. Expecting the child to verbalize an understanding for the hospitalization is an inappropriate outcome. Parents staying with the child throughout a hospitalization is an inappropriate outcome. Although children benefit from parental involvement, parents may not have the support structure to stay in the room with the child at all times. 11. A nurse has completed a teaching session with parents of preschool aged children. Which statement made by the parent identifies an appropriate level of language development for a 4year-old child? a. The child has a vocabulary of 300 words and uses simple sentences. b. The child uses correct grammar in sentences. c. The child is able to pronounce consonants clearly. d. The child uses language to express abstract thought. ANS: B The 4-year-old child is able to use correct grammar in sentence structure and typically has difficulty in pronouncing consonants. Simple sentences and a 300-word vocabulary are
CURRENT DIAGNOSIS AND TREATMENT PEDIATRICS 24TH LATEST appropriate for a 2-year-old child. The use of language to express abstract thought is developmentally appropriate for the adolescent. 12. Which should the nurse evaluate before administering the Denver Developmental Screening Test II (DDST-II)? a. The childs height and weight b. The parents ability to comprehend the results c. The childs mood d. The parentchild interaction ANS: C The results of the screening test are valid if the child acted in a normal and expected manner. The childs height and weight are not relevant to the DDST-II screening process. The parents ability to understand the results of the screening is not relevant to the validity of the test. The parentchild interaction is not significantly relevant to the test results. 13. Which children are at greater risk for not receiving immunizations? a. Children who attend licensed day care programs b. Children entering school c. Children who are home schooled d. Young adults entering college ANS: C Home schooled children are at risk for being underimmunized and need to be monitored. All states require immunizations for children in day care programs and entering school. Most colleges require a record of immunizations as part of a health history. 14. Which developmental assessment instrument is appropriate to assess a 5-year-old child? a. Brazelton Behavioral Scale
CURRENT DIAGNOSIS AND TREATMENT PEDIATRICS 24TH LATEST b. Denver Developmental Screening Test II (DDST-II) c. Dubowitz Scale d. New Ballard Scale ANS: B The DDST-II is used for infants and children between birth and 6 years of age. Brazeltons Behavioral Scale is used for newborn assessment. The Dubowitz Scale is used for estimation of gestational age. The New Ballard Scale is used for newborn screening. 15. A 2-month-old child has not received any immunizations. Which immunizations should the nurse give? a. DTaP, Hib, HepB, IPV, varicella b. DTaP, Hib, HepB, MMR, IPV c. DTaP, Hib, HepB, PCV, IPV, rotavirus d. DTaP, Hib, HepB, PCV, IPV, HepA ANS: C DTaP, Hib, HepB, PCV, IPV, and rotavirus are appropriate immunizations for an unimmunized 2-month-old child. The child should not receive varicella until at or after 12 months of age. MMR is not given to children until at or after 12 months of age. HepA is recommended for all children at 1 year of age. 16. You are preparing immunizations for a 12-month-old child who is immunocompromised. Which immunization cannot be given? a. DTaP b. HepA c. IPV d. Varicella ANS: D
CURRENT DIAGNOSIS AND TREATMENT PEDIATRICS 24TH LATEST Children who are immunologically compromised should not receive live viral vaccines. Varicella is a live vaccine, and should not be given except in special circumstances. DTaP, HepA, and IPV can be safely given. 17. Which immunization can cause fever and rash to occur 1 to 2 weeks after administration? a. HepB b. DTaP c. Hib d. MMR ANS: D MMR is a live virus vaccine and can cause fever and rash 1 to 2 weeks after administration. HepB, DTaP, and Hib do not cause fever or rash. MULTIPLE RESPONSE 1. The nurse is preparing immunizations for a healthy 11-year-old boy who has received all his primary immunizations. Which immunizations will the nurse consider? Select all that apply. a. Meningococcal b. DTaP c. OPV d. Smallpox ANS: A, B Meningococcal conjugate vaccine should be given to all children at age 11 to 12 years. The American Academy of Pediatrics recommends one dose of DTaP vaccine for children at age 11 to 12 years, as long as they have received the primary DTaP series. Oral polio vaccine is no longer administered in the U.S. The current smallpox vaccine is not recommended for healthy, low-risk children younger than 18 years of age.
CURRENT DIAGNOSIS AND TREATMENT PEDIATRICS 24TH LATEST 2. Parents of a 4-month-old child ask the nurse what they can do to help relieve the discomfort of teething. The nurse should include which suggestions for the parents? Select all that apply. a. Provide warm liquids. b. Rub the gums with aspirin. c. Over-the-counter topical medications for gum pain relief can be used as directed. d. Administer acetaminophen (Tylenol) as directed. e. Provide a hard food such as a frozen bagel for chewing. ANS: C, D, E To help parents cope with teething, nurses can suggest that they provide cool liquids and hard foods (e.g., dry toast, Popsicles, frozen bagels) for chewing. Hard, cold teethers and ice wrapped in cloth may also provide comfort for inflamed gums. Nurses should explain to parents that overthe-counter topical medications for gum pain relief should be used only as directed. Home remedies, such as rubbing the gums with whiskey or aspirin, should be discouraged, but acetaminophen administered as directed for the childs age can relieve discomfort. OTHER 1. Place in order the gross motor developmental milestones a nurse expects to assess in an infant. Begin with the earliest gross motor milestone expected and progress to the last gross motor milestone attained. a. Turns from abdomen to back b. Lifts head off of bed when in a prone position c. Walks holding on to furniture d. Turns from back to abdomen e. Sits unsupported ANS:
CURRENT DIAGNOSIS AND TREATMENT PEDIATRICS 24TH LATEST B, A, D, E, C The infant lifts its head off of the bed when in a prone position at 3 months, turns from abdomen to back at 4 to 5 months, turns from back to abdomen at 6 to 7 months, sits unsupported at 8 to 9 months, and can walk holding on to furniture at 10 to 12 months.
Chapter 3. Adolescence 1. A nurse is teaching an adolescent about Tanner stages. Which statement best describes Tanner staging? a. Predictable stages of puberty that are based on chronological age b. Staging of puberty based on the initiation of menarche and nocturnal emissions c. Predictable stages of puberty that are based on primary and secondary sexual characteristics d. Staging of puberty based on the initiation of primary sexual characteristics ANS: C Tanner sexual-maturing ratings are based on the development of stages of primary and secondary sexual characteristics. They are not based on chronological age. The age at which an adolescent enters puberty is variable. The puberty stage in girls begins with breast development. The puberty stage in boys begins with genital enlargement. Primary sexual characteristics are not the basis of Tanner staging. 2. Which behavior suggests appropriate psychosocial development in the adolescent?
CURRENT DIAGNOSIS AND TREATMENT PEDIATRICS 24TH LATEST a. The adolescent seeks validation for socially acceptable behavior from older adults. b. The adolescent is self-absorbed and self-centered and has sudden mood swings. c. Adolescents move from peers and enjoy spending time with family members. d. Conformity with the peer group increases in late adolescence. ANS: B During adolescence, energy is focused within. Adolescents concentrate on themselves in an effort to determine who they are or who they will be. Adolescents are likely to be impulsive and impatient. The peer group validates acceptable behavior during adolescence. Adolescents move from family and enjoy spending time with peers. Adolescents also spend time alone; they need this time to think and concentrate on themselves. Conformity becomes less important in late adolescence. 3. The parents of a 14-year-old girl are concerned that their adolescent spends too much time looking in the mirror. Which statement is the most appropriate for the nurse to make? a. Your teenager needs clearer and stricter limits about her behavior. b. Your teenager needs more responsibility at home. c. During adolescence, this behavior is not unusual. d. The behavior is abnormal and needs further investigation. ANS: C Narcissistic behavior is normal during this period of development. The teenager is seeking a personal identity. Stricter limits are not an appropriate response for a behavior that is part of normal development. More responsibility at home is not an appropriate response for this situation. 4. Which factor contributes to early adolescents engaging in risk-taking behaviors? a. Peer pressure b. A desire to master their environment
CURRENT DIAGNOSIS AND TREATMENT PEDIATRICS 24TH LATEST c. Engagement in the process of separation from their parents d. A belief that they are invulnerable ANS: D During early to middle adolescence, children feel that they are exempt from the consequences of risk-taking behaviors: they believe negative consequences happen only to others. Impressing peers is more typically the factor influencing behavior of older school-age children. Mastering the environment is the task of young school-age children. Emancipation is a major issue for the older adolescent. The process is accomplished as the teenager gains an education or vocational training. 5. Which statement is the most appropriate advice to give parents of a 16-year-old teenager who is rebellious? a. You need to be stricter so that your teenager feels more secure. b. You need to allow your teenager to make realistic choices while using consistent and structured discipline. c. Increasing your teens involvement with his peers will improve his self-esteem. d. Allow your teenager to choose the type of discipline that is used in your home. ANS: B Allowing teenagers to choose between realistic options and offering consistent and structured discipline typically enhances cooperation and decreases rebelliousness. Setting stricter limits typically does not decrease rebelliousness or increase feelings of security. Increasing peer involvement does not typically increase self-esteem. Allowing teenagers to choose the method of discipline is not realistic and typically does not reduce rebelliousness. 6. Which statement by the nurse is most appropriate to a 15-year-old adolescent whose friend has mentioned suicide? a. Tell your friend to come to the clinic immediately. b. You need to gather details about your friends suicide plan.
CURRENT DIAGNOSIS AND TREATMENT PEDIATRICS 24TH LATEST c. Your friends threat needs to be taken seriously and immediate help for your friend is important. d. If your friend mentions suicide a second time, you will want to get your friend some help. ANS: C Suicide is the third most common cause of death among American adolescents. A suicide threat from an adolescent serves as a dramatic message to others and should be taken seriously. Adolescents at risk should be targeted for supportive guidance and counseling before a crisis occurs. Instructing a 15-year-old adolescent to tell a friend to come to the clinic immediately provides the teen with limited information and does not address the concern. It is important to determine whether a person threatening suicide has a plan of action; however, the best information for the 15-year-old adolescent to have is that all threats of suicide should be taken seriously and immediate help is important. It is imperative that help is provided immediately for a teenager who is talking about suicide. Waiting until the teen discusses it a second time may be too late. 7. When planning care for adolescents, the nurse should: a. teach parents first, and they, in turn, will teach the teenager. b. provide information for their long-term health needs because teenagers respond best to long-range planning. c. maintain the parents role by providing explanations for treatment and procedures to the parents only. d. give information privately to adolescents about how they can manage the specific problems that they identify. ANS: D Problems that teenagers identify and are interested in are typically the problems that they are the most willing to address. Confidentiality is important to adolescents. Adolescents prefer to confer privately (without parents) with the nurse and healthcare provider. Teenagers are socially and cognitively at the developmental stage where the healthcare provider can teach them and can
CURRENT DIAGNOSIS AND TREATMENT PEDIATRICS 24TH LATEST receive explanations about healthcare directly from the nurse. Teenagers are more interested in immediate healthcare needs than in long-term needs. 8. A 17-year-old adolescent tells the nurse that he is not having sex because it would make his parents very angry. This response indicates that the adolescent has a developmental lag in which area? a. Cognitive development b. Moral development c. Psychosocial development d. Psychosexual development ANS: B The appropriate moral development for a 17-year-old adolescent would include evidence that the teenager has internalized a value system and does not depend on parents to determine right and wrong behaviors. Cognitive development is related to moral development, but it is not the pivotal point in determining right and wrong behaviors. Identity formation is the psychosocial development task. Energy is focused within the adolescent, who exhibits behavior that is selfabsorbed and egocentric. Although a task during adolescence is the development of a sexual identity, the teenagers dependence on the parents sanctioning of right or wrong behavior is more appropriately related to moral development. 9. The best response a nurse can make to a 15-year-old girl who has verbalized a desire to have a baby is: a. Have you talked with your parents about this? b. Do you have plans to continue school? c. Will you be able to support the baby? d. Can you tell me how your life will change if you have an infant? ANS: D
CURRENT DIAGNOSIS AND TREATMENT PEDIATRICS 24TH LATEST Having the teenager describe how the infant will affect her life will allow the teen to think more realistically. Her description will allow the nurse to assess the teens perception and reality orientation. Asking the teenager if she has talked to her parents is not particularly helpful to the teen or the nurse and may terminate the communication. A direct question about continuing school and how the teenager will support the child will not facilitate communication. Openended questions encourage communication. 10. In an interview with the nurse, a mother states that she is concerned that her 14-year-old teen is critical and finding fault with her. The nurse counsels the mother that: a. the family needs to change its value system to meet the teenagers changing needs. b. the parentteen relationship is important for the teenager and conflicts are to be expected. c. teenagers create psychological distance from the parent to separate from the parent. d. parents need to relinquish their relationship with their teenager to the teens peers. ANS: C The teenager uses critical and fault-finding behavior as a mechanism to separate from the parent and become independent. Changing the familys value system to meet the teenagers needs is not realistic and will result only in the teenagers being critical of the new system. The parentteen relationship is not as important to the teenager as it was in earlier years. Friends and peers become more important. Parents should not relinquish their relationship with their teenager to the teens peers. Maintaining a consistent parental relationship with the adolescent is important. 11. A nurse is teaching adolescent boys about pubertal changes. Which is the first sign of pubertal change seen with boys? a. Testicular enlargement b. Facial hair c. Scrotal enlargement d. Voice deepens
CURRENT DIAGNOSIS AND TREATMENT PEDIATRICS 24TH LATEST ANS: A The first sign of pubertal changes in boys is testicular enlargement in response to testosterone secretion, which usually occurs in Tanner stage 2. Slight pubic hair is present and the smooth skin texture of the scrotum is somewhat altered. As testosterone secretion increases, the penis, testes, and scrotum enlarge. During Tanner stages 4 and 5, rising levels of testosterone cause the voice to deepen and facial hair appears at the corners of the upper lip and chin. MULTIPLE RESPONSE 1. Parents of a teenager ask the nurse what signs they should look for if their child is in a gang. The nurse should include which signs when answering? Select all that apply. a. Plans to try out for the debate team at school b. Skips classes to go to the mall c. Hangs out with friends they have had since childhood d. Has an unexplained source of money e. Fears the police ANS: B, D, E Signs of gang involvement include skipping classes, unexplained sources of money, and fear of the police. Associating with new friends while ignoring old friends is another sign, so hanging out with friends they have had since childhood is not a sign of gang involvement. A change in attitude toward participating in activities is another sign of gang involvement. Plans to try out for the debate team at school are not a sign of gang involvement. OTHER 1. Place in order the signs of female sexual maturity beginning with the first sign and ending with the last. Use the following format for your answers: A, B, C, D a. Growth of pubic hair b. Menarche
CURRENT DIAGNOSIS AND TREATMENT PEDIATRICS 24TH LATEST c. Appearance of breast buds d. Ovulation is established ANS: C, A, B, D Sexual maturation in girls begins with the appearance of breast buds (thelarche), which is the first sign of ovarian function. Thelarche occurs at approximately age 8 to 11 years and is followed by the growth of pubic hair. Linear growth slows, and menarche begins approximately 1 year after the peak height velocity (PHV). Ovulation occurs with menarche but may or may not accompany the first 2 years of menarche so it is not established until later.
Chapter 4. Adolescent Substance Abuse 42. The nurse assessing a typical family with an adolescent child would more often than not find
the arguments between caregivers and child to be about: a.
adolescent sexuality
c.
drugs and drug abuse
b.
religion or politics
d.
ordinary family matters
ANS: D Feedback
A
B
C
Incorrect. The nurse assessing a typical family with an adolescent child would not find the arguments between caregivers and child to be about adolescent sexuality. Incorrect. The nurse assessing a typical family with an adolescent child would not find the arguments between caregivers and child to be about religion or politics. Incorrect. The nurse assessing a typical family with an adolescent child would not find the arguments between caregivers and child to be about drugs and drug abuse.
CURRENT DIAGNOSIS AND TREATMENT PEDIATRICS 24TH LATEST
D
Correct. The nurse assessing a typical family with an adolescent child would more often than not find the arguments between caregivers and child to be about ordinary family matters.
Chapter 5. Eating Disorders 1. The nurse is assessing a child admitted to the hospital for abdominal pain. Which of the
following findings by the nurse would be typical of those seen in appendicitis?
a.
anorexia, nausea, and vomiting preceded the pain according to the history provided by the caregivers
b.
a subnormal temperature for the last 2 days
c.
pain that was vague and somewhat localized to the periumbilical area and gradually migrated to the right lower quadrant
d.
night time chills for 1 week
ANS: C Feedback
A
B
C
Incorrect. Anorexia, nausea, and vomiting preceding the pain according to the history provided by the caregivers is not the typical finding seen in appendicitis. Incorrect. A subnormal temperature for the last 2 days is not the typical finding seen in appendicitis. Correct. Pain that was vague and somewhat localized to the periumbilical area and gradually migrated to the right lower quadrant is the typical finding seen in appendicitis. Incorrect. Night time chills for 1 week is not the typical finding seen in appendicitis.
D 2.A pediatric client presents to the emergency department with acute abdominal pain followed by anorexia and nausea. The nurse suspects appendicitis. Upon palpation, the nurse anticipates identifying pain localized in which area(s) of the abdomen? Select all that apply.
CURRENT DIAGNOSIS AND TREATMENT PEDIATRICS 24TH LATEST a.
costovertebral angle
c.
left lower quadrant
b.
right lower quadrant
d.
periumbilical area
ANS: B, D Feedback The pain associated with appendicitis typically is localized at the periumbilical area, gradually migrating to the right lower quadrant.
Correct
The pain associated with appendicitis typically is localized at the periumbilical area, gradually migrating to the right lower quadrant. The pain of appendicitis is not associated with discomfort in the costovertebral angle.
Incorrect
The pain of appendicitis is not associated with discomfort in the left lower quadrant.
3. The infant with hypertrophic pyloric stenosis will eventually:
a.
experience spontaneous recovery in 75% of cases
b.
stop eating and go into a life-threatening decline
c.
have projectile vomitus propelled up to several feet
d.
pass an unusually large bowel movement
ANS: C Feedback Incorrect. The infant with hypertrophic pyloric stenosis will not eventually experience spontaneous recovery in 75% of cases. Treatment for pyloric stenosis is a surgical procedure called a pyloromyotomy. A
CURRENT DIAGNOSIS AND TREATMENT PEDIATRICS 24TH LATEST
B C
D
Incorrect. The infant with hypertrophic pyloric stenosis is hungry and wants to be fed again, in spite of feeding and vomiting. Correct. The infant with hypertrophic pyloric stenosis will eventually have projectile vomitus propelled up to several feet. Incorrect. Because food does not pass through the pylorus, bowel movements are small.
4. Which of the following is an early warning sign of hypertrophic pyloric stenosis?
a.
the infant looking and acting somewhat sick
b.
the infant being hungry and wanting to feed again very soon after vomiting
c.
milk running out of the infants mouth periodically during the feeding
d.
unusually loud burping sounds
ANS: B Feedback Incorrect. The infant with hypertrophic pyloric stenosis does not act or look sick. A B
Correct. An early warning sign of hypertrophic pyloric stenosis is the infant being hungry and wanting to be fed again very soon after vomiting.
C
Incorrect. An early warning sign of hypertrophic pyloric stenosis is not milk running out of the infants mouth periodically during the feeding. The infant experiences vomiting which eventually becomes projectile, being propelled up to several feet.
D
Incorrect. An early warning sign of hypertrophic pyloric stenosis is not unusually loud burping sounds.
5. Diagnosis of hypertrophic pyloric stenosis can be made on history and what other finding?
a.
epigastric tenderness over several days
c.
failure to gain weight
CURRENT DIAGNOSIS AND TREATMENT PEDIATRICS 24TH LATEST
b.
crying without producing any tears
d.
olive-shaped mass in the epigastrium
ANS: D Feedback
A
Incorrect. Epigastric tenderness over several days is not a manifestation of hypertrophic pyloric stenosis. Incorrect. Crying without producing any tears is a sign of dehydration.
B
C
D
Incorrect. Failure to gain weight accompanies hypertrophic pyloric stenosis but is not a diagnostic indicator. Correct. Diagnosis of hypertrophic pyloric stenosis can be made on history and palpation of an olive-shaped mass in the epigastrium.
6. Which of the following are appropriate goals in working with a child or adolescent with a
diagnosis of anorexia nervosa? Select all that apply. a.
weight gain to within 5% of ideal body weight within 1 weeks time frame
b.
weight gain to within 10% of ideal body weight at a gain of 1-2 pounds per week
c.
eat 100% of a 2,500-calorie diet each day
d.
eat a balanced diet with sufficient calories to allow weight gain
ANS: B, D Feedback When working with a child or adolescent with anorexia nervosa, appropriate goals include weight gain to within 10% of body weight at 1-2 pounds of weight gain per week.
Correct
The client should be encouraged to eat a balanced diet with sufficient calories to allow weight gain.
CURRENT DIAGNOSIS AND TREATMENT PEDIATRICS 24TH LATEST Weight gain to within 5% of the ideal body weight within 1 weeks time would require rapid weight to be gained too quickly. This rapid weight gain is medically unsafe and may lead to cardiac overload and death. Weight gain should not be too rapid, such as that which would occur with eating 100% of a 2,500-calorie diet daily.
Chapter 6. Child and Adolescent Psychiatric Disorders 1. A nurse is assessing a child with a depressive disorder. Which symptom is likely to be manifested by the child? a. Increased nighttime waking b. Impulsivity and distractibility c. Carelessness and inattention to details d. Refusal to leave the house ANS: A Sleep pattern disturbances are often associated with depression. These include insomnia or hypersomnia. Impulsivity and distractibility are manifestations of attention-deficit hyperactivity disorder (ADHD). A diminished ability to think or concentrate, carelessness, and inattention to details are clinical manifestations of a depressive disorder. A refusal to leave the house, even to play with friends, is characteristic of separation anxiety disorder. 2. A nurse is teaching parents about symptoms associated with suicide. Which statement about suicide should the nurse include in the teaching plan?
CURRENT DIAGNOSIS AND TREATMENT PEDIATRICS 24TH LATEST a. Children younger than 10 years of age do not attempt suicide. b. A child who attempts suicide is usually depressed and has low self-esteem. c. Suicide is usually an isolated event in a school community. d. The suicide rate among females is higher than among males. ANS: B Poor self-concept and depression contribute significantly to suicidal behaviors. Children as young as 3 years of age who have attempted suicide have been evaluated and found to be cognizant of their actions. It is common for suicide to occur in a cluster within a community (e.g., schools). Males have a higher incidence of both suicide attempts and completed suicides. 3. What is the best response for the nurse to make to an adolescent who states, I am very sad. I wish I wasnt alive.? a. Everyone feels sad once in a while. b. You are just trying to escape your problems. c. Have you told your parents how you feel? d. Have you thought about hurting yourself? ANS: D Have you thought about hurting yourself? acknowledges the adolescents suicide gesture and further assesses the adolescents condition. Everyone feels sad once in a while is a judgmental response that ignores the adolescents obvious statement indicating a need for professional help. The parents should be made aware of an adolescents precarious mental state; however, Have you told your parents how you feel? does not address the adolescents statement. 4. A nurse is teaching parents about family relationship patterns associated with eating disorders. Which family relationship pattern should the nurse teach that is consistent for an adolescent female diagnosed with an eating disorder? a. The adolescent is viewed as an extension of the parent. b. There is an overprotective mother and an emotionally distant father.
CURRENT DIAGNOSIS AND TREATMENT PEDIATRICS 24TH LATEST c. The mother is domineering and the father is passive. d. The adolescent is the youngest child or is an only child. ANS: A One of the most salient factors associated with eating disorders is enmeshed family relationships in which the child is considered to be an extension of the parent or is viewed as a means of meeting the parents needs. The family dynamics for males with anorexia are reported to include a mother who is overinvolved with the child and a father who typifies a strong, cultural image. A domineering mother and passive father are not characteristic of the family dynamics associated with eating disorders. Birth order and number of children in the family were not identified as factors in enmeshed family relationships. 5. The long-term treatment plan for an adolescent with an eating disorder focuses on: a. managing the effects of malnutrition. b. establishing sufficient caloric intake. c. improving family dynamics. d. restructuring the perception of body image. ANS: D The focus of treatment in individual therapy for an eating disorder involves restructuring cognitive perceptions about the individuals body image. The treatment of eating disorders is initially focused on reestablishing physiological homeostasis. Once body systems are stabilized, the next goal of treatment for eating disorders is maintaining adequate caloric intake. Although family therapy is indicated when dysfunctional family relationships exist, the primary focus of therapy for eating disorders is to help the adolescent cope with complex issues. 6. A parent of a child with a psychosocial disorder states, I dont know how my child developed this problem. The nurse should base a response on which information? a. Neurobiological, family, and sociocultural factors can contribute to the development of psychosocial disorders in children.
CURRENT DIAGNOSIS AND TREATMENT PEDIATRICS 24TH LATEST b. Like many conditions affecting children, the etiology of psychosocial disorders is unknown. c. The majority of psychosocial disorders have a clear pattern of genetic inheritance. d. Dysfunctional family patterns are usually identified as the cause of a psychosocial disorder. ANS: A Psychosocial disorders are responses to stress and may be manifested as disturbances in feeling, body functions, behavior, or performance. The etiology of many psychosocial disorders in children can be identified. Some psychosocial disorders are inheritable disorders. Others have been identified as having a familial predisposition. Research consistently shows that psychosocial disorders are caused by a combination of predisposing or inherent factors and environmental or interactional factors. 7. A nurse is caring for a child admitted for substance abuse. The nurse plans care with the recognition that substance abuse primarily affects which organ of the body? a. Heart b. Liver c. Brain d. Lungs ANS: C The primary affect of substance abuse is on the brain and residually on the rest of the body. Although an excessive amount of a chemical can cause cardiac abnormalities, the brain is the most commonly affected organ. Long-term alcohol use is known to impair the liver; however, brain function is decreased by any amount of alcohol intake. The pulmonary system is not the primary target; however, one commonly abused drug known to cause pulmonary problems is tobacco. 8. A 14-year-old child admits to using marijuana every day. Which phase of substance abuse should the nurse assess for?
CURRENT DIAGNOSIS AND TREATMENT PEDIATRICS 24TH LATEST a. Experimentation b. Early drug use c. True drug addiction d. Severe drug addiction ANS: C True drug addiction is identified as regular use of drugs. Physical dependence may be present. Social functioning has a drug focus. With experimentation, the individual tries the drug to see what it is like or to satisfy peers. Early drug use is identified as using drugs with some degree of regularity for their desirable effects. In severe drug addiction, the physical condition of the individual deteriorates and all activities are related to drug use. 9. The school nurse observes an unkempt child dressed in inappropriate clothing who repeatedly asks for food. The nurse is concerned about which problem? a. Physical abuse b. Physical neglect c. Emotional abuse d. Sexual abuse ANS: B These physical and behavioral indicators suggest that parental attention is not being given to the childs physical needs. The child is being neglected. There are no physical indicators of actual abuse in this description. Behavioral indicators of physical abuse reflect an impaired relationship with parents and other adults. Emotional abuse is manifested by developmental problems or maladaptive behaviors. Physical indicators of sexual abuse are focused on the genitourinary system. A variety of behavioral indicators range from bizarre sexual behavior to eating and sleeping disturbances. 10. Which should be the most appropriate nursing intervention for the infant who is not gaining weight?
CURRENT DIAGNOSIS AND TREATMENT PEDIATRICS 24TH LATEST a. Instruct the primary caregiver on proper feeding techniques. b. Observe and document the parentinfant interaction. c. Assign different nurses to care for the infant. d. Feed the infant on a predetermined schedule. ANS: B Observation and documentation of the parentinfant interaction may offer insight into the cause of malnutrition. Instruction alone is not the best teaching strategy. Role modeling and supervised practice along with parental instruction will facilitate the parents learning to feed the infant. A consistent caregiver will facilitate trust in the infant. The infants caloric intake is increased by feeding the infant on demand rather than on a schedule. 11. Which statement made by a parent of a toddler who is not gaining weight indicates the need for education about feeding small children? a. He doesnt want to eat, so I put the cereal in his bottle. b. I put him in a high chair for meals and snacks. c. I turn off the television and we eat together for every meal. d. I try to feed him at the same times every day. ANS: A Large quantities of cereal or baby food in bottles do not provide sufficient nutritional intake for the small child. The young child should be placed in a high chair for feeding. Distraction during feedings, such as watching television, is identified as a reason for inadequate nutritional intake in young children. Having the parents or others eat with the child makes meals and snacks a pleasant time. A regular pattern or schedule for meals facilitates nutritional intake. 12. Which intervention should the nurse teach parents about caring for an infant experiencing drug withdrawal? a. Keep rooms in the home well lighted. b. Play music or the television continuously.
CURRENT DIAGNOSIS AND TREATMENT PEDIATRICS 24TH LATEST c. Organize care to minimize disruptions. d. Let the infant calm himself if irritable. ANS: C The infants care should be coordinated to limit the number of times the infant is disturbed. Light levels should be maintained at the minimum necessary level. Sound levels should be kept to the minimum necessary level. Comfort measures should be provided immediately when the infant exhibits irritability. 13. A child who has symptoms of irritable mood and changes in sleep and appetite patterns lasting 3 weeks meets the criteria for which depressive disorder? a. Major depressive disorder b. Dysthymic disorder c. Cyclothymic disorder d. Panic disorder ANS: A A 2-week (or longer) episode of depressed or irritable mood in addition to disturbances in appetite, sleep, energy, or self-esteem meets the criteria for a major depressive disorder. A dysthymic disorder is associated with a depressed or irritable mood for at least a year. A cyclothymic or bipolar mood disorder is characterized by chronic, fluctuating mood disturbances between depressive lows and highs for a year. A panic disorder is a type of anxiety disorder. 14. What is the goal of therapeutic management for a child diagnosed with attention-deficit hyperactivity (ADHD) disorder? a. Administer stimulant medications. b. Assess the child for other psychosocial disorders. c. Correct nutritional imbalances. d. Reduce the frequency and intensity of unsocialized behaviors.
CURRENT DIAGNOSIS AND TREATMENT PEDIATRICS 24TH LATEST ANS: D The primary goal of therapeutic management for the child with ADHD is to reduce the intensity and frequency of unsocialized behaviors. Although medications are effective in managing behaviors associated with ADHD, all families do not choose to give their child medication. Administering medication is not the primary goal. Children with ADHD may have other psychosocial or learning problems; however, diagnosing these is not the primary goal. Interventions to correct nutritional imbalances are the primary focus of care for eating disorders. 15. Which behavior demonstrated by an adolescent should alert the school nurse to a problem of substance abuse? a. States feelings of worthlessness b. Increased desire for social conformity c. Does not feel the need for peer approval d. Rebellious behavior ANS: D Rebellious or aggressive behavior is a behavior that may indicate substance abuse. Feelings of worthlessness are suggestive of a depressive disorder. An adolescent with a substance abuse problem may be depressed, but this behavior is not a manifestation of substance abuse. The clinical manifestations of substance abuse are marked by an increase in antisocial behavior as the desire for social conformity decreases and the need for the substance increases. The adolescent with a substance abuse problem may demonstrate an excessive dependence on peer influence. 16. A nurse is caring for an infant with neonatal abstinence syndrome. Which manifestation should the nurse expect to assess? a. Weight gain b. Respiratory acidosis c. High-pitched persistent cry d. Hypotonus
CURRENT DIAGNOSIS AND TREATMENT PEDIATRICS 24TH LATEST ANS: C A high-pitched persistent cry is one of the many manifestations of infant drug withdrawal. The infant undergoing drug withdrawal may lose weight or fail to gain weight. Respiratory alkalosis and respiratory distress are manifestations of withdrawal. An infant undergoing drug withdrawal would have hypertonus, hyperreflexia, and hyperactivity. 17. Which finding noted by the nurse on a physical assessment may suggest that a child has been sexually abused? a. Swelling of the genitalia and pain on urination b. Smooth philtrum and thin upper lip c. Speech and physical development delays d. History of constipation, drowsiness, and constricted pupils ANS: A Physical indicators of sexual abuse may include swelling or itching of the genitalia and pain on urination. Other indicators may include bruises, bleeding, or lacerations of the external genitalia, vagina, or anal area. The infant with fetal alcohol syndrome may have microphthalmia or abnormally small eyes or short palpebral fissures, a thin upper lip, and a poorly developed philtrum. Children who have been emotionally abused may exhibit speech disorders, lags in physical development, failure to thrive, or hyperactive and disruptive behaviors. Opiates can cause these behaviors: detachment and apathy, drowsiness, constricted pupils, constipation, slurred speech, and impaired judgment. 18. A nurse is assessing a child with attention-deficit hyperactivity disorder (ADHD). Which manifestation should the nurse not expect to assess? a. Talking incessantly b. Blurting out the answers to questions before the questions have been completed c. Acting withdrawn in social situations d. Fidgeting with hands or feet
CURRENT DIAGNOSIS AND TREATMENT PEDIATRICS 24TH LATEST ANS: C The child with ADHD tends to be talkative, often interrupting conversations, rather than withdrawn in social situations. Talking excessively is a characteristic of impulsivity/hyperactivity. Blurting out the answers to questions before the questions have been completed is an indication of the impulse control that is often lacking in children with ADHD. The child with ADHD tends to be talkative, often interrupting conversations, rather than withdrawn in social situations. MULTIPLE RESPONSE 1. A nurse working on the pediatric unit should be aware that children admitted with which assessment findings are suggestive of physical child abuse? Select all that apply. a. Bruises in various stages of healing b. Bruises over the shins or bony prominences c. Burns on the palms of the hands d. A fracture of the right wrist from a sports accident e. Rib fractures in an infant ANS: A, C, E Bruises in various stages of healing and burns on the palms of the hand may be indicative of physical abuse. Rib fractures in an infant are another indicator of physical abuse. Bruises over the shins or bony prominences are seen in children beginning to walk. A fracture of the right wrist can occur as the child begins to participate in sports activities. 2. The nurse is aware that suicide risk increases if the child displays which characteristics? Select all that apply. a. Previous suicide attempt b. No previous exposure to violence in the home c. Recent loss d. Effective social network
CURRENT DIAGNOSIS AND TREATMENT PEDIATRICS 24TH LATEST e. History of physical abuse ANS: A, C, E The risk of suicide increases if the child has had a previous suicide attempt, a recent loss, or a history of physical abuse. No previous violence in the home or having an effective social network decreases the risk of suicide.
Chapter 7. Child Abuse and Neglect 1. Which of the following statements best defines the term child maltreatment? a. intentional injury of a child
c.failure to provide what a child needs b.not giving a child what he or she wantsd.accidental harm to a child by someone ANS: A Feedback ACorrect: Child maltreatment is the intentional injury of a child. BIncorrect: Child maltreatment is not considered failure to give a child what the child wants. CIncorrect: Child maltreatment is not considered the failure to provide what a child needs. DIncorrect: Child maltreatment is not considered accidental harm to a child by someone else. The harm or injury is intentional, not accidental. 2. Which of the following statements best defines the term physical abuse?
a. bodily injury to a person that seems to have been inflicted by other than accidental means b.purposefully beating a child so that there are highly visible marks on the childs body c. use of the hands applied to a child in an excessively forceful manner d.any damage to a child that involves the use of muscle-applied force ANS: A Feedback
CURRENT DIAGNOSIS AND TREATMENT PEDIATRICS 24TH LATEST ACorrect: Physical abuse is defined as bodily injury to a person that seems to have been inflicted by other than accidental means. Such injuries may include damage to the skin, including bruise, burns, bite marks, and lacerations; damage to the head; or shaken baby syndrome. BIncorrect: In physical abuse, marks may not be visible on the childs body. For example, with shaken baby syndrome, there may not be any visible signs of injury. CIncorrect: Physical abuse may include more than the use of the hands applied to a child in an excessively forceful manner. DIncorrect: Muscle-applied force may not be the etiology of physical abuse in the child. 3. The school nurse observes parents interacting with a school-aged child and notices that they do
not show any affection toward the child and there is no evidence of emotional support or supervision. Later the nurse learns from the child that he must take care of all his own hygiene tasks, has to find something to eat on his own, and his parents never say anything nice about him. The nurse at this point believes that the parents are engaging in: a. physical abuse c.poor parenting b.psychological abused.withholding of love ANS: B Feedback AIncorrect: These characteristics are not specific for physical abuse. BCorrect: Psychological abuse results from the recurrent lack of attention to the childs needs, and includes lack of affection, emotional support, or supervision. CIncorrect: These characteristics are not specific for poor parenting. DIncorrect: These characteristics are not specific for withholding of love. 4. The majority of perpetrators of abuse to children reported to state Child Protective Service
agencies as suspected victims of abuse and neglect are: a. neighbors within one blockc. parents b.strangers d.relatives other than parents ANS: C Feedback AIncorrect: Neighbors within one block are not the majority of perpetrators. BIncorrect: Strangers are not the majority of perpetrators. CCorrect: Three-fourths of all perpetrators of abuse to children reported to the Child Protective Service agencies are caregivers. DIncorrect: Relatives other than parents are not the majority of perpetrators.
CURRENT DIAGNOSIS AND TREATMENT PEDIATRICS 24TH LATEST 5. The majority of child abuse victims fall into which of the following age ranges?
a. over 10 yearsc.6 to 8 years b.8 to 10 years d.under 6 years ANS: D Feedback AIncorrect: The majority of child abuse victims are under 6 years, not over 10 years. BIncorrect: The majority of child abuse victims are under 6 years, not 8 to 10 years. CIncorrect: The majority of child abuse victims are under 6 years, not 6 to 8 years. DCorrect: The majority of child abuse victims are under 6 years. 6. Which of the following is the most common type of mistreatment of children?
a. physical abusec.sexual abuse b.neglect d.emotional maltreatment ANS: B Feedback AIncorrect: The most common type of mistreatment of children is not physical abuse. Twentyfive percent of child victims are physically abused. BCorrect: The most common type of mistreatment of children is neglect. Fifty-five percent of child victims suffer from neglect. CIncorrect: The most common type of mistreatment of children is not sexual abuse. Twelve percent of victims are sexually abused. DIncorrect: The most common type of mistreatment of children is not emotional maltreatment. Six percent of victims are emotionally abused. 7. In the sociological model of family violence, family violence is viewed as:
a. a pattern of behavior that is passed from generation to generation b.a pattern of harm within the nuclear family with no outside persons involved c. any harmful action(s) between related persons no matter how distant the relationship is d.harmful activities within a group designated as family no matter what the relationship ANS: A Feedback ACorrect: In the sociological model of family violence, family violence is viewed as a pattern of behavior that is passed from generation to generation. Although the form of abuse may change, the pattern continues. In one generation the abuse may be physical, but may change to
CURRENT DIAGNOSIS AND TREATMENT PEDIATRICS 24TH LATEST sexual abuse in the next generation. Caregivers will construct similar abusive relationships with their own children based on their experiences of having violent caregivers. BIncorrect: In the sociological model of family violence, family violence is not viewed as a pattern of harm within the nuclear family with no outside persons involved. CIncorrect: In the sociological model of family violence, family violence is not viewed as any harmful action(s) between related persons no matter how distant the relationship is. DIncorrect: In the sociological model of family violence, family violence is not viewed as harmful activities within a group designated as family no matter what the relationship. 8. When family violence is passed from generation to generation, this type of abuse has which of
the following characteristics? a. It continues without changing to another type.
b.The form of abuse may change. c. It usually gets more violent. d.It is more detrimental to the child from generation to generation. ANS: B Feedback AIncorrect: Intergenerational patterns of violence do not necessarily continue without changing to another type of abuse. BCorrect: When family violence is passed from generation to generation, the form of abuse may change. CIncorrect: When family violence is passed from generation to generation, it does not necessarily get more violent. DIncorrect: This type of abuse is not more detrimental to the child from generation to generation. 9. Which of the following children is in a high-risk family for less nurturing and more hurtful
behavior? a. a child being raised by grandparents b.a child who is forbidden from playing with other children c. a child being raised by two men d.a child in a nuclear family there both parents have full-time jobs ANS: B Feedback AIncorrect: A child being raised by grandparents is not necessarily in a high-risk family for abuse. The characteristics of the grandparents would have to be known. BCorrect: Children at high risk for less nurturing and more hurtful behavior live in a family system that has little contact with community groups such as school or church. These children
CURRENT DIAGNOSIS AND TREATMENT PEDIATRICS 24TH LATEST are not involved in after-school activities and are not allowed to play with other neighborhood children. CIncorrect: A child being raised by two men is not necessarily in a high-risk situation for abuse. DIncorrect: A child in a nuclear family there both parents have full-time jobs is probably not in a high-risk family for abuse. One of the risk factors for abuse is stress. Families who spend increased time together under stressful conditions may not have appropriate social supports. Examples of this type of stress are poverty and unemployment. 10. The social-interactional systemic perspective of child abuse and neglect says that the
legitimization of violence in the family is due to which of the following factors? a. family pathology of a genetic nature b.increased availability of pornography c. emphasis on hiding sexuality and not being open d.societys attitudes, beliefs, and values ANS: D Feedback AIncorrect: According to the social-interactional systemic perspective of child abuse, the legitimization of violence in the family is not due to family pathology of a genetic nature. BIncorrect: The legitimization of violence in the family is not due to increased availability of pornography. CIncorrect: The legitimization of violence in the family is not due to emphasis on hiding sexuality and not being open. DCorrect: The social-interactional systemic perspective suggests society contains the attitudes, values, and beliefs that legitimize violence in families. For example, violent acts, interactions, and roles are part of society, and television, movies and videos demonstrate and illustrate the value society places on violence. Our society tends to accept violence as a matter of course, treating it casually or exploiting it to gain an audience. 11. According to the social-interactional systemic perspective of child abuse and neglect, four
factors place the family members at risk for abuse. These risk factors are the family itself, the caregiver, the child, and: a. chronic povertyc.the presence of a family crisis b.genetics d.the national emphasis on sex ANS: C Feedback AIncorrect: These four factors are the family itself, the caregiver, the child, but not chronic poverty. BIncorrect: These four factors are the family itself, the caregiver, the child, but not genetics.
CURRENT DIAGNOSIS AND TREATMENT PEDIATRICS 24TH LATEST CCorrect: These four factors are the family itself, the caregiver, the child, and the presence of a family crisis. DIncorrect: These four factors are the family itself, the caregiver, the child, but not the national emphasis on sex. 12. When caregivers lack knowledge about parenting, lack parenting skills, and are emotionally
immature, the child often assumes which of the following roles? a. victim
b.caregiver role toward the caregiver c. regressed child of regressed caregivers d.scapegoat ANS: B Feedback AIncorrect: The child does not assume the role of victim in this situation. BCorrect: When caregivers lack knowledge about parenting, lack parenting skills, and are emotionally immature, the child often assumes a caregiver role toward the caregiver. An example of this is then a 5-year old consoles an unemployed parent. CIncorrect: The child does not assume the role of regressed child of regressed caregivers. DIncorrect: The child does not assume the role of scapegoat. 13. Which of the following examples best defines the term role reversal?
a. The child assumes a caregiver role toward the caregiver. b.The good child takes on a bad child role. c. A person who has been a good provider quits his or her job. d.A lazy person becomes very productive in the family. ANS: A Feedback ACorrect: Role reversal occurs then the child assumes a caregiver role toward the caregiver. BIncorrect: Role reversal is not exemplified by the good child taking on a bad child role. CIncorrect: Role reversal is not defined as a person who has been a good provider quitting his or her job. DIncorrect: Role reversal is not defined as a lazy person becoming very productive in the family. 14. When there is a depressed parent in the family, it is most likely that the depression will have
which of the following effects? a. will not affect the persons ability or performance in parenting b.will cause the parent who is depressed to try harder to be a good parent
CURRENT DIAGNOSIS AND TREATMENT PEDIATRICS 24TH LATEST c. places the parent at risk for physically or emotionally abusing the children d.will be seen in the child during the growing-up years ANS: C Feedback AIncorrect: Depression will affect the persons ability or performance in parenting. BIncorrect: Depression will not cause the parent who is depressed to try harder to be a good parent. CCorrect: A depressed parent is at risk for physically or emotionally abusing the children. Such an individual may be irritable or withdrawn from family members, not have the emotional energy to manage the demanding needs of a child, or become provoked and use harsher punishment than is necessary or appropriate. DIncorrect: The most likely effect of depression in a parent is not that depression will be seen in the child during the growing-up years. 15. The nurse working in the pediatric clinic notices that a newborn seems particularly fussy. The
mother verifies that this is a very fussy baby and that it is impossible to soothe the baby. In thinking through what to further assess and what to teach the mother, the nurse will keep in mind that fussy babies are: a. often in some kind of pain c.often victims of the mothers drug use b.somewhat neurologically unstabled.at greater risk for abuse ANS: D Feedback AIncorrect: Fussy babies are not necessarily in some kind of pain. Some infants are born with a fussy temperament. BIncorrect: Fussy babies are not somewhat neurologically unstable. CIncorrect: Fussy babies are not necessarily the victim of the mothers drug use. DCorrect: Infants with a fussy, unsoothable, difficult temperament are most likely to resist a caregivers attempt at comforting, placing them at greater risk for abuse. 16. Which of the following children are at greatest risk of abuse or neglect by the parents?
a. postmaturity babies b.13-year-old children c. children with high intelligent quotient (IQ) d.premature infants ANS: D
CURRENT DIAGNOSIS AND TREATMENT PEDIATRICS 24TH LATEST Feedback AIncorrect: Postmature babies are not at greatest risk of abuse or neglect by the parents. BIncorrect: 13-year-old children are not at greatest risk of abuse or neglect by the parents. Instead young children (particularly those younger than 3 years) are at high risk for abuse or neglect. CIncorrect: Children with high intelligent quotient (IQ) are not at greatest risk of abuse or neglect by the parents. DCorrect: Premature infants are at greatest risk of abuse or neglect by the parents because they place great demands on the parents. 17. An infant is separated from his parents for a few minutes while the nurse weighs him. The
infant seems distressed and looks around for the mother. The mother soothes the infant then the nurse is through weighing him. This type of attachment between mother and baby is most likely which of the following types of attachment? a. avoidant c. secure b.disorganizedd.strange ANS: C Feedback AIncorrect: This type of attachment between mother and baby is not likely an avoidant attachment. In avoidant attachment the infant realizes the caregiver will not always be available to provide comfort. The infant exhibits independent behavior without acknowledging the caregiver prior to separation, exhibits minimal distress during separation, and avoids emotional support offered by the caregiver upon reunion. BIncorrect: This type of attachment is not disorganized attachment. In this situation the infant is confused and cannot understand how to get physiological and psychological needs met by the caregiver. This infant may be rejected or comforted by the caregiver. CCorrect: This type of attachment between mother and baby is most likely a secure attachment. DIncorrect: This type of attachment is not strange attachment. Strange is not a type of attachment. 18. The nurse is caring for an infant who shows no distress then her parents leave, and then they
return the infant ignores her parents. There was no evidence of distress while the parents were gone. This type of attachment between the infant and parents is most likely which of the following types of attachment? a. secure c.ambivalent b.avoidantd.detached ANS: B
CURRENT DIAGNOSIS AND TREATMENT PEDIATRICS 24TH LATEST
Chapter 8. Ambulatory / Office Pediatrics 1. In which situation should the nurse address anxiety as a priority problem in planning care for the child and family? a. Twenty-four hour observation b. Emergency hospitalization c. Outpatient admission d. Rehabilitation admission ANS: B Emergency hospitalization involves (1) limited time for preparation both for the child and family, (2) situations that cause fear for the family that the child may die or be permanently disabled, and (3) a high level of activity, which can foster further anxiety. Although preparation time may be limited with a 24-hour observation, this situation does not usually involve the acuteness of the situation and the high levels of anxiety associated with emergency admission. Outpatient admission generally involves preparation time for the family and child. Because of the lower level of acuteness in these settings, anxiety levels are not as high. Rehabilitation admission follows a serious illness or disease. This type of unit may resemble a home environment, which decreases the childs and familys anxiety. 2. What is the primary disadvantage associated with outpatient and day facility care? a. Increased cost b. Increased risk of infection c. Lack of physical connection to the hospital d. Longer separation of the child from the family ANS: C Outpatient and day facility care do not provide extended care; therefore, a child requiring extended care would have to be transferred to the hospital, causing increased stress to the child
CURRENT DIAGNOSIS AND TREATMENT PEDIATRICS 24TH LATEST and parents. Outpatient and day facility care decreases cost, decreases the risk of infection, and minimizes separation of the child from the family. 3. In planning care, the nurse recognizes that which child should have the most difficulty with separation from family during hospitalization? a. A 5-month-old infant b. A 15-month-old toddler c. A 4-year-old child d. A 7-year-old child ANS: B Separation is the major stressor for children hospitalized between the ages of 6 and 30 months. Infants younger than 6 months of age will generally adapt to hospitalization if their basic needs for food, warmth, and comfort are met. Although separation anxiety occurs in hospitalized preschoolers, it is usually less obvious and less serious than that experienced by the toddler. The school-age child is accustomed to separation from parents. Although hospitalization is a stressor, the 7-year-old child will have less separation anxiety than a 15-month-old toddler. 4. A 2-year-old child has been hospitalized for 4 days. The nurse notes the child is quiet and withdrawn. Which is the best explanation for this behavior? a. The child is protesting because of separation from caregivers. b. The child has adjusted to the hospitalization. c. The child is experiencing the despair stage of separation. d. The child has reached the stage of detachment. ANS: C In the despair stage of separation, the child exhibits signs of hopelessness and becomes quiet, withdrawn, and apathetic. The detachment stage occurs after prolonged separation. During this phase, the child becomes interested in the environment and begins to play. In the protest stage,
CURRENT DIAGNOSIS AND TREATMENT PEDIATRICS 24TH LATEST the child would be agitated, crying, resistant to caregivers, and inconsolable. Toddlers do not readily adjust to hospitalization and separation from caregivers. 5. A 3-year-old child cries, kicks, and clings to the father when the parents try to leave the hospital room. What is the nurses best response to the parents about this behavior? a. Your child is showing a normal response to the stress of hospitalization. b. Your child is not coping effectively with hospitalization. Well need to get a psychological consult from the doctor. c. It is helpful for parents to stay with children during hospitalization. d. You can avoid this if you wait to leave after your child falls asleep. ANS: A The child is exhibiting a healthy attachment to the father. The childs behavior represents the protest stage of separation and does not represent maladaptive behavior. Suggesting that the parents stay during hospitalization places undue stress and guilt on the parents. It fosters the childs mistrust when the parent waits to leave after the child falls asleep. 6. A preschool aged child tells the nurse I was bad, thats why I got sick. Which is the best rationale for this childs statement? a. The child has a fear that mutilation will lead to death. b. The childs imagination is very active, and he may believe the illness is a result of something he did. c. The child has a general understanding of body integrity at this age. d. The child will not have fear related to an IV catheter initiation but will have fear of an impending surgery. ANS: B The child has imaginative thoughts at this stage of growth and development. The child may believe that an illness occurred as a result of some personal deed or thought or perhaps because he touched something or someone. Preschoolers do not have the cognitive ability to connect mutilation to death and do not have a sound understanding of body integrity. The preschooler
CURRENT DIAGNOSIS AND TREATMENT PEDIATRICS 24TH LATEST fears all types of intrusive procedures whether undergoing a simple procedure such as an IV start or something more invasive such as surgery. 7. A nurse caring for a hospitalized adolescent should implement which most developmentally appropriate intervention? a. Encouraging peers to call and visit when the adolescents condition allows b. Being sure the adolescent wears a hospital gown or pajamas throughout the hospitalization c. Discouraging questions and concerns about the effects of the illness on the adolescents appearance d. Asking the parents how the adolescent usually copes in new situations ANS: A The peer group is important to the adolescents sense of belonging and identity; therefore, separation from friends is a major source of anxiety for the hospitalized adolescent. Adolescents should be encouraged to wear their own clothes to foster their sense of identity. Questions and concerns about the adolescents appearance and the effects of illness on appearance should be encouraged. How the adolescent copes should be asked directly of the adolescent. 8. The nurse is discussing toddler development with the mother of a 2 1/2-year-old child. Which statement by the mother indicates she has an understanding of the toddlers developmental task? a. I always help my daughter complete tasks to help her achieve a sense of accomplishment. b. I provide many opportunities for my daughter to play with other children her age. c. I consistently stress the difference between right and wrong to my daughter. d. I encourage my daughter to do things for herself when she can. ANS: D The toddlers developmental task is to achieve autonomy. Encouraging toddlers to do things for themselves assists with this developmental task. Toddlers participate in parallel play. They play
CURRENT DIAGNOSIS AND TREATMENT PEDIATRICS 24TH LATEST next to rather than with age mates. Excessive stress on the differences between right and wrong can stifle autonomy in the toddler and foster shame and doubt. 9. Which interventions would best help a hospitalized toddler feel a sense of control? a. Assign the same nurse to care for the child. b. Put a cover over the childs crib. c. Require parents to stay with the child. d. Follow the childs usual routines for feeding and bedtime. ANS: D Familiar rituals and routines are important to toddlers and give the child a sense of control. Following the childs usual routines during hospitalization minimizes feelings of loss of control. Providing consistent caregivers is most applicable for the very young child, such as the neonate and infant. Placing a cover over the childs crib may increase feelings of loss of control. Parents are encouraged, rather than expected, to stay with the child during hospitalization. 10. Parents ask the nurse why observation for 24 hours in an acute-care setting is often appropriate for children. Which is the best response by the nurse? a. Longer hospital stays are more costly. b. Children become ill quickly and recover quickly. c. Children feel less separation anxiety when hospitalized for just 24 hours. d. Families experience less disruption during short hospital stays. ANS: B Children become ill quickly and recover quickly; therefore, they can require acute care for a shorter period of time. A childs state of wellness, rather than cost, determines the length of stay. Separation anxiety is primarily a factor of the stage of development not the length of the hospital stay. Family disruption is a secondary outcome of a childs hospitalization; it does not determine length of stay.
CURRENT DIAGNOSIS AND TREATMENT PEDIATRICS 24TH LATEST 11. The nurse is aware that separation is the major stressor for which age group? a. Newborns and infants b. Infants and toddlers c. Toddlers and preschoolers d. Preschoolers and school-age children ANS: B Separation anxiety is at its peak during the infant and toddler ages. Newborns feel little separation anxiety as long as their comfort needs are met. Preschoolers are most fearful of injury and pain. Loss of control is the primary stressor for school-age children. 12. The nurse recognizes that the stress of hospitalization is increased by an active imagination during unfamiliar experiences for which age group? a. Toddlers b. Preschoolers c. School-age children d. Adolescents ANS: B Active imagination is a primary characteristic of preschoolers. A toddlers primary response to hospitalization is separation anxiety. School-age children experience stress with loss of control. Adolescents experience stress from separation from their peers. 13. Which play activity should the nurse implement to enhance deep breathing exercises for a toddler? a. Blowing bubbles b. Throwing a Nerf ball c. Using a spirometer d. Keeping a chart of deep breathing
CURRENT DIAGNOSIS AND TREATMENT PEDIATRICS 24TH LATEST ANS: A Age-appropriate play for a toddler to enhance deep breathing would be blowing bubbles. Throwing a Nerf ball would not enhance deep breathing. Using a spirometer and keeping a chart of deep breathing are more appropriate for a school-age child. 14. The nurse should plan to explain procedures and encourage selection of their own meals from hospital menus for which age group of patients? a. Toddlers b. Preschoolers c. School-age children d. Adolescents ANS: C School-age children are developmentally ready to accept detailed explanations. They can select their own menus and become actively involved in other areas of their care. Toddlers need routine and parent involvement for coping. Preschoolers need simple explanations of procedures. Detailed explanations and support of peers help adolescents cope. 15. What is the best action for the nurse to take when a 5-year-old child cries, screams, and resists having his IV restarted because an IV antibiotic is due? a. Exit the room and leave the child alone until he stops crying. b. Tell the child big boys and girls dont cry. c. Let the child decide which color arm board to use with his IV. d. Proceed quickly with the IV insertion to decrease stress. ANS: C Giving the preschooler some choice and control, while maintaining boundaries of treatment, supports the childs coping skills. Leaving the child alone robs the child of support when a coping
CURRENT DIAGNOSIS AND TREATMENT PEDIATRICS 24TH LATEST difficulty exists. Crying is a normal response to stress. The child needs time to adjust and support to cope with unfamiliar and painful procedures during hospitalization. 16. What is the best nursing response to the mother of a 4-year-old child who asks what she can do to help the child cope with a siblings repeated hospitalizations? a. Recommend that the child be sent to visit the grandmother until the sibling returns home. b. Inform the parent that the child is too young to visit the hospital. c. Assume the child understands that the sibling will soon be discharged because the child asks no questions. d. Help the mother give the child a simple explanation of the treatment and encourage the mother to have the child visit the hospitalized sibling. ANS: D Needs of a sibling will be better met with factual information and contact with the ill child. Separation from family and home may intensify fear and anxiety. Parents are experts on their children and need to determine when their child can visit a hospital. Children may have difficulty expressing questions and fears and need the support of parents and other caregivers. 17. Parents are expressing concerned about their preschoolers current sleep habits, which were disrupted by a recent hospitalization, stating that the child now awakens frequently at night. Which response by the nurse best addresses this problem? a. Regressive behavior after a hospitalization is normal and is usually short term. b. The child is probably expressing anger. c. Egocentric behavior often manifests itself when the child is left alone to sleep. d. The child is probably feeling pain and needs further evaluation. ANS: A Regression is manifested in a variety of ways, is normal, and usually is short term. Nighttime waking is not associated with anger. Egocentric behavior is not an explanation for nighttime waking. More information is needed before an assessment of pain can be made.
CURRENT DIAGNOSIS AND TREATMENT PEDIATRICS 24TH LATEST 18. Which nursing intervention is appropriate for the hospitalized neonate? a. Assign the neonate to a room with other neonates. b. Provide play activities in the hospital room. c. Offer the neonate a pacifier between feedings. d. Request that parents bring a security object from home. ANS: C The neonate needs opportunities for nonnutritive sucking and oral stimulation with a pacifier. The neonate is not aware of other children. The choice of roommate will not affect the neonate socially. It is important for older children to room with similar-age children. Formal play activities would not be relevant for the neonate. Having parents bring a security object from home is applicable to older children. 19. Which approach will best help a 7-year-old child cope with a lengthy course of intravenous antibiotic therapy? a. Arrange for the child to go to the playroom daily. b. Ask the child to draw you a picture about himself. c. Allow the child to participate in injection play. d. Give the child stickers for cooperative behavior. ANS: C Injection play is an appropriate intervention for the child who has to undergo frequent blood work, injections, intravenous therapy, or any other therapy involving syringes and needles. The hospitalized child should have opportunities to go to the playroom each day if his condition warrants. This free play does not have any specific therapeutic purpose. Children can express their thoughts and beliefs through drawing. Asking the child to draw a picture of himself may not elicit the childs feelings about his treatment. Rewards such as stickers may enhance cooperative behavior. They will not address coping with painful treatments.
CURRENT DIAGNOSIS AND TREATMENT PEDIATRICS 24TH LATEST 20. A 6-year-old child tells the nurse that she does not like the food at the hospital. A review of intake reveals she has eaten very little for the past 2 days. Which intervention is appropriate for the nursing diagnosis: Imbalanced Nutrition: Less than body requirements? a. Select nutritious foods on the menu for the child. b. Permit the child to eat junk foods at snack times. c. Arrange the childs meal tray with generous portions of food. d. Encourage family members to bring foods from home. ANS: D Having the parents bring foods that the child likes and is familiar with will increase the likelihood that she will eat. A 6-year-old child should be permitted to make her own menu selections with the assistance of an adult as needed. Allowing the child to select foods gives the child control and provides an opportunity to select foods that the child likes. Junk foods have little or no nutritional value. If the child is permitted to eat junk food, she may refuse to eat nutritious food at mealtimes. Meals served to children should contain small portions. Children may feel overwhelmed by large portions and refuse to eat any of the food. 21. A 3 1/2-year-old child who is toilet trained has had several accidents since hospital admission. What is the nurses best action in this situation? a. Find out how long the child has been toilet trained at home. b. Tell the parent it is necessary to begin toilet training again. c. Explain how to use a bedpan and place it close to the child. d. Follow home routines of elimination. ANS: D Cooperation will increase and anxiety will decrease if the childs normal routine and rituals are maintained. Some regression to previous behaviors is normal during hospitalization, even when the child has been practicing the skill for some time. Hospitalization is a stressful experience and is not an appropriate time to learn or relearn a skill. Developmentally, the 3 1/2-year-old child cannot use a bedpan independently.
CURRENT DIAGNOSIS AND TREATMENT PEDIATRICS 24TH LATEST 22. Which question would most likely elicit information about how a family is coping with a childs hospitalization? a. Was this admission an emergency? b. How has your childs hospitalization affected your family? c. Who is taking care of your other children while you are here? d. Is this the childs first hospitalization? ANS: B Open-ended questions encourage communication. Ensuring a positive outcome from the hospital experience can be optimized by the nurse addressing the health needs of family members as well as the needs of the child. Was this admission an emergency?, Who is taking care of your other children while you are here?, and Is this the childs first hospitalization? are closed-ended questions. The nurse would have to ask other questions to gather additional information. 23. What would the nurse advise the mother of a 4-year-old child to bring for the child on the day of outpatient surgery? a. Snacks b. Fruit juice boxes c. All of the childs medications d. One of the childs favorite toys ANS: D A familiar toy can be effective in decreasing a childs stress in an unfamiliar environment. The child will be NPO before surgery; therefore, including snacks for the child is contraindicated, however unnecessary stress will result when the child is denied the juice. It is not necessary to bring all medications on the day of surgery. The medication the child has been receiving should have been noted during the preoperative workup. The parent should be knowledgeable concerning which medications the child has been taking if further information is necessary. MULTIPLE RESPONSE
CURRENT DIAGNOSIS AND TREATMENT PEDIATRICS 24TH LATEST 1. Which developmental approaches should the nurse implement for the hospitalized adolescent? Select all that apply. a. Expect regression. b. Provide privacy. c. Encourage peers to call. d. Answer questions with simple concrete explanations. e. Encourage questions about appearance. ANS: B, C, E The developmental appropriate approaches for an adolescent include providing privacy for care and visiting, encouraging questions about appearance and the effects of illness, and encouraging peers to call and visit if the adolescents condition can tolerate this action. The adolescent requires the use of scientific terminology and detailed explanations; answering questions with simple concrete explanations is appropriate for the preschool age. Regression occurs for the school age and younger child, but is not common in the adolescent. 2. Parents of a school-age child ask the nurse about services provided by school-based clinics. Which statement made by the nurse is accurate with regard to school-based clinic services? Select all that apply. a. Vision, hearing, and growth screening are provided. b. Education about health-related topics is provided to children and parents. c. Minor outpatient surgical procedures can be performed. d. Emergency first aid treatment is provided. e. Casts can be applied to broken bones. ANS: A, B, D School-based clinics provide screening, emergency care, healthcare education, and immunization services. They are primary care clinics and cannot perform outpatient surgical procedures or apply casts to broken bones.
CURRENT DIAGNOSIS AND TREATMENT PEDIATRICS 24TH LATEST
Chapter 9. Immunization 1. Which of the following is a consideration in the administration of immunizations? 1. Immunizations can only be administered if a child is free from any illnesses. 2. Caregivers should administer aspirin prior to the injection to prevent any side effects. 3. Children should be isolated from other children for 24 hours after receiving immunizations. 4. Caregivers should be advised that immunizations may cause a mild fever or soreness and redness at the injection site. ANS: 4 Feedback 1. Children with mild signs and symptoms of a cold may receive immunizations. However, if they are febrile, it is better to hold off on the immunization until later. 2. Acetaminophen or ibuprofen are recommended to relieve any pain or fever following immunizations. 3. Generally, children do not need to be isolated from others after immunizations. 4. Some immunizations may cause a mild fever, or soreness and redness at the injection site. Caregivers may give acetaminophen or ibuprofen to relieve any pain or fever. Warm compresses may also be applied to the injection site.
2. A vaccine that contains fragments of cells that stimulate an immune response, but does not contain the whole cell is known as a: 1. Toxoid. 2. Subunit. 3. Acellular.
CURRENT DIAGNOSIS AND TREATMENT PEDIATRICS 24TH LATEST 4. Attenuated. ANS: 3 Feedback 1.A toxoid vaccine is made from toxins produced by bacteria that are inactivated so that they cannot cause harm, but can still stimulate an immune response. 2.A subunit is produced from small fragments of viral protein. 3.An acellular vaccine contains fragments of cells that stimulate an immune response, but do not contain the whole cell. 4.An attenuated vaccine is one produced from the weakened virus.
3. Which of the following is true about immunizations containing thimersal and autism? 1. Thimersal has been removed from all vaccines. 2. Only the MMR vaccine is associated with autism. 3. Thimersal has been linked to learning disabilities, but not autism. 4. Researchers have found no evidence that thimersal is linked to autism. ANS: 4 Feedback 1. Even though researchers have found no evidence that thimersal is linked to autism, the Advisory Committee on Immunization Practices has recommended that thimersal be removed from all vaccines. 2. Currently, this preservative is only found in very small amounts in some flu vaccines. Also, no evidence has been found that the MMR vaccine is associated with autism. 3. Even though researchers have found no evidence that thimersal is linked to autism, the Advisory Committee on Immunization Practices has recommended that thimersal be removed from all vaccines. 4. Even though researchers have found no evidence that thimersal is linked to autism, the Advisory Committee on Immunization Practices has recommended that thimersal be removed from all vaccines.
CURRENT DIAGNOSIS AND TREATMENT PEDIATRICS 24TH LATEST 4. A 27-year-old mother has just delivered a healthy 7 lb. 3 oz. baby boy. Which of the following is true regarding the babys immunization schedule for Hepatitis B? 1. The infant will need two doses administered at least 6 months apart. 2. The infant should not receive the vaccine until at least 12 months of age. 3. The infant will not need any vaccines at this time because of passive immunity from the mother. 4. If the mother is Hepatitis B surface antigen positive, the baby will need 0.5ml of Hepatitis B Immune Globulin. ANS: 4 Feedback 1. Three doses of Hepatitis B are given before age 2: at birth, at 1 to 2 months of age, and after 24 weeks of age. 2. Three doses of hepatitis B are given before age 2: at birth, at 1 to 2 months of age, and after 24 weeks of age. 3. Immunity will not occur passively. 4. If the mother is Hepatitis B surface antigen (HbsAg) positive, 0.5 ml of Hepatitis B Immune Globulin (HBIG) is also given.
5. A mother is bringing her 2-month-old son in for his well-child visit. He is due to receive his
Diphtheria, Tetanus, and Pertussis vaccine. The nurse is educating the mother on the possible side effects of the vaccine, which include: 1. Pain and swelling in the joints. 2. Irritability, loss of appetite, and seizures. 3. Nausea, vomiting, and abdominal pain. 4. Fever and decreased white blood cell count. ANS: 2
CURRENT DIAGNOSIS AND TREATMENT PEDIATRICS 24TH LATEST Feedback 1. Swelling occurs at the site, not the joints. 2. Side effects of the DTaP vaccine are irritability, loss of appetite, localized swelling, and seizures, which are rare. 3. Nausea and vomiting are rare. 4. Fever and a decreased white blood cell count are rare.
6. When a fetus receives antibodies from the mother by way of the placenta, this is known as: 1. Active immunity. 2. Natural Passive immunity. 3. Transmission immunity. 4. Attenuated immunity. ANS: 2 Feedback 1. Active immunity is when a person is exposed to the disease organism
and makes his or her own antibodies. 2. Natural Passive immunity, a form of passive immunity, is when antibodies are passed from mother to fetus by way of the placenta 3. Transmission immunity does not exist. 4. Attenuated immunity occurs when a vaccine is given.
7. Which of the following is a high risk group among children and adolescents for contracting Hepatitis B? 1. Those living in institutions 2. Individuals infected by sexual partners 3. Children who are hemophiliacs or are receiving frequent blood transfusions 4. All of the above
CURRENT DIAGNOSIS AND TREATMENT PEDIATRICS 24TH LATEST ANS: 4 Feedback 1. High risk groups include those living in institutions. 2. High risk groups include those living in institutions, those involved in IV drug use, and those infected by sexual partners. 3. Hepatitis B is transmitted by blood or blood products, or by sexual contact. Children who are hemophiliacs or are receiving frequent blood transfusions are at risk. Individuals who have traveled to Africa or Asia are also at higher risk. 4. Hepatitis B is transmitted by blood or blood products, or by sexual contact. High risk groups include those living in institutions, those involved in IV drug use, those infected by sexual partners, and children who are hemophiliacs or are receiving frequent blood transfusions. Individuals who have traveled to Africa or Asia are also at higher risk
8. A 15-year-old girl has been diagnosed with Hepatitis B and will be starting treatment with Interferon-alpha (IFN). The nurse needs to educate the patient on this medication. Which of the following should be included in this education? 1. The medication reduces the replication of the Hepatitis B virus. 2. The patient needs to remain on treatment for one year. 3. Patients may develop resistance to Interferon-alpha over time. 4. Interferon-alpha has fewer side effects than Lamivudine, which is also used to treat Hepatitis B. ANS: 1 Feedback 1. Interferon-alpha reduces the replication of the HBV virus. The medication is given as a subcutaneous injection three times a week for 4 to 6 months. Lamivudine, the other drug used to treat Hepatitis, has fewer side effects, but may patients develop a resistance. 2. The medication is given as a subcutaneous injection three times a week for 4 to 6 months. 3. Lamivudine, the other drug used to treat Hepatitis, has fewer side effects, but may patients develop a resistance.
CURRENT DIAGNOSIS AND TREATMENT PEDIATRICS 24TH LATEST 4. Lamivudine, the other drug used to treat Hepatitis, has fewer side
effects, but may patients develop a resistance.
9. A 6-year-old boy with Varicella-zoster (chickenpox) and a visible rash is being admitted to the hospital for treatment of dehydration. Which of the following isolation precautions needs to be taken? 1. Contact 2. Airborne 3. Contact and droplet 4. Contact and airborne ANS: 4 Feedback 1. Transmission of the Varicella-zoster virus is from contact with fluid from vesicles of an infected person, secretions from the nose, mouth and eyes, and airborne from coughing and sneezing. 2. Transmission of the Varicella-zoster virus is from contact with fluid from vesicles of an infected person, secretions from the nose, mouth and eyes, and airborne from coughing and sneezing. 3. The transmission does not occur with droplets. 4. Transmission of the Varicella-zoster virus is from contact with fluid from vesicles of an infected person, secretions from the nose, mouth and eyes, and airborne from coughing and sneezing.
10. The mother of a 4-year-old girl brings her daughter in to be evaluated for a fever of 101 degrees and a headache that has lasted for the past three days. The mother was concerned this morning when the child woke up and had reddened cheeks that looked like she had been slapped. The nurse suspects the child has: 1. Impetigo.
CURRENT DIAGNOSIS AND TREATMENT PEDIATRICS 24TH LATEST 2. Fifths disease. 3. German measles. 4. Hand-Foot-Mouth Disease ANS: 2 Feedback 1. Crustations develop and serous drainage is present 2. The prodromal signs and symptoms of Fifths disease include fever, aching, and a headache. The child will also have erythema of the cheeks, giving the appearance of slapped cheeks. 3. Swelling of the glands and a rash occur 4. The hands and feet become very red, and a fever is usually present.
11. Which of the following should be considered in the administration of the influenza vaccine? 1. The vaccine is contraindicated for individuals who are allergic to eggs or egg products. 2. The Live Attenuated Influenza Virus (LAIV) is a safe alternative for children of any age. 3. The influenza vaccine is contraindicated in children with asthma and wheezing within the past 12 months. 4. Since influenza has a bimodal seasonal pattern, children under 18 years of age should receive two doses of the vaccine. ANS: 1 Feedback 1. The influenza vaccine is manufactured from the protein of eggs;
therefore, the vaccine is contraindicated for those individuals who are allergic to eggs or egg products. 2. The live vaccine should not be given to children with asthma issues.
CURRENT DIAGNOSIS AND TREATMENT PEDIATRICS 24TH LATEST 3. The nasal spray, live attenuated influenza virus is contraindicated for
children with asthma and should not be given to children ages 2 to 4 who have been wheezing within the past 12 months. 4. Children through 8 years of age who are receiving their first influenza immunization need two doses, at least four weeks apart.
12. The period of time when a child begins to have mild signs and symptoms until the main illness appears, and which is associated with increased communicability, is referred to as: 1. Transmission. 2. Susceptibility. 3. Prodromal period. 4. Incubation period. ANS: 3 Feedback 1.Transmission is the mode of infecting others. 2.Susceptibility is the ability of a person to contract the illness. 3. Prodromal signs and symptoms may appear before a rash or the main illness appears and should be included in the physical assessment of a child with a communicable disease. The prodromal period is often associated with increased communicability of the disease. Prodromal signs and symptoms may include coryza, cough, fever, and malaise. 4. The incubation period is the time it takes for an individual to contract the disease to the start of the prodromal signs and symptoms.
13. Reyes Syndrome can be a life-threatening disease affecting the brain and liver of children. The cause of this disease comes from the ingestion of: 1. Ibuprofen during a viral illness. 2. Ibuprofen during a bacterial illness.
CURRENT DIAGNOSIS AND TREATMENT PEDIATRICS 24TH LATEST 3. Aspirin or other salicylates during a viral illness. 4. Aspirin or other salicylates during a bacterial illness. ANS: 3 Feedback 1.Ibuprofen is safe for viral illnesses. 2.Ibuprofen is safe for bacterial illnesses. 3. Reyes Syndrome typically follows a viral infection, such as chickenpox, influenza, or an upper respiratory infection. The ingestion of aspirin or other medication containing salicylates during a viral illness greatly increases the probability of developing the syndrome. Caregivers should be taught not to give aspirin or salicylate products to any child or adolescent during a febrile illness. 4. Reyes Syndrome typically follows a viral infection, so it is acceptable to give aspirin or other salicylates for a bacterial infection. 14. A mother brings in her 4-year-old daughter to be evaluated for a rash all over her torso. The mother states that the 4 year old was recently around her grandmother, who is presently recovering from shingles. The mother is now concerned that the child has contracted shingles. Which of the following would be an appropriate response by the nurse to the mother? 1. It is possible that she has contracted shingles from her grandmother. 2. She has contracted chickenpox since both diseases are from the same virus. 3. She can only get shingles from someone who has chickenpox. 4. This rash does follow the normal pattern along a dermatome or particular nerve fiber. ANS: 1 Feedback 1. However, a child may contract chickenpox from an individual with
shingles. 2. Shingles is a reactivation of the Varicella-zoster virus that causes
chickenpox. Caregivers should be educated that a child cannot get shingles from someone with chickenpox. However, a child may contract chickenpox from an individual with shingles. The chickenpox rash first appears on the trunk and face, whereas shingles follows dermatomes or particular nerve fibers.
CURRENT DIAGNOSIS AND TREATMENT PEDIATRICS 24TH LATEST 3. Caregivers should be educated that a child cannot get shingles from
someone with chickenpox. However, a child may contract chickenpox from an individual with shingles. The chickenpox rash first appears on the trunk and face, whereas shingles follows dermatomes or particular nerve fibers. 4. The chickenpox rash first appears on the trunk and face, whereas shingles follows dermatomes or particular nerve fibers. 15. A 6-year-old boy is admitted to the hospital with a severe case of chickenpox with persistent fever. The nurse monitors the child for which of the following complications? 1. Pneumonia 2. Kidney failure 3. Seizure activity 4. Rheumatic fever ANS: 1 Feedback 1. Complications from varicella-zoster or chickenpox include bacterial infections of the skin, pneumonia, septicemia, encephalitis, and bleeding problems. 2. Kidney issues are not usually present with chickenpox. 3.Seizures are rare with chickenpox. 4.Rheumatic fever is a complication of other bacteria.
16. A nurse is working on a pediatric unit and notices that many of the children on the unit are admitted with a communicable disease. She realizes that the best action she can take to prevent the spread of communicable diseases is: 1. Hand washing. 2. Wearing gloves. 3. Wearing gloves and a gown.
CURRENT DIAGNOSIS AND TREATMENT PEDIATRICS 24TH LATEST 4. Placing the child in isolation. ANS: 1 Feedback 1. The key to preventing the spread of communicable diseases is hand
washing. Caregivers and children must be taught to wash hands before and after caring for infants and children, before eating and handling food, after using the bathroom or changing diapers, after handling animals, after playing in water or in the sand, and after using tissues to wipe eyes or noses. 2. Wearing gloves is only effective if hand washing occurs before and after use. 3. Gowning and gloving are the not primary barriers for preventing the spread of disease. 4. The child does not need isolation at this time.
17. An 18-year-old boy is getting ready to leave for college and will be living in a dormitory. Which of the following vaccines is recommended prior to his departure? 1. Hepatitis A 2. Pneumococcal polysaccharide (PPSV23) 3. Pneumococcal conjugate vaccine (PCV13) 4. Meningococcal conjugate vaccine (MCV4) ANS: 4 Feedback 1.Hepatitis A is not recommended at this time. 2.The vaccine is not recommended at this time. 3.The vaccine is not recommended at this time. 4.College students are an at risk population for contracting meningitis since they live in close proximity to each other. The meningococcal conjugate vaccine (MCV4) is recommended for college students for the prevention of meningitis.
CURRENT DIAGNOSIS AND TREATMENT PEDIATRICS 24TH LATEST 18. A 16-year-old girl with mononucleosis is being evaluated for the development of a maculopapular rash. The nurse should ask the patient if she has taken which of the following medications? 1. Acetaminophen 2. Acyclovir 3. Amoxicillin 4. Aspirin ANS: 3 Feedback 1. Acetaminophen will not cause the side effects and is an acceptable
medication. 2. Acyclovir is contraindicated.
3.A maculopapular rash will appear if amoxicillin or ampicillin is given to an individual with mononucleosis. 4.Aspirin should not be given to a child or an adolescent at any time.
19. An 8-year-old boy was just diagnosed with the mumps. The nurse is developing a plan of care for the boy, which should include which of the following interventions? 1. Administer intravenous Acyclovir as ordered. 2. Drink plenty of fluids, including citrus juices high in Vitamin C. 3. Apply ice packs or warm compresses to the neck for comfort and pain relief. 4. Place the patient in contact precautions. ANS: 3 Feedback 1.Acyclovir is not an effective treatment for mumps 2.Citrus juice should not be provided during this time.
CURRENT DIAGNOSIS AND TREATMENT PEDIATRICS 24TH LATEST 3. Interventions for mumps include respiratory isolation; acetaminophen or
ibuprofen for fever and pain; bland, soft foods; hydration with intravenous fluids or bland liquids, avoiding citrus juices; ice packs or warm compresses for the neck for comfort and pain relief; and snugfitting underwear and warmth to provide comfort and pain relief for orchitis. Acyclovir is ineffective for the treatment of mumps. 4. Good hand washing is needed, not contact precautions.
20. Which of the following vaccines is contraindicated for those who are pregnant or immunocompromised? 1. Tdap and Rotarix 2. MMR and varicella 3. Hepatitis and pneumococcal 4. Pertussis and hemophilus influenza type b ANS: 2 Feedback 1. Tdap and Rotarix are not active viruses and can be given. 2. The MMR and varicella vaccines are manufactured from live attenuated
or weakened viruses, and are contraindicated for those that are pregnant or immunosuppressed. 3. Hepatitis and pneumococcal are not live viruses. 4.Pertussis is not a live vaccine.
21. Small red spots in the mouth with a bluish-white center, also known as Koplik spots, are a characteristic sign of: 1. Roseola 2. Rotavirus 3. Rubella
CURRENT DIAGNOSIS AND TREATMENT PEDIATRICS 24TH LATEST 4. Rubeola ANS: 4 Feedback 1.Roseola will cause a fine pin point rash on the trunk of the body. 2.Rotovirus will cause massive diarrhea stools. 3.Rubella cause the swelling of lymph nodes and a fine rash. 4.The prodromal signs and symptoms include coryza, cough, conjunctivitis, fever, malaise, and Koplik spots, small red spots in the mouth with a bluish-white center.
22. A 6 month old is brought in to the office for a well-child visit. The baby was born prematurely at 34 weeks gestation. Which of the following vaccines is recommended for the infant to reduce the risk of Respiratory Syncytial Virus Bronchiolitis? 1. Synagis 2. RotaTeq 3. Relenza 4. Immune globulin ANS: 1 Feedback 1. Synagis has significantly reduced the incidence of RSV Bronchiolitis in infants. The immunization is recommended for infants who were born at less than 35 weeks gestational age or who have chronic lung or heart disease. 2. RotaTeq is used for the prevention of rotovirus, not RSV 3.Relenza is used for the treatment of influenza, not RSV. 4.The immune globulin is used in situations for patients that are not able to have the synagis vaccine.
CURRENT DIAGNOSIS AND TREATMENT PEDIATRICS 24TH LATEST 23. Which of the following supplements has been recommended by the World Health Organization and has been found to reduce the risk of pneumonia and death from measles in children under age 2? 1. Iron 2. Folic Acid 3. Vitamin A 4. Vitamin D ANS: 3 Feedback 1. Iron deficiency does not influence the occurrence of pneumonia or
measles. 2. Folic acid does not influence the occurrence of pneumonia or measles.
3.A systematic review and meta-analysis has found that Vitamin A therapy reduced the risk of pneumonia and death as a complication of measles for children under age 2 in areas of the world where Vitamin A deficiency was present. 4.Vitamin D does not influence the occurrence of pneumonia or measles.
24. A 6-week-old infant is brought into the emergency department with a cough that ends in a crowing sound, followed by vomiting. The nurse suspects which of the following illnesses? 1. Pertussis 2. Diptheria 3. Bronchiolitis 4. Respiratory Syncytial Virus ANS: 1 Feedback
CURRENT DIAGNOSIS AND TREATMENT PEDIATRICS 24TH LATEST 1. Characteristic signs and symptoms of pertussis, or whooping cough,
include cold symptoms with coryza, mild cough and fever in the early stages, progressing to a cough ending with a crowing (whooping) that may be severe enough to cause vomiting and cyanosis. 2. The child will have diarrhea and fevers with diphtheria. 3. The child will have symptoms of wheezing and difficulty breathing with Bronchiolitis. 4. The child will have large amounts of mucous and wheezing present with RSV.
25. A 17-year-old boy is admitted with meningitis and for treatment of dehydration. The patient needs to be placed on droplet precautions. Which of the following interventions should be taken for droplet precautions? 1. 1. Anyone entering the room must wear gloves and a gown. 2. 2. The patient needs to be placed in a negative pressure room. 3. 3. Anyone entering the room needs to wear a respirator mask upon entering. 4. 4. The patient must wear a mask when being transported outside of the room. ANS: 4 Feedback 1.Gloves and a gown are required for contact precautions. 2.A negative pressure room is required for those on airborne precautions. 3.Respirator masks are required for some airborne illnesses. 4.Droplet precautions prevent the spread of diseases by close respiratory contact or respiratory secretions. Those in close contact with the patient must wear a mask. The patient must wear a mask when being transported outside of the room. A negative pressure room is required for those on airborne precautions. Respirator masks are required for some airborne illnesses. Gloves and gown are required for contact precautions.
26. A 5-year-old girl is being admitted for dehydration from influenza. The girl is started on Tamiflu. Which of the following is true about the administration of this medication? 1. Tamiflu is only effective for Influenza type A.
CURRENT DIAGNOSIS AND TREATMENT PEDIATRICS 24TH LATEST 2. Tamiflu is only effective for Influenza type B. 3. The medication must be given within 48 hours of the onset of symptoms. 4. The medication is not recommended for children with airway diseases, such as asthma. ANS: 3 Feedback 1. Two FDA approved antiviral medications have been recommended for influenza: Tamiflu and Relenza. These medications must be given within 48 hours of the onset of symptoms and are effective for both Influenza types A and B. 2. Two FDA approved antiviral medications have been recommended for influenza: Tamiflu and Relenza. These medications must be given within 48 hours of the onset of symptoms and are effective for both Influenza types A and B. 3. Two FDA approved antiviral medications have been recommended for influenza: Tamiflu and Relenza. These medications must be given within 48 hours of the onset of symptoms and are effective for both Influenza types A and B. Tamiflu may be given to children over 1 year of age, whereas Relenza is recommended for children over 7 years old. Relenza is administered as an inhalation medication and is not recommended for children with airway diseases, such as asthma. 4. Relenza is administered as an inhalation medication and is not recommended for children with airway diseases, such as asthma.
27. The agent responsible for the Hand-Foot-Mouth Disease is: 1. Paramyxovirus. 2. Coxsackie Virus. 3. Human Herpesvirus 6. 4. Human Parvovirus B 19. ANS: 2 Feedback
CURRENT DIAGNOSIS AND TREATMENT PEDIATRICS 24TH LATEST 1. Paramyxovirus is the agent responsible for mumps. 2. The Coxsackie, or enterovirus, is the responsible agent for Hand-Foot-
Mouth Disease. 3. The Human Herpesvirus 6 is responsible for Roseola. 4. The Human Parvovirus B 19 is the agent responsible for Fifths disease.
28. A 16-year-old girl comes into the office for a sports physical. The recommendation is made for the girl to receive the Human Papillomavirus vaccine. Her mother is upset about the recommendation because she is sure her daughter is not yet sexually active. Which of the following responses would be most appropriate for the nurse to give? 1. Your daughter should receive the vaccine even though she has a yeast allergy. 2. We need to give her a pregnancy test as well prior to administering the vaccine. 3. She is overdue for the vaccine because she should have received it when she was 9. 4. It is best if the vaccine is given before she becomes sexually active in order to prevent genital warts and cervical cancer in the future. ANS: 4 Feedback 1.The vaccine should not be given if the child is allergic to yeast. 2.The nurse should educate the mother and daughter that the vaccine is best administered prior to becoming sexually active to prevent genital warts and cervical cancer from an HPV infection. 3. The HPV vaccine is generally given to girls ages 11 to 18, but can be given as early as age 9. 4. The HPV vaccine protects girls against HPV infections that may lead to genital warts and cervical cancer. It is generally given to girls ages 11to 18 prior to being sexually active, but can be given as early as age 9. The vaccine should not be administered if the girl is allergic to yeast. Healthcare workers should check for pregnancy before giving the vaccine. Boys ages 9 to 18 may also receive the vaccine.
CURRENT DIAGNOSIS AND TREATMENT PEDIATRICS 24TH LATEST 29. A 2-month-old infant is presenting for a well-child visit and is due to receive the Rotavirus vaccine. The nurse recognizes that obtaining the patients history is essential in the administration of immunizations and preventing communicable disease. Which of the following should the nurse assess for prior to administering this vaccine? 1. Lung disease 2. Cardiac defects 3. Seizure disorder 4. Gastrointestinal disorders ANS: 4 Feedback 1.The immunization does not interfere with lung disease. 2.The immunization does not interfere with cardiac defects. 3.The immunization does not interfere with seizure disorders. 4.Rotavirus causes severe diarrhea and dehydration. The first immunization is given between 6 and 14 weeks. The immunization should be avoided if the child has a history of intussusception or other gastrointestinal disorders.
30. A 3 year old is brought to the emergency department with a fever that has lasted for the past three days. The mother is concerned because the toddler is very lethargic. The child is crying, but the nurse notices no tears, and her eyes look sunken. The nurse anticipates which of the following priority interventions? 1. Administering intravenous fluids 2. Placing the child in contact precautions 3. Decreasing stimulation and providing time to rest 4. Administering antibiotics to reduce the childs fever
CURRENT DIAGNOSIS AND TREATMENT PEDIATRICS 24TH LATEST ANS: 1 Feedback 1. This child is demonstrating signs and symptoms of dehydration and needs immediate intervention with intravenous fluids. The nurse should also assess if the child has had a decreased urine output and continue to monitor the childs intake and output. 2. The child is past the transmission state of illness. 3. These are important factors, but they are not the priority at this time. 4.Viral infections can cause fevers, and no antibiotics should be used at that time.
31. The Pneumococcal (PPSV23) vaccine is recommended for children with which of the following high-risk conditions? 1. Asplenia 2. Cochlear implants 3. Congenital heart defects 4. All of the above ANS: 4 Feedback 1. The Pneumococcal (PPSV23) vaccine is recommended for children and
adolescents with high-risk conditions, such as asplenia. The vaccination should be repeated after five years. 2. The Pneumococcal (PPSV23) vaccine is recommended for children and adolescents with high-risk conditions, such as cochlear implants. The vaccination should be repeated after five years. 3. The Pneumococcal (PPSV23) vaccine is recommended for children and adolescents with high-risk conditions, such as congenital heart disease. The vaccination should be repeated after five years. 4. The Pneumococcal (PPSV23) vaccine is recommended for children and adolescents with high-risk conditions, such as cochlear implants, sickle cell anemia, congenital heart disease, or asplenia. The vaccination should be repeated after five years.
CURRENT DIAGNOSIS AND TREATMENT PEDIATRICS 24TH LATEST
32. The parents of a 1 year old have brought their child in to receive his vaccinations. The child is due for his fourth and final dose of Haemophilus Influenzae type B (HIB). The nurse reviews with the parents the importance of the vaccine in the prevention of: 1. Pneumonia. 2. Otitis media. 3. Liver disease. 4. Vomiting and diarrhea. ANS: 2 Feedback 1. The vaccine prevents conjunctivitis, otitis media, sinusitis, and has
greatly reduced meningitis in young children. 2. Haemophilus Influenzae type B is a bacteria that causes infection in
various parts of the body. The vaccine prevents conjunctivitis, otitis media, sinusitis, and has greatly reduced meningitis in young children. 3. The vaccine does not prevent against liver disease. 4.The vaccine will not stop vomiting or diarrhea.
33. A family recently returned from a trip to Mexico. About one month after returning, the parents noticed that their 6-year-old child has dark urine and has been complaining of a stomachache and a poor appetite. Considering the history and clinical presentation of the child, the nurse suspects that the child may have which of the following? 1. Malaria 2. Hepatitis A 3. Hepatitis B 4. Clostridium Difficile
CURRENT DIAGNOSIS AND TREATMENT PEDIATRICS 24TH LATEST ANS: 2 Feedback 1.A child with malaria would have constant diarrhea and fevers. 2.The clinical presentation in this scenario is that of Hepatitis A. The disease is often contracted when children travel outside of the country and ingest contaminated food or water. They may present with malaise, poor appetite, nausea, jaundice, abdominal pain, and dark urine. Children under the age of 6 may have mild or no symptoms. Children who are at risk for the disease or are traveling should receive the Hepatitis A vaccine. 3. The child would have signs of liver issues if the diagnosis was Hepatitis B. 4. The child is at low risk of contracting C. Diff in a foreign country. 34. A mother and father are reading a pamphlet about the Hepatitis B series of vaccines for their newborn daughter. The father asks the nurse how many shots their baby will need to receive. The nurse knows that her teaching has been effective when the parents respond with: 1. A shot will be given today, then at her two month checkup, and again at her six month checkup. 2. The shot cannot be given until she is at least 6 months old because she will need to build some immunity to the virus first. 3. The injection is given in just one does, and then a booster is administered before she goes to school. 4. She does not need the immunization because her mother was Hepatitis B negative. ANS: 1 Feedback 1. This is the correct time frame for the vaccine administration for a newborn. 2. The vaccine can be given at birth, and the parents do not need to wait. 3.The vaccine requires a series of three injections. 4.Her mothers status does not influence the need for the vaccine.
CURRENT DIAGNOSIS AND TREATMENT PEDIATRICS 24TH LATEST 35. A 9-year-old girl is scheduled to receive a series of the Hepatitis B vaccines because she did not receive them at birth. The child comes to the outpatient clinic for her second dose. Before offering the vaccine, the nurse assesses the child and asks the parents if she has an allergy to: 1. Eggs. 2. Penicillin. 3. The previous dose of the Hepatitis B vaccine. 4. Sulfonamides. ANS: 3 Feedback 1. This allergy does not affect the administration of the Hepatitis B
vaccine. 2. This allergy does not affect the administration of the Hepatitis B
vaccine. 3. If a reaction occurred with the first dose, there is a high probability the child is allergic to the vaccine, thus it should not be repeated. 4. This allergy does not affect the administration of the Hepatitis B vaccine. 36. The family is asking questions about the differences between Hepatitis A and Hepatitis B vaccines for their 2-year-old daughter. The nurse knows that the family understands the difference between the vaccines when the father states: 1. The Hepatitis A vaccine should be given at birth, then repeated six months later. 2. The Hepatitis A vaccine is given in two doses, at least six months apart. 3. The Hepatitis B vaccine requires a live virus, and the Hepatitis A vaccine does not. 4. The Hepatitis B vaccine should not be given because our child is allergic to eggs. ANS: 2 Feedback
CURRENT DIAGNOSIS AND TREATMENT PEDIATRICS 24TH LATEST 1. The Hepatitis A vaccine should not be given until after the age of 12
months. 2. The Hepatitis A vaccine should be given in this manner.
3.Neither vaccine is live. 4.Hepatitis B vaccines can be given, even if the child is allergic to eggs.
38. The pediatric nurse is discussing vaccines for a well-child check at 1 year of age. Common vaccines to give at this time include: 1. Hib (Hemophilus Influenzae Type B) 2. RotaTeq. 3. MCV4 (Meningococcal). 4. DRaP (Diphtheria, Tetanus, Pertussis). ANS: 1 Feedback 1. Recommended at this time 2. Not all children are required to have this vaccine. 3. Recommended later in life 4. Recommended earlier in life
39. A nurse is reviewing the immunization schedule for a 9-month-old infant. The schedule notes that the child is due to receive the Rotarix vaccine today. During the assessment, the mother tells the nurse that the child had surgery three weeks ago for intussusception. The nurse knows that: 1. The child should wait to get the vaccine because of the recent surgery. 2. The child should not receive the vaccine because of the past history of intussusception. 3. The child should be positioned so that the medication can be given orally. 4. The child should receive the medication regardless of past history.
CURRENT DIAGNOSIS AND TREATMENT PEDIATRICS 24TH LATEST ANS: 2 Feedback 1. The timing of the vaccine is not the priority. 2. A child with a GI issue should not receive Rotarix. 3. Does not come in oral form 4. Past history influences if the child can have the vaccine.
40. Lacey, a 9-year-old African American girl, has a history of Sickle Cell disease. She is scheduled to receive her polio vaccine today. The nurse knows that she should give the Inactive Poliovirus Vaccine (IPV) because: 1. Laceys race has a higher risk of reaction to the oral vaccine. 2. Lacey is too old for the oral vaccine. 3. Lacey is at high risk for paralysis because of her immunodeficiency. 4. The oral vaccine is not as effective for females as it is for males. ANS: 3 Feedback 1.Race does not cause the child to be at a higher risk for polio. 2.Age is not a disqualification for the vaccine. 3. Autoimmune deficiency can increase the occurrence of paralysis if the vaccine if given. 4. Gender does not determine the effectiveness of the vaccine. 41. Keegan, a 9-year-old patient, has been brought to the pediatric clinic for an influenza vaccine. The nurse knows that this is Keegans first time receiving the vaccine. The nurse knows the mother understands the instructions for the vaccine when she states: 1. I will not need to make sure he receives the vaccine every year. 2. I cannot let him be around other children for at least 12 hours because he is contagious. 3. I will need to watch him to see if he has an allergic reaction to eggs after this vaccine.
CURRENT DIAGNOSIS AND TREATMENT PEDIATRICS 24TH LATEST 4. I will need to bring him back in for a second immunization in four weeks. ANS: 4 Feedback 1.A two-step flu vaccine is required when receiving the vaccine for the first time. 2.The vaccine does not spread the flu virus. 3.Reactions with eggs do not occur with this vaccine. 4.A two-step vaccination process is needed when receiving the flu vaccine for the first time.
42. A nurse has given instructions to the mother of a 5 year old child on common side effects after the measles, mumps, and rubella (MMR) vaccine. The common reactions include all but: 1. Fever. 2. Swollen cheeks. 3. Pin point rash. 4. Joint pain. ANS: 3 Feedback 1. Common side effect of the MMR vaccine 2. Common side effect of the MMR vaccine 3. An uncommon side effect and further assessment is needed 4. Common side effect of the MMR vaccine 43. A 1 year old has received her MMR and Varivax vaccines at the clinic today. The nurse explains the chance for rash and gives visual aids to the mother to take home. The nurse knows that the mother understands the normal reaction to Varivax when she points out: 1. A varicella-type rash. 2. A maculopapular rash.
CURRENT DIAGNOSIS AND TREATMENT PEDIATRICS 24TH LATEST 3. A pin point rash. 4. A strawberry-patterned rash on her tongue. ANS: 1 Feedback 1. This is a normal reaction 2. An abnormal reaction with the Varivax vaccine 3. An abnormal reaction with the Varivax vaccine 4. An abnormal reaction with the Varivax vaccine 44. A nurse is assessing a pediatric patient prior to giving the meningococcal vaccine. Which statement by the father would be an indication not to give the patient the vaccine? 1. She is allergic to those latex gloves you are using, so can you please use the latex-free ones? 2. She is allergic to those latex-free gloves, so please use the latex gloves. 3. She is allergic to gluten, so this vaccine should not be given. 4. She is allergic to rubbing alcohol, so please do not cleanse the site with it. ANS: 1 Feedback 1. The meningococcal vaccine should not be given to a child with a latex
allergy. 2. Latex-free gloves do not cause hypersensitivity to the vaccine. 3.Gluten is not in the meningococcal vaccine. 4.Rubbing alcohol will not cause a major allergic reaction. 45. Iona has been brought to the clinic for her well-child checkup and immunizations. When the nurse completes the assessment, Iona is noted to have a low-grade fever. The nurse knows that: 1. Iona should not receive the immunizations because her body is ill. 2. Iona can receive the immunizations since the fever is low grade.
CURRENT DIAGNOSIS AND TREATMENT PEDIATRICS 24TH LATEST 3. Iona can receive the immunizations because the low-grade fever will increase the effectiveness of the vaccine. 4. It is important to remain on schedule with the immunizations. ANS: 1 Feedback 1.A patient should be in good health prior to the administration of vaccines to help reduce the side effects. 2.A patient should be in good health prior to the administration of vaccines to help reduce the side effects. 3.A low-grade fever will increase the occurrence of the child obtaining the disease process. 4.It is important to remain on schedule with immunizations, but typical childhood illnesses last 10 to 14 days, and this will not change the schedule enough to make a difference. 46. The purpose of the vaccine information statement is: 1. To give the purpose of the vaccine. 2. To provide a list of possible side effects. 3. To answer some questions for the caregiver. 4. All of the above ANS: 4 Feedback 1.Correct, along with other answers 2.Correct, along with other answers 3.Correct, along with other answers 4.All of the statements should be included during teaching about immunizations. 47. Stella, a 16year-old girl, has come to the clinic to receive the HPV vaccine. Prior to the administration of the vaccine, the nurse must assess for: 1. Allergies to latex.
CURRENT DIAGNOSIS AND TREATMENT PEDIATRICS 24TH LATEST 2. Allergies to eggs. 3. The last sexual encounter. 4. Pregnancy. ANS: 4 Feedback 1.Latex is not a concern with this vaccination. 2.An allergy to eggs would not be a concern. 3.The last sexual encounter will not influence administration of the vaccine. 4.A woman who is pregnant should not receive the vaccine. 48. Identify the vaccine that is given to students prior to living in college dormitories. 1. Influenza 2. Meningococcal vaccine 3. HPV 4. MMR ANS: 2 Feedback 1. The influenza vaccine is required every year. 2. This disease is highly contagious in close living quarters and is required by many universities priority to living in the dormitories. 3. The HPV vaccine should be given prior to living in the dorms. 4.MMR is wanted, but not a requirement for many universities. 49. Micah has come for his pre-kindergarten checkup at the pediatric clinic. His mother reports that Micah is behind on his immunizations because of issues with extreme prematurity. She also reports that Micah had chicken pox three months ago. The nurse knows that Micah will have which type of immunity toward the Varicella-zoster virus? 1. Vaccine-induced immunity
CURRENT DIAGNOSIS AND TREATMENT PEDIATRICS 24TH LATEST 2. Natural active immunity 3. Passive active immunity 4. None of the above ANS: 2 Feedback 1. The varicella-zoster vaccine was not given in order to produce immunity. 2. Since the child had the disease, the body is now immune to the disease. 3.The child did not receive immunity from his mother. 4.At least one of the above is correct. 50. In a nursing class, the instructor is discussing the differences between acellular vaccines and toxoids. The students are able to identify acellular vaccines because: 1. The vaccines produce a toxin to get an immune response. 2. The vaccine has a whole viral protein cell, unlike the toxoids. 3. The vaccine has fragments of cells to stimulate an immune response. 4. The vaccine is inactive and produced by whole cell bacteria. ANS: 3 Feedback 1.The toxin does not produce the immune response. 2.The cell proteins are not whole. 3. Fragments of the cell cause the immune response. 4. The vaccine is active and requires stimulation to create the immune response. 51. A nursing student is about to give a newborn baby a Hepatitis B injection. The student knows that the injection must be given IM. The best place to give an IM injection on a newborn is: 1. The deltoid.
CURRENT DIAGNOSIS AND TREATMENT PEDIATRICS 24TH LATEST 2. The abdomen. 3. The rectus femoris. 4. The forearm. ANS: 3 Feedback 1.The deltoid lacks a large area of muscle in a newborn for IM injections. 2.IM injections should not be given in the abdomen. 3. The rectus femoris is easily identified and large enough for an IM injection for a newborn. 4. The forearm is not used for IM injections. 52. Which of the following vaccines can be given subcutaneously? 1. MCV4 2. Varicella 3. MMR 4. Rotavirus ANS: 2 1. 2. 3. 4.
Feedback Given IM Given IM Given subcutaneously Given via IV infusion or IM
53. The pediatric nurse is teaching a young mother prodromal signs of communicable diseases at an in-home visit. The nurse knows that the mother understands the prodromal signs and symptoms when she states: 1. I will watch for coryza and vomiting to begin. 2. I will need to watch for cough, coryza, and fevers.
CURRENT DIAGNOSIS AND TREATMENT PEDIATRICS 24TH LATEST 3. I will need to watch for a change in behavior and a low-grade fever. 4. I will watch for a rash, fever, and vomiting. ANS: 2 Feedback 1. Vomiting is not part of the prodromal signs. 2. Cough, coryza, and fevers are prodromal signs. 3. A low-grade fever is not a prodromal sign. 4. A rash and vomiting are not seen as prodromal signs. 54. Anne goes into her pediatric patients room for her 0400 assessment of a child who was admitted with dehydration 12 hours ago. Anne begins her assessment and knows that there is a medical emergency when the findings indicate: 1. A child with a regular heart rate and a respiratory rate at the high end of normal. 2. A child who is awake and wanting to play on the PS3. 3. A child who has a purple rash in his extremities, and it does not blanch. 4. A child who complains that he is tired and vomits thick mucus. ANS: 3 Feedback 1. A regular heart rate is not a medical emergency. 2. The child may not sleep well because of the environment. 3. The purple rash is a concern for the childs circulatory system. 4. The thick mucus occurs because the child has been dehydrated. 55. Lizas mother calls the pediatric triage nurse because she is concerned about her usually healthy daughters rosy, lacey cheeks, fever, and fussiness. The mother reports that this illness has been going around the day care. The nurse anticipates that the doctors orders will include: 1. Acetaminophen, adequate hydration, and to let the virus run its course. 2. Having the child admitted to the emergency room because this is a medical emergency.
CURRENT DIAGNOSIS AND TREATMENT PEDIATRICS 24TH LATEST 3. Having the mother speak with the day care and discuss good hand washing techniques. 4. Giving the child TLC and hydration. ANS: 1 Feedback 1. The child is exhibiting a viral infection. 2. The signs and symptoms are not a medical emergency. 3. This would be appropriate to educate the mother, but this is not the primary answer for this question. 4. This would be appropriate, but it does not answer the question for the mother. 56. A pregnant school teacher has been notified that there is a case of erythema infectiosum in her classroom. The school nurse speaks to the teacher because if she contracts erythema infectiosum, the mother and baby are at risk for: 1. Growth restrictions in an unborn baby. 2. The mother to have a miscarriage. 3. The mother to have slow weight gain and affect the placenta. 4. All of the above. ANS: 2 Feedback 1. Growth restriction is not a risk for the fetus. 2. The virus can cause a miscarriage in a pregnant woman. 3. Weight gain and the placenta are not affected by the virus. 4. One of the answers is correct. 57. Tabitha has been admitted with influenza. The nurse discusses the plan of care with the family. The child will be in droplet isolation. The reason for the droplet isolation is because: 1. The goal is to not bring in any outside infections to the child, as pneumonia can be a complication.
CURRENT DIAGNOSIS AND TREATMENT PEDIATRICS 24TH LATEST 2. The goal is to prevent the spread of the virus to any of the health-care workers and other patients. 3. The goal is to prevent a massive outbreak within the community. 4. The goal is to provide a quiet environment for the child to rest. ANS: 2 Feedback 1. The childs immune system is compromised, but the concern in droplet transfer to others. 2. Influenza is highly contagious through droplet formation. The isolation will help prevent the spread of the disease. 3. Influenza can be widespread, but the question is focusing on the familys needs. 4. The child needs rest to recover, but the answer addresses the droplet communicability of Influenza. 58. The doctor has prescribed Zanamivir for a 4-year-old patient. The nurse questions this order because: 1. The medication is only to be given to adults with pneumonia. 2. The medication is only recommended for infants with Hand-Foot-Mouth Disease. 3. The medication is only recommended for ages 7 and older for Influenza. 4. The medication has proven to be ineffective in the management of Influenza. ANS: 3 Feedback 1. The medication is recommended for adults to treat Influenza. 2. The medication should not be used in infants and is used for Influenza, not Hand-Foot-Mouth Disease. 3. The medication is for Influenza in children over the age of 7. 4. The medication is effective for a child over the age of 7 for Influenza. 59. A 12-year-old girl presents to the emergency room with complaints of a sore throat, fever, and malaise for the past six days. The parents have noticed that she has been progressively
CURRENT DIAGNOSIS AND TREATMENT PEDIATRICS 24TH LATEST sleeping more. The patient is diagnosed with mononucleosis. The caregiver education done by the nurse should include all of the following except: 1. Rest periods should follow play/activity periods. 2. After the girl goes back to school, she should not participate in ice hockey for 6 to 8 weeks. 3. Keep her well hydrated, and provide nutrition that is high in protein. 4. Give acetaminophen or ibuprofen for fever management. ANS: 4 Feedback 1. Rest periods should be provided for the child. 2. The child should not participate in contact sports for 6 to 8 weeks after
having mononucleosis. 3. Hydration and a diet high in protein are needed for increasing the immune system. 4. Fever is not present at this stage of the healing process. 60. Logan, a, 11-year-old boy, was diagnosed with parotitis three days ago at the pediatric outpatient clinic. His mother calls the triage nurse to report that Logan is complaining of pain in his peritoneal area. The nurse knows that: 1. This is a medical emergency, and Logan should be brought to the emergency room right away. 2. The virus can cause orchitis. 3. The virus can cause oophoritis. 4. This is a common complaint because it is hard for a child with parotitis to urinate. ANS: 2 Feedback 1. Pain in the peritoneal area in common with this condition. 2. The virus is known to cause orchitis and has these signs and symptoms.
3.Oophoritis is not a sign or symptom of parotitis. 4.Urination is not an issue for a child with parotitis.
CURRENT DIAGNOSIS AND TREATMENT PEDIATRICS 24TH LATEST 61. RSV can be life threatening for infants and young children. Children who are at high risk for vulnerability to RSV include all of the following except: 1. Immunocompromised school-age children. 2. Infants with congenital heart defects. 3. Premature infants. 4. Respiratory compromised toddlers. ANS: 1 Feedback 1. RSV tends to affect younger children. 2. Children with a congenital heart defect are at a higher risk for
susceptibility to RSV. 3.A premature infant is at a higher risk due to an immature immune system. 4.A toddler with respiratory issues has a compromised immune system, placing him/her at higher risk. 62. Jackson, a 9-month-old infant, is an inpatient on a pediatric floor due to a diagnosis of RSV. His mother is concerned because everyone comes into the room with a gown, mask, and gloves. The nurse attempts to educate his mother on the process of the disease. The nurse knows that the mother has an understanding of the illness when she states: 1. I can catch the virus just like a cold. 2. The virus is spread by droplets, and the health-care workers need to make sure not to carry Jacksons illness to other patients. 3. This is to protect Jackson from getting an illness from the health-care providers. 4. This is standard procedure on a pediatric floor during the RSV season. ANS: 2 Feedback 1. Adults rarely have RSV.
CURRENT DIAGNOSIS AND TREATMENT PEDIATRICS 24TH LATEST 2. RSV is spread by droplets, and the parent acknowledges that the disease
can be spread through the health-care workers. 3. The child is contagious, not the health-care workers.
4.Standard procedures are not always known to the parents. 63. Synagis is being prescribed for Kenyon, a 60-day-old, premature infant, prior to leaving the NICU. The nurse explains the reason for the medication to his mother. The mother understands that the medication will need: 1. To be given in the pediatricians office every 28 to 30 days for the next year. 2. To be given in the pediatricians office every 28 to 30 days for the next 3 to 5 months while it is RSV season. 3. To be given in the pediatricians office at 6 months and again at 9 months. 4. To be given in the pediatricians office at 1 year of age. ANS: 2 Feedback 1. The medication is not needed for an entire year. 2. The medication is given during the months when RSV is most prevalent.
3.The medication needs to be given monthly during the RSV season. 4.The medication needs to be given monthly during the RSV season. 64. An outbreak of German measles has occurred at the middle school. The school nurse has been notifying the parents of the children, along with the staff. The best prevention against contracting the highly contagious disease is: 1. To keep ill children at home. 2. To let children come to school after 16 days from the last known day of exposure because if there are no signs and symptoms, the child will not become ill. 3. Receiving the MMR vaccine. 4. Good hand washing.
CURRENT DIAGNOSIS AND TREATMENT PEDIATRICS 24TH LATEST ANS: 3 Feedback 1. Keeping the ill child at home is preferred, but not the priority intervention. 2. Incorrect. This is the incubation period for the German measles. 3.The best prevention is an immunization. 4.Hand washing is important for stopping the spread of the disease, but it is not the best option. 65. Photophobia can occur in which communicable disease process? 1. German measles 2. Influenza 3. Rubeola 4. Hand-Foot-Mouth Disease ANS: 3 Feedback 1.Photophobia is not present in German measles. 2.Photophobia is not usually present in Influenza. 3.Photophobia is common with Rubeola. 4.Hand-Foot-Mouth Disease does not have photophobia as a symptom. 66. Jasmines mom calls the pediatric triage nurse and asks if she can give aspirin because of a fever and achiness due to chicken pox. The nurse knows that the mother understands the reason not to give the aspirin in this situation when she states: 1. If I give aspirin, she is as risk for major issues with her brain and liver. 2. I can give the aspirin in a low dose and be safe. 3. I can give the aspirin in three separate doses throughout the day. 4. I should not give her aspirin because this can make her blood thin and not help with her get better.
CURRENT DIAGNOSIS AND TREATMENT PEDIATRICS 24TH LATEST ANS: 1 Feedback 1. Aspirin is not recommended for children due to the risk for liver and brain damage. 2. Any dose of aspirin for a child can cause damage to the liver and brain. 3.Any dosing schedule can cause damage to the childs liver and brain. 4.The asprin may cause blood to thin, but the main concern is the development of Reyes Syndrome. 67. A complication of scarlet fever is glomerulonephritis. This is because: 1. The high doses of penicillin used to treat the disease process are hard for the kidneys to process. 2. The group A beta-hemolytic streptococcus can affect the kidneys if not treated. 3. The Epstein-Barr virus can affect the kidneys if not treated. 4. The HIV irus can affect the kidneys if not treated. ANS: 2 Feedback 1. Penicillin does not affect kidney function. 2. Beta-hemolytic streptococcus can cause damage to the kidneys if not
treated. 3. The Epstein-Barr virus is not present in scarlet fever. 4.The HAV virus is not present in scarlet fever. 68. A 2-month-old presents to the pediatric clinic with a cough with crowing at the end, coryza, and vomiting. The diagnosis is whooping cough, and the child is taken immediately to the pediatric unit at the nearby hospital. The nurse explains the plan of care for the infant to the mother. The nurse should say: (Select all that apply.) 1. Feedings will be small in an attempt to keep the infant from vomiting. 2. We will be watching how the infant breathes and will provide extra support if needed.
CURRENT DIAGNOSIS AND TREATMENT PEDIATRICS 24TH LATEST 3. We plan to wake the infant every hour in order to assess and feed the infant to make sure that he/she is getting enough calories. 4. Cool mist humidification will be continuous to help keep the fever down. 5. An IV will be placed for antibiotics. ANS: 1, 2, 5 Feedback 1. Small, frequent feedings will decrease the chance for an emesis when the infant coughs. 2. Respiratory compromise occurs with whooping cough. It is important to provide support if needed. 3. Letting the infant rest is important to help with the healing process. 4.Cool mist humidification is used for keeping secretions thin, not for fever reduction. 5.IV antibiotics are an effective treatment for this disease process. 69. Malcolm has given his pediatric patient three vaccines. Documentation is being placed in the patients chart. Malcolm knows that he must document: (Select all that apply.) 1. The lot number of the vaccine. 2. The site of the injection. 3. The date of administration. 4. The consent for administration. 5. The side effects the patient exhibited. ANS: 1, 2, 3, 4 Feedback 1.Needed for documentation 2.Needed for documentation 3.Needed for documentation 4.Needed for documentation 5.This is not required initially unless the patient exhibits a side effect.
CURRENT DIAGNOSIS AND TREATMENT PEDIATRICS 24TH LATEST 70. A day-care center calls the pediatric triage nurse because there has been an outbreak of Hand-Foot-Mouth Disease in the preschool room. It is important to teach the day-care center to: (Select all that apply.) 1. Clean surfaces and toys with a mixture of disinfectant and bleach water. 2. Practice good hand hygiene. 3. Have the children stay home if they are active cases. 4. Send information home with the parents. 5. All of the above information should be given to the day-care center. ANS: 5 Feedback 1. Cleaning and disinfecting is needed. 2. Hand hygiene helps prevent the spread of illness.
3.An infected child should stay home. 4. The parents should receive education about the disease and the presence of it in the daycare. 5. All of the information should be provided and completed in the day-care center. 71. Joshua is sent to the nurses office because the teacher thinks he has pink eye. The school nurse begins to assess his eye. She should: (Number the following in the correct order.) Wash the eye from the inside corner to outside corner. Provide a cool cloth to comfort the eye. Wash hands and put on gloves. Call his parents so they can take Joshua to the doctor for treatment. Sanitize the nurses office, including door handles. Sanitize Joshuas classroom work area.
CURRENT DIAGNOSIS AND TREATMENT PEDIATRICS 24TH LATEST ANS: 2, 3, 1, 4, 5, 6 True/False 72. Children may be given a calculated dose of acetaminophen or ibuprofen following an immunization. ANS: T Feedback 1. The medication will aid in comfort for the child. 2. The medication will aid in comfort and should be given to the child.
Chapter 10. Travel Medicine 1. Which should the nurse use to prepare liquid medication in volumes less than 5 milliliters? a. Calibrated syringe b. Paper measuring cup c. Plastic measuring cup d. Household teaspoon ANS: A To ensure accuracy, a calibrated syringe without a needle should be used to prepare a liquid dosage less than 5 milliliters. Paper and plastic measuring cups are not calibrated for liquid volumes less than 5 milliliters. A household teaspoon is not accurate enough to measure small amounts of medication. 2. Which food choice is appropriate to mix with medication? a. Formula or milk b. Applesauce c. Syrup d. Orange juice
CURRENT DIAGNOSIS AND TREATMENT PEDIATRICS 24TH LATEST ANS: B To prevent the child from developing a negative association with an essential food, a nonessential food such as applesauce is best for mixing with medications. Formula and milk are essential foods in a childs diet. Medications may alter their flavor and cause the child to avoid them in the future. Syrup is not used to mix with medications because of its high sugar content. Orange juice is considered an essential food; therefore, the nurse should not mix medications with it. 3. Which physiological difference would affect the absorption of oral medications administered to a 3-month-old infant? a. More rapid peristaltic activity b. More acidic gastric secretions c. Usually more rapid gastric emptying d. Variable pancreatic enzyme activity ANS: D Pancreatic enzyme activity is variable in infants for the first 3 months of life as the gastrointestinal system matures. Medications that require specific enzymes for dissolution and absorption might not be digested to a form suitable for intestinal action. Infants up to 8 months of age tend to have prolonged motility. The longer the intestinal transit time, the more medication is absorbed. The gastric secretions of infants are less acidic than in older children or adults. Gastric emptying is usually slower in infants. 4. Which factor should the nurse remember when administering topical medication to an infant? a. Infants require a larger dosage because of a greater body surface area. b. Infants have a thinner stratum corneum that absorbs more medication. c. Infants have a smaller percentage of muscle mass compared with adults. d. The skin of infants is less sensitive to allergic reactions. ANS: B
CURRENT DIAGNOSIS AND TREATMENT PEDIATRICS 24TH LATEST Infants and young children have a thinner outer skin layer (stratum corneum), which increases the absorption of topical medication. A similar dose of a topical medication administered to an infant compared with an adult is approximately three times greater in the infant because of the greater body surface area. The smaller muscle mass in infants affects site selection for injected medications. The young childs skin is more prone to irritation, making contact dermatitis and other allergic reactions more common. 5. What is the appropriate nursing response to a parent who asks, What should I do if my child cannot take a tablet? a. You can crush the tablet and put it in some food. b. Find out if the medication is available in a liquid form. c. If the child cant swallow the tablet, tell the child to chew it. d. Let me show you how to get your child to swallow tablets. ANS: B A tablet should not be crushed without knowing whether it will alter the absorption, effectiveness, release time, or taste. Therefore telling the parent to find out whether the medication is available in liquid form is the most appropriate response. A chewed tablet may have an offensive taste, and chewing it may alter its absorption, effectiveness, or release time. Forcing a child, or anyone, to swallow a tablet is not acceptable and may be dangerous. 6. What is the maximum safe volume that an infant (aged 1 to 12 months) can receive in an intramuscular injection? a. 0.25 milliliter b. 0.5 milliliter c. 1 milliliter d. 1.5 milliliters ANS: C
CURRENT DIAGNOSIS AND TREATMENT PEDIATRICS 24TH LATEST The maximum volume of medication for an intramuscular injection to an infant is 1 mL. The neonate should receive no more than 0.5 mL per intramuscular injection. 1.5 milliliters is not appropriate for an infant. It is appropriate for an intramuscular injection to a child 3 to 14 years of age.
7. Which muscle would the nurse select to give a 6-month-old infant an intramuscular injection? a. Deltoid b. Ventrogluteal c. Dorsogluteal d. Vastus lateralis ANS: D The vastus lateralis is not located near any vital nerves or blood vessels. It is the best choice for intramuscular injections for children younger than 3 years of age. The deltoid muscle is not used for intramuscular injections in young children. The ventrogluteal muscle is safe for intramuscular injections for children older than 18 months. The dorsogluteal muscle does not develop until a child has been walking for at least 1 year. 8. The nurse is planning to administer an intramuscular injection to a 13-year-old child. What is the maximum volume of medication that can be injected into the ventrogluteal site? a. 0.5 to 1 milliliter b. 1 to 1.5 milliliters c. 1.5 to 2 milliliters d. 2 to 2.5 milliliters ANS: C The maximum volume of medication for an intramuscular injection to an older child (6 to 14 years) is 1.5 to 2.0 milliliters. 0.5 to 1.4 milliliters are acceptable volumes to inject, but they are
CURRENT DIAGNOSIS AND TREATMENT PEDIATRICS 24TH LATEST not the maximum. 2 to 2.5 milliliters exceeds the amount that can be safely injected into one site for a 13-year-old child. 9. Which parameter should guide the nurse when administering a subcutaneous injection? a. Do not give injections in edematous areas. b. Attach a clean 1-inch needle to the syringe. c. The maximum volume injected into one site is 2 milliliters. d. Do not pinch up tissue before inserting the needle. ANS: A Subcutaneous injections should never be given in areas of edema because absorption is unreliable. A short (no more than 5/8 inch) needle should be used to deposit medication into subcutaneous tissue. Volumes for subcutaneous injections are small, usually averaging 0.5 milliliters. The skin is pinched up for a subcutaneous injection to raise the fatty tissue away from the muscle. 10. Which action is correct when administering ear drops to a 2-year-old child? a. Administer the ear drops straight from the refrigerator. b. Pull the pinna of the ear back and down. c. Massage the pinna after administering the medication. d. Pull the pinna of the ear back and up. ANS: B For children younger than 3 years, the pinna, or lower lobe, of the ear should be pulled back and down to straighten the ear canal. Medication should be at room temperature because cold solutions in the ear will cause pain. The tragus of the ear should be massaged to ensure the drops reach the tympanic membrane. For a child 3 years or older, the pinna is pulled up and back.
CURRENT DIAGNOSIS AND TREATMENT PEDIATRICS 24TH LATEST 11. A nurse is preparing to start a continuous IV infusion on a child. The nurse selects a Buretrol (volume-control) attachment as part of the IV tubing set-up. The main purpose for selecting a Buretrol attachment is to: a. avoid fluid overload. b. aid in measuring intake. c. administer antibiotics. d. ensure adequate intravenous fluid intake. ANS: A A volume-control device such as a Buretrol or an infusion pump allows the nurse to set a specific volume of fluid to be given in a specific period of time (usually 1 hour) and decreases the risk of inadvertently administering a large amount of fluid. Although the use of a volume-control device allows for accurate measurement of intake, the primary purpose for using this equipment is to prevent fluid overload. Medications such as antibiotics can be administered with a volumecontrol device; however, this is not the primary purpose. 12. Which is the most important nursing action before discharge for a mother who is apprehensive about giving her child insulin? a. Review the side effects of insulin with the mother. b. Have the mother verbalize that she knows the importance of follow-up care. c. Observe the mother while she administers an insulin injection. d. Help the mother devise a rotation schedule for injections. ANS: C It is important that the nurse evaluate the mothers ability to give the insulin injection prior to discharge. Watching her give the injection to the child will give the nurse an opportunity to offer assistance and correct any errors. Although reviewing side effects is important, this could be done over the phone or by the pharmacist when the medication is picked up. Having the mother verbalize her knowledge of the importance of follow-up care is important but not directly
CURRENT DIAGNOSIS AND TREATMENT PEDIATRICS 24TH LATEST relevant to the mothers concern. Helping the mother devise a rotation schedule for injections is important but not as important as having the mother demonstrate the procedure. 13. A nurse has just initiated an intravenous piggyback of gentamicin (Garamycin). What is the best time for a trough serum level to be measured? a. Just before the next dose b. When the infusion is finished c. One hour after the medication is administered d. Depends on the specific medication ANS: A The medication trough is the level at which the serum concentration is lowest. Trough levels are usually obtained just before the next medication dose. The serum concentration would be increasing as the infusion finishes. This is not the concentration trough. The peak concentration, or the concentration after the medication has been distributed, varies according to the specific medication. Trough is always the lowest just before the next medication dose. 14. A nurse should routinely ask a colleague to double-check a medication calculation and the actual medication before administering which medications? a. Antibiotics b. Acetaminophen c. Anticonvulsants d. Anticoagulants ANS: D The nurse should ask another nurse to check the dosage calculation and the medication before administering anticoagulants. The nurse always double-checks a dosage calculation, but it is not necessary to have a second nurse check the medication before administering antibiotics, acetaminophen, or anticonvulsant medications.
CURRENT DIAGNOSIS AND TREATMENT PEDIATRICS 24TH LATEST 15. Which nursing action is correct when administering heparin subcutaneously? a. Insert the needle with the bevel up at a 15-degree angle. b. Insert the needle into the skin at a 45-degree angle. c. Inject the needle into the tissue on the upper back. d. Massage the injection site when the injection is complete. ANS: B For a subcutaneous injection, the nurse would pinch the skin and inject at a 45-degree angle. Inserting the needle with the bevel up at a 15-degree angle is the technique used for an intradermal injection. The upper back is used for intradermal injections. The nurse would not massage the site after administering heparin. 16. Which indicates that a school-age child is using a metered-dose inhaler correctly? a. The child uses his inhaled steroid before the bronchodilator. b. The child exhales forcefully as he squeezes the inhaler. c. The child holds his breath for 10 seconds after the first puff. d. The child waits 10 minutes before taking a second puff. ANS: C After a puff, the child should hold his breath for about 10 seconds or until he counts slowly to 5. If one of the childs medications is an inhaled steroid, it should be administered last. The child should inhale slowly as the inhaler is squeezed or depressed. The child does not need to wait this long to take a second puff of medication. He can take a second puff after holding his breath for 10 seconds. 17. Which step is appropriate when using EMLA cream before intravenous catheter insertion? a. Rub a liberal amount of cream into the skin thoroughly. b. Cover the skin with a gauze dressing after applying the cream. c. Leave the cream on the skin for 1 to 2 hours before the procedure.
CURRENT DIAGNOSIS AND TREATMENT PEDIATRICS 24TH LATEST d. Use the smallest amount of cream necessary to numb the skin surface. ANS: C The cream should be left in place for a minimum of 1 hour and up to 2 hours. The EMLA cream should not be rubbed into the skin. After the cream is applied to the skin surface, it is covered with a transparent occlusive dressing. The nurse would use a liberal amount of EMLA cream. 18. A child is receiving intravenous fluids. How frequently should the nurse assess and document the condition of the childs intravenous site? a. Every hour b. Every 2 hours c. Every 4 hours d. Every shift ANS: A The nurse assesses and documents an IV site at least every hour for signs and symptoms of infiltration and phlebitis. The nurse should assess a childs IV site more frequently than every 2 to 4 hours or every shift. Serious complications could occur during this time interval. 19. What is the hourly maintenance fluid rate for an intravenous infusion in a child weighing 19.3 kilograms? a. 19 milliliters b. 45 milliliters c. 61 milliliters d. 95 milliliters ANS: C The formula for calculating daily fluid requirements is 0 to 10 kg: 100 mL/kg/day; 10 to 20 kg: 1000 mL for the first 10 kg of body weight plus 50 mL/kg/day for each kg between 10 and 20. To determine an hourly rate, divide the total milliliters per day by 24.
CURRENT DIAGNOSIS AND TREATMENT PEDIATRICS 24TH LATEST 20. The nurse administering an IV piggyback medication to a preschool child should take which action? a. Dilute the medication in at least 20 milliliters and infuse over at least 15 minutes. b. Flush the IV tubing before and after the infusion with normal saline solution. c. Inject the medication into the IV catheter using the port closest to the child. d. Inject the medication into the IV tubing in the direction away from the child. ANS: A Medications given by IV piggyback are diluted in at least 20 milliliters of IV solution and administered over at least 15 minutes. When administering medications by IV piggyback, the nurse flushes the tubing after the medication has infused, usually with 16 to 20 milliliters of IV solution. The nurse is using the IV push method when injecting medication into the IV tubing using the port closest to the child. The IV retrograde method involves clamping the IV tubing below the injection port and injecting medication into the tubing in a direction away from the child, causing it to flow into the tubing above the injection port. 21. What nursing action is indicated when a child receiving a unit of packed red blood cells complains of chills, headache, and nausea? a. Continue the infusion and take the childs vital signs. b. Stop the infusion immediately and notify the physician. c. Slow the infusion and assess for cessation of symptoms. d. Start a dextrose solution and stay with the child. ANS: B If a reaction is suspected, as in this case, the transfusion is stopped immediately and the physician is notified. If the child is displaying signs of a transfusion reaction, the transfusion cannot continue. Dextrose solutions are never infused with blood products because the dextrose causes hemolysis. This action does not address the blood infusion. 22. What is the best action for the nurse to take when giving medications to a 3-year-old child?
CURRENT DIAGNOSIS AND TREATMENT PEDIATRICS 24TH LATEST a. Tell the child to take the medication right now. b. Tell the child to take the medication or he will have to get a shot. c. Allow the child to choose fruit punch or apple juice when giving the medication. d. Tell the child that another child his age just took his medication like a good boy. ANS: C Realistic choices allow the child to feel some control. Direct confrontation typically results in a no response. Threatening a child with a shot is inappropriate. Comparisons are not helpful in getting a child to cooperate. MULTIPLE RESPONSE 1. Which are advantages of using an Electronic Medical Record System (EMR) for medication administration to children? Select all that apply. a. Eliminates the need to perform the six rights of medication administration b. Reduces medication errors c. Is a cost effective means of medication administration d. Improves communication of patient medication lists e. Improves communication of patient allergies ANS: B, D, E One of the major reasons for the increasing use of computer systems such as patient electronic medical records (EMRs) is to reduce medication errors. EMRs, which include electronic medication administration records (EMARs), can improve communication of patient medication lists and other information such as allergies between different healthcare providers working in the same facility or in other settings. These systems do not replace the responsibility of physicians and nurses for clear and complete medication orders, accurate dose calculations, and correct administration of medications to children. The systems are not more cost effective than administering medications without the use of a computer scanning system. SHORT ANSWER
CURRENT DIAGNOSIS AND TREATMENT PEDIATRICS 24TH LATEST 1. You need to administer ibuprofen, 120 mg, to your 4-year-old patient. Ibuprofen comes in liquid form in a dose of 100 mg/5 mL. How many milliliters will you give? ANS: 6 6 mL 5 mL = x mL 100 mg 120 mg 100 x = 600; x = 6.0 mL. 2. You need to administer ceftriaxone sodium to your 3-year-old patient who weighs 33 pounds. The physicians order states that you should administer ceftriaxone sodium 50 mg/kg once a day. How many milligrams will you prepare? ANS: 750750 mg 33 lbs 0.4536 = 15 kg; 15 50 = 750 mg. 3. Calculate the daily maintenance fluid 24-hour total requirement for a child weighing 21.5 kg. ANS: 1530 1530 mL 1500 mL/day + 20 mL/kg/day for each additional kg over 20 kg. 1500 + 30 = 1530.
Chapter 11. Normal Childhood Nutrition and Its Disorders
CURRENT DIAGNOSIS AND TREATMENT PEDIATRICS 24TH LATEST 1. If a patient with multiple sclerosis starts coughing frequently during meals and starts to eat significantly less food than normal, the patient may have a. pneumonia. b.gastroesophageal reflux disease. c. peptic ulcer. d.dysphagia. ANS: D Patients with multiple sclerosis may develop dysphagia; coughing during meals and eating less than usual are common symptoms. Dysphagia may cause pneumonia if food enters the lungs and causes infection. Gastroesophageal reflux disease causes burning and pain but typically not coughing. Peptic ulcer causes intestinal pain but not coughing. 2. For a patient with dysphagia, the food that would be most difficult to swallow is a. applesauce. b.mashed potatoes. c. chocolate pudding. d.chicken noodle soup. ANS: D Patients with dysphagia often have difficulty swallowing thin liquids, such as chicken noodle soup. Thicker, pureed foods such as applesauce, mashed potatoes, and chocolate pudding are usually easier to swallow. 3. If a patient has difficulty swallowing, the best position for eating is a. propped up in bed with a caregiver by the bedside. b.sitting upright opposite a caregiver. c. sitting at a dining table with social dining companions. d.lying on the left side, with a caregiver by the side. ANS: B The safest eating position for someone who has trouble swallowing is sitting upright; the caregiver should sit opposite to help provide cues and reminders while the patient is eating. It is usually best for patients with dysphagia not to interact socially during mealtimes because talking
CURRENT DIAGNOSIS AND TREATMENT PEDIATRICS 24TH LATEST may make swallowing more difficult. Sitting propped up in bed and leaning back would make it easier for food to enter the airway. Lying on ones side while eating is very likely to cause aspiration. 4. The most acute risk for patients with dysphagia is a. constipation. b.dehydration. c. dry mouth. d.panic attacks. ANS: B An acute daily concern for patients with dysphagia is dehydration. Patients are not usually allowed to drink fluids without supervision, and so fluid intake must be monitored throughout the day. Constipation and dry mouth may occur with poor fluid and food intake but are less acute. Patients with dysphagia may experience feelings of panic, but this is not the most acute concern. 5. The most helpful recommendation for a client who often experiences heartburn at night would be to a. avoid eating within 4 hours of going to bed. b.increase the fiber content of the diet. c. decrease the fiber content of the diet. d.take antacid medications before going to bed. ANS: A Gastroesophageal reflux disease (GERD) usually occurs 1 to 4 hours after eating; therefore, waiting at least 4 hours after eating before going to bed would help prevent heartburn at night. Antacid medications may be helpful but should not be used on a regular basis before other strategies have been tried. Fiber content of the diet does not affect heartburn. 6. An example of a meal that is likely to relax the lower esophageal sphincter and allow gastroesophageal reflux is a. pasta with marinara sauce and sourdough bread. b.ham with rice pilaf.
CURRENT DIAGNOSIS AND TREATMENT PEDIATRICS 24TH LATEST c. fried chicken and pasta salad. d.chicken and spinach tortilla wrap and salsa. ANS: C High-fat foods and beverages relax the lower esophageal sphincter (allowing stomach contents to back up). Both fried chicken and pasta salad are high in fat and would exacerbate gastroesophageal reflux. The other meals are relatively low in fat and would be much less likely to cause reflux. 7. Peptic ulcer disease (PUD) would be most like to occur in someone who uses medication to treat a. gastroesophageal reflux disease. b.constipation. c. celiac disease. d.joint pain or arthritis. ANS: D Risk of PUD increases with chronic use of nonsteroidal anti-inflammatory drugs (NSAIDs), often used to treat joint pain or arthritis. Use of antacids to treat GERD would decrease risk of PUD. Laxatives and stool softeners used to treat constipation do not affect risk of PUD. Celiac disease is treated by avoiding dietary sources of gluten, not with medications. 8. Nutrition therapy for peptic ulcers should be individualized, depending on a. type of drug treatment. b.location of the ulcer. c. patient tolerance. d.the cause of the ulcer. ANS: C There are no specific dietary recommendations for treatment of peptic ulcer, and so foods eaten depend on the patients symptoms and tolerance. The type of drug treatment and location of the ulcer do not significantly affect food tolerance. Factors that contribute to formation of peptic ulcers are mostly nonnutritional (stress, skipping meals, chronic use of use of NSAIDs), and so dietary intake is not a significant factor in treatment.
CURRENT DIAGNOSIS AND TREATMENT PEDIATRICS 24TH LATEST 9. If a patient who has undergone gastric bypass surgery for treatment of extreme obesity experiences sudden sweating, nausea, and stomach cramps after meals, the patient may have a. gastroesophageal reflux. b.pancreatitis. c. peptic ulcer disease. d.dumping syndrome. ANS: D Patients who have undergone gastric bypass may experience dumping syndrome as a large volume of hyperosmolar food is dumped into the small intestine because the stomach reservoir is missing or too small. GERD and peptic ulcers are not necessarily more common after gastrectomy. Pancreatitis causes extreme pain and vomiting but is not related to gastric bypass. 10. An appropriate meal for someone with dumping syndrome would be a. a small vanilla milkshake. b.cream of mushroom soup. c. half a turkey sandwich. d.popcorn and sugar-free soda. ANS: C Individual tolerances vary, but the turkey half-sandwich would probably be tolerated best. Half a sandwich is probably small enough not to cause symptoms. A milkshake is high in sugar and probably too cold. Cream of mushroom soup is probably too hot, and liquids should be taken between rather than with meals. The carbonation in the soda may increase gastrointestinal distention because of gas. 11. If a patient has esophagitis and also has worn tooth enamel, he or she may have a. a hiatal hernia. b.fluoride deficiency. c. an eating disorder. d.gastroesophageal reflux disease. ANS: C
CURRENT DIAGNOSIS AND TREATMENT PEDIATRICS 24TH LATEST The eating disorder bulimia nervosa causes esophagitis and eroded tooth enamel because of the effects of stomach acid being vomited back into the esophagus and mouth. A hiatal hernia and GERD may cause esophagitis, but stomach contents would enter only the esophagus, not the mouth, and so the teeth would not be worn. Fluoride deficiency may cause loss of tooth enamel but would not cause esophagitis. 12. An example of a meal that may contain gluten is a. baked chicken breast with herbed rice. b.pork chop with sweet potatoes and butter. c. grilled steak with baked potato and sour cream. d.roast beef with mashed potatoes and gravy. ANS: D Gravy is often made with wheat flour, which contains gluten. Rice, potatoes, and sweet potatoes do not contain gluten. Baked chicken, pork chops, and grilled steak do not contain gluten unless they are breaded or coated with wheat flour before being cooked. 13. A patient with lactose intolerance would be most likely to experience symptoms if he or she ate a a. cup of pudding. b.turkey sandwich with Swiss cheese. c. biscuit with butter and honey. d.cup of sherbet. ANS: A A cup of pudding would contain a cup of milk; most individuals with lactose intolerance can only tolerate to cup milk at one time. Sherbet, Swiss cheese, biscuits, and butter all have low levels of lactose. 14. Milk may be made suitable for patients with lactose intolerance by treating it with tablets that contain a. yogurt. b.lactase enzyme. c. vitamin D and calcium.
CURRENT DIAGNOSIS AND TREATMENT PEDIATRICS 24TH LATEST d.Lactobacillus acidophilus. ANS: B Lactase enzyme tables may be added to milk 24 hours before it is drunk to digest the lactose; this makes the milk suitable for patients with lactose intolerance. Lactobacillus acidophilus and yogurt are not necessarily better tolerated than regular milk. Vitamin D and calcium do not affect lactose intolerance, although patients with lactose intolerance may benefit from supplements of these nutrients if their intake is low because they avoid milk. 15. For patients with inflammatory bowel disease, nutrition therapy is a. the primary mode of treatment. b.rarely necessary. c. an important adjunct to drugs and surgery. d.highly specialized. ANS: C Nutrition therapy is important to help maintain nutritional status of patients with inflammatory bowel disease, but is used in conjunction with drug and surgical treatments; it is not the primary mode of treatment. Nutrition therapy is often needed to maintain weight and micronutrient status. Awareness of deficiencies that occur in these patients is important, but the RD does not need specialized knowledge to work with patients with IBD. 16. Because patients with active Crohns disease have intestinal inflammation, they benefit from high intake of a. fat. b.dietary fiber. c. simple sugars. d.protein. ANS: D Patients with inflammatory bowel disease benefit from a diet high in protein to compensate for malabsorption. Fat intake should be moderate to provide adequate kilocalories within a healthful
CURRENT DIAGNOSIS AND TREATMENT PEDIATRICS 24TH LATEST diet; high fat intake may cause steatorrhea. Dietary fiber may irritate the inflamed gut. Simple sugars provide a readily digested and absorbed source of energy but no other nutrients. 17. Patients with inflammatory bowel disease may benefit from a high-fiber diet during a. acute episodes. b.times of remission. c. recovery from surgery. d.preparation for surgery. ANS: B A high-fiber diet may help stimulate peristalsis during times of remission in patients with inflammatory bowel disease. During acute episodes, the diet should be low in fiber and should promote bowel rest. In general, the diet should be high calorie, high protein, and nutrient dense to compensate for malabsorption. 18. Fluid loss is most likely to be a problem for a patient with a. a colostomy. b.a hiatal hernia. c. an ileostomy. d.Crohns disease. ANS: C Fluid loss is most likely to be a problem for a patient with an ileostomy because food exits the intestines before water is absorbed in the colon. A colostomy may cause some fluid loss, depending on where the colostomy is located and how much of the colon the intestinal contents pass through for fluid absorption. A hiatal hernia does not cause fluid loss. Crohns disease may cause some fluid loss during acute periods of inflammation because of malabsorption. 19. The most significant nutritional concern for a patient with an ileostomy is a. fat malabsorption and steatorrhea. b.dumping syndrome. c. loss of fluid and electrolytes. d.malabsorption of protein and carbohydrates. ANS: C
CURRENT DIAGNOSIS AND TREATMENT PEDIATRICS 24TH LATEST For a patient with an ileostomy, digestive output exits the body instead of passing through the colon, the major site of absorption of water and electrolytes. Digestion and absorption of protein, carbohydrate, and fat are not usually affected. Dumping syndrome is caused by a partial or total gastrectomy. 20. For patients with ostomies, a lower amount of output occurs a. if the ostomy is closer to the rectum. b. if the ostomy is closer to the rectum.
c. with high fiber intake. d.with high fat intake. ANS: A As effluent progresses through the colon, water is absorbed, and the effluent becomes more solid; therefore, an ostomy closer to the rectum would have a smaller volume. Fiber intake may increase output slightly, but less so than the site of the ostomy; fat intake does not significantly affect the amount of output.
Chapter 12. Emergencies and Injuries 1. Which nursing action would facilitate care being provided to a child in an emergency situation? a. Encourage the family to remain in the waiting room. b. Assist parents in distracting the child during a procedure. c. Always reassure the child and family. d. Give explanations using professional terminology. ANS: B Include parents as partners in the childs treatments. Parents may need direct guidance in concrete terms to help distract the child. Allowing the parents to remain with the child may help calm the child. Telling the truth is the most important thing. False reassurance does not facilitate a trusting
CURRENT DIAGNOSIS AND TREATMENT PEDIATRICS 24TH LATEST relationship. Professional terminology may not be understood. Speak to the child and family in language that they will understand. 2. The father of a child in the emergency department is yelling at the physician and nurses. Which action would be contraindicated in this situation? a. Provide a nondefensive response. b. Encourage the father to talk about his feelings. c. Speak in simple, short sentences. d. Tell the father he must wait in the waiting room. ANS: D Because a parent who is upset may be aggravated by observers, he should be directed to a quiet area. When dealing with parents who are upset, it is important not to be defensive or attempt to justify anyones actions. Encouraging the father to talk about his feelings may assist him to acknowledge his emotions and may defuse his angry reaction. People who are upset need to be spoken to with simple words (no longer than five letters) and short sentences (no more than five words). 3. Which would be an appropriate nursing intervention for a 6-month-old infant in the emergency department? a. Distract the infant with noise or bright lights. b. Avoid warming the infant. c. Remove any pacifiers from the baby. d. Encourage the parent to hold the infant. ANS: D Parents should be encouraged to hold the infant as much as possible while in the emergency department. Having the parent hold the infant may help to calm the child. Distraction with noise or bright lights would be most appropriate for a preschool-age child. In an emergency healthcare
CURRENT DIAGNOSIS AND TREATMENT PEDIATRICS 24TH LATEST facility, it is important to keep infants warm. Infants use pacifiers to comfort themselves; therefore, the pacifier should not be taken away. 4. Which action should the nurse working in the emergency department initiate to decrease fear in a 2-year-old child? a. Keep the child physically restrained during nursing care. b. Allow the child to hold a favorite toy or blanket. c. Direct the parents to remain outside the treatment room. d. Let the child decide whether to sit up or lie down for procedures. ANS: B Allowing a child to hold a favorite toy or blanket is comforting. It may be necessary to restrain the toddler for some nursing care or procedures. Because toddlers need autonomy and do not respond well to restrictions, the nurse should remove any restriction or restraint as soon as safety permits. Parents should remain with the child as much as possible to calm and reassure the child. The toddler should not be given the overwhelming choice of deciding which position she prefers. 5. Which nursing action would be most appropriate to assist a preschool-age child in coping with the emergency department experience? a. Explain the procedures and give the child some time to prepare. b. Remind the child that she is a big girl. c. Avoid the use of bandages. d. Use positive terms and avoid terms such as shot and cut. ANS: D Using positive terms and avoiding words that have frightening connotations assist the child in coping. Preschool-age children should be told about procedures immediately before they are done. Time to prepare only allows time for fantasies and increased anxiety. Children should not be shamed into cooperation. Bandages are important to preschool-aged children. Children in this age group believe that their insides can leak out and that bandages stop this from happening.
CURRENT DIAGNOSIS AND TREATMENT PEDIATRICS 24TH LATEST 6. Which action should the nurse incorporate into a care plan for a 14-year-old child in the emergency department? a. Limit the number of choices to be made by the adolescent. b. Insist that parents remain with the adolescent. c. Provide clear explanations and encourage questions. d. Give rewards for cooperation with procedures. ANS: C Adolescents are capable of abstract thinking and can understand explanations. They should be offered the opportunity to ask questions and make decisions. Adolescents should have the choice of whether parents remain with them. They are very modest, and this modesty should be respected. Giving rewards such as stickers for cooperation with treatments or procedures is more appropriate for the younger child. 7. The emergency department nurse notices that the mother of a young child is making a lot of phone calls and getting advice from her friends about what she should do. This behavior is an indication of which of the following? a. Stress b. Healthy coping skills c. Attention-getting behaviors d. Low self-esteem ANS: A Hyperactive behavior such as making a lot of phone calls and enlisting everyones opinions is a sign of stress. This is not a healthy coping skill and may be an attention-getting behavior or indicative of the mother having low self-esteem, but is more likely an indicator of stress. 8. A preschool child in the emergency department has a respiratory rate of 10 breaths per minute. How should the nurse interpret this finding?
CURRENT DIAGNOSIS AND TREATMENT PEDIATRICS 24TH LATEST a. The child is relaxed. b. Respiratory failure is likely. c. This child is in respiratory distress. d. The childs condition is improving. ANS: B
Chapter 13. Poisoning 1. A nurse has completed a teaching session for parents about baby-proofing the home. Which statements made by the parents indicate an understanding of the teaching? Select all that apply. a. We will put plastic fillers in all electrical plugs. b. We will place poisonous substances in a high cupboard. c. We will place a gate at the top and bottom of stairways. d. We will keep our household hot water heater at 130 degrees. e. We will remove front knobs from the stove. ANS: A, C, E By the time babies reach 6 months of age, they begin to become much more active, curious, and mobile. Putting plastic fillers on all electrical plugs can prevent an electrical shock. Putting gates at the top and bottom of stairways will prevent falls. Removing front knobs form the stove can prevent burns. Poisonous substances should be stored in a locked cabinet not in a cabinet that children can reach when they begin to climb. The household hot water heater should be turned down to 120 degrees or less.
15. A child is brought to the emergency department because he ingested an unknown quantity of Tylenol. After gastric lavage is completed, the nurse might expect this child to receive: a. Activated charcoal
CURRENT DIAGNOSIS AND TREATMENT PEDIATRICS 24TH LATEST b. N-Acetylcysteine c. Vitamin K d. Syrup of ipecac ANS: B Gastric lavage is followed by N-acetylcysteine (Mucomyst), the antidote for acetaminophen. 16. The nurse, planning a parent education program about lead poisoning prevention, would include the information that the sources of lead in the community are most likely: a. Increased lead content of air b. Use of aluminum cookware c. Deteriorating paint in older buildings d. Inhaling smog ANS: C The primary source of lead is paint from old, deteriorating buildings. 17. A frightened mother calls a neighbor because her child swallowed dishwashing detergent. The most appropriate action that the neighbor can advise is: a. Induce vomiting by giving the child syrup of ipecac. b. Take the child to the local emergency department. c. Give the child activated charcoal mixed with juice. d. Give the child milk to soothe affected mucous membranes. ANS: B Inducing vomiting is no longer recommended because it may pose additional problems. The child should be taken immediately to the nearest emergency department. 18. A child has been diagnosed with ascariasis (roundworm). The statement made by her mother that may suggest a cause for her condition is:
CURRENT DIAGNOSIS AND TREATMENT PEDIATRICS 24TH LATEST a. Ive been airing out the house on these nice breezy days. b. My child often goes out to the garden and pulls up a carrot to eat. c. She runs barefoot so much I have to wash her feet at least twice a day. d. We just remodeled our bathroom at home. ANS: B The child can ingest roundworm eggs from contaminated soil.
Chapter 14. Critical Care 1. A child in the ICU exhibits tachycardia, tachypnea, hypertension, and low pulse pressure in the extremities. Which of these signs is the best indicator of inadequate perfusion of blood? A)
Tachycardia
B)
Tachypnea
C)
Hypertension
D)
Low pulse pressure
2. A 12-month-old child is admitted to the ICU after being found abandoned in a hot car. The child is responsive, but the nurse notices that the anterior fontanelle of the child is sunken. Which of the following conditions should the nurse suspect in this situation? A)
Dehydration
B)
Increased intracranial pressure
C)
Fluid overload
D)
Hypertension
3. An ICU nurse, in testing the reflexes of a 3-month-old baby girl, strokes the lateral aspect of the sole of the foot to test for possible nerve damage. In response, the child fans her toes and dorsiflexes her big toe. What is the name of this reflex, and is the childs response normal? A)
Moro reflex; abnormal
B)
Rooting reflex; normal
CURRENT DIAGNOSIS AND TREATMENT PEDIATRICS 24TH LATEST C)
Grasp reflex; abnormal
D)
Babinski reflex; normal
4. A nurse in the ICU observes paradoxical irritability, meningeal irritability, and nuchal rigidity in a 5-year-old girl. She suspects meningitis. What other finding or findings would support this belief? Select all that apply. A)
Positive Babinski reflex
B)
Positive Moro reflex
C)
Positive Brudzinskis sign
D)
Positive Kernigs sign
5. A newborn baby involved in an auto accident has been admitted to the ICU and is currently being assessed by a nurse in a drafty, cool room. The nurse notes mottling on the infants skin, a progression of coolness toward the trunk, peripheral cyanosis, normal blood pressure, low pulse pressure, and tachycardia. Given the circumstances, which of the following are reliable signs that the infant is in shock? Select all that apply. A)
Mottled skin
B)
Progression of coolness toward the trunk
C)
Peripheral cyanosis
D)
Normal blood pressure
E)
Low pulse pressure
F)
Tachycardia
6. A nurse working with an infant patient observes that the child, who has a nasogastric tube, is grunting on expiration, wheezing, and seesaw breathing. The nurse recognizes that the infant is most likely experiencing which condition? A)
Respiratory distress due to obstructed nasal passages
B)
Respiratory arrest due to obstruction of the glottis
C)
Epileptic seizure due to overstimulation
D)
Shock due to loss of blood
7. A 2-year-old is admitted to the ICU due to severe dehydration. The mother of the child is distraught and feels guilty that she took a long walk with her child in the sun earlier in the day. She said that she and the rest of the family had no issues with dehydration. What would be the best response for the nurse to give the mother? A)
Children his age should have limited sun exposure and should be covered with sunscreen.
CURRENT DIAGNOSIS AND TREATMENT PEDIATRICS 24TH LATEST B)
The mother should regularly test the specific gravity of the childs urine to determine whether he is adequately hydrated.
C)
Children are at increased risk for dehydration due to a higher percentage of totalbody water.
D)
The mother should look for taut skin and edema as signs of dehydration in her child.
8. A 6-year-old girl who is in the ICU for an unrelated reason confides in the nurse that her father sometimes hits her. Which of the following findings would most likely indicate abuse in this situation? A)
A bruise on her knee
B)
A bruise on the bottom of her foot
C)
A bruise on her abdomen
D)
A bruise on her face
9. A 12-year-old boy is experiencing severe pain following emergency surgery in the ICU. He is visibly frustrated and restless. His parents would like to achieve a steady state of pain relief for their son and to give him a sense of some control over the pain. Which pain relief method would be most appropriate in this situation? A)
Distraction by means of a video game
B)
Massage
C)
Opioids
D)
PCA
10. The nurse observes that a child in the ICU who has been admitted following an auto accident is visibly stressed and crying. The child has a broken arm, which has been splinted, and is receiving analgesics. No other injuries have been found. When the nurse performs a pain assessment test with the child, the child refuses to cooperate. The patients mother is hysterical and is frantically pacing next to the bed. What is the most likely cause of the childs distress? A)
Fear of further medical interventions
B)
The mothers communicated anxiety
C)
Pain from the broken arm
D)
Shock
11. The nurse is caring for a critically ill child. The child has been experiencing tachypnea for several hours despite appropriate interventions. Suddenly, the child becomes bradypneic. What is the most appropriate nursing action? A)
Continue the current plan of care, as this change indicates improvement.
CURRENT DIAGNOSIS AND TREATMENT PEDIATRICS 24TH LATEST B)
Initiate positive-pressure ventilation with a manual bag-mask device.
C)
Increase intravenous sedation and narcotic dosages to continue improvement.
D)
Ask the family to leave the room in anticipation of further action.
12. A child is admitted to the ICU after being in a motor vehicle crash. The nurse assesses a sudden decrease in the pulse rate. The nurse knows that this change in pulse rate is most likely related to what pathology? A)
Anxiety
B)
Hyperventilation
C)
Shock
D)
Hypoxemia
13. A critically ill child who is experiencing internal bleeding from a motor vehicle accident demonstrates a sudden significant drop in blood pressure. What intervention should the nurse implement first? A)
Administer 20 mL/kg intravenous crystalloid fluid.
B)
Assess for oversedation with benzodiazepines or narcotics.
C)
Increase frequency of vital sign assessment to every 15 minutes.
D)
Reassess mental status and level of consciousness.
14. A 6-month-old infant, who was admitted for gastroenteritis, has depressed fontanelle at rest and while in a supine position. What additional assessments should the nurse perform or facilitate? A)
Denver Developmental Test
B)
Deep tendon reflexes
C)
Number of wet diapers per day
D)
Measure the frontal occipital circumference
15. An infant with a history of diarrhea and poor feeding for 2 days is admitted to a CCU. The infant has progression of coolness toward the trunk, prolonged capillary refill, and moderate tachycardia and is somewhat difficult to arouse. What additional assessment data should the nurse collect first? A)
Cardiac monitor pattern
B)
Serial blood pressure
C)
Deep tendon reflexes
D)
Status of anterior fontanelle
CURRENT DIAGNOSIS AND TREATMENT PEDIATRICS 24TH LATEST 16. The nurse notices that a 5-month-old patient is having difficulty breathing, with nasal flaring, audible grunting, and sternal and intercostal retractions. The infants respiratory rate is rapid, and the infant is restless and obtunded. What intervention should the nurse implement first? A)
Position the baby with head in a neutral midline position and apply the jaw-thrust maneuver.
B)
Facilitate administration of an inhaled bronchodilator.
C)
Administer intravenous sedation or pain medication.
D)
Ask the parent to use comforting measures for the infant.
17. A young child has repeated upper respiratory infections. The mother is distressed and states that she is sure she is taking care of her child properly and does not understand why her child is constantly ill. Nursing assessment validates that the mother is providing a balanced diet, the child eats well, the mother is careful with hand hygiene, and the child has no genetic or chronic diseases. The child does attend day care while the mother works. On what rationale should the nurse base her reassurance to the mother? A)
All small children are sick all the time, and they outgrow this behavior in a few years.
B)
The day care center is probably the cause of the constant illness, and the child should be moved.
C)
Small children have an immature immune system and are more susceptible to viral illness.
D)
The mother should be referred for counseling as she is exhibiting a pathological lack of coping.
18. The nurse enters the room of a critically ill child who is newly unresponsive. What is the next nursing action? A)
Open the airway.
B)
Place on side.
C)
Give abdominal thrusts.
D)
Start rescue breathing.
19. A child experiences a respiratory arrest and is intubated. What is the best nursing assessment parameter to immediately verify proper placement of the endotracheal tube? A)
Symmetrical rise and fall of the chest
B)
Chest x-ray shows right bronchial intubation.
C)
Tidal volume is within limits for age.
CURRENT DIAGNOSIS AND TREATMENT PEDIATRICS 24TH LATEST D)
Absence of sternal retractions and grunting
20. The nurse is preparing to administer a medication to a child. The medication is ordered as 10 mg/kg. When the nurse looks up the childs weight on the vital sign record, no unit of measure is documented. What is the best nursing action? A)
Assume that the weight was measured in kilograms, since that is unit policy.
B)
Call the staff member who weighed the child and verify the unit of measure.
C)
Use arithmetic analysis to determine the probable unit of measure used.
D)
Reweigh the child to ensure accuracy of weight determination for the day.
21. Since most medications in pediatrics are calculated based on the childs weight, considerable nursing time is required to verify correct dosages. This time constraint can be a problem in emergent situations. What mechanism can be used to facilitate accuracy of dosing during emergencies? A)
Precalculate doses of emergency drugs for each child admitted.
B)
Estimate childs weight and drug doses in emergencies to save time.
C)
Use the standard doses from the Pediatric Advanced Life Support recommendations.
D)
Add a pharmacist to the emergency response team for pediatrics.
22. The nurse is caring for a 1-year-old child who has had major surgery and suspects that the child is in pain. What method should the nurse use to assess the child for pain? A)
A one-dimensional system such as the Faces scale
B)
A multidimensional system such as COMFORT
C)
Ask the parents to decide if the child is in pain.
D)
Assess vital signs, restlessness, crying, and diaphoresis.
23. A young child is experiencing pain secondary to surgical repair of a fractured femur. The child is on supplemental oxygen and is breathing spontaneously and crying. The nurse wishes to administer the most effective pain relief with the least risk of adverse effects. Which combination of relief measures is most likely to meet this goal? A)
Antipsychotic medication and sedative
B)
Opioid and distraction strategies
C)
Sedative and opioid
D)
Muscle relaxant and massage
24. A pediatric critical care unit wishes to ensure family-centered care. What nursing actions will best support family-centered care?
CURRENT DIAGNOSIS AND TREATMENT PEDIATRICS 24TH LATEST A)
Asking parents to leave during painful procedures for the child
B)
Assuming that the childs illness is secondary to parental ignorance
C)
Encouraging a family member to be constantly present at the childs side
D)
Restricting visits by family to ensure that the child can rest
Answer Key 1. D 2. A 3. D 4. C, D 5. B, E 6. A 7. C 8. C 9. D 10. B 11. B 12. D 13. A 14. C 15. D 16. A 17. C 18. A 19. A 20. D 21. A 22. B 23. B
CURRENT DIAGNOSIS AND TREATMENT PEDIATRICS 24TH LATEST 24. C
Chapter 15. Skin 1. What should be included in teaching a parent about the management of small red macules and vesicles that become pustules around the childs mouth and cheek? a. Keep the child home from school for 24 hours after initiation of antibiotic treatment. b. Clean the rash vigorously with Betadine three times a day. c. Notify the physician for any itching. d. Keep the child home from school until the lesions are healed. ANS: A To prevent the spread of impetigo to others, the child should be kept home from school for 24 hours after treatment is initiated. Good hand washing is imperative in preventing the spread of impetigo. The lesions should be washed gently with a warm soapy washcloth three times a day. Washcloth should not be shared with other members of the family. Itching is common and does not necessitate medical treatment. Rather, parents should be taught to clip the childs nails to prevent maceration of the lesions. The child may return to school 24 hours after initiation of antibiotic treatment. 2. When taking a history of a child with cellulitis, which information would be most pertinent for the nurse to assess? a. Any medication the child is taking b. Enlarged, mobile, and nontender lymph nodes c. Childs urinalysis results d. Recent infections or signs of infection ANS: D
CURRENT DIAGNOSIS AND TREATMENT PEDIATRICS 24TH LATEST Cellulitis may follow an upper respiratory infection, sinusitis, otitis media, or a tooth abscess. The affected area is red, hot, tender, and indurated. Medication history is important, but the history of recent infections is more relevant to the diagnosis. Lymph nodes may be enlarged (lymphadenitis), but they are not mobile and are nontender. Lymphangitis may be seen, with red streaking of the surrounding area. An abnormal urinalysis result is not usually associated with cellulitis. 3. Which statement made by a parent indicates an understanding about the management of a child with cellulitis? a. I am supposed to continue the antibiotic until the redness and swelling disappear. b. I have been putting ice on my sons arm to relieve the swelling. c. I should call the doctor if the redness disappears. d. I have been putting a warm soak on my sons arm every 4 hours. ANS: D Warm soaks applied every 4 hours while the child is awake increase circulation to the infected area, relieve pain, and promote healing. The parent should not discontinue antibiotics when signs of infection disappear. To ensure complete healing, the parent should understand that the entire course of antibiotics should be given as prescribed. A warm soak is indicated for the treatment of cellulitis. Ice will decrease circulation to the affected area and inhibit the healing process. The disappearance of redness indicates healing and is not a reason to seek medical advice. 4. What should the parents of an infant with thrush (oral candidiasis) be taught about medication administration? a. Give nystatin suspension with a syringe without a needle. b. Apply nystatin cream to the affected area twice a day. c. Give nystatin just before the infant is fed. d. Rub nystatin suspension onto the oral mucous membranes with a gloved finger after feedings. ANS: D
CURRENT DIAGNOSIS AND TREATMENT PEDIATRICS 24TH LATEST It is important to apply the nystatin suspension to the affected areas, which is best accomplished by rubbing it onto the gums and tongue, after feedings, every 6 hours, until 3 to 4 days after symptoms have disappeared. Medication may not reach the affected areas when it is squirted into the infants mouth. Rubbing the suspension onto the gum ensures contact with the affected areas. Nystatin cream is used for diaper rash caused by Candida. To prolong contact with the affected areas, the medication should be administered after a feeding. 5. What beverage should the parents of a child with ringworm be taught to give along with the prescribed griseofulvin (Fulvicin)? a. Water b. A carbonated drink c. Milk d. Fruit juice ANS: C Griseofulvin is insoluble in water. Giving the medication with a high-fat meal or milk increases absorption. Carbonated drinks do not contain fat, which aids in the absorption of griseofulvin. Fruit juice does not contain any fat; fat aids absorption of the medication. 6. Which assessment is applicable to the care of a child with herpetic gingivostomatitis? a. Comparison of range of motion for the upper and lower extremities b. Urine output, mucous membranes, and skin turgor c. Growth pattern since birth d. Bowel elimination pattern ANS: B The child with herpetic gingivostomatitis is at risk for fluid volume deficit. Painful lesions on the mouth make drinking unpleasant and undesirable, with subsequent dehydration becoming a real danger. An oral herpetic infection does not affect joint function. Herpetic gingivostomatitis is not a chronic disorder that would affect the childs long-term growth pattern. Although constipation
CURRENT DIAGNOSIS AND TREATMENT PEDIATRICS 24TH LATEST could be caused by dehydration, it is more important to assess urine output, skin turgor, and mucous membranes to identify dehydration before constipation is a problem. 7. Parents of the child with lice infestation should be instructed carefully in the use of antilice products because of which potential side effect? a. Nephrotoxicity b. Neurotoxicity c. Ototoxicity d. Bone marrow depression ANS: B Because of the danger of absorption through the skin and potential for neurotoxicity, antilice treatment must be used with caution. A child with many open lesions can absorb enough to cause seizures. Antilice products are not known to be nephrotoxic or ototoxic. Products that treat lice are not known to cause bone marrow depression. 8. When assessing the child with atopic dermatitis, the nurse should ask the parents about a history of which problem? a. Asthma b. Nephrosis c. Lower respiratory tract infections d. Neurotoxicity ANS: A Most children with atopic dermatitis have a family history of asthma, hay fever, or atopic dermatitis and up to 80% of children with atopic dermatitis have asthma or allergic rhinitis. Complications of atopic dermatitis relate to the skin. The renal system is not affected by atopic dermatitis. There is no link between lower respiratory tract infections and atopic dermatitis. Atopic dermatitis does not have a relationship to neurotoxicity.
CURRENT DIAGNOSIS AND TREATMENT PEDIATRICS 24TH LATEST 9. What should the nurse teach an adolescent who is taking tretinoin (Retin-A) for treatment acne? a. The medication should be taken with meals. b. Apply sunscreen before going outdoors. c. Wash with benzoyl peroxide before application. d. The effect of the medication should be evident within 1 week. ANS: B Tretinoin causes photosensitivity and sunscreen should be applied before sun exposure. Tretinoin is a topical medication. Application is not affected by meals. If applied together, benzoyl peroxide and tretinoin have reduced effectiveness and a potentially irritant effect. Optimal results from tretinoin are not achieved for 3 to 5 months. 10. When changing an infants diaper, the nurse notices small bright red papules with satellite lesions on the perineum, anterior thigh, and lower abdomen. This rash is characteristic of which condition? a. Primary candidiasis b. Irritant contact dermatitis c. Intertrigo d. Seborrheic dermatitis ANS: A Small red papules with peripheral scaling in a sharply demarcated area involving the anterior thighs, lower abdomen, and perineum are characteristic of primary candidiasis. A shiny, parchment-like erythematous rash on the buttocks, medial thighs, mons pubis, and scrotum, but not in the folds, is suggestive of irritant contact dermatitis. Intertrigo is identified by a red macerated area of sharp demarcation in the groin folds. It can also develop in the gluteal and neck folds. Seborrheic dermatitis is recognized by salmon-colored, greasy lesions with a yellowish scale found primarily in skin-fold areas or on the scalp.
CURRENT DIAGNOSIS AND TREATMENT PEDIATRICS 24TH LATEST 11. The depth of a burn injury may be classified as: a. localized or systemic. b. superficial, superficial partial thickness, deep partial thickness, or full thickness. c. electrical, chemical, or thermal. d. minor, moderate, or major. ANS: B The vocabulary to classify the depth of burn is superficial, partial thickness, or full thickness. These terms refer to the effect of the burn injury. For example, is there a reaction in the area of the burn (localized) or throughout the body (systemic)? Electrical, chemical, or thermal are terms that refer to the cause of the burn injury. Minor, moderate, or major are terms that refer to the severity of the burn injury. 12. What is the major difference between caring for an infant with burns and an adolescent with burns? a. An increased risk of cardiovascular problems in the infant b. A decreased need for caloric intake in the infant c. An increased risk for hypervolemia in the adolescent d. A decreased need for electrolyte replacement in the infant ANS: A The higher proportion of body fluid to body mass in infants increases the risk of cardiovascular problems because of a less effective cardiovascular response to changing intravascular volume. Infants are at an increased risk for protein and calorie deficiency because they have smaller muscle mass and lower body fat. Hypovolemia is a risk for all burn patients; however, the risk is higher for the infant than for the adolescent. There is an increased risk for electrolyte loss in the infant because of the larger body surface area. 13. Which procedure is contraindicated in the care of a child with a minor partial-thickness burn injury wound?
CURRENT DIAGNOSIS AND TREATMENT PEDIATRICS 24TH LATEST a. Cleaning the affected area with mild soap and water b. Applying antimicrobial ointment to the burn wound c. Changing dressings daily d. Leaving all loose tissue or skin intact ANS: D All loose skin and tissue should be debrided because it can become a breeding ground for infectious organisms. Cleaning with mild soap and water is important to the healing process. Antimicrobial ointment is used on the burn wound to fight infection. Clean dressings are applied daily to prevent wound infection. When dressings are changed, the condition of the burn wound can be assessed. 14. The process of burn shock continues until which physiological mechanism occurs? a. Heart rate returns to normal. b. Airway swelling decreases. c. Body temperature regulation returns to normal. d. Capillaries regain their seal. ANS: D Within minutes of the burn injury, the capillary seals are lost with a massive fluid leakage into the surrounding tissue, resulting in burn shock. The process of burn shock continues for approximately 24 to 48 hours, when capillary seals are restored. The heart rate will be increased throughout the healing process because of increased metabolism. Airway swelling subsides over a period of 2 to 5 days after injury. Body temperature regulation will not be normal until healing is well under way. 15. To assess the child with severe burns for adequate perfusion, the nurse monitors which area? a. Distal pulses b. Skin turgor c. Urine output
CURRENT DIAGNOSIS AND TREATMENT PEDIATRICS 24TH LATEST d. Mucous membranes ANS: C Urine output reflects the adequacy of end-organ perfusion. Distal pulses may be affected by many variables. Urine output is the most reliable indicator of end-organ perfusion. Skin turgor is often difficult to assess on burn patients because the skin is not intact. Mucous membranes do not reflect end-organ perfusion. 16. Which medication would be best for the nurse to administer before a dressing change for the severely burned child? a. Codeine b. Benadryl c. Morphine d. Acetaminophen ANS: C Morphine is the drug of choice for pain management in the severely burned child. It should be administered intravenously. Codeine may be used to diminish pain between dressing changes. Benadryl is administered to relieve discomfort from itching. Acetaminophen can be given for discomfort between painful procedures. 17. Which nursing assessment and care holds the highest priority in the initial care of a child with a major burn injury? a. Establishing and maintaining the childs airway b. Establishing and maintaining intravenous access c. Insertion of a catheter to monitor hourly urine output d. Insertion of a nasogastric tube into the stomach to supply adequate nutrition ANS: A
CURRENT DIAGNOSIS AND TREATMENT PEDIATRICS 24TH LATEST Establishing and maintaining the childs airway are always the priority focus for assessment and care. Establishing intravenous access is the second priority in this situation, after the airway has been established. Inserting a catheter and monitoring hourly urine output are the third most important nursing intervention. Nasogastric feedings are not begun initially on a child with major or severe burns. The initial assessment and care focus for a child with major burn injuries are the ABCs. MULTIPLE RESPONSE 1. A nurse is teaching parents about prevention of diaper dermatitis. Which should the nurse include in the teaching plan? Select all that apply. a. Clean the diaper area gently after every diaper change with a mild soap. b. Use a protective ointment to clean dry intact skin. c. Use a steroid cream after each diaper change. d. Use rubber or plastic pants over the diaper. e. Wash cloth diapers in hot water with a mild soap and double rinse. ANS: A, B, E Prompt, gentle cleaning with water and mild soap (Dove, Neutrogena Baby Soap) after each voiding or defecation rids the skin of ammonia and other irritants and decreases the chance of skin breakdown and infection. A bland, protective ointment (A&D, Balmex, Desitin, zinc oxide) can be applied to clean, dry, intact skin to help prevent diaper rash. If cloth diapers are laundered at home, the parents should wash them in hot water, using a mild soap and double rinsing. Occlusion increases the risk of systemic absorption of steroid; thus steroid creams are rarely used for diaper dermatitis because the diaper functions as an occlusive dressing. Rubber or plastic pants increase skin breakdown by holding in moisture and should be used infrequently. 2. A nurse is instructing parents on treatment of pediculosis (head lice). Which should the nurse include in the teaching plan? Select all that apply. a. Bedding should be washed in warm water and dried on a low setting.
CURRENT DIAGNOSIS AND TREATMENT PEDIATRICS 24TH LATEST b. After treating the hair and scalp with a pediculicide, shampoo the hair with regular shampoo. c. Retreat the hair and scalp with a pediculicide in 7 to 10 days. d. Items that cannot be washed should be dry cleaned or sealed in plastic bags for 2 to 3 weeks. e. Combs and brushes should be boiled in water for at least 10 minutes. ANS: C, D, E An over-the-counter pediculicide, permethrin 1% (Nix, Elimite, Acticin), kills head lice and eggs with one application and has residual activity (i.e., it stays in the hair after treatment) for 10 days. Nix Creme Rinse is applied to the hair after it is washed with a conditioner-free shampoo. The product should be rinsed out after 10 minutes. The hair should not be shampooed for 24 hours after the treatment. Even though the kill rate is high and there is residual action, retreatment should occur after 7 to 10 days. Combs and brushes should be boiled or soaked in antilice shampoo or hot water [greater than 60 C (140 F)] for at least 10 minutes. Advise parents to wash clothing (especially hats and jackets), bedding, and linens in hot water and dry at a hot dryer setting.
Chapter 16. Eye 1. When examining the eye, the nurse notices that the patients eyelid margins approximate completely. The nurse recognizes that this assessment finding: a. Is expected. b. May indicate a problem with extraocular muscles. c. May result in problems with tearing. d. Indicates increased intraocular pressure. ANS: A The palpebral fissure is the elliptical open space between the eyelids, and, when closed, the lid margins approximate completely, which is a normal finding.
CURRENT DIAGNOSIS AND TREATMENT PEDIATRICS 24TH LATEST 2. During ocular examinations, the nurse keeps in mind that movement of the extraocular muscles is: a. Decreased in the older child. b. Impaired in a patient with cataracts. c. Stimulated by cranial nerves (CNs) I and II. d. Stimulated by CNs III, IV, and VI. ANS: D Movement of the extraocular muscles is stimulated by three CNs: III, IV, and VI. 3. The nurse is performing an external eye examination. Which statement regarding the outer layer of the eye is true? a. The outer layer of the eye is very sensitive to touch. b. The outer layer of the eye is darkly pigmented to prevent light from reflecting internally. c. The trigeminal nerve (CN V) and the trochlear nerve (CN IV) are stimulated when the outer surface of the eye is stimulated. d. The visual receptive layer of the eye in which light waves are changed into nerve impulses is located in the outer layer of the eye. ANS: A The cornea and the sclera make up the outer layer of the eye. The cornea is very sensitive to touch. The middle layer, the choroid, has dark pigmentation to prevent light from reflecting internally. The trigeminal nerve (CN V) and the facial nerve (CN VII) are stimulated when the outer surface of the eye is stimulated. The retina, in the inner layer of the eye, is where light waves are changed into nerve impulses. 4. When examining a patients eyes, the nurse recalls that stimulation of the sympathetic branch of the autonomic nervous system: a. Causes pupillary constriction.
CURRENT DIAGNOSIS AND TREATMENT PEDIATRICS 24TH LATEST b. Adjusts the eye for near vision. c. Elevates the eyelid and dilates the pupil. d. Causes contraction of the ciliary body. ANS: C Stimulation of the sympathetic branch of the autonomic nervous system dilates the pupil and elevates the eyelid. Parasympathetic nervous system stimulation causes the pupil to constrict. The muscle fibers of the iris contract the pupil in bright light to accommodate for near vision. The ciliary body controls the thickness of the lens. 5. The nurse is reviewing causes of increased intraocular pressure. Which of these factors determines intraocular pressure? a. Thickness or bulging of the lens b. Posterior chamber as it accommodates increased fluid c. Contraction of the ciliary body in response to the aqueous within the eye d. Amount of aqueous produced and resistance to its outflow at the angle of the anterior chamber ANS: D Intraocular pressure is determined by a balance between the amount of aqueous produced and the resistance to its outflow at the angle of the anterior chamber. The other responses are incorrect. 6. The nurse is conducting a visual examination. Which of these statements regarding visual pathways and visual fields is true? a. The right side of the brain interprets the vision for the right eye. b. The image formed on the retina is upside down and reversed from its actual appearance in the outside world. c. Light rays are refracted through the transparent media of the eye before striking the pupil. d. Light impulses are conducted through the optic nerve to the temporal lobes of the brain.
CURRENT DIAGNOSIS AND TREATMENT PEDIATRICS 24TH LATEST ANS: B The image formed on the retina is upside down and reversed from its actual appearance in the outside world. The light rays are refracted through the transparent media of the eye before striking the retina, and the nerve impulses are conducted through the optic nerve tract to the visual cortex of the occipital lobe of the brain. The left side of the brain interprets vision for the right eye. 7. The nurse is testing a patients visual accommodation, which refers to which action? a. Pupillary constriction when looking at a near object b. Pupillary dilation when looking at a far object c. Changes in peripheral vision in response to light d. Involuntary blinking in the presence of bright light ANS: A The muscle fibers of the iris contract the pupil in bright light and accommodate for near vision, which also results in pupil constriction. The other responses are not correct. 8. A patient has a normal pupillary light reflex. The nurse recognizes that this reflex indicates that: a. The eyes converge to focus on the light. b. Light is reflected at the same spot in both eyes. c. The eye focuses the image in the center of the pupil. d. Constriction of both pupils occurs in response to bright light. ANS: D The pupillary light reflex is the normal constriction of the pupils when bright light shines on the retina. The other responses are not correct. 9. A mother asks when her newborn infants eyesight will be developed. The nurse should reply:
CURRENT DIAGNOSIS AND TREATMENT PEDIATRICS 24TH LATEST a. Vision is not totally developed until 2 years of age. b. Infants develop the ability to focus on an object at approximately 8 months of age. c. By approximately 3 months of age, infants develop more coordinated eye movements and can fixate on an object. d. Most infants have uncoordinated eye movements for the first year of life. ANS: C Eye movements may be poorly coordinated at birth, but by 3 to 4 months of age, the infant should establish binocularity and should be able to fixate simultaneously on a single image with both eyes. 10. The nurse is reviewing in age-related changes in the eye for a class. Which of these physiologic changes is responsible for presbyopia? a. Degeneration of the cornea b. Loss of lens elasticity c. Decreased adaptation to darkness d. Decreased distance vision abilities ANS: B The lens loses elasticity and decreases its ability to change shape to accommodate for near vision. This condition is called presbyopia. 11. Which of these assessment findings would the nurse expect to see when examining the eyes of a black patient? a. Increased night vision b. Dark retinal background c. Increased photosensitivity d. Narrowed palpebral fissures
CURRENT DIAGNOSIS AND TREATMENT PEDIATRICS 24TH LATEST ANS: B An ethnically based variability in the color of the iris and in retinal pigmentation exists, with darker irides having darker retinas behind them. 12. A 52-year-old patient describes the presence of occasional floaters or spots moving in front of his eyes. The nurse should: a. Examine the retina to determine the number of floaters. b. Presume the patient has glaucoma and refer him for further testing. c. Consider these to be abnormal findings, and refer him to an ophthalmologist. d. Know that floaters are usually insignificant and are caused by condensed vitreous fibers. ANS: D Floaters are a common sensation with myopia or after middle age and are attributable to condensed vitreous fibers. Floaters or spots are not usually significant, but the acute onset of floaters may occur with retinal detachment. 13. The nurse is preparing to assess the visual acuity of a 16-year-old patient. How should the nurse proceed? a. Perform the confrontation test. b. Ask the patient to read the print on a handheld Jaeger card. c. Use the Snellen chart positioned 20 feet away from the patient. d. Determine the patients ability to read newsprint at a distance of 12 to 14 inches. ANS: C The Snellen alphabet chart is the most commonly used and most accurate measure of visual acuity. The confrontation test is a gross measure of peripheral vision. The Jaeger card or newspaper tests are used to test near vision.
CURRENT DIAGNOSIS AND TREATMENT PEDIATRICS 24TH LATEST 14. A patients vision is recorded as 20/30 when the Snellen eye chart is used. The nurse interprets these results to indicate that: a. At 30 feet the patient can read the entire chart. b. The patient can read at 20 feet what a person with normal vision can read at 30 feet. c. The patient can read the chart from 20 feet in the left eye and 30 feet in the right eye. d. The patient can read from 30 feet what a person with normal vision can read from 20 feet. ANS: B The top number indicates the distance the person is standing from the chart; the denominator gives the distance at which a normal eye can see. 15. A patient is unable to read even the largest letters on the Snellen chart. The nurse should take which action next? a. Refer the patient to an ophthalmologist or optometrist for further evaluation. b. Assess whether the patient can count the nurses fingers when they are placed in front of his or her eyes. c. Ask the patient to put on his or her reading glasses and attempt to read the Snellen chart again. d. Shorten the distance between the patient and the chart until the letters are seen, and record that distance. ANS: D If the person is unable to see even the largest letters when standing 20 feet from the chart, then the nurse should shorten the distance to the chart until the letters are seen, and record that distance (e.g., 10/200). If visual acuity is even lower, then the nurse should assess whether the person can count fingers when they are spread in front of the eyes or can distinguish light perception from a penlight. If vision is poorer than 20/30, then a referral to an ophthalmologist or optometrist is necessary, but the nurse must first assess the visual acuity.
CURRENT DIAGNOSIS AND TREATMENT PEDIATRICS 24TH LATEST 16. A patients vision is recorded as 20/80 in each eye. The nurse interprets this finding to mean that the patient: a. Has poor vision. b. Has acute vision. c. Has normal vision. d. Is presbyopic. ANS: A Normal visual acuity is 20/20 in each eye; the larger the denominator, the poorer the vision. 17. When performing the corneal light reflex assessment, the nurse notes that the light is reflected at 2 oclock in each eye. The nurse should: a. Consider this a normal finding. b. Refer the individual for further evaluation. c. Document this finding as an asymmetric light reflex. d. Perform the confrontation test to validate the findings. ANS: A Reflection of the light on the corneas should be in exactly the same spot on each eye, or symmetric. If asymmetry is noted, then the nurse should administer the cover test. 18. The nurse is performing the diagnostic positions test. Normal findings would be which of these results? a. Convergence of the eyes b. Parallel movement of both eyes c. Nystagmus in extreme superior gaze d. Slight amount of lid lag when moving the eyes from a superior to an inferior position
CURRENT DIAGNOSIS AND TREATMENT PEDIATRICS 24TH LATEST ANS: B A normal response for the diagnostic positions test is parallel tracking of the object with both eyes. Eye movement that is not parallel indicates a weakness of an extraocular muscle or dysfunction of the CN that innervates it. 19. During an assessment of the sclera of a black patient, the nurse would consider which of these an expected finding? a. Yellow fatty deposits over the cornea b. Pallor near the outer canthus of the lower lid c. Yellow color of the sclera that extends up to the iris d. Presence of small brown macules on the sclera ANS: D Normally in dark-skinned people, small brown macules may be observed in the sclera.
20. A 2-week-old infant can fixate on an object but cannot follow a light or bright toy. The nurse would: a. Consider this a normal finding. b. Assess the pupillary light reflex for possible blindness. c. Continue with the examination, and assess visual fields. d. Expect that a 2-week-old infant should be able to fixate and follow an object. ANS: A By 2 to 4 weeks an infant can fixate on an object. By the age of 1 month, the infant should fixate and follow a bright light or toy. 21. The nurse is assessing color vision of a male child. Which statement is correct? The nurse should: a. Check color vision annually until the age of 18 years. b. Ask the child to identify the color of his or her clothing. c. Test for color vision once between the ages of 4 and 8 years. d. Begin color vision screening at the childs 2-year checkup. ANS: C
CURRENT DIAGNOSIS AND TREATMENT PEDIATRICS 24TH LATEST Test boys only once for color vision between the ages of 4 and 8 years. Color vision is not tested in girls because it is rare in girls. Testing is performed with the Ishihara test, which is a series of polychromatic cards. 22. The nurse is performing an eye-screening clinic at a daycare center. When examining a 2year-old child, the nurse suspects that the child has a lazy eye and should: a. Examine the external structures of the eye. b. Assess visual acuity with the Snellen eye chart. c. Assess the childs visual fields with the confrontation test. d. Test for strabismus by performing the corneal light reflex test. ANS: D Testing for strabismus is done by performing the corneal light reflex test and the cover test. The Snellen eye chart and confrontation test are not used to test for strabismus. Chapter 17. Oral Medicine and Dentistry 6. The mother of a 7-month-old reports that the first lower central incisor has erupted. She asks the nurse, How many teeth will he have by his first birthday? The nurse would explain that by 1 year of age, the infant usually has: a. Two teeth b. Four teeth c. Six teeth d. Eight teeth ANS: C The 1-year-old infant usually has about six teeth, four above and two below. 7. At a well-baby visit, parents of a 6-month-old ask when to take the infant for the first dental visit. The nurses best response would be: a. If the teeth are brushed regularly, the child should see a dentist by 3 years of age. b. The first dental visit should be arranged after the first tooth erupts. c. The child should have a dental examination when all deciduous teeth have erupted. d. A dental visit by 1 year of age is recommended by the American Academy of Pediatric Dentistry.
CURRENT DIAGNOSIS AND TREATMENT PEDIATRICS 24TH LATEST ANS: D The Academy of Pediatric Dentistry recommends that the first dental visit occur by 1 year of age. 8. The nurse planning anticipatory guidance for the caregiver of a preschool-age child would explain that permanent teeth begin erupting about the age of: a. 4 years b. 6 years c. 8 years d. 10 years ANS: B Permanent teeth do not erupt through the gums until the sixth year. 9. A mother asks the nurse how much food should be offered to her 2-year-old. The nurse responds that a good rule of thumb for serving size would be: a. 2 tablespoons b. 3 tablespoons c. 4 tablespoons d. 5 tablespoons ANS: A The rule of thumb for serving sizes is to offer 1 tablespoon per year of age. 10. An assessment of a childs nutritional status reveals the child is alert, with shiny hair, firm gums, firm mucous membranes, and regular elimination. This childs nutritional status would be described as: a. Overnourished b. Undernourished
CURRENT DIAGNOSIS AND TREATMENT PEDIATRICS 24TH LATEST c. Well nourished d. Borderline ANS: C Well-nourished children show steady gains in height and weight and have shiny hair, firm gums and mucous membranes, and regular elimination. 11. The nurse encourages a Puerto Rican family to bring food to a child because he is not eating the food served on his tray at the hospital. The nurse would expect the child to eat: a. Dried beans mixed with rice b. Crisp vegetables c. Spaghetti and meatballs d. Wild berries, roots, and seeds ANS: A A common food choice of Americans of Puerto Rican descent is dried beans mixed with rice. 3. The nurse cautions that children who are put to sleep with a bottle are at risk for a dental problem called . ANS: milk caries NOT: Rationale: The bacteriocidal effects of saliva decrease during sleep; therefore, when the saliva and the milk combine, they bathe the teeth in a mixture that encourages dental caries.
Chapter 18. Ear, Nose, and Throat 1. The nurse needs to pull the portion of the ear that consists of movable cartilage and skin down and back when administering eardrops. This portion of the ear is called the: a. Auricle. b. Concha. c. Outer meatus. d. Mastoid process.
CURRENT DIAGNOSIS AND TREATMENT PEDIATRICS 24TH LATEST ANS: A The external ear is called the auricle or pinna and consists of movable cartilage and skin. 2. The nurse is examining a patients ears and notices cerumen in the external canal. Which of these statements about cerumen is correct? a. Sticky honey-colored cerumen is a sign of infection. b. The presence of cerumen is indicative of poor hygiene. c. The purpose of cerumen is to protect and lubricate the ear. d. Cerumen is necessary for transmitting sound through the auditory canal. ANS: C The ear is lined with glands that secrete cerumen, which is a yellow waxy material that lubricates and protects the ear. 3. When examining the ear with an otoscope, the nurse notes that the tympanic membrane should appear: a. Light pink with a slight bulge. b. Pearly gray and slightly concave. c. Pulled in at the base of the cone of light. d. Whitish with a small fleck of light in the superior portion. ANS: B The tympanic membrane is a translucent membrane with a pearly gray color and a prominent cone of light in the anteroinferior quadrant, which is the reflection of the otoscope light. The tympanic membrane is oval and slightly concave, pulled in at its center by the malleus, which is one of the middle ear ossicles. 4. The nurse is reviewing the structures of the ear. Which of these statements concerning the eustachian tube is true?
CURRENT DIAGNOSIS AND TREATMENT PEDIATRICS 24TH LATEST a. The eustachian tube is responsible for the production of cerumen. b. It remains open except when swallowing or yawning. c. The eustachian tube allows passage of air between the middle and outer ear. d. It helps equalize air pressure on both sides of the tympanic membrane. ANS: D The eustachian tube allows an equalization of air pressure on each side of the tympanic membrane so that the membrane does not rupture during, for example, altitude changes in an airplane. The tube is normally closed, but it opens with swallowing or yawning. 5. A patient with a middle ear infection asks the nurse, What does the middle ear do? The nurse responds by telling the patient that the middle ear functions to: a. Maintain balance. b. Interpret sounds as they enter the ear. c. Conduct vibrations of sounds to the inner ear. d. Increase amplitude of sound for the inner ear to function. ANS: C Among its other functions, the middle ear conducts sound vibrations from the outer ear to the central hearing apparatus in the inner ear. The other responses are not functions of the middle ear. 6. The nurse is reviewing the function of the cranial nerves (CNs). Which CN is responsible for conducting nerve impulses to the brain from the organ of Corti? a. I b. III c. VIII d. XI ANS: C
CURRENT DIAGNOSIS AND TREATMENT PEDIATRICS 24TH LATEST The nerve impulses are conducted by the auditory portion of CN VIII to the brain. 7. The nurse is assessing a child who may have hearing loss. Which of these statements is true concerning air conduction? a. Air conduction is the normal pathway for hearing. b. Vibrations of the bones in the skull cause air conduction. c. Amplitude of sound determines the pitch that is heard. d. Loss of air conduction is called a conductive hearing loss. ANS: A The normal pathway of hearing is air conduction, which starts when sound waves produce vibrations on the tympanic membrane. Conductive hearing loss results from a mechanical dysfunction of the external or middle ear. The other statements are not true concerning air conduction. 8. A patient has been shown to have a sensorineural hearing loss. During the assessment, it would be important for the nurse to: a. Speak loudly so the patient can hear the questions. b. Assess for middle ear infection as a possible cause. c. Ask the patient what medications he is currently taking. d. Look for the source of the obstruction in the external ear. ANS: C A simple increase in amplitude may not enable the person to understand spoken words. Sensorineural hearing loss may be caused by presbycusis, which is a gradual nerve degeneration that occurs with aging and by ototoxic drugs, which affect the hair cells in the cochlea. 9. During an interview, the patient states he has the sensation that everything around him is spinning. The nurse recognizes that the portion of the ear responsible for this sensation is the: a. Cochlea.
CURRENT DIAGNOSIS AND TREATMENT PEDIATRICS 24TH LATEST b. CN VIII. c. Organ of Corti. d. Labyrinth. ANS: D If the labyrinth ever becomes inflamed, then it feeds the wrong information to the brain, creating a staggering gait and a strong, spinning, whirling sensation called vertigo. 10. The mother of a 2-year-old is concerned because her son has had three ear infections in the past year. What would be an appropriate response by the nurse? a. It is unusual for a small child to have frequent ear infections unless something else is wrong. b. We need to check the immune system of your son to determine why he is having so many ear infections. c. Ear infections are not uncommon in infants and toddlers because they tend to have more cerumen in the external ear. d. Your sons eustachian tube is shorter and wider than yours because of his age, which allows for infections to develop more easily. ANS: D The infants eustachian tube is relatively shorter and wider than the adults eustachian tube, and its position is more horizontal; consequently, pathogens from the nasopharynx can more easily migrate through to the middle ear. The other responses are not appropriate. 11. The primary purpose of the ciliated mucous membrane in the nose is to: a. Warm the inhaled air. b. Filter out dust and bacteria. c. Filter coarse particles from inhaled air. d. Facilitate the movement of air through the nares. ANS: B
CURRENT DIAGNOSIS AND TREATMENT PEDIATRICS 24TH LATEST The nasal hairs filter the coarsest matter from inhaled air, whereas the mucous blanket filters out dust and bacteria. The rich blood supply of the nasal mucosa warms the inhaled air. 12. The projections in the nasal cavity that increase the surface area are called the: a. Meatus. b. Septum. c. Turbinates. d. Kiesselbach plexus. ANS: C The lateral walls of each nasal cavity contain three parallel bony projections: the superior, middle, and inferior turbinates. These increase the surface area, making more blood vessels and mucous membrane available to warm, humidify, and filter the inhaled air. 13. The nurse is reviewing the development of the newborn infant. Regarding the sinuses, which statement is true in relation to a newborn infant? a. Sphenoid sinuses are full size at birth. b. Maxillary sinuses reach full size after puberty. c. Frontal sinuses are fairly well developed at birth. d. Maxillary and ethmoid sinuses are the only sinuses present at birth. ANS: D Only the maxillary and ethmoid sinuses are present at birth. The sphenoid sinuses are minute at birth and develop after puberty. The frontal sinuses are absent at birth, are fairly well developed at age 7 to 8 years, and reach full size after puberty. 14. The tissue that connects the tongue to the floor of the mouth is the: a. Uvula. b. Palate.
CURRENT DIAGNOSIS AND TREATMENT PEDIATRICS 24TH LATEST c. Papillae. d. Frenulum. ANS: D The frenulum is a midline fold of tissue that connects the tongue to the floor of the mouth. The uvula is the free projection hanging down from the middle of the soft palate. The palate is the arching roof of the mouth. Papillae are the rough, bumpy elevations on the tongues dorsal surface. 15. The salivary gland that is the largest and located in the cheek in front of the ear is the gland. a. Parotid b. Stensens c. Sublingual d. Submandibular ANS: A The mouth contains three pairs of salivary glands. The largest, the parotid gland, lies within the cheeks in front of the ear extending from the zygomatic arch down to the angle of the jaw. The Stensens duct (not gland) drains the parotid gland onto the buccal mucosa opposite the second molar. The sublingual gland is located within the floor of the mouth under the tongue. The submandibular gland lies beneath the mandible at the angle of the jaw. 16. In assessing the tonsils of a 13 year old, the nurse notices that they are involuted, granular in appearance, and appear to have deep crypts. What is correct response to these findings? a. Refer the patient to a throat specialist. b. No response is needed; this appearance is normal for the tonsils. c. Continue with the assessment, looking for any other abnormal findings. d. Obtain a throat culture on the patient for possible streptococcal (strep) infection.
CURRENT DIAGNOSIS AND TREATMENT PEDIATRICS 24TH LATEST ANS: B The tonsils are the same color as the surrounding mucous membrane, although they look more granular and their surface shows deep crypts. Tonsillar tissue enlarges during childhood until puberty and then involutes. 17. The nurse is obtaining a health history on a 3-month-old infant. During the interview, the mother states, I think she is getting her first tooth because she has started drooling a lot. The nurses best response would be: a. Youre right, drooling is usually a sign of the first tooth. b. It would be unusual for a 3 month old to be getting her first tooth. c. This could be the sign of a problem with the salivary glands. d. She is just starting to salivate and hasnt learned to swallow the saliva. ANS: D In the infant, salivation starts at 3 months. The baby will drool for a few months before learning to swallow the saliva. This drooling does not herald the eruption of the first tooth, although many parents think it does. 18. The nurse is performing an oral assessment on a 14-year-old Black patient and notices the presence of a 1 cm, nontender, grayish-white lesion on the left buccal mucosa. Which one of these statements is true? This lesion is: a. Leukoedema and is common in dark-pigmented persons. b. The result of hyperpigmentation and is normal. c. Torus palatinus and would normally be found only in smokers. d. Indicative of cancer and should be immediately tested. ANS: A Leukoedema, a grayish-white benign lesion occurring on the buccal mucosa, is most often observed in Blacks.
CURRENT DIAGNOSIS AND TREATMENT PEDIATRICS 24TH LATEST 19. While obtaining a health history, a patient tells the nurse that he has frequent nosebleeds and asks the best way to get them to stop. What would be the nurses best response? a. While sitting up, place a cold compress over your nose. b. Sit up with your head tilted forward and pinch your nose. c. Just allow the bleeding to stop on its own, but dont blow your nose. d. Lie on your back with your head tilted back and pinch your nose. ANS: B With a nosebleed, the person should sit up with the head tilted forward and pinch the nose between the thumb and forefinger for 5 to 15 minutes. 20. The nurse notices that the mother of a 2-year-old boy brings him into the clinic quite frequently for various injuries and suspects there may be some child abuse involved. During an inspection of his mouth, the nurse should look for: a. Swollen, red tonsils. b. Ulcerations on the hard palate. c.
Bruising on the buccal mucosa or gums.
d. Small yellow papules along the hard palate. ANS: C The nurse should notice any bruising or laceration on the buccal mucosa or gums of an infant or young child. Trauma may indicate child abuse from a forced feeding of a bottle or spoon. 21. The nurse is assessing a 3 year old for drainage from the nose. On assessment, a purulent drainage that has a very foul odor is noted from the left naris and no drainage is observed from the right naris. The child is afebrile with no other symptoms. What should the nurse do next? a. Refer to the physician for an antibiotic order. b.
Have the mother bring the child back in 1 week.
c. Perform an otoscopic examination of the left nares. d. Tell the mother that this drainage is normal for a child of this age. ANS: C Children are prone to put an object up the nose, producing unilateral purulent drainage with a foul odor. Because some risk for aspiration exists, removal should be prompt.
Chapter 19. Respiratory Tract and Mediastinum
CURRENT DIAGNOSIS AND TREATMENT PEDIATRICS 24TH LATEST 1. A nurse in the labor and delivery room is assessing respirations on a newborn. The nurse understands that which change in the respiratory system occurs postnatally? a. Respirations are stimulated by hypoxemia. b. It takes up to 48 hours for most of the alveoli to expand. c. Surfactant in the lungs interferes with lung expansion. d. Pulmonary blood flow decreases after birth. ANS: A A postnatal change in the respiratory system is the stimulation of respiration by hypoxemia, hypercarbia, cold, tactile stimulation, and a possible decrease in the concentration of prostaglandin E2. Inflation of the normal lung is complete within a few breaths, and most alveoli have expanded within the first hour of life. Surfactant in the lungs lowers surface tension and facilitates lung expansion. Pulmonary blood flow increases after birth. 2. Which information should the nurse teach families about reducing exposure to pollens and dust? a. Replace wood and tile floors with wall-to-wall carpeting. b. Do not use an air conditioner. c. Put dust-proof covers on pillows and mattresses. d. Keep humidity in the house above 60%. ANS: C Covering mattresses and pillows with dust-proof covers will reduce exposure to dust. Carpets retain dust. To reduce exposure to dust, carpeting should be replaced with wood, tile, slate, or vinyl. These floors can be cleaned easily. For anyone with pollen allergies, it is best to keep windows closed and to run the air conditioner. A humidity level above 60% promotes dust mites. It is recommended that household humidity be kept between 40% and 50% to reduce dust mites inside the house.
CURRENT DIAGNOSIS AND TREATMENT PEDIATRICS 24TH LATEST 3. A child has had cold symptoms for more than 2 weeks, a headache, nasal congestion with purulent nasal drainage, facial tenderness, and a cough that increases during sleep. The nurse recognizes these symptoms are characteristic of which respiratory condition? a. Allergic rhinitis b. Bronchitis c. Asthma d. Sinusitis ANS: D Sinusitis is characterized by signs and symptoms of a cold that do not improve after 14 days, a low-grade fever, nasal congestion and purulent nasal discharge, headache, tenderness, a feeling of fullness over the affected sinuses, halitosis, and a cough that increases when the child is lying down. The classic symptoms of allergic rhinitis are watery rhinorrhea, itchy nose, eyes, ears, and palate, and sneezing. Symptoms occur as long as the child is exposed to the allergen. Bronchitis is characterized by a gradual onset of rhinitis and a cough that is initially nonproductive but may change to a loose cough. The manifestations of asthma may vary, with wheezing being a classic sign. The symptoms presented in the question do not suggest asthma. 4. The child with chronic otitis media with effusion should be evaluated for which problem? a. Brain abscess b. Meningitis c. Hearing loss d. Perforation of the tympanic membrane ANS: C Chronic otitis media with effusion is the most common cause of hearing loss in children. The infection of acute otitis media can spread to surrounding tissues, causing a brain abscess or meningitis. Inflammation and pressure from acute otitis media may result in perforation of the tympanic membrane.
CURRENT DIAGNOSIS AND TREATMENT PEDIATRICS 24TH LATEST 5. The nurse should expect the initial plan of care for a 9-month-old child with an acute otitis media infection to include: a. symptomatic treatment and observation for 48 to 72 hours after diagnosis. b. an oral antibiotic, such as amoxicillin, several times a day for 7 days. c. pneumococcal conjugate vaccine. d. myringotomy with tympanoplasty tubes. ANS: A For select children 6 months of age or older with acute otitis media, as an alternative to initiating antibiotic therapy, once diagnosed, acute otitis media is treated by initiating symptomatic treatment and observation for 48 to 72 hours. Acute otitis media may be treated with a 5- to 10day course of oral antibiotics. When treatment is indicated, amoxicillin at a divided dose of 80 to 90 mg/kg/day given either every 8 or 12 hours for 5 to 10 days may be ordered. Pneumococcal conjugate vaccine helps to prevent ear infections but is not included in the initial plan of care for a child with acute otitis media. Surgical intervention is considered when the child has persistent ear infections despite antibiotic therapy or with otitis media with effusion that persists for more than 3 months and is associated with hearing loss. 6. Which statement made by a parent indicates understanding about treatment of streptococcal pharyngitis? a. I guess my child will need to have his tonsils removed. b. A couple of days of rest and some ibuprofen will take care of this. c. I should give the penicillin three times a day for 10 days. d. I am giving my child prednisone to decrease the swelling of the tonsils. ANS: C Streptococcal pharyngitis is best treated with oral penicillin two to three times daily for 10 days. Surgical removal of the tonsils is a controversial issue. It may be warranted in cases of recurrent tonsillitis. It is not indicated for the treatment of acute tonsillitis. Comfort measures such as rest
CURRENT DIAGNOSIS AND TREATMENT PEDIATRICS 24TH LATEST and analgesics are indicated, but these will not treat the bacterial infection. Corticosteroids are not used in the treatment of streptococcal pharyngitis. 7. A nurse is planning care for a child with laryngomalacia. Which symptom should the nurse plan to assess that is characteristic of laryngomalacia? a. Stridor b. High-pitched cry c. Nasal congestion d. Irritability ANS: A Stridor is usually present at birth but may begin as late as 2 months. Symptoms increase when the infant is supine or crying. High-pitched cries are consistent with neurological abnormalities and are not usually respiratory in nature. Nasal congestion is nonspecific in relation to laryngomalacia. Irritability often occurs with respiratory illnesses; however, it is not the most characteristic symptom of laryngomalacia. 8. The nurse should assess a child who has had a tonsillectomy for which problem? a. Frequent swallowing b. Inspiratory stridor c. Rhonchi d. Elevated white blood cell count ANS: A Frequent swallowing is indicative of postoperative bleeding. Inspiratory stridor is characteristic of croup. Rhonchi are lower airway sounds indicating pneumonia. Assessment of blood cell counts is part of a preoperative workup. 9. A nurse is teaching parents about manifestations of spasmodic croup. Which is a distinguishing manifestation of spasmodic croup that should be included in the teaching session?
CURRENT DIAGNOSIS AND TREATMENT PEDIATRICS 24TH LATEST a. It has a gradual onset. b. It is characterized a harsh barky cough. c. It is bacterial in nature. d. The child has a high fever. ANS: B Spasmodic croup is viral in origin with a sudden onset usually at night of a harsh, metallic, barky cough, sore throat, inspiratory stridor, and hoarseness. A gradual onset is indicative of laryngotracheobronchitis. A high fever is not usually present. 10. Which intervention for treating croup at home should be taught to parents? a. Have a decongestant available to give the child when an attack occurs. b. Have the child sleep in a dry room. c. Sit with the child in the bathroom with the shower on when an attack occurs. d. Give the child an antibiotic at bedtime. ANS: C Sitting in the bathroom with the shower on provides humidity for the child, which usually improves symptoms in croup. Decongestants are inappropriate for croup, which affects the mid airway level. A dry environment may contribute to symptoms. Croup is caused by a virus. Antibiotic treatment is not indicated. 11. A 5-year-old child is brought to the emergency department with copious drooling and a croaking sound on inspiration. Her mother states that the child is very agitated and only wants to sit upright. What should be the nurses first action in this situation? a. Prepare intubation equipment and call the physician. b. Examine the childs oropharynx and call the physician. c. Obtain a throat culture for respiratory syncytial virus (RSV). d. Obtain vital signs and listen to breath sounds.
CURRENT DIAGNOSIS AND TREATMENT PEDIATRICS 24TH LATEST ANS: A The 5-year-old child has symptoms of epiglottitis, is acutely ill, and requires emergency measures. If epiglottitis is suspected, the nurse would never examine the childs throat. Inspection of the epiglottis is done only by a physician because it could trigger airway obstruction. A throat culture could precipitate a complete respiratory obstruction. Vital signs can be assessed after emergency equipment is readied. 12. Which action for care can be taught to the parents of a 3-year-old child with pneumonia who is not hospitalized? a. Offer the child only cool liquids. b. Offer the child a favorite warm liquid drink. c. Use a warm mist humidifier. d. Call the physician for a respiratory rate less than 28 breaths/minute. ANS: B Offering the child favorite fluids will facilitate oral intake. Warm liquids are preferable as they help loosen secretions. Cool mist humidifiers are preferred to warm mist humidifiers. Warm mist is a safety concern and could cause burns if touched by the child. Typically parents are not taught to count their childrens respirations and report abnormalities to the physician. Even if this were the case, a respiratory rate of less than 28 breaths/minute is normal for a 3-year-old child. The expected respiratory rate for a 3-year-old child is 20 to 30 breaths/minute. 13. Which sign is indicative of respiratory distress in infants? a. Nasal flaring b. Respiratory rate of 55 breaths/minute c. Irregular respiratory pattern d. Abdominal breathing ANS: A
CURRENT DIAGNOSIS AND TREATMENT PEDIATRICS 24TH LATEST Infants have difficulty breathing through their mouths; therefore, nasal flaring is usually accompanied by extra respiratory efforts. It also allows more air to enter as the nares flare. A respiratory rate of 55 breaths/minute would be a normal assessment for an infant. Tachypnea would be a respiratory rate of 60 to 80 breaths/minute. Irregular respirations are normal in the infant. Abdominal breathing is common because the diaphragm is the neonates major breathing muscle. 14. Once an allergen is identified in a child with allergic rhinitis, what would be the treatment of choice? a. Use appropriate medications. b. Begin desensitization injections. c. Eliminate the allergen. d. Remove the adenoids. ANS: C The first priority is to attempt to remove the causative agent from the childs environment. Medications are not a first-line treatment but can be helpful in controlling allergic rhinitis. Immunotherapy is usually the final component of controlling allergic rhinitis. Adenoids are tissues that can swell with constant rhinitis; however, a surgical procedure is not indicated for allergic rhinitis. Dealing with the cause is the first priority. 15. A child has returned to the postsurgical floor after having had a tonsillectomy. Which assessment finding should the nurse report to the physician? a. Vomiting bright red blood b. Pain at the surgical site c. Pain on swallowing d. The ability to only take small sips of liquids ANS: A
CURRENT DIAGNOSIS AND TREATMENT PEDIATRICS 24TH LATEST Vomiting bright red blood and swallowing frequently are signs of bleeding postoperatively and should be reported to the physician. It is normal for the child to have pain at the surgical site and on swallowing. Only clear liquids are offered immediately after surgery, and small sips would be preferred. 16. Teaching safety precautions with the administration of antihistamines is important due to which common side effect? a. Dry mouth b. Excitability c. Drowsiness d. Dry mucous membranes ANS: C Drowsiness is a safety hazard when alertness is needed, especially with a teenage driver. Nonsedating brands should be used. A dry mouth is not a safety issue. Excitability may affect rest or sleep, but drowsiness is the most important safety hazard. Dry mucous membranes are not a safety issue. 17. Which is an appropriate beverage for the nurse to give to a child who had a tonsillectomy earlier in the day? a. Chocolate ice cream b. Orange juice c. Fruit punch d. Apple juice ANS: D The child can have clear, cool liquids when fully awake. The child can have full liquids on the second postoperative day. Citrus drinks are not offered because they can irritate the throat. Red liquids are avoided because they give the appearance of blood if vomited.
CURRENT DIAGNOSIS AND TREATMENT PEDIATRICS 24TH LATEST 18. Which intervention should the nurse implement as a priority in the management of a child with epiglottitis? a. Adequate hydration b. Maintaining a patent airway c. Cessation of coughing d. Decreasing fever ANS: B Epiglottitis can rapidly progress to complete airway obstruction and death. The goal of treatment is to maintain a patent airway. The child with epiglottitis will not be able to take fluids orally. Hydration is a concern, but not the priority. Cough is not a symptom of epiglottitis. The child with epiglottitis will have an elevated temperature. Reducing fever is not the priority of care. 19. What information should the nurse teach workers at a day care center about respiratory syncytial virus (RSV)? a. RSV is transmitted through particles in the air. b. RSV can live on skin or paper for up to a few seconds after contact. c. RSV can survive on nonporous surfaces for about 60 minutes. d. Frequent hand washing can decrease the spread of the virus. ANS: D Meticulous hand washing can decrease the spread of organisms. RSV infection is not airborne. It is acquired mainly through contact with contaminated surfaces. RSV can live on skin or paper for up to 1 hour and can live on cribs and other nonporous surfaces for up to 6 hours. 20. Which intervention is appropriate for the infant hospitalized with bronchiolitis? a. Position on the side with neck slightly flexed. b. Administer antibiotics as ordered. c. Restrict oral and parenteral fluids if tachypneic.
CURRENT DIAGNOSIS AND TREATMENT PEDIATRICS 24TH LATEST d. Give cool, humidified oxygen. ANS: D Cool, humidified oxygen is given to relieve dyspnea, hypoxemia, and insensible fluid loss from tachypnea. The infant should be positioned with the head and chest elevated at a 30- to 40-degree angle and the neck slightly extended to maintain an open airway and decrease pressure on the diaphragm. The etiology of bronchiolitis is viral. Antibiotics are given only if there is a secondary bacterial infection. Tachypnea increases insensible fluid loss. If the infant is tachypneic, fluids are given parenterally to prevent dehydration. 21. The nurse is caring for a child hospitalized for status asthmaticus. Which assessment finding suggests that the childs condition is worsening? a. Hypoventilation b. Thirst c. Bradycardia d. Clubbing ANS: A The nurse would assess the child for signs of hypoxia, including restlessness, fatigue, irritability, and increased heart and respiratory rate. As the child tires from the increased work of breathing hypoventilation occurs leading to increased carbon dioxide levels. The nurse would be alert for signs of hypoxia. Thirst would reflect the childs hydration status. Bradycardia is not a sign of hypoxia; tachycardia is. Clubbing develops over a period of months in response to hypoxia. The presence of clubbing does not indicate the childs condition is worsening. 22. Which finding is expected when assessing a child hospitalized for asthma? a. Inspiratory stridor b. Harsh, barky cough c. Wheezing d. Rhinorrhea
CURRENT DIAGNOSIS AND TREATMENT PEDIATRICS 24TH LATEST ANS: C Wheezing is a classic manifestation of asthma. Inspiratory stridor is a clinical manifestation of croup. A harsh, barky cough is characteristic of croup. Rhinorrhea is not associated with asthma. 23. Which statement indicates that a parent of a toddler needs more education about preventing foreign body aspiration? a. I keep objects with small parts out of reach. b. My toddler loves to play with balloons. c. I wont permit my child to have peanuts. d. I never leave coins where my child could get them. ANS: B Latex balloons account for a significant number of deaths from aspiration every year. Keeping toys with small parts and other small objects out of reach can prevent foreign body aspiration. Peanuts are just one of the foods that pose a choking risk if given to young children. Small objects, such as coins, need to be put out of the small childs reach. 24. A nurse is teaching a class to parents on respiratory distress syndrome (RDS). Which statement about RDS indicates the parents understood the teaching? a. Factors causing chronic fetal stress increase the incidence of RDS. b. RDS is the leading cause of respiratory failure in premature infants. c. RDS is caused by an overproduction of surfactant in fetal development. d. The incidence of RDS is significantly lower as gestational age decreases. ANS: B RDS is the leading cause of respiratory failure in premature infants. Things that tend to cause chronic fetal stress, such as maternal hypertension, drug abuse, and prolonged rupture of membranes, decrease the incidence of hyaline membrane disease. RDS occurs in infants with
CURRENT DIAGNOSIS AND TREATMENT PEDIATRICS 24TH LATEST insufficient amounts of surfactant or immature lung development. The incidence of RDS increases as gestational age decreases. 25. What is a common trigger for asthma attacks in children? a. Febrile episodes b. Dehydration c. Exercise d. Seizures ANS: C Exercise is one of the most common triggers for asthma attacks, particularly in school-age children. Febrile episodes are consistent with other problems, for example, seizures. Dehydration occurs as a result of diarrhea; it does not trigger asthma attacks. Viral infections are triggers for asthma. Seizures can result from a too-rapid intravenous infusion of theophyllinea therapy for asthma. 26. Which child would require a Mantoux test? a. The child who has episodes of nighttime wheezing and coughing b. The child who has a history of allergic rhinitis c. The child whose babysitter is diagnosed with tuberculosis d. The premature infant who is being treated for apnea of infancy ANS: C The Mantoux test is the initial screening mechanism for patients exposed to tuberculosis. Nighttime wheezing and coughing are consistent with a diagnosis of asthma. Allergic rhinitis would require an allergy workup. The premature infant who is being treated for apnea of infancy would require a sleep study as part of the evaluation. 27. What explanation should the nurse give to a parent of a child with asthma about using a peak flowmeter?
CURRENT DIAGNOSIS AND TREATMENT PEDIATRICS 24TH LATEST a. It is used to monitor the childs breathing capacity. b. It measures the childs lung volume. c. It will help the medication reach the childs airways. d. It measures the amount of air the child breathes in. ANS: A The peak flowmeter is a device used to monitor breathing capacity in the child with asthma; it measures the flow of air in a forced exhalation in liters per minute. A child with asthma would have a pulmonary function test to measure lung volume. A spacer used with a metered dose inhaler prolongs medication transit so medication reaches the airways. 28. What is the best nursing response to the parent of a child with asthma who asks if his child can still participate in sports? a. Children with asthma are usually restricted from physical activities. b. Children can usually play any type of sport if their asthma is well controlled. c. Avoid swimming because breathing underwater is dangerous for people with asthma. d. Even with good asthma control, I would advise limiting the child to one athletic activity per school year. ANS: B Sports that do not require sustained exertion, such as gymnastics, baseball, and weight lifting, are well tolerated. Children can usually play any type of sport if their asthma is well controlled. Children with asthma should not be restricted from physical activity. Swimming is recommended as the ideal sport for children with asthma because the air is humidified and exhaling underwater prolongs exhalation and increases end-expiratory pressure. 29. Which response indicates that a school-age child understands the interpretation of peak flowmeter results? a. When my peak flow is in the green zone, I need more medication.
CURRENT DIAGNOSIS AND TREATMENT PEDIATRICS 24TH LATEST b. The red zone means my peak flow is 80% to 100%. c. When my peak flow is in the yellow zone, I might need to take more medicine. d. The yellow zone means I need to take albuterol right away. ANS: C The yellow zone indicates caution. The childs peak flow is 50% to 80% of his personal best. A temporary increase in medication may be indicated. The green zone indicates all clear. There are no asthma symptoms present. The childs exhalation is 80% to 100% of his personal best. The red zone is interpreted as a medical alert. The childs exhalation is below 50% of his personal best. The child should use a bronchodilator immediately. The child would take an immediate bronchodilator if his peak flow is in the red zone. 30. A nurse is admitting a client with an asthma exacerbation. Which drug should the nurse be prepared to administer to relieve an acute asthma episode? a. Systemic corticosteroids b. Inhaled corticosteroids c. Leukotriene blockers d. Long-acting bronchodilators ANS: A Systemic corticosteroids decrease airway inflammation in an acute asthma attack. They are given for short burst courses of 5 to 7 days. Inhaled corticosteroids are used for long-term, routine control of asthma. Leukotriene blockers diminish the mediator action of leukotrienes and are used for long-term, routine control of asthma in children older than 12 years. A long-acting bronchodilator would not relieve acute symptoms. 31. The nurse is getting an end-of-shift report on a child with status asthmaticus. Which intervention should the nurse question? a. Administer oxygen at 6 liters by nasal cannula. b. Assess intravenous (IV) maintenance fluids and site every hour.
CURRENT DIAGNOSIS AND TREATMENT PEDIATRICS 24TH LATEST c. Notify the physician for signs of increasing respiratory distress. d. Organize care to allow for uninterrupted rest periods. ANS: A Supplemental oxygen should not exceed 2 L/min and 6 L/min exceeds the recommended flow rate for a nasal cannula. Administration of oxygen to a child with chronic carbon dioxide retention may lead to respiratory depression by decreasing the stimulus to breathe. When the child cannot take oral fluids because of respiratory distress, IV fluids are administered. The child with a continuous IV infusion must be assessed hourly to prevent complications. A physician should be notified of any changes indicating increasing respiratory distress. A child in respiratory distress is easily fatigued. Nursing care should be organized so the child can get needed rest without being disturbed. 32. Which intervention should be included in the plan of care for a child hospitalized for an exacerbation of cystic fibrosis? a. Perform chest physiotherapy 30 minutes after meals. b. Administer low-flow oxygen (less than 2 L/min). c. Position with the head lower than the rest of the body. d. Provide privacy during coughing episodes. ANS: B Low-flow oxygen is administered because in children who are chronically hypoxic, too much oxygen can depress respirations. Chest physiotherapy should be scheduled at least 1 hour before or 2 hours after meals to reduce gastrointestinal upset. The head of the bed should be elevated and the child positioned upright to facilitate breathing. The nurse should stay with the child during coughing episodes. 33. What should the nurse teach a child about using an albuterol metered-dose inhaler for exercise-induced asthma? a. Take two puffs every 6 hours around the clock.
CURRENT DIAGNOSIS AND TREATMENT PEDIATRICS 24TH LATEST b. Use the inhaler only when he is short of breath. c. Use the inhaler 15 minutes before exercise. d. Take one to two puffs every morning on awakening. ANS: C The appropriate time to use an inhaled beta2-agonist or cromolyn is before an event that could trigger an attack. Two puffs every 6 hours around the clock will not relieve exercise-induced asthma. Waiting until symptoms are severe is too late to begin using a metered-dose inhaler. One to two puffs every morning on awakening may be the childs usual schedule for medication. If exercise causes symptoms, additional medication is indicated. 34. A preterm infant is being discharged from the hospital. Which immunization should the nurse prepare to administer to protect the infant from the respiratory syncytial virus (RSV)? a. Pavilizumab (Synagis) b. Ribaviran (Virazole) c. Hemophilus influenza type B (HIB) d. Pneumococcal (Prevnar) ANS: A Intramuscular pavilizumab (Synagis) is the immunization administered monthly throughout the RSV season for premature infants (less than 35 weeks of gestation) younger than 6 months. Ribaviran is an antiviral medication used for treatment of severe cases of RSV. The HIB immunization is given to prevent infections caused by Hemophilus influenza type B. The Prevnar vaccination prevents pneumonia by the pneumococcal virus. 35. Which statement, if made by parents of a child with cystic fibrosis, indicates that they understood the nurses teaching on pancreatic enzyme replacement? a. Enzymes will improve my childs breathing. b. I should give the enzymes 1 hour after meals. c. Enzymes should be given with meals and snacks.
CURRENT DIAGNOSIS AND TREATMENT PEDIATRICS 24TH LATEST d. The enzymes are stopped if my child begins wheezing. ANS: C Children with cystic fibrosis need to take enzymes with all food for adequate absorption of nutrients. Pancreatic enzymes do not affect the respiratory system and are taken within 30 minutes of eating all meals and snacks. Giving the medication 1 hour after meals is inappropriate and ineffective for absorption of nutrients. Wheezing is not a reason to stop taking enzyme replacements. 36. Which vitamin supplements are necessary for children with cystic fibrosis? a. Vitamin C and calcium b. Vitamins B6 and B12 c. Magnesium d. Vitamins A, D, E, and K ANS: D Fat-soluble vitamins are poorly absorbed because of deficient pancreatic enzymes in children with cystic fibrosis; therefore, supplements are necessary. Vitamin C and calcium are not fat soluble. B6 and B12 are not fat-soluble vitamins. Magnesium is not a vitamin. 37. Why do infants and young children develop respiratory distress more quickly in acute and chronic alterations of the respiratory system? a. They have a widened, shorter airway. b. There is a defect in their sucking ability. c. The gag reflex increases mucus production. d. Mucus and edema obstruct small airways. ANS: D The airway in infants and young children is narrower, not wider; respiratory distress can occur quickly because mucus and edema can cause obstruction to their small airways. Sucking is not
CURRENT DIAGNOSIS AND TREATMENT PEDIATRICS 24TH LATEST necessarily related to problems with the airway. The gag reflex is necessary to prevent aspiration. It does not produce mucus.
38. Which statement made by a parent would indicate an understanding about the genetic transmission of cystic fibrosis (CF)? a. Only one parent carries the cystic fibrosis gene. b. Both parents are carriers of the cystic fibrosis gene. c. The presence of the disease is most likely the result of a genetic mutation. d. The mother is usually the carrier of the cystic fibrosis gene. ANS: B Cystic fibrosis follows a pattern of autosomal recessive transmission. Both parents must be carriers of the gene for the disease to be transmitted to the child. If both parents carry the CF gene, each pregnancy has a 25% chance of producing a CF-affected child. Cystic fibrosis will not be present if only one parent is a carrier of the cystic fibrosis gene and is known to have a definite pattern of transmission. It is transmitted as an autosomal recessive trait. A carrier parent can transmit the carrier gene to the child. The disease is present when the carrier gene is transmitted from both parents. 39. A small child with cystic fibrosis cannot swallow pancreatic enzymes capsules. The nurse would teach parents to mix enzymes with which food? a. Macaroni and cheese b. Tapioca c. Applesauce d. Hot chocolate ANS: C
CURRENT DIAGNOSIS AND TREATMENT PEDIATRICS 24TH LATEST Enzymes can be mixed with a small amount of nonprotein foods, such as applesauce. Macaroni and cheese and tapioca are not good choices because enzymes are inactivated by heat and starchy foods. Enzymes are less effective if mixed with foods that are hot, such as hot chocolate. 40. The nurse should teach parents of a child with cystic fibrosis to adjust enzyme dosage according to which indicator? a. Stool formation b. Vomiting c. Weight d. Urine output ANS: A When there is constipation, less enzyme is needed; with steatorrhea, more enzyme is needed for digestion of nutrients. Vomiting does not affect enzyme dosaging. The childs weight does not affect enzyme dosaging. Urine output is not relevant to enzyme replacement. 41. Which finding would confirm a diagnosis of cystic fibrosis? a. A chest radiograph shows alveolar hyperinflation. b. Stool analysis indicates significant amounts of fecal fat. c. Sweat chloride is greater than 60 mEq/L. d. Liver function levels are abnormal. ANS: C The diagnosis of cystic fibrosis requires a positive sweat test. A chloride level greater than 60 mEq/L is considered diagnostic for cystic fibrosis. Hyperinflation is one of the first findings on a chest radiograph of a child with cystic fibrosis. It does not confirm a diagnosis. A 72-hour fecal fat determination may be included in a diagnostic workup. Inability to secrete digestive enzymes causes steatorrhea. Liver function tests may be part of the diagnostic workup for cystic fibrosis. MULTIPLE RESPONSE
CURRENT DIAGNOSIS AND TREATMENT PEDIATRICS 24TH LATEST 1. The nurse should provide what information to the parents of a healthy newborn infant to help prevent sudden infant death syndrome (SIDS)? Select all that apply. a. Place the infant on his back to sleep. b. Use a soft mattress. c. Offer a pacifier for sleep. d. Bed sharing with parents is recommended. ANS: A, C The American Academy of Pediatrics (AAP) recommends the following to help prevent SIDS in infants: place healthy infants on their backs to sleep, use mattresses with a firm sleeping surface, avoid exposing the infant to second-hand smoke, and offer a pacifier for sleep. In addition, bed sharing is not recommended, and parents are advised to put the infant in a safe bassinet or crib in the parents room for sleeping. 2. The nurse should implement which interventions for an infant experiencing apnea? Select all that apply. a. Stimulate the infant by gently tapping the foot. b. Shake the infant vigorously. c. Have resuscitative equipment available. d. Suction the infant. e. Maintain a neutral thermal environment. ANS: A, C, E An infant having apnea should be stimulated by gently tapping the foot. Resuscitative equipment should be available and the infant should be maintained in a neutral thermal environment. The infant should not be shaken vigorously or suctioned. 3. A nurse is planning care for an asymptomatic child with a positive tuberculin test. The nurse should include which in the plan? Select all that apply.
CURRENT DIAGNOSIS AND TREATMENT PEDIATRICS 24TH LATEST a. Administration of daily isoniazid (INH) b. Instructing family members about administration of isoniazid (INH) to all close contacts of the child c. Administration of the Bacillus Calmette-Gurin vaccine d. Reporting the case to the Health Department e. Administration of isoniazid (INH) and rifampin (Rifadin) simultaneously ANS: A, B, D After a chest radiograph is obtained, asymptomatic children with positive tuberculin tests and no previous history of TB receive daily isoniazid (INH) for 9 months. Asymptomatic contacts should receive INH for at least 8 to 10 weeks after contact has been broken or until a negative skin test can be confirmed (a second test is taken at least 10 weeks after the last exposure). Reporting cases of TB is required by law in all states in the United States. The Bacillus Calmette-Gurin vaccine is the only anti-TB vaccine available, but it is given only to children who test negative. For asymptomatic TB only isoniazid is administered, not both isoniazid and rifampin together. Rifampin is used if the child has resistance to isoniazid. SHORT ANSWER 1. Which childhood vaccine has dramatically reduced the incidence of epiglottitis? ANS: H. influenzae type B
Chapter 20. Sleep Medicine MULTIPLE CHOICE 1. The nurse finds that a patient is extremely agitated, yells frequently, and is attempting to get out of bed without assistance. What is the nurses initial action? a. Administer zolpidem after taking the patients vital signs.
CURRENT DIAGNOSIS AND TREATMENT PEDIATRICS 24TH LATEST b. Close the patients door for privacy after administering Tylenol. c. Administer benzodiazepine before calling the health care provider. d. Spend uninterrupted time listening to the patient. ANS: D Assessing the patients level of anxiety is important. The patient may only need someone to listen to what stressors he or she is facing. The nurse must assess the patient before medication can be administered. 2. An older child patient received a hypnotic agent at 9:00 PM. At 2:00 AM, the nurse discovers that the patient has removed her gown and is attempting to get out of bed without assistance. What type of medication effect is the patient exhibiting? a. Allergic b. Hypersensitivity c. Paradoxical d. Therapeutic ANS: C A paradoxical effect may occur in older adult patients. This includes periods of excitement, confusion, restlessness, and euphoria. Allergies to medications tend to manifest in skin or respiratory symptoms. A patient who is hypersensitive to a hypnotic would be difficult to rouse. A therapeutic effect for a hypnotic would be sedation. 3. For what conditions are benzodiazepines prescribed? a. Chronic amnesia b. Chronic insomnia c. Preoperative sedation d. Psychotic episodes ANS: C
CURRENT DIAGNOSIS AND TREATMENT PEDIATRICS 24TH LATEST The sedative hypnotic effect of benzodiazepines facilitates surgical sedation. Short acting benzodiazepines are administered intramuscularly for preoperative sedation. They are also given intravenously for conscious sedation before short diagnostic procedures or for the induction of general anesthesia. Benzodiazepines are not recommended for long term use and do not affect amnesia. Benzodiazepines are a poor choice for the treatment of chronic insomnia because of their risk for habituation. Benzodiazepines do not have an antipsychotic effect. 4. A patient receiving diazepam (Valium) is complaining of nausea and vomiting and is becoming jaundiced. Which type of blood work will be performed? a. Renal function tests b. Liver function tests c. Clotting times d. Electrolyte panels ANS: B Liver function tests will be performed because nausea, vomiting, and jaundice can be indicative of hepatotoxicity. Abnormal liver function test results (i.e., elevated bilirubin, aspartate transaminase [AST], alanine transaminase [ALT], gamma glutamyl transferase [GGT], and alkaline phosphatase levels, as well as prothrombin time) are indicative of hepatotoxicity. Benzodiazepines do not affect the kidneys, clotting times, and electrolytes. 5. In addition to facilitating sleep, what is another benefit of sedatives? a. Increased pain control postoperatively b. Reduced bronchial secretions c. Decreased patient anxiety d. Increased patient alertness ANS: C
CURRENT DIAGNOSIS AND TREATMENT PEDIATRICS 24TH LATEST Preoperatively, sedatives will help decrease patient anxiety and facilitate sleep. Sedatives are not long acting enough to affect postoperative pain control. Sedatives do not affect bronchial secretions. Sedatives diminish patient alertness. 6. Which two phases make up normal sleep? a. Hypnagogic and hypnopompic b. Rapid eye movement (REM) and non REM c. Alpha and beta d. Delta and theta ANS: B Normal sleep can be divided into two phases, REM and non REM. Hypnagogic is the state between waking and sleeping; hypnopompic is the transition between sleeping and waking states. Beta waves are those associated with day to day wakefulness. During periods of relaxation while still awake, our brain waves become slower, increase in amplitude, and become more synchronous. These types of waves are called alpha waves. The first stage of sleep is characterized by theta waves. During a normal nights sleep, a sleeper passes from the theta waves of stages 1 and 2 to the delta waves of stages 3 and 4. Delta waves are the slowest and highest amplitude brain waves. 7. Which sleep pattern stage diminishes as an effect of aging? a. Stage I b. Stage II c. Stage III d. Stage IV ANS: D As we age, stage IV sleep diminishes. Many people older than 75 years do not demonstrate any stage IV sleep patterns. Between 2% and 5% of sleep is stage I. Stage II comprises about 50% of normal sleep time. Stage III is a transition between lighter sleep and deeper sleep.
CURRENT DIAGNOSIS AND TREATMENT PEDIATRICS 24TH LATEST 8. A patient has been prescribed lorazepam (Ativan), a benzodiazepine used to treat insomnia. Which action will the nurse take? a. Advise the patient to take the medication with food. b. Assess the patients blood pressure in sitting and lying positions. c. Inform the patient to discontinue the medication once sleep improves. d. Instruct the patient to lie down before taking the medication. ANS: B Measuring blood pressure in sitting and lying positions is important to assess for transient hypotension. Ativan does not have to be taken with food. Rapid discontinuance of the medication after long term use may result in symptoms similar to those of alcohol withdrawal. Gradual withdrawal of benzodiazepines is over 2 to 4 weeks. Medications should be taken sitting up. 9. Which disease is associated with insufficient sleep? a. Cancer b. Glaucoma c. Myocardial infarction d. Renal failure ANS: C Individuals who sleep less than 5 hours a night have a threefold increased risk of heart attacks. Cancer, glaucoma, and renal failure are not associated with lack of sleep. 10. The nurse is caring for an older patient recently admitted to an assisted living center who is experiencing insomnia associated with the recent relocation. At bedtime, which nursing action will assist the patient to sleep? a. Offering the patient hot tea b. Encouraging the patient to ambulate in the hallway c. Performing back massage
CURRENT DIAGNOSIS AND TREATMENT PEDIATRICS 24TH LATEST d. Administering an analgesic ANS: C Providing a back rub will promote relaxation and reduce anxiety associated with a new environment. This would also provide an opportunity for the nurse to encourage the patient to express feelings. The patient should avoid products containing caffeine, such as coffee, tea, soft drinks, and chocolate. A quiet unwinding activity before bedtime is helpful for sleep promotion. Administering an analgesic is not an appropriate action. 11. The nurse is explaining the use of medications to a patient with insomnia. Which statement about sedatives is true? a. A sedative will produce sleep. b. Sedatives increase the total time in REM sleep. c. Increased relaxation occurs with sedatives. d. Sedatives are more potent than hypnotics. ANS: C A hypnotic is a drug that produces sleep; a sedative quiets the patient and gives a feeling of relaxation and rest, not necessarily accompanied by sleep. Sedatives do not increase the total time in REM sleep. A small dose of a drug may act as a sedative, whereas a larger dose of the same drug may act as a hypnotic and produce sleep. 12. The nurse is assessing a patient who is being evaluated in an outpatient clinic for complaints of back pain. The patient reports taking diphenhydramine for insomnia related to job stress. Which statement by the nurse is accurate regarding this medication? a. This medication should only be taken for 1 week. b. This medication can cause nausea. c. The medication should not be taken after eating a high fat meal. d. This is an herbal medication that has been used for hundreds of years.
CURRENT DIAGNOSIS AND TREATMENT PEDIATRICS 24TH LATEST ANS: A Antihistamines (particularly diphenhydramine and doxylamine) have sedative properties that may be used for short term treatment of mild insomnia. They are common ingredients in over the counter (OTC) sleep aids. Tolerance develops after only a few nights of use; increasing the dose actually causes a more restless and irregular sleep pattern. Diphenhydramine does not cause nausea. There is no restriction on taking diphenhydramine after a high fat meal. Diphenhydramine is not an herbal medication. 13. When reviewing a patients history and physical information, the nurse notes that the patient has physicians orders for chloral hydrate and warfarin. During assessment of this patient, the nurse observes areas of petechiae and ecchymosis on the upper and lower extremities. The most appropriate lab work for the nurse to assess next is: a. liver function studies. b. C-reactive protein. c. sedimentation rate. d. prothrombin time. ANS: D Chloral hydrate may enhance the anticoagulant effects of warfarin. The patient should be observed for petechiae, ecchymoses, nosebleeds, bleeding gums, dark tarry stools, and bright red or coffee ground emesis. Prothrombin time should be monitored, and the physician should reduce the dosage of warfarin, if necessary. 14. The nurse is administering Somnote to a patient. When providing medication education to the patient, the nurse will include that Somnote should be: a. thoroughly chewed. b. taken with a full glass of water. c. taken on an empty stomach. d. taken only before bedtime.
CURRENT DIAGNOSIS AND TREATMENT PEDIATRICS 24TH LATEST ANS: B Somnote is available in capsule form and should not be chewed, should be taken with a full glass of water, should be taken after meals, and is usually ordered to be taken three times a day after meals.
MULTIPLE RESPONSE 15. Why are benzodiazepines often preferred over barbiturates? (Select all that apply.) a. They have selective action at specific receptor sites. b. There is a wide range of safety between therapeutic and lethal levels. c. REM sleep is decreased to a lesser extent. d. Accidental overdoses are well tolerated. e. There are no hypotensive episodes when rising to a sitting position. ANS: A, B, C, D The selectivity of specific drugs at receptor sites accounts for the wide variety of uses. There is a wide safety margin between the therapeutic and lethal dosages for these drugs. Benzodiazepines decrease REM sleep to a lesser extent. Intentional and unintentional overdoses of benzodiazepines are often well tolerated and not fatal. There are transient hypotensive episodes with benzodiazepine therapy. 16. Drug used to induce sleep in children is: a. Melatonin b. Ativan c. omenaprazole
CURRENT DIAGNOSIS AND TREATMENT PEDIATRICS 24TH LATEST d. Xanax
Chapter 21. Cardiovascular Diseases 1. A nurse is conducting a class for nursing students about fetal circulation. Which statement is accurate about fetal circulation and should be included in the teaching session?
a. Oxygen is carried to the fetus by the umbilical arteries. b. Blood from the inferior vena cava is shunted directly to the right ventricle through the foramen ovale. c. Pulmonary vascular resistance is high because the lungs are filled with fluid. d. Blood flows from the ductus arteriosus to the pulmonary artery.
ANS: C
CURRENT DIAGNOSIS AND TREATMENT PEDIATRICS 24TH LATEST Resistance in the pulmonary circulation is very high because the lungs are collapsed and filled with fluid. Oxygen and nutrients are carried to the fetus by the umbilical vein. The inferior vena cava empties blood into the right atrium. The direction of blood flow and the pressure in the right atrium propel most of this blood through the foramen ovale into the left atrium. Most of the blood in the pulmonary artery flows though the ductus arteriosus into the descending aorta.
2. Which postoperative intervention should be questioned for a child after a cardiac catheterization?
a. Continue intravenous (IV) fluids until the infant is tolerating oral fluids. b. Check the dressing for bleeding. c. Assess the peripheral circulation on the affected extremity. d. Keep the affected leg flexed and elevated.
ANS: D The child should be positioned with the affected leg straight for 4 to 6 hours after the procedure. IV fluid administration continues until the child is taking and retaining adequate amounts of oral fluids. The insertion site dressing should be observed frequently for bleeding. The nurse should also look under the child to check for pooled blood. Peripheral perfusion is monitored after catheterization. Distal pulses should be palpable, although they may be weaker than in the contralateral extremity.
3. Which information should be included in the nurses discharge instructions for a child who underwent a cardiac catheterization earlier in the day?
a.
CURRENT DIAGNOSIS AND TREATMENT PEDIATRICS 24TH LATEST The pressure dressing is changed daily for the first week. b. The child may soak in the tub beginning tomorrow. c. Contact sports can be resumed in 2 days. d. The child can return to school on the third day after the procedure.
ANS: D The child can return to school on the third day after the procedure. It is important to emphasize follow-up with the cardiologist. The day after the cardiac catheterization, the pressure dressing is removed and replaced with a Band-Aid. The catheter insertion site is assessed daily for healing. Any bleeding or sign of infection, such as drainage, must be reported to the cardiologist. Bathing is limited to a shower, sponge bath, or a brief tub bath (no soaking) for the first 1 to 3 days after the procedure. Strenuous exercise such as contact sports, swimming, or climbing trees is avoided for up to 1 week after the procedure.
4. A nurse is preparing to assess a 9-month-old infant admitted to the hospital for further evaluation of an atrial septal defect (ASD). Which should the nurse do first for the cardiac assessment?
a. Percussion b. Palpation c. Auscultation d. History and inspection
CURRENT DIAGNOSIS AND TREATMENT PEDIATRICS 24TH LATEST ANS: D The assessment should begin with the least threatening interventionsthe history and inspection. Assessment progression includes inspection, auscultation, and palpation because each step includes more touching. Percussion of the chest is usually deferred. Palpation can be threatening to the child because it requires a significant amount of physical contact. For this reason it is not the initial step in a cardiac assessment. Auscultation requires touching the child and is not the initial step in a cardiac assessment.
5. In which situation is there a risk that a newborn infant will have a congenital heart defect (CHD)?
a. Trisomy 21 detected on amniocentesis b. Family history of myocardial infarction c. Father has type 1 diabetes mellitus d. Older sibling was born with Turners syndrome
ANS: A The incidence of congenital heart disease is approximately 50% in children with trisomy 21 (Down syndrome). A family history of congenital heart disease, not acquired heart disease, increases the risk of giving birth to a child with CHD. Infants born to mothers who are insulin dependent have an increased risk of CHD. Infants identified as having certain genetic defects, such as Turners syndrome, have a higher incidence of CHD. A family history is not a risk factor.
6. Before giving a dose of digoxin (Lanoxin), the nurse checked an infants apical heart rate and it is 114 beats per minute. What should the nurse do next?
CURRENT DIAGNOSIS AND TREATMENT PEDIATRICS 24TH LATEST a. Administer the dose as ordered. b. Hold the medication until the next dose. c. Wait and recheck the apical heart rate in 30 minutes. d. Notify the physician about the infants heart rate.
ANS: A The infants heart rate is above the lower limit for which the medication is held. The dose can be given. It is unnecessary to recheck the heart rate at a later time. A dose of Lanoxin is withheld for a heart rate less than 100 beats per minute in an infant and a physician should be notified.
7. Which intervention should be included in the plan of care for an infant with the nursing diagnosis Fluid volume excess related to congestive heart failure?
a. Weigh the infant every day on the same scale at the same time. b. Notify the physician when weight gain exceeds more than 20 g/day. c. Put the infant in a car seat to minimize movement. d. Administer digoxin (Lanoxin) as ordered by the physician.
ANS: A Excess fluid volume may not be overtly visible. Weight changes may indicate fluid retention. Weighing the infant on the same scale at the same time each day ensures consistency. An excessive weight gain for an infant is an increase of more than 50 g/day. With fluid volume
CURRENT DIAGNOSIS AND TREATMENT PEDIATRICS 24TH LATEST excess, skin will be edematous. The infants position should be changed frequently to prevent undesirable pooling of fluid in certain areas. Lanoxin is used in the treatment of congestive heart failure to improve cardiac function. Diuretics will help the body get rid of excess fluid.
8. The nurse assessing a premature newborn infant auscultates a continuous machinery-like murmur. This finding is associated with which congenital heart defect?
a. Pulmonary stenosis b. Patent ductus arteriosus c. Ventricular septal defect d. Coarctation of the aorta
ANS: B The classic murmur associated with patent ductus arteriosus is a machinery-like one that can be heard throughout both systole and diastole. A systolic ejection murmur that may be accompanied by a palpable thrill is a manifestation of pulmonary stenosis. The characteristic murmur associated with ventricular septal defect is a loud, harsh holosystolic murmur. A systolic murmur that is accompanied by an ejection click may be heard on auscultation when coarctation of the aorta is present.
9. Which is an expected assessment finding in a child with coarctation of the aorta?
a. Orthostatic hypotension b.
CURRENT DIAGNOSIS AND TREATMENT PEDIATRICS 24TH LATEST Systolic hypertension in the lower extremities c. Blood pressure higher on the left side of the body d. Disparity in blood pressure between the upper and lower extremities
ANS: D The classic finding in children with coarctation of the aorta is a disparity in pulses and blood pressures between the upper and lower extremities. Orthostatic hypotension is not present with coarctation of the aorta. Systolic hypertension may be detected in the upper extremities. The left arm may not accurately reflect systolic hypertension because the left subclavian artery can be involved in the coarctation.
10. A nurse is assessing an infant with chronic hypoxia due to tetralogy of Fallot. Which finding does the nurse expect to assess?
a. Polycythemia b. Pulmonary hypotension c. Dehydration d. Anemia
ANS: A The body attempts to improve tissue oxygenation by producing additional red blood cells and thereby increases the oxygen-carrying capacity of the blood. Pulmonary hypertension is a clinical consequence of cyanosis. Dehydration can occur rapidly in cyanotic heart disease. It is
CURRENT DIAGNOSIS AND TREATMENT PEDIATRICS 24TH LATEST not a compensatory mechanism to chronic hypoxia. Anemia may develop as a result of increased blood viscosity.
11. Which statement made by a parent indicates understanding of activity restrictions for a child after cardiac surgery?
a. My child needs to get extra rest for a few weeks. b. My son is really looking forward to riding his bike next week. c. Im so glad we can attend services as a family this coming Sunday. d. I am going to keep my child out of day care for 6 weeks.
ANS: D Settings in which large groups of people are present should be avoided for 4 to 6 weeks after discharge, including day care. The child should resume his regular bedtime and sleep schedule after discharge. Activities during which the child could fall, such as riding a bicycle, are avoided for 4 to 6 weeks after discharge. Large crowds of people should be avoided for 4 to 6 weeks after discharge, including public worship.
12. A child had a ventricular septal defect surgically repaired 3 months ago. Which antibiotic prophylaxis is indicated for an upcoming dental procedure?
a. No antibiotic prophylaxis is necessary. b. Amoxicillin is taken orally 1 hour before the procedure.
CURRENT DIAGNOSIS AND TREATMENT PEDIATRICS 24TH LATEST c. Oral penicillin is given for 7 to 10 days before the procedure. d. Parenteral antibiotics are administered for 4 to 8 weeks after the procedure.
ANS: B The standard prophylactic agent is amoxicillin given orally 1 hour before the procedure. Antibiotic prophylaxis is indicated for the first 6 months after surgical repair. Antibiotic prophylaxis is not given for this period of time. The treatment for infective endocarditis involves parenteral antibiotics for 4 to 8 weeks.
13. A nurse is assessing a 7-day-old infant. The nurse detects a soft murmur. The nurse notifies the primary care physician because the nurse is aware that fetal shunts are closed in the infant at what point in time?
a. When the umbilical cord is cut b. Within several days of birth c. Within a month after birth d. By the end of the first year of life
ANS: B In the normal neonate, fetal shunts functionally close in response to pressure changes in the systemic and pulmonary circulations and to increased oxygen content. This process may take several days to complete. With the neonates first breath, gas exchange is transferred from the placenta to the lungs. The separation of the fetus from the umbilical cord does not contribute to the establishment of neonatal circulation.
CURRENT DIAGNOSIS AND TREATMENT PEDIATRICS 24TH LATEST
14. When assessing a child for possible congenital heart defects, where should the nurse measure blood pressure?
a. The right arm b. The left arm c. All four extremities d. Both arms while the child is crying
ANS: C When a congenital heart defect is suspected, the blood pressure should be measured in all four extremities while the child is quiet. Discrepancies between upper and lower extremities may indicate cardiac disease. Blood pressure measurements when the child is crying are likely to be elevated; thus, the readings will be inaccurate. Blood pressure measurements for upper and lower extremities are compared during an assessment for congenital heart defects.
15. What should be the nurses first action when planning to teach the parents of an infant with a congenital heart defect?
a. Assess the parents readiness to learn. b. Gather literature for the parents. c.
CURRENT DIAGNOSIS AND TREATMENT PEDIATRICS 24TH LATEST Secure a quiet place for teaching. d. Discuss the plan with the nursing team.
ANS: A Any effort to organize the right environment, plan, or literature is of no use if the parents are not ready to learn. A baseline assessment of prior knowledge should be taken into consideration before developing any teaching plan. Locating a quiet place for meeting with parents is appropriate; however, an assessment should be done before any teaching is done. Discussing a teaching plan with the nursing team is appropriate after an assessment of the parents knowledge and readiness.
16. A nurse is explaining a patent ductus arteriosus defect to the parents of a preterm infant. The parents indicate understanding of the defect when they state that a patent ductus arteriosus:
a. involves a defect that results in a right-to-left shunting of blood in the heart. b. involves a defect in which the fetal shunt between the aorta and the pulmonary artery fails to close. c. is a stenotic lesion that must be surgically corrected at birth. d. causes an abnormal opening between the four chambers of the heart.
ANS: B Patent ductus arteriosus is failure of the fetal shunt between the aorta and the pulmonary artery to close. A patent ductus arteriosus allows blood to flow from the high-pressure aorta to the lowpressure pulmonary artery, resulting in a left-to-right shunt. Patent ductus arteriosus is not a stenotic lesion. Patent ductus arteriosus can be closed both medically and surgically.
CURRENT DIAGNOSIS AND TREATMENT PEDIATRICS 24TH LATEST Atrioventricular defect occurs when fetal development of the endocardial cushions is disturbed, resulting in abnormalities in the atrial and ventricular septa and the atrioventricular valves.
17. Why might a newborn infant with a cardiac defect, such as coarctation of the aorta resulting in a right-to-left shunt, receive prostaglandin E1? a. To decrease inflammation b. To control pain c. To decrease respirations d. To keep the ductus arteriosus patent
ANS: D Prostaglandin E1 is given to infants with a right-to-left shunt to keep the ductus arteriosus patent to increase pulmonary blood flow.
18. Which congenital heart defect results in increased pulmonary blood flow?
a. Ventricular septal defect b. Coarctation of the aorta c. Tetralogy of Fallot d. Pulmonary stenosis
CURRENT DIAGNOSIS AND TREATMENT PEDIATRICS 24TH LATEST
ANS: A Ventricular septal defect causes a left-to-right shunting of blood, thus increasing pulmonary blood flow. Coarctation of the aorta is a stenotic lesion that causes increased resistance to blood flow from the proximal to distal aorta. The defects associated with tetralogy of Fallot result in a right-to-left shunting of blood, thus decreasing pulmonary blood flow. Pulmonary stenosis causes obstruction of blood flow from the right ventricle to the pulmonary artery. Pulmonary blood flow is decreased.
19. Which statement suggests that a parent understands how to correctly administer digoxin?
a. I measure the amount I am supposed to give with a measuring spoon. b. I put the medicine in the babys bottle. c. When she spits up right after I give the medicine, I give her another dose. d. I give the medicine at 8 in the morning and evening every day.
ANS: D For maximum effectiveness, the medication should be given at the same time every day and should be measured with a syringe. The medication should not be mixed with formula or food. It is difficult to judge whether the child received the proper dose if the medication is placed in food or formula. To prevent toxicity, the parent should not repeat the dose without contacting the childs physician.
20. What nursing action is appropriate to take when an infant with a congenital heart defect has an increased respiratory rate and sweating and is not feeding well?
CURRENT DIAGNOSIS AND TREATMENT PEDIATRICS 24TH LATEST a. Check the infants temperature. b. Alert the physician. c. Withhold oral feeding. d. Increase the oxygen rate.
ANS: B An increased respiratory rate, sweating, and not feeling well are signs of early congestive heart failure and the physician should be notified; they do not suggest a febrile process. Withholding the infants feeding is an incomplete response to the problem. Increasing oxygen may alleviate symptoms, but medications such as digoxin and furosemide are necessary to improve heart function and fluid retention.
21. Nursing care for the child in congestive heart failure includes which action?
a. Counting the number of saturated diapers b. Putting the infant in the Trendelenburg position c. Removing oxygen while the infant is crying d. Organizing care to provide rest periods
ANS: D
CURRENT DIAGNOSIS AND TREATMENT PEDIATRICS 24TH LATEST Nursing care should be planned to allow for periods of undisturbed rest. Diapers must be weighed for an accurate record of output. The head of the bed should be raised to decrease the work of breathing. Oxygen should be administered during stressful periods such as when the child is crying. 22. Which strategy is appropriate when feeding the infant with congestive heart failure?
a. Continue the feeding until a sufficient amount of formula is taken. b. Limit feedings to no more than 30 minutes. c. Always bottle feed every 4 hours. d. Feed larger volumes of concentrated formula less frequently.
ANS: B The infant with congestive heart failure may tire easily so the feeding should not continue beyond 30 minutes. If inadequate amounts of formula are taken, gavage feedings should be considered. Infants with congestive heart failure may be breast-fed or fed a smaller volume of concentrated formula. Feedings every 3 hours is a frequently used interval. If the infant were fed less frequently than every 3 hours, more formula would need to be consumed and would tire the infant.
23. A nurse is teaching an adolescent about primary hypertension. Which statement made by the adolescent indicates an understanding about primary hypertension?
a. Primary hypertension should be treated with diuretics as soon as it is detected. b.
CURRENT DIAGNOSIS AND TREATMENT PEDIATRICS 24TH LATEST Congenital heart defects are the most common cause of primary hypertension. c. Primary hypertension may be treated with weight reduction. d. Primary hypertension is not affected by exercise.
ANS: C Primary hypertension in children may be treated with weight reduction and exercise programs. If ineffective, pharmacological intervention may be needed. Primary hypertension is considered to be an inherited disorder.
24. A nurse is planning care for a child with secondary hypertension. The nurse plans to include which initial treatment of secondary hypertension?
a. Weight control and diet b. Treating the underlying disease c. Administration of digoxin d. Administration of beta-adrenergic receptor blockers
ANS: B Identification of the underlying disease should be the first step in treating secondary hypertension. Weight control and diet are a nonpharmacological treatment for primary hypertension. Digoxin is indicated in the treatment of congestive heart failure. Beta-adrenergic receptor blockers are indicated in the treatment of primary hypertension.
CURRENT DIAGNOSIS AND TREATMENT PEDIATRICS 24TH LATEST 25. Which should the nurse include in discharge teaching for the child with a cardiac arrhythmia?
a. CPR instructions b. Repeating digoxin if the child vomits c. Resting if dizziness occurs d. Checking the childs pulse after digoxin administration
ANS: A The parents and significant others in the childs life should have CPR training. The digoxin dose is not repeated if the child vomits. Dizziness is a symptom the child should be taught to report to adults so the physician can be notified. The childs pulse should be counted before the medication is given. The dose is withheld if the pulse is below the parameters set by the physician.
26. A nurse is assigned to care for an infant with an unrepaired tetralogy of Fallot. What should the nurse do first when the baby is crying and becomes severely cyanotic? a. Place the infant in a knee-chest position. b. Administer oxygen. c. Administer morphine sulfate. d. Notify the physician.
ANS: A
CURRENT DIAGNOSIS AND TREATMENT PEDIATRICS 24TH LATEST Placing the infant in a knee-chest position will decrease venous return so that smaller amounts of highly saturated blood reach the heart. Administering oxygen is indicated after placing the infant in a knee-chest position. Administering morphine sulfate calms the infant. It is indicated after the infant has been placed in a knee-chest position. The physician should be notified after the infant has been placed in a knee-chest position. 27. The nurse caring for a child with a diagnosis of rheumatic fever should assess the child for which finding?
a. Sore throat b. Elevated blood pressure c. Desquamation of the fingers and toes d. Tender, warm, inflamed joints
ANS: D Arthritis, characterized by tender, warm, erythematous joints, is one of the major manifestations of rheumatic fever. The child may have had a sore throat previously associated with a group A beta-hemolytic streptococcal infection a few weeks earlier. A sore throat is not a manifestation of rheumatic fever. Hypertension is not associated with rheumatic fever. Desquamation of the fingers and toes is a manifestation of Kawasaki syndrome. 28. A nurse is caring for a child admitted to the hospital with Kawasaki disease. Which cardiac complication of Kawasaki disease should the nurse monitor for?
a. Cardiac valvular disease b. Cardiomyopathy
CURRENT DIAGNOSIS AND TREATMENT PEDIATRICS 24TH LATEST c. Coronary aneurysm d. Rheumatic fever
ANS: C Coronary aneurysm formation begins early in the second phase of Kawasaki syndrome. Coronary artery aneurysms are seen in 20% of children with untreated Kawasaki disease. Cardiac valvular disease can occur in rheumatic fever. Cardiomyopathies are diseases of the heart muscle, which can occur as a result of congenital heart disease, coronary artery disease, or other systemic disease. Rheumatic fever is not a complication of Kawasaki syndrome. MULTIPLE RESPONSE 1. Which congenital heart disease causes cyanosis when not repaired? Select all that apply.
a. Patent ductus arteriosus (PDA) b. Tetralogy of Fallot c. Pulmonary atresia d. Transposition of the great arteries
ANS: B, C, D Tetralogy of Fallot is a cyanotic lesion with decreased pulmonary blood flow. The hypoxia results in baseline oxygen saturations as low as 75% to 85%. Even with oxygen administration, saturations do not reach the normal range. Pulmonary atresia is a cyanotic lesion with decreased pulmonary blood flow. The hypoxia results in baseline oxygen saturations as low as 75% to 85%. Even with oxygen administration, saturations do not reach the normal range. Transposition of the great arteries is a cyanotic lesion with increased pulmonary blood flow. PDA is failure of the fetal shunt between the aorta and the pulmonary artery to close. PDA is not classified as a
CURRENT DIAGNOSIS AND TREATMENT PEDIATRICS 24TH LATEST cyanotic heart disease. Prostaglandin E1 is often given to maintain ductal patency in children with cyanotic heart diseases. 2. A child has a total cholesterol level of 180 mg/dL. What dietary recommendations should the nurse make to the child and the childs parents? Select all that apply.
a. Replace whole milk for 2% or 1% milk b. Increase servings of red meat c. Increase servings of fish d. Avoid excessive intake of fruit juices e. Limit servings of whole grain
ANS: A, C, D A low-fat diet includes using nonfat or low-fat dairy products, limiting red meat intake, and increasing intake of fish, vegetables, whole grains, and legumes. Children should avoid excessive intake of fruit juices and other sweetened drinks, sugars, and saturated fats.
3. A nurse is conducting discharge teaching to parents about the care of their infant after cardiac surgery. The nurse instructs the parents to notify the physician if which occurs? Select all that apply.
a. Respiratory rate of 36 at rest b.
CURRENT DIAGNOSIS AND TREATMENT PEDIATRICS 24TH LATEST Appetite slowly increasing c. Temperature above 37.7 C (100 F). d. New, frequent coughing e. Turning blue or bluer than normal
ANS: C, D, E The parents should be instructed to notify the physician after their infants cardiac surgery for a temperature above 37.7 C; new, frequent coughing; and any episodes of the infant turning blue or bluer than normal. A respiratory rate of 36 at rest for an infant is within normal expectations and it is expected that the appetite will increase slowly.
Chapter 22. Gastrointestinal Tract 1. Which is the best nursing response to a mother asking about the cause of her infants bilateral cleft lip? a. Did you have trouble with this pregnancy? b. Do you know of anyone in your or the fathers family born with cleft lip or palate problems? c. This defect is associated with intrauterine infection during the second trimester. d. Was your husband in the military and involved in chemical warfare? ANS: B Cleft lip and palate result from embryonic failure resulting from multiple genetic and environmental factors. A genetic pattern or familial risk seems to exist. A troublesome pregnancy has not been associated with bilateral cleft lip. The defect occurred at approximately 6
CURRENT DIAGNOSIS AND TREATMENT PEDIATRICS 24TH LATEST to 8 weeks of gestation. Second-trimester intrauterine infection is not a known cause of bilateral cleft lip. Chemical warfare is not significantly associated with bilateral cleft lip and palate. 2. Which nursing intervention is most helpful to parents of a neonate with bilateral cleft lip? a. Assure the parents that the correction will be immediate and uncomplicated. b. Show the parents before-and-after pictures of an infant whose cleft lip has been successfully repaired. c. Teach the parents about long-term enteral feedings. d. Refer the parents to a community agency that addresses this problem. ANS: B Showing the parents pictures of successful lip repair promotes bonding and enhances coping ability. Correction is usually done around 4 weeks but may be done as early as 2 to 3 days after birth. The infant with a bilateral cleft lip can be fed orally using a compressible, longer nipple, and by making a larger hole in the nipple. Long-term enteral feedings are not usually indicated. A community agency referral is not appropriate at this time and may not be indicated long term. 3. The postoperative care plan for an infant with surgical repair of a cleft lip includes which intervention? a. A clear liquid diet for 72 hours b. Nasogastric feedings until the sutures are removed c. Elbow restraints to keep the infants fingers away from the mouth d. Rinsing the mouth after every feeding ANS: C Keeping the infants hands away from the incision reduces potential complications at the surgical site. The infants diet is advanced from clear liquid to soft foods within 48 hours of surgery. After surgery, the infant can resume preoperative feeding techniques. Rinsing the mouth after feeding is an inappropriate intervention. Feeding a small amount of water after feedings will help keep
CURRENT DIAGNOSIS AND TREATMENT PEDIATRICS 24TH LATEST the mouth clean. A cleft lip repair site should be cleansed with a wet sterile cotton swab after feedings. 4. A nurse is teaching a group of parents about tracheoesophageal fistula. Which statement, made by the nurse, is accurate about tracheoesophageal fistula (TEF)? a. This defect results from an embryonal failure of the foregut to differentiate into the trachea and esophagus. b. It is a fistula between the esophagus and stomach that results in the oral intake being refluxed and aspirated. c. An extra connection between the esophagus and trachea develops because of genetic abnormalities. d. The defect occurs in the second trimester of pregnancy. ANS: A When the foregut does not differentiate into the trachea and esophagus during the fourth to fifth week of gestation, a TEF occurs. TEF is an abnormal connection between the esophagus and trachea. There is no connection between the trachea and esophagus in normal fetal development. Tracheoesophageal fistula occurs early in pregnancy during the fourth to fifth week of gestation. 5. Which maternal assessment is related to the infants diagnosis of TEF? a. Maternal age more than 40 years b. First term pregnancy for the mother c. Maternal history of polyhydramnios d. Complicated pregnancy ANS: C A maternal history of polyhydramnios is associated with TEF. Advanced maternal age is not a risk factor for TEF. The first term pregnancy is not a risk factor for an infant with TEF. Complicated pregnancy is not a risk factor for TEF.
CURRENT DIAGNOSIS AND TREATMENT PEDIATRICS 24TH LATEST 6. What clinical manifestation should a nurse should be alert for when a diagnosis of esophageal atresia is suspected? a. A radiograph in the prenatal period indicates abnormal development. b. It is visually identified at the time of delivery. c. A nasogastric tube fails to pass at birth. d. The infant has a low birth weight. ANS: C Atresia is suspected when a nasogastric tube fails to pass 10 to 11 centimeters beyond the gum line. Abdominal radiographs will confirm the diagnosis. Prenatal radiographs do not provide a definitive diagnosis. The defect is not externally visible. Bronchoscopy and endoscopy can be used to identify this defect. Infants with esophageal atresia may have been born prematurely and with a low birth weight, but neither is suggestive of the presence of an esophageal atresia. 7. The nurse admits an infant with vomiting and the diagnosis of hypertrophic pyloric stenosis. Which metabolic alteration should the nurse plan to assess for with this infant? a. Metabolic alkalosis b. Metabolic acidosis c. Respiratory acidosis d. Respiratory alkalosis ANS: A Frequent projectile vomiting, characteristic of pyloric stenosis, results in a loss of nonvolatile acids that decreases hydrogen ion concentration. This results in an excess of bicarbonate that increases arterial pH above 7.45 (metabolic alkalosis). Metabolic acidosis, respiratory acidosis, and respiratory alkalosis do not result from vomiting. 8. What is the most important information to be included in the discharge planning for an infant with gastroesophageal reflux?
CURRENT DIAGNOSIS AND TREATMENT PEDIATRICS 24TH LATEST a. Teach the parents to position the infant on the left side. b. Reinforce the parents knowledge of the infants developmental needs. c. Teach the parents how to do infant cardiopulmonary resuscitation (CPR). d. Have the parents keep an accurate record of intake and output. ANS: C Risk of aspiration is a priority nursing diagnosis for the infant with gastroesophageal reflux. The parents must be taught infant CPR. Correct positioning minimizes aspiration. The correct position for the infant is on the right side after feeding and supine for sleeping. Knowledge of developmental needs should be included in discharge planning for all hospitalized infants but is not the most important in this case. Keeping a record of intake and output is not a priority and may not be necessary. 9. Which information does the nurse include when teaching the parents of a 5-week-old infant about pyloromyotomy? a. The infant will be in the hospital for a week. b. The surgical procedure is routine and no big deal. c. The prognosis for complete correction with surgery is good. d. They will need to ask the physician about home care nursing. ANS: C Pyloromyotomy is the definitive treatment for pyloric stenosis. Prognosis is good with few complications. These comments reassure parents. The infant will remain in the hospital for a day or two postoperatively. Although the prognosis for surgical correction is good, telling the parents that surgery is no big deal minimizes the infants condition. Home care nursing is not necessary after pyloromyotomy. 10. A nurse has admitted a child to the hospital with a diagnosis of rule out peptic ulcer disease. Which test will the nurse expect to be ordered to confirm the diagnosis of a peptic ulcer? a. A 24-hour dietary history
CURRENT DIAGNOSIS AND TREATMENT PEDIATRICS 24TH LATEST b. A positive Hematest result on a stool sample c. A fiberoptic upper endoscopy d. An abdominal ultrasound ANS: C Endoscopy provides direct visualization of the stomach lining and confirms the diagnosis of peptic ulcer. Dietary history may yield information suggestive of a peptic ulcer, but the diagnosis is confirmed through endoscopy. Blood in the stool indicates a gastrointestinal abnormality, but it does not conclusively confirm a diagnosis of peptic ulcer. An abdominal ultrasound is used to rule out other gastrointestinal alterations such as gallstones, tumor, or mechanical obstruction. 11. What should the nurse teach a school-age child and his parents about the management of ulcer disease? a. Eat a bland, low-fiber diet in small frequent meals. b. Eat three balanced meals a day with no snacking between meals. c. The child needs to eat alone to avoid stress. d. Do not give antacids 1 hour before or after antiulcer medications. ANS: D Antacids can interfere with antiulcer medication if given less than 1 hour before or after antiulcer medications. A bland diet is not indicated for ulcer disease. The diet should be a regular diet that is low in caffeine, and the child should eat a meal or snack every 2 to 3 hours. Eating alone is not indicated. 12. Which prescribed formula should the nurse plan to provide for an infant with lactose intolerance? a. Isomil b. Enfamil c. Similac d. Good Start
CURRENT DIAGNOSIS AND TREATMENT PEDIATRICS 24TH LATEST ANS: A The treatment for lactose intolerance is removal of lactose from the diet. Formulas that do not contain lactose (Isomil, Nursoy, Nutramigen, Prosobee, and other soy-based formulas) may be given to the infant suspected of having lactose intolerance. Enfamil, Similac, and Good Start are all milk-based formulas. 13. Which dietary foods high in calcium should the nurse encourage a lactose intolerant child to eat? a. Yogurt b. Green leafy vegetables c. Cheese d. Rice ANS: B The child between 1 and 10 years requires a minimum of 800 milligrams of calcium daily. Because high-calcium dairy products containing lactose are restricted from the childs diet, alternate sources such as egg yolk, green leafy vegetables, dried beans, and cauliflower must be provided to prevent hypocalcemia. Yogurt and cheese contain lactose. Rice is not high in calcium. 14. Which food choice by a parent of a 2-year-old child with celiac disease indicates a need for further teaching? a. Oatmeal b. Rice cake c. Corn muffin d. Meat patty ANS: A
CURRENT DIAGNOSIS AND TREATMENT PEDIATRICS 24TH LATEST The child with celiac disease is unable to fully digest gluten, the protein found in wheat, barley, rye, and oats. Oatmeal contains gluten and is not an appropriate food selection. Rice is an appropriate choice because it does not contain gluten. Corn is digestible because it does not contain gluten. Meats do not contain gluten and can be included in the diet of a child with celiac disease. 15. Which assessment finding should the nurse expect in an infant with Hirschsprungs disease? a. Currant jelly stools b. Constipation with passage of foul-smelling, ribbon-like stools c. Foul-smelling, fatty stools d. Diarrhea ANS: B Constipation results from the absence of ganglion cells in the rectum and colon and is present since the neonatal period with passage of frequent foul-smelling, ribbon-like, or pellet-like stools. Currant jelly stools are associated with intussusception. Foul-smelling, fatty stools are associated with cystic fibrosis and celiac disease. Diarrhea is not typically associated with Hirschsprungs disease but may result from impaction. 16. Which would be an expected outcome for the parents of a child with encopresis? a. The parents will give the child an enema daily for 34 months. b. The parents will develop a plan to achieve control over incontinence. c. The parents will have the child launder soiled clothes. d. The parents will supply the child with a low-fiber diet. ANS: B Parents of the child with encopresis often feel guilty and believe that encopresis is willful on the part of the child. The family functions effectively by openly discussing problems and developing a plan to achieve control over incontinence. Stool softeners or laxatives, along with dietary changes, are typically used to treat encopresis. Enemas are indicated when a fecal impaction is
CURRENT DIAGNOSIS AND TREATMENT PEDIATRICS 24TH LATEST present. Having the child launder soiled clothes is a punishment and will increase the childs shame and embarrassment. The child should not be punished for an action that is not willful. Increasing fiber in the diet and fluid intake results in greater bulk in the stool, making it easier to pass. 17. Which intervention should be included in the nurses plan of care for a 7-year-old child with encopresis who has cleared the initial impaction? a. Have the child sit on the toilet 30 minutes when he gets up in the morning and at bedtime. b. Increase sugar in the childs diet to promote bowel elimination. c. Use a Fleet enema daily. d. Give the child a choice of beverage to mix with a laxative. ANS: D Offering realistic choices is helpful in meeting the school-age childs sense of control. To facilitate bowel elimination, the child should sit on the toilet for 5 to 10 minutes after breakfast and dinner. Decreasing the amount of sugar in the diet will help keep stools soft. Daily Fleet enemas can result in hypernatremia and hyperphosphatemia and are used only during periods of fecal impaction. 18. A nurse is assisting a child with inflammatory bowel disease to choose items from the dietary menu. Which dietary item should be avoided because it is high in residue? a. Eggs b. Cheese c. Grapes d. Jello ANS: C
CURRENT DIAGNOSIS AND TREATMENT PEDIATRICS 24TH LATEST Fruits with skins or seeds should be avoided because they are high in residue. Cooked or canned fruits and vegetables without skins are allowed. Eggs, cheese, and Jello would be allowed on a low residue diet. 19. What is an expected outcome for the child with irritable bowel disease? a. Decreasing symptoms b. Adherence to a low-fiber diet c. Increasing milk products in the diet d. Adapting the lifestyle to the lifelong problems ANS: A Management of irritable bowel disease is aimed at identifying and decreasing exposure to triggers and decreasing bowel spasms, which will decrease symptoms. Management includes maintenance of a healthy, well-balanced, moderate-fiber, lower-fat diet. A moderate amount of fiber in the diet is indicated for the child with irritable bowel disease. No modification in dairy products is necessary unless the child is lactose intolerant. Irritable bowel syndrome is typically self-limiting and resolves by age 20 years. 20. An infant has been admitted to the Neonatal Intensive Care Unit (NICU) with a congenital gastroschisis. Which intervention should the nurse perform first upon admission to the unit? a. Place the infant flat and prone. b. Cover the defect with sterile warm, moist gauze and wrap with plastic. c. Begin a gestational age assessment. d. Wrap the infant in a warm blanket and allow the father to hold the infant briefly. ANS: B Gastroschisis is the protrusion of intraabdominal contents through a defect in the abdominal wall lateral to the umbilical ring. There is no peritoneal sac. The defect should be immediately wrapped in warm, moist, sterile gauze and covered with plastic to keep moist. The infant cannot be placed prone as more damage could occur to the defect. Movement of the infant should be
CURRENT DIAGNOSIS AND TREATMENT PEDIATRICS 24TH LATEST minimized so gestational age assessment and parental holding would be done after the infant is stabilized. 21. What is an appropriate statement for the nurse to make to parents of a child who has had a barium enema to correct an intussusception? a. I will call the physician when the baby passes his first stool. b. I am going to dilate the anal sphincter with a gloved finger to help the baby pass the barium. c. I would like you to save all the soiled diapers so I can inspect them. d. Add cereal to the babys formula to help him pass the barium. ANS: C The nurse needs to inspect diapers after a barium enema because it is important to document the passage of barium and note the characteristics of the stool. The physician does not need to be notified when the infant passes the first stool. Dilating the anal sphincter is not appropriate for the child after a barium enema. After reduction, the infant is given clear liquids and the diet is gradually increased. 22. Which is the best response for the nurse to make to parents who ask why their infant has a nasogastric tube to intermittent suction after abdominal surgery? a. The nasogastric tube decompresses the abdomen and decreases vomiting. b. We can keep a more accurate measure of intake and output with the nasogastric tube. c. The tube is used to decrease postoperative diarrhea. d. Believe it or not, the nasogastric tube makes the baby more comfortable after surgery. ANS: A The nasogastric tube provides decompression and decreases vomiting. A nursing responsibility when a patient has a nasogastric tube is measurement of accurate intake and output, but this is
CURRENT DIAGNOSIS AND TREATMENT PEDIATRICS 24TH LATEST not why nasogastric tubes are inserted. Nasogastric tube placement does not decrease diarrhea. The presence of a nasogastric tube can be perceived as a discomfort by the patient. 23. Which stool characteristic should the nurse expect to assess with a child diagnosed with intussusception? a. Ribbon-like stools b. Hard stools positive for guaiac c. Currant jelly stools d. Loose, foul-smelling stools ANS: C Pressure on the bowel from obstruction leads to passage of currant jelly stools. Ribbon-like stools are characteristic of Hirschsprungs disease. With intussusception, passage of bloody mucus stools occurs. Stools will not be hard. Loose, foul-smelling stools may indicate infectious gastroenteritis. 24. Which is a priority concern for a 14-year-old child with inflammatory bowel disease? a. Compliance with antidiarrheal medication therapy b. Long-term complications c. Dealing with the embarrassment and stress of diarrhea d. Home schooling ANS: C Embarrassment and stress from chronic diarrhea are real concerns for the adolescent with inflammatory bowel disease. Antidiarrheal medications are not typically ordered for a child with inflammatory bowel disease. Long-term complications are not a priority concern for the adolescent with inflammatory bowel disease. Exacerbations may interfere with school attendance, but home schooling is not a usual consideration for the adolescent with inflammatory bowel disease.
CURRENT DIAGNOSIS AND TREATMENT PEDIATRICS 24TH LATEST 25. A nurse is conducting a teaching session to adolescents about Crohns disease. Which statement, made by the nurse, is the most accurate? a. Crohns disease is responsive to dietary modifications. b. Crohns disease can occur anywhere in the gastrointestinal tract. c. Edema usually accompanies this disease. d. Symptoms of Crohns disease usually disappear by late adolescence. ANS: B Crohns disease can occur anywhere in the GI tract from the mouth to the anus and is most common in the terminal ileum. Maintaining a low-fiber, low-residue, and milk-free diet may give the child some relief; however, strict restrictions may not alleviate symptoms. Diarrhea and malabsorption from Crohns disease cause weight loss, anorexia, dehydration, and growth failure. Edema does not accompany this disease. Crohns disease is a long-term health problem. Symptoms do not typically disappear by adolescence. 26. A child is admitted to the pediatric floor for appendicitis. Which assessment finding will the nurse monitor that indicates the appendix has ruptured? a. Abdominal pain shifts from the left to the right side. b. Vomiting and diarrhea become more intense. c. Elevated temperature decreases to normal. d. Abdominal pain is relieved. ANS: D Abdominal pain is relieved when appendix rupture occurs. Pain in the right lower quadrant is suggestive of appendicitis. Abdominal pain does not shift from one side to the other. The child with appendicitis may have vomiting and diarrhea. A rupture does not intensify symptoms. Because peritonitis is associated with a ruptured appendix, the temperature would be elevated in the presence of infection.
CURRENT DIAGNOSIS AND TREATMENT PEDIATRICS 24TH LATEST 27. What is the most important action to prevent the spread of gastroenteritis in a daycare setting? a. Administering prophylactic medications to children and staff b. Frequent hand washing c. Having parents bring food from home d. Directing the staff to wear gloves at all times ANS: B Hand washing is the most the important measure to prevent the spread of infectious diarrhea. Prophylactic medications are not helpful in preventing gastroenteritis. Bringing food from home will not prevent the spread of infectious diarrhea. Gloves should be worn when changing diapers, soiled clothing, or linens. They do not need to be worn for interactions that do not involve contact with secretions. 28. What is an expected outcome for a 1-month-old infant with biliary atresia? a. Correction of the defect with the Kasai procedure b. Adequate nutrition and age-appropriate growth and development c. Increased blood pressure and adherence to a salt-free diet d. Adequate protein intake ANS: B Adequate nutrition, preventing skin breakdown, adequate growth and development, and family education and support are expected outcomes in an infant with biliary atresia. The goal of the Kasai procedure is to allow for adequate growth until a transplant can be done. It is not a curative procedure. Although blood pressure typically is elevated, a modified salt diet is appropriate. Protein intake may need to be restricted to avoid hepatic encephalopathy. 29. Which assessment findings would be significant for a child with cirrhosis? a. Weight loss
CURRENT DIAGNOSIS AND TREATMENT PEDIATRICS 24TH LATEST b. Change in level of consciousness c. Soft, smooth skin d. Pallor and cyanosis ANS: B The child with cirrhosis must be assessed for encephalopathy, which is characterized by a change in level of consciousness. Encephalopathy can result from a buildup of ammonia in the blood from the incomplete breakdown of protein. One complication of cirrhosis is ascites. The child needs to be assessed for increasing abdominal girth and edema. A child who is retaining fluid will not exhibit weight loss. Biliary obstruction can lead to intense pruritus. The skin will be irritated from frequent scratching. A skin assessment would likely reveal jaundice. Pallor and cyanosis are associated with a cardiac problem. 30. Which nursing diagnosis has the highest priority for the child with celiac disease? a. Pain related to chronic constipation b. Altered growth and development related to obesity c. Fluid volume excess related to celiac crisis d. Imbalanced nutrition: Less than body requirements related to malabsorption ANS: D Imbalanced nutrition: Less than body requirements related to malabsorption is the highest priority nursing diagnosis because celiac disease causes gluten enteropathy, a malabsorption condition. The pain associated with celiac disease is associated with diarrhea, not constipation. Celiac disease causes altered growth and development associated with malnutrition, not obesity. Celiac crisis causes fluid volume deficit. 31. The nurse notes on assessment that a 1-year-old child is underweight, with abdominal distention, thin legs and arms, and foul-smelling stools. The nurse suspects failure to thrive associated with which condition? a. Celiac disease
CURRENT DIAGNOSIS AND TREATMENT PEDIATRICS 24TH LATEST b. Intussusception c. Irritable bowel syndrome d. Imperforate anus ANS: A These are classic symptoms of celiac disease. Intussusception is not associated with failure to thrive or underweight, thin legs and arms, and foul-smelling stools. Stools are like currant jelly. Irritable bowel syndrome is characterized by diarrhea and pain, and the child does not typically have thin legs and arms. Imperforate anus is the incomplete development or absence of the anus in its normal position in the perineum. Symptoms are evident in early infancy. 32. A 10-year-old boy is admitted to the hospital with a diagnosis of appendicitis. He is nauseated, febrile, and complaining of severe abdominal pain radiating to the right lower quadrant. During a routine nursing check, he states that his stomach doesnt hurt anymore. The nurse should suspect that: a. he is anxious about surgery. b. his appendix has ruptured. c. he does not communicate effectively about pain. d. his nausea and vomiting have decreased, thereby relieving his abdominal pain. ANS: B A classic symptom indicating appendix rupture is the sudden relief of pain. The boy may be anxious, but this will not cause his pain to disappear. There is no evidence to substantiate the assumption that he does not communicate effectively about pain. His nausea and vomiting have not decreased, nor will this affect his abdominal pain. 33. The nurse caring for a child with suspected appendicitis should question which physician prescriptions? a. Keep patient NPO. b. Start IV of D5/0.45 normal saline at 60 mL/hr.
CURRENT DIAGNOSIS AND TREATMENT PEDIATRICS 24TH LATEST c. Apply K-pad to abdomen prn for pain. d. Obtain CBC on admission to the nursing unit. ANS: C A K-pad (moist heat device) is contraindicated for suspected appendicitis because it may contribute to the rupture of the appendix. NPO status is appropriate for the potential appendectomy client. An IV is appropriate both as a preoperative intervention and to compensate for the short-term NPO status. Because appendicitis is frequently reflected in an elevated WBC, laboratory data are needed. 34. Which order should the nurse question when caring for a child after surgery for Hirschsprungs disease? a. Monitor rectal temperature every 4 hours and report an elevation greater than 38.5 C. b. Assess stools after surgery. c. Keep the child NPO until bowel sounds return. d. Maintain IV fluids at an ordered rate. ANS: A Rectal temperatures should not be taken after this surgery. Rectal temperatures are generally not the route of choice for children because of the routes traumatic nature. Assessing stools after surgery is an appropriate intervention postoperatively. Stools should be soft and formed. Keeping the child NPO until bowel sounds return is an appropriate intervention postoperatively. Maintaining IV fluids at an ordered rate is an appropriate postoperative order. 35. Which diagnosis has the highest priority for the child with irritable bowel syndrome? a. Alteration in nutrition: Less than body requirements related to malabsorption b. Altered growth and development related to inadequate nutrition c. Pain related to hyperperistalsis d. Constipation related to maldigestion
CURRENT DIAGNOSIS AND TREATMENT PEDIATRICS 24TH LATEST ANS: C Diffuse abdominal pain unrelated to activity or meals is a common clinical manifestation of irritable bowel syndrome. Normal physical growth and development usually occur with this disorder. Constipation may occur with irritable bowel syndrome, usually alternating with diarrhea. 36. A 7-year-old child is admitted to the hospital with severe abdominal pain, bloody currant jelly diarrhea, and fever. What is his probable diagnosis? a. Hirschsprungs disease b. Celiac disease c. Ruptured appendix d. Intussusception ANS: D Severe abdominal pain, bloody currant jelly diarrhea, and fever are common clinical manifestations of intussusception. Hirschsprungs disease usually manifests as bowel obstruction. Severe abdominal pain, bloody currant jelly diarrhea, and fever are not common symptoms of celiac disease. Although a child with a ruptured appendix will probably be febrile, the other symptoms are not indicative of a ruptured appendix. 37. Which goal has the highest priority for a child with malabsorption associated with lactose intolerance? a. The child will experience no abdominal spasms. b. The child will not experience constipation associated with malabsorption syndrome. c. The child will not experience diarrhea associated with malabsorption syndrome. d. The child will receive adequate nutrition as evidenced by a weight gain of 1 kg/day. ANS: C
CURRENT DIAGNOSIS AND TREATMENT PEDIATRICS 24TH LATEST The highest priority goal is that the child will not experience diarrhea associated with malabsorption syndrome; this goal is correct for a child with malabsorption associated with lactose intolerance. A child usually has abdominal cramping, pain, and distention rather than spasms. The child usually has diarrhea, not constipation. One kilogram a day is too much weight gain with no time parameters. 38. What would be an appropriate meal for a school-age child with celiac disease? a. Baked chicken and cornbread b. Hot dog and bun c. Bean with barley soup and rice cakes d. Cheeseburger on rye bread ANS: A Children with celiac disease must eliminate all wheat, rye, barley, oats, and hydrolyzed vegetable proteins from their diet. Cornbread does not contain glutens. Most buns, barley, and rye bread contain glutens. 39. What should the nurse stress in a teaching plan for the mother of an 11-year-old boy with ulcerative colitis? a. Preventing the spread of illness to others b. Nutritional guidance and preventing constipation c. Teaching daily use of enemas d. Coping with stress and adjusting to a chronic illness ANS: D Coping with the stress of a chronic illness and the clinical manifestations associated with ulcerative colitis (diarrhea, pain) are important teaching foci. Ulcerative colitis is not infectious. Although nutritional guidance is a priority teaching focus, diarrhea is a problem with ulcerative colitis, not constipation. Teaching daily use of enemas is not part of the therapeutic plan of care.
CURRENT DIAGNOSIS AND TREATMENT PEDIATRICS 24TH LATEST 40. An infant with Hirschsprungs disease has a temporary colostomy. Which statement by the infants mother indicates she understands how to care for the infants colostomy at home? a. I need to be careful to check the skin around the colostomy for breakdown and be sure I keep it clean. b. Ill call my home health nurse if the colostomy bag needs to be changed. c. Ill call the doctor if I notice that the colostomy stoma is pink. d. Ill have my mother help me with the care of the colostomy. ANS: A Preventing skin breakdown is a priority concern when caring for a colostomy. The mother should be taught the basics of colostomy care, including how to change the appliance. The colostomy stoma should be pink in color, not pale or discolored. There is no evidence that her mother knows how to care for a colostomy. 41. Careful hand washing before and after contact can prevent the spread of
in day care
and school settings. a. irritable bowel syndrome b. ulcerative colitis c. hepatic cirrhosis d. hepatitis A ANS: D Hepatitis A is spread person to person, by the fecal-oral route and through contaminated food or water. Good hand washing is critical in preventing its spread. The virus can survive on contaminated objects for weeks. Irritable bowel syndrome is the result of increased intestinal motility and is not contagious. Ulcerative colitis and cirrhosis are not infectious. MULTIPLE RESPONSE 1. Which interventions should a nurse implement when caring for a child with hepatitis? Select all that apply.
CURRENT DIAGNOSIS AND TREATMENT PEDIATRICS 24TH LATEST a. Provide a well-balanced low-fat diet. b. Schedule play time in the playroom with other children. c. Teach parents not to administer any over-the-counter medications. d. Arrange for home schooling as the child will not be able to return to school. e. Instruct parents on the importance of good hand washing. ANS: A, C, E The child with hepatitis should be placed on a well-balanced low-fat diet. Parents should be taught to not give over-the-counter medications because of impaired liver function. Hand hygiene is the most important preventive measure for the spread of hepatitis. The child will be in contact isolation in the hospital so play time with other hospitalized children is not scheduled. The child will be on contact isolation for at least 1 week after the onset of jaundice, but after that period, will be allowed to return to school. 2. The nurse is providing home care instructions to the parents of an infant being discharged after repair of a bilateral cleft lip. Which instructions should the nurse include? Select all that apply. a. Acetaminophen (Tylenol) should not be given to your infant. b. Feed your infant in an upright position. c. Place your infant prone for a period of time each day. d. Burp your child frequently during feedings. e. Apply antibiotic ointment to the lip as prescribed. ANS: B, D, E After cleft lip surgery the parents are taught to feed the infant in an upright position to decrease the chance of choking. The parents are taught to burp the infant frequently during feedings because excess air is often swallowed. Parents are taught to cleanse the suture line area with a cotton swab using a rolling motion and apply antibiotic ointment with the same technique. Tylenol is used for pain and the child should never be placed prone as that can damage the suture line.
CURRENT DIAGNOSIS AND TREATMENT PEDIATRICS 24TH LATEST
Chapter 23. Liver and Pancreas 1. 1. What is being assessed when auscultating the gastrointestinal system? 1. 1. Changes in the abdominal appearance 2. 2. Presence or absence of bowel sounds 3. 3. Distension as well as spleen and liver size 4. 4. Presence of a hernia ANS: 2 Feedback 1.A visual inspection is done or this. 2.Auscultation allows the examiner to assess the bowel sounds and assess for any changes that may occur in the bowel sounds due to a GI problem. 3.Palpation assesses the distension and size. 4.Presence of a hernia is detected with palpation and a visual inspection. 2. Which of the following symptoms may be found in Celiac Disease? 1. 1. Abdominal pain with bloating 2. 2. Weight gain with very skinny extremities 3. 3. Small, hard stools 4. 4. Normal growth ANS: 1 Feedback 1. The most common complaint of celiac patients is abdominal bloating
that is usually painful. 2. Patients with this disorder usually appear skinny with thin extremities.
3.Patients with this disorder tend to have diarrhea and foul smelling stools. 4. This can affect growth because of villi damage due to gluten intolerance.
CURRENT DIAGNOSIS AND TREATMENT PEDIATRICS 24TH LATEST 1. 3. When teaching the family about a gluten-free diet, what are the recommendations for the family about diet? 1. 1. Many gluten-free products are available, so it is important to read labels. 2. 2. Participation in a support group may help with identifying stores that carry gluten-free food. 3. 3. Communion wafers contain gluten and may need to be avoided. 4. 4. All of the above. ANS: 4 Feedback 1. Reading labels will help identify food that can cause difficulty, as well as make the family aware of foods that the child will not tolerate. 2. Support groups are excellent resources for information about gluten-free foods and stores in the area that carry these products. 3. Speaking with the clergy may be important due to the gluten in communion wafers. 4. All are important factors for teaching about a gluten-free diet. 4. Appendicitis may have abdominal pain as a symptom. Where does the abdominal pain occur? 1. 1. Left upper quadrant 2. 2. Right lower quadrant 3. 3. Periumbilical 4. 4. 2 and 3 only ANS: 4 Feedback 1.Pain usually is not in the upper left quadrant. 2.Advances to the right lower quadrant 3.Begins in the periumbilical area 4. Pain usually is not in the upper left quadrant. It begins in the periumbilical area. 5. The most commonly used measure to diagnose obesity is:
1. 1. Body mass index. 2. 2. Ultrasound.
CURRENT DIAGNOSIS AND TREATMENT PEDIATRICS 24TH LATEST 3. 3. Weight measurement. 4. 4. Cholesterol measurement. ANS: 4 Feedback 1. Body mass index is the most commonly used measurement for
diagnosing obesity. The CDC web site allows for the computation of body mass index for children and adults. 2. Ultrasounds do not assess obesity. 3. Weight measurements may be part of the diagnosis, but are not as definitive as the body mass index. 4. Cholesterol measurements may be part of the diagnosis, but are not as definitive as the body mass index. 1. 6. Gallstones are occurring more often in children. Which of the following is not a treatment for children with gallstones? 1. 1. Surgery is always the treatment for gallstones. 2. 2. Infants do not need treatment. 3. 3. Crohns patients may have an ERCP, only without surgery. 4. 4. Laser lithotripsy may be an effective option. ANS: 1 Feedback 1.Surgery is not the only treatment for gallstones. 2.Infants usually resolve gallstones on their own. 3. Children with Crohns disease need a functioning gallbladder, and an ERCP may help them retain the gallbladder while removing the stones. 4. Laser lithotripsy has been found to be an effective treatment and is being used in children as an alternative treatment. 1. 7. The first symptom in an infant of the gastrointestinal manifestation of cystic fibrosis is: 1. 1. Constipation. 2. 2. Meconium ileus. 3. 3. Rapid weight gain. 4. 4. Inability to breastfeed.
CURRENT DIAGNOSIS AND TREATMENT PEDIATRICS 24TH LATEST ANS: 2 Feedback 1. Newborns with cystic fibrosis fail to pass meconium in the first 48 hours. 2. Newborns with cystic fibrosis fail to pass meconium in the first 48 hours and develop meconium ileus. That may be followed by diarrhea and poor weight gain. 3. Rapid weight loss may occur due to diarrhea and poor weight gain. 4.Mothers of children with cystic fibrosis are encouraged to breastfeed because children with cystic fibrosis can breastfeed. 1. 8. Treatment for a child with cystic fibrosis with gastrointestinal symptoms may include: 1. 1. Pancreatic enzymes. 2. 2. Fat-soluble vitamins. 3. 3. No immunizations. 4. 4. 1 and 2. ANS: 4 Feedback 1.A child with cystic fibrosis will require replacement pancreatic enzymes. 2.A child with cystic fibrosis will require fat-soluble vitamins for proper growth and nutrition. 3. Regular immunizations are recommended. 4.More than one answer applies. 1. 9. Symptoms of Biliary Atresia would include which of the following? 1. 1. Prolonged jaundice (appearing for longer than two weeks) 2. 2. Elevated direct bilirubin (greater than 20 percent of the total bilirubin measurement) 3. 3. Very dark stools 4. 4. 1 and 2 ANS: 4 Feedback 1. Biliary Atresia causes prolonged jaundice, lasting well past two weeks.
CURRENT DIAGNOSIS AND TREATMENT PEDIATRICS 24TH LATEST 2. The direct bilirubin remains elevated and is always greater than 20% of
the total bilirubin measurement. 3. Stools become light and urine becomes dark. 4. Biliary Atresia causes a prolonged jaundice, lasting well past two
weeks. The direct bilirubin remains elevated and is always greater than 20% of the total bilirubin measurement. 1. 10. The main complication of neonatal jaundice is: 1. 1. Lack of voiding and stooling. 2. 2. Bilirubin encephalopathy. 3. 3. The need to immediately stop breastfeeding. 4. 4. Increased risk of infection. ANS: 2 Feedback 1. Jaundice does not affect voiding and stooling, but lack of voiding and
stooling may increase the jaundice level. 2. Bilirubin encephalopathy can cause hypotonia, opisthotonic posturing,
and brain damage at high levels of jaundice, which make early intervention important. 3. Moms may continue to breastfeed if the baby is jaundiced, but may sometimes need to supplement if intake or output is poor. 4. Babies with an infection may have a higher jaundice level, but jaundice does not cause the infection. 1. 11. Which of the following statements about gastroschisis and omphalocele is true? 1. 1. Only omphalocele has a malrotation. 2. 2. Omphalocele contain stomach and intestines within a sac of amnion and is associated with anomalies. 3. 3. Gastroschisis opens to the left of the umbilical cord, contains the liver, and is associated with anomalies. 4. 4. 1 and 2 ANS: 2 Feedback 1. Both defects are associated with malrotation.
CURRENT DIAGNOSIS AND TREATMENT PEDIATRICS 24TH LATEST 2. Omphalocele may have the stomach and intestines, as well as other GI
organs, and is covered by a sac that may rupture in utero. Both defects are associated with malrotation. 3. Gastroschisis arises to the right of the umbilical cord and has the stomach and intestines, but rarely the liver. It is also rarely associated with other anomalies. 4. Both defects are associated with malrotation. Omphalocele may have the stomach and intestines, as well as other GI organs, and is covered by a sac that may rupture in utero. 1. 12. When caring for a newborn with an abdominal wall defect at birth, it is important to do all of the following except: 1. 1. Wrap and support the defect to prevent rupture. 2. 2. Start immediate feedings to support nutrition. 3. 3. Insert an orogastric tube to decompress the stomach. 4. 4. Provide parenteral nutrition. ANS: 2 Feedback 1.Keeping the defect moist and supported helps to prevent a rupture. 2.Feeding will distend the abdomen and could cause damage. 3. An orogastric tube will help keep the stomach decompressed and help prevent a rupture. 4. Parenteral nutrition is needed for caloric intake and growth.
1. 13. What serologic tests in Hepatitis B would indicate the presence of a chronic infection? 1. 1. Hepatitis B surface antigen (HBsAg) 2. 2. Anti-HBe 3. 3. Anti-HBc subtotal 4. 4. HB surface antibody (anti-HBs Ag) ANS: 1 Feedback
CURRENT DIAGNOSIS AND TREATMENT PEDIATRICS 24TH LATEST 1. Hepatitis B surface antigen with AntiHBc total indicates a chronic
infection. 2. HB surface antibody indicates recovery from an infection. 3. Anti-HBe indicates decreasing infectivity, and recovery from infection
gives a total, not a subtotal. 4. HB surface antibody indicates recovery from an infection.
1. 14. Gastroesophageal reflux should be treated when a child or infant exhibits all of the following except: 1. 1. When a baby spits up frequently. 2. 2. If a baby develops a chronic cough or pneumonia. 3. 3. When a baby has excessive irritability after meals. 4. 4. When a child has a persistent sore throat without illness. ANS: 1 Feedback 1.All babies spit up, but can have reflux that needs treatment. 2.Infants or children with respiratory symptoms should be treated. 3.Infants or children with excessive irritability should be treated. 4. Infants or children with persistent sore throat should be treated, as this makes eating difficult and will affect nutrition. 1. 15. What are symptoms that may indicate an inguinal hernia? 1. 1. Right lower quadrant pain with rebound tenderness 2. 2. A feeling of weakness or pressure in the groin 3. 3. The lack of a hydrocele in a newborn 4. 4. Pain after internal rotation of a flexed thigh ANS: 2 Feedback 1. Right lower quadrant pain with rebound and pain with internal rotation
are indicative of an appendicitis. 2. Weakness and pressure in the groin area is common. 3. An infant with hydroceles should be examined for possible inguinal
hernias. 4. Pain with an internal rotation of flexed thigh is not a sign of a hernia.
CURRENT DIAGNOSIS AND TREATMENT PEDIATRICS 24TH LATEST 1. 16. What information should be taught to parents of children with peptic ulcers? 1. 1. Stress reduction techniques, such as relaxation 2. 2. Minimal consumption of food that may aggravate condition 3. 3. Child may stop medication when symptoms are gone. 4. 4. The child may use Motrin for headaches. ANS: 1 Feedback 1. Children should use appropriate relaxation techniques to reduce stress.
This may include visualization and hypnosis. 2. Children should learn the foods that may aggravate the condition and avoid foods that can cause aggravation. 3. The course of medication for an ulcer should be completed and not stopped prematurely because of symptom abatement. 4. NSAIDS should not be used by children with ulcers as it aggravates the condition. 1. 17. What does an Irritable Bowel Syndrome (IBS) patient need to know to help reduce his/her symptoms? 1. 1. Keep a food diary to identify triggers for symptoms 2. 2. Decrease the amount of fiber in his/her diet 3. 3. Fewer larger meals may reduce incidence of symptoms 4. 4. Avoid supplements, such as fiber supplements or probiotics ANS: 1 Feedback 1. IBS patients should keep a food diary to identify the foods that aggravate their condition. 2. IBS patients should also increase the amount of fiber in their diet. 3.IBS patients should eat more frequent, smaller meals. 4. Fiber and probiotic supplements are encouraged. 1. 18. Crohns disease may present with which of the following symptoms? 1. 1. Right lower quadrant pain 2. 2. Joint pain
CURRENT DIAGNOSIS AND TREATMENT PEDIATRICS 24TH LATEST 3. 3. Increased growth 4. 4. Skin lesions ANS: 1 Feedback 1. Crohns disease may mimic appendicitis with right lower quadrant pain.
It is manifested by diarrhea, usually bloody, not constipation. 2. Joint pain is a symptom seen in ulcerative colitis, not Crohns disease. 3.Growth is slow due to malabsorption of nutrients 4.Skin lesions are a symptom usually seen in ulcerative colitis. 19. What is the recommended treatment for gastroenteritis? 1. 1. Child should drink to thirst to replace fluids. 2. 2. Child should take Imodium and Pepto-Bismol to stop diarrhea. 3. 3. Mothers should stop breastfeeding until the illness is gone. 4. 4. The child should have a bland diet for 24 hours after the vomiting stops, then return to a regular diet. ANS: 4 Feedback 1.The child should only have small amounts of fluid, as large amounts may cause more vomiting. 2.Pepto-Bismol and Imodium should not be used for gastroenteritis in children. 3.Breastfeeding can continue even with gastroenteritis. 4.After vomiting stops and the child is tolerating fluids, then a bland diet is recommended with a return to regular diet in another 24 hours. 20. Treatment for constipation may be difficult. What is a strategy that may be included in the treatment of constipation? 1. Use of a reward system for successful toileting. 2. The child uses the toilet whenever he/she feels the urge. 3. Have the parents make a mental note about when the child uses the toilet.
CURRENT DIAGNOSIS AND TREATMENT PEDIATRICS 24TH LATEST 4. None of the above are correct. ANS: 1 Feedback 1.The child should have a record to keep track of taking his/her medication and his/her success with having a stool. 2.A child needs a regular schedule to sit on the toilet. He/she needs a regular time to sit on the toilet, usually twice a day, not just when the urge hits him/her. 3. The child should have a consistent reward system for successful toileting. He/she needs a regular time to sit on the toilet, usually twice a day, not just when the urge hits him/her. 4. One answer is correct. 1. 21. Hirschsprungs Disease may present with which of the following symptoms? 1. 1. No meconium passage in the first 24 hours 2. 2. Persistent diarrhea 3. 3. Excellent weight gain with little stooling 4. 4. Hard, pellet-like stools ANS: 1 Feedback 1.Newborns with Hirschsprungs Disease may not pass meconium until well after the first 24 hours. The stools are thin and ribbon-like, and the child suffers from constipation. The child usually has poor weight gain. 2.The stools are thin and ribbon-like and the child suffers from constipation. 3.The child usually has poor weight gain. 4.The stools are thin and ribbon-like and the child suffers from constipation. 1. 22. What are recommended interventions for a patient with Fatty Liver Disease? 1. 1. Rapid weight loss 2. 2. Metformin administration 3. 3. Group-based therapy 4. 4. 2 and 3
CURRENT DIAGNOSIS AND TREATMENT PEDIATRICS 24TH LATEST ANS: 4 Feedback 1.Slow, measured weight loss is better than a rapid dropping of weight. 2.Metformin is an effective therapy for this condition. 3. Group-based therapy with the adolescent age group for weight loss as support during this process is beneficial and effective. 4. Group-based therapy with the adolescent age group for weight loss as support during this process is beneficial and effective. Slow, measured weight loss is better than a rapid dropping of weight. Metformin is an effective therapy for this condition. 1. 23. Intestinal obstruction can be caused by an Intussusception or a Volvulus. What are the important points to know about these conditions? 1. Intussusception and Volvulus are both immediately treated with surgery. 2. Intussusception cannot be reduced with a barium enema. 3. Both conditions present with continuous pain. 4. Both Intussusception and Volvulus may first be present with bilious vomiting. ANS: 4 Feedback 1. Intussusception may be reduced with a barium enema or air
insufflations. 2. Intussusception may be reduced with a barium enema or air insufflation, saving the baby from a surgical intervention. 3. Because of Malrotation, the conditions result in cramping pains. 4.Volvulus is always treated with surgery. Both conditions may have bilious vomiting as the presenting symptom. The pain in Volvulus is continuous; it may be intermittent with increasing frequency in an Intussusception. 24. A neonate has been taken to the nursery because of a cyanotic event while feeding due to choking. As the baby lies on the warmer bed, he/she is noted to have froth around the mouth. The priority action by the nurse would be to: 1. Assess vital signs.
CURRENT DIAGNOSIS AND TREATMENT PEDIATRICS 24TH LATEST 2. Use suction to remove the secretions. 3. Hold the neonate in an upright position. 4. Feed the neonate. ANS: 2 Feedback 1. Vital signs will need to be taken prior to informing the doctor, but is not the priority at this time. 2. Suctioning the secretions to maintain an open airway is the priority at this time. 3. Holding the neonate upright will force secretions down because of gravity and increase the risk for aspiration. 4. Feeding the neonate could increase the risk for aspiration. The child needs an evaluation before feedings are continued. 25. An infant is brought to the emergency department with the following clinical manifestations: poor skin turgor, weight loss, lethargy, tachycardia, and tachypnea. This is suggestive of which of the following? 1. Water excess 2. Sodium excess 3. Water depletion 4. Potassium excess ANS: 3 Feedback 1. Water excess would cause crackles in the lungs and a doughy skin turgor with weight gain. 2. Sodium excess would cause the child to have sunken eyes and gain weight. 3. Water depletion will cause these signs and symptoms. 4.Potassium excess will cause irregular heart rhythms and nausea. 26. Clinical manifestations of sodium excess (hypernatremia) include which of the following?
CURRENT DIAGNOSIS AND TREATMENT PEDIATRICS 24TH LATEST 1. Hyperreflexia 2. Abdominal cramps 3. Cardiac dysrhythmias 4. Dry, sticky mucous membranes ANS: 4 Feedback 1.Hyperreflexia occurs with high magnesium levels. 2.Abdominal cramping may occur with high potassium levels. 3.Cardiac dysrhythmias will occur with hypercalcemia. 4.High sodium concentrations cause water loss and create dry, sticky mucous membranes. 27. A 7 year old with acute diarrhea and mild dehydration is being given oral rehydration. The childs mother calls the clinic nurse because he is also occasionally vomiting. The nurse should recommend which of the following? 1. Bring the child to the hospital as soon as possible for IV fluids 2. Alternate between oral rehydration and soda pop 3. Continue giving the oral rehydration in small amounts 4. Do not allow the child to drink anything for the next eight hours, then restart fluids if the vomiting has stopped. ANS: 3 Feedback 1. Because this is acute and only occasional vomiting, the child can be taken care of at home with close monitoring. 2. Soda has sodium and can contribute to the electrolyte imbalance. 3.Small amounts of fluids will decrease the urge to vomit because it is not filling the entire stomach, and the hydration will help the electrolytes find balance. 4.Waiting eight hours can further the dehydration and cause more issues with vomiting because the electrolyte imbalance is not being addressed.
CURRENT DIAGNOSIS AND TREATMENT PEDIATRICS 24TH LATEST 28. A newborn has been diagnosed with tracheoesophageal fistula. The nurse should ask for clarification of the doctors orders if the chart states to: 1. Place an IV for fluids and antibiotics. 2. Use an NG tube placed into the proximal pouch. 3. Take aspiration precautions. 4. Place the newborn in the prone position with his/her head elevated. ANS: 4 Feedback 1.IV fluids will be needed to maintain electrolyte balance because the newborn will not be receiving oral feedings. 2. The NG will help drain any secretions in the pouch. 3. The newborn is at high risk for aspiration because of the lack of connection to the stomach. 4. The newborn should be in a supine position to prevent aspiration and allow for the secretions to drain. 29. Comfort care for a neonate with a new diagnosis of tracheoesophageal fistula would consist of all of the following except: 1. Holding the neonate. 2. Swaddling the neonate. 3. Nonnutritive sucking on a pacifier. 4. Placing sucrose on a pacifier for the baby to have nonnutritive sucking. ANS: 4 Feedback 1. Neonates are comforted when being held. 2. Swaddling helps a neonate find his/her boundaries and allows for him/her to feel comfort. 3. The sucking is a normal reaction and soothes the newborn. 4. The neonate should remain NPO to prevent chances for aspiration.
CURRENT DIAGNOSIS AND TREATMENT PEDIATRICS 24TH LATEST 30. A family has just received the diagnosis of Celiac Disease for their 5-year-old son. The nurse knows that the family understands the teaching when the father states: 1. We will need to read labels, and anything with white flour should be avoided. 2. My child will not be able to participate in hockey anymore because of the diagnosis. 3. The family will need to change its eating out habits and only cook at home. 4. We will need to notify the school so that other children will not catch the disease. ANS: 1 Feedback 1.White flour contains gluten, which is the allergy issue for Celiac Disease. 2.A child can participate in any sports with this disease. 3.The family will need to modify what is ordered when eating out. 4.Celiac Disease is not contagious. 31. An appendectomy has been performed on an 8-year-old boy. When the child arrives to the pediatric unit, the top nursing priority will be: 1. Measuring the childs urine output. 2. Making the parents comfortable in the room. 3. Starting the child on a liquid diet. 4. Monitoring for pain. ANS: 4 Feedback 1. Kidney function will be assessed after the patient has adequate pain
management. 2. The parents will need to be comfortable in the room, but this is not the top priority for the boy. 3. The child will remain NPO for a few hours after the surgery. 4. Pain management will be the top priority because of the surgical site.
CURRENT DIAGNOSIS AND TREATMENT PEDIATRICS 24TH LATEST 32. A child that is exhibiting signs of appendicitis will have: 1. Pain after internal rotation of a flexed thigh. 2. Rebound pain on the left quadrant. 3. A high fever. 4. Acute pain episodes for 3 to 4 days in a row. ANS: 1 Feedback 1.Rotation of the leg may cause pain in a child with appendicitis. 2.Rebound pain happens in the left quadrant. 3.A high fever is usually not noted in appendicitis until rupture occurs. 4.Acute pain is short in duration. 33. A 3-month-old infant has had surgery to repair an inguinal hernia. The nurse is giving discharge instructions for care of the surgical site. Instructions should include: 1. Cleansing the surgical site by placing the infant in a bathtub twice a day. 2. After each diaper change, check to make sure the area is clean and dry. 3. Apply a dressing to the area to keep it clean. 4. Remove the stitches in one week. ANS: 2 Feedback 1. Placing the child in the bathtub may saturate the site and increase the risk for infection. 2. Keeping the site clean and dry will aid in healing without infection. 3.A dressing may keep the area moist and not facilitate healing. 4.The stitches may need to be removed by a health-care provider. 34. When palpating an inguinal hernia, a nurse would be concerned when feeling:
CURRENT DIAGNOSIS AND TREATMENT PEDIATRICS 24TH LATEST 1. Crepitus. 2. Normal skin. 3. An edematous area surrounding the hernia. 4. A lump with erythema and edema. ANS: 4 Feedback 1.Crepitus is not noted when palpating an inguinal hernia. 2.Normal skin will not be noted with an inguinal hernia. 3.The area will have edema along with erythema. 4.Erythema and edema would be cause for concern with an inguinal hernia. It may indicate strangulation of tissue or loss of circulation. 35. A nurse working on a pediatric floor has three patients. Which patient should be seen first? 1. A four hour old post-op appendectomy with IV fluids and an antibiotic due 2. The child with abdominal pain and rectal bleeding with IV fluids. 3. An infant that had hernia repair surgery yesterday. 4. A family needing teaching about diet restrictions related to Celiac Disease ANS: 2 Feedback 1.The patient is beyond the crucial two hour time frame after surgery. 2.This patient should be seen first because of the active bleeding. 3. The hernia repair assessment can be done at any time because the patient is not exhibiting risk factors. 4. Teaching can be done at various times throughout the childs inpatient stay and is not a top priority at this time. 36. A pediatric patient with ulcerative colitis is receiving long-term corticosteroid therapy. A side effect of this type of therapy can include all of the following except: 1. A higher than average heart rate.
CURRENT DIAGNOSIS AND TREATMENT PEDIATRICS 24TH LATEST 2. Mood swings. 3. Easy bruising. 4. A moon face. ANS: 1 Feedback 1. The heart rate is not affected by the long-term use. 2. Mood swings are apparent in children with long-term use. 3. Easily bruising occurs with long-term use. 4. The moon face is a side effect of long-term corticosteroid therapy. 37. A common sign of an infant having a peptic ulcer is: 1. Abdominal distension. 2. Frequent stools. 3. Anorexia. 4. Gastritis. ANS: 1 Feedback 1.This is a common sign of peptic ulcer disease. 2.This is seen in Crohns disease. 3. The infant will want to eat and can digest foods, so they do not have a wasted appearance. 4. The infant will not have inflammation of the GI tract. 38. The mother of a teen with Irritable Bowel Syndrome is asking what types of food should be part of his diet. Identify a food that would be appropriate for the teen. 1. Wheat Chex cereal 2. Hamburger
CURRENT DIAGNOSIS AND TREATMENT PEDIATRICS 24TH LATEST 3. Spinach 4. Peaches ANS: 1 Feedback 1.High fiber should be part of the diet to help with bowel movements. 2.A hamburger does not provide the needed fiber content for bowel movements. The protein can be irritating to the stomach. 3.The spinach does not have the high fiber content needed. 4.The peaches do not have the high fiber content needed. 39. When assessing an emesis of an infant, it is important to note which of the following? 1. Curdled milk 2. Amount 3. The timing of the emesis 4. All of the above should be documented. ANS: 4 Feedback 1. Curdled milk is an indication of the digestion process. 2. The amount can indicate the adoption of the previous intake. 3. The timing may indicate what the infant is doing to cause the emesis. 4.Curdled milk, amount, and timing can give indications as to how to treat the infants condition. 40. A 6 month old is exhibiting signs of gastroesophageal reflux. A nursing intervention to aid in decreasing pain would be: 1. Elevating the head of the bed 30 degrees. 2. Providing large amounts of formula every three hours. 3. Thinning formula so it decreases occurrences.
CURRENT DIAGNOSIS AND TREATMENT PEDIATRICS 24TH LATEST 4. Keep the baby held upright for an hour after feedings. ANS: 1 Feedback 1. Elevating the head of the bed will help take pressure off of the
diaphragm. 2. Small amounts of food decrease the occurrence of gastroesophageal
reflux. 3. Thinning formula decreases the calories needed and does not decrease
the occurrence of gastroesophageal reflux. 4. Keeping a baby upright for an hour is not a realistic expectation.
41. A new baby has been diagnosed with gastroesophageal reflux. As the nurse is feeding the baby, she notes that the baby is twisting and arching. The nurse knows that the baby is exhibiting: 1. Torsion. 2. Flexion. 3. Sandifers Syndrome. 4. Hashimoto Disease. ANS: 3 Feedback 1. Torsion is the twisting of the gut. 2. Flexion is the extension of the body. The baby is arching. 3. The infant is exhibiting Sandifers Syndrome with the twisting and arching. 4. Hashimoto Disease is a thyroid disorder. 42. A previously healthy 3-week-old baby has been admitted to the pediatric unit for pyloric stenosis. On admission, what would the nurse anticipate being told? 1. History of watery stools 2. History of projectile vomiting
CURRENT DIAGNOSIS AND TREATMENT PEDIATRICS 24TH LATEST 3. History of increased stools 4. History of vomiting with large amount of bile ANS: 2 Feedback 1. Normal stools are present with pyloric stenosis. 2. The projectile vomiting is an indication that the pyloric sphincter is not holding the stomach contents in properly. 3. The infant may have decreased stools. 4. The vomiting consists of formula/milk that appears similar to what was in the bottle. 43. A child with unresolved pyloric stenosis may exhibit signs of: 1. Crying. 2. Irritability. 3. Poor weight gain. 4. Tachycardia. ANS: 3 Feedback 1. The baby may cry because he/she is hungry, but this is not the major,
long-term issue. 2. The baby may be irritable because he/she is hungry, but this is not the major, long-term issue 3. Poor weight gain is a major, long-term issue that needs to be resolved quickly so that there are no neurological effects. 4. Tachycardia is not common with long-term pyloric stenosis issues. 44. The nurse is assessing an infant with a history of projectile vomiting. The nurse is palpating the abdomen and notes an olive sign in the upper abdomen. This is the cardinal sign of: 1. Pyloric stenosis. 2. GERD.
CURRENT DIAGNOSIS AND TREATMENT PEDIATRICS 24TH LATEST 3. Intussusception. 4. Hirschsprungs disease. ANS: 1 Feedback 1. The olive sign is the actual shape of the pyloric sphincter and the stomach meeting. 2. Palpation does not help diagnose GERD. 3. Intussusception is noted by a raised bowel in the lower abdomen. 4.Palpation for Hirschsprungs will note an obtunded abdomen, not the olive sign. 45. The nurse has received orders for a 4 month old with a diagnosis of Volvulus. To provide comfort for the infant, the nurse should: 1. Encourage the mother to hold the infant and provide a pacifier. 2. Keep the infant swaddled tightly. 3. Place the infant in a prone position. 4. Assess the pain level of the infant using the FACES scale. ANS: 1 Feedback 1.The holding will place the patient in a position to take pressure off of the abdomen, and the pacifier may help with decreasing the need for food. 2.Swaddling the baby may cause more distress because of the constriction around the abdomen. 3.The prone position will place too much pressure on the abdomen. 4.FACES is used for assessing pain, not providing a source of comfort. 46. An infant with intussusception has returned to the pediatric unit from the operating room. The nurse should anticipate providing all of the following care except: 1. Maintaining an IV.
CURRENT DIAGNOSIS AND TREATMENT PEDIATRICS 24TH LATEST 2. Starting gradual oral feedings right away. 3. Monitoring for pain. 4. Maintaining the nasogastric tube. ANS: 2 Feedback 1. The infant will not be taking in large amounts of fluids, so IV fluid maintenance will be important. 2. The infants intestinal tract will need time to heal prior to starting feedings. 3. Pain management will be needed for healing. 4. The NG tube will help with draining gastric contents and air to decrease the risk for an upset stomach. 47. Megan is caring for a neonate. Megan assesses the possibility of necrotizing enterocolitis (NEC) because the premie is exhibiting the generalized assessment of: 1. Hypertonia, tachycardia, and metabolic alkalosis. 2. Abdominal distention, temperature instability, and bloody stools. 3. Hypertension, apnea, and ruddy skin color. 4. No residual feedings and increased urinary output. ANS: 2 Feedback 1.Metabolic alkalosis cannot be seen in a generalized assessment. 2.These are the cardinal signs of a neonate with a generalized assessment of necrotizing enterocolitis. 3. Hypotension usually occurs, along with the neonate appearing gray or pale in color. 4. Feedings will have a high residual and a decreased urinary output when necrotizing enterocolitis is developing.
CURRENT DIAGNOSIS AND TREATMENT PEDIATRICS 24TH LATEST 48. A toddler has been diagnosed with short bowel syndrome because of a past history of necrotizing enterocolitis. The nurse should encourage the parents to do all of the following except: 1. Give the child juice at least once a day to help with Vitamin C consumption. 2. Give enteral feedings. 3. Introduce solid food. 4. Keep follow-up appointments. ANS: 1 Feedback 1. Juice may cause more diarrhea and should be avoided. 2. The child may need to have enteral feedings to increase the calorie
content for growth because of the lack of absorption in the GI tract. 3. Solid food should be started slowly and be in higher calorie content
because of the lack of absorption for the GI tract. 4. Appointments are needed to make sure the child is growing and
receiving the proper nutritional content. 59. Which of the following conditions is often associated with severe diarrhea of gastroenteritis? 1. Metabolic acidosis 2. Metabolic alkalosis 3. Respiratory acidosis 4. Respiratory alkalosis ANS: 1 Feedback 1. The body is in an acidic state and because of the electrolyte imbalance in the GI tract, it is metabolic acidosis. 2. Alkalosis is not occurring because of the lack of fluid. 3.The condition is not affecting the respiratory tract. 4.The condition is not affecting the respiratory tract.
CURRENT DIAGNOSIS AND TREATMENT PEDIATRICS 24TH LATEST 50. A nurse taking care of a child with severe gastroenteritis knows that compensation for fluid loss will occur. What causes the compensation to occur? 1. Vasodilation of the peripheral vascular system increases the perfusion. 2. Fluid shifts from the interstitial space to the intravascular space 3. The renal-aldosterone system is activated. 4. The body reserves the fluid in the vasculature so that the heart does not have an increased workload. ANS: 2 Feedback 1. Vasoconstriction can occur because of the lack of fluid. 2. The fluid shift occurs because the cells are dehydrated and want the
fluid. 3. The renal-aldosterone system is not activated at this time. 4. The heart has an increased workload because the blood becomes thicker.
The thickening occurs because of the lack of fluid. 51. An infant is brought to the emergency department with the following clinical signs: poor skin turgor, weight loss, lethargy, tachycardia, and tachypnea. This is suggestive of which of the following? 1. Sodium excess 2. Water depletion 3. Potassium Excess 4. Fluid overload ANS: 2 Feedback 1. An infant with sodium excess would have weight gain because of fluid
retention as well as a doughy skin turgor.
CURRENT DIAGNOSIS AND TREATMENT PEDIATRICS 24TH LATEST 2. The weight loss and poor skin turgor indicate the lack of fluid in the
body. The high heart and respiratory rates are because the blood is thicker, making the heart and lungs work harder. 3. The infant is not exhibiting cardiac arrhythmias that are part of hyperkalemia. 4. The infant would have increased weight, a doughy skin turgor, and a sluggish pulse if fluid overload was occurring. 52. A preschooler that is being potty trained is refusing to have bowel movements. The mother brought the child to the clinic because now the child has constipation. The nurse speaks to the mother about behavior modification efforts to have the preschooler defecate. All of the following may help except: 1. Rewarding the child for defecating in the toilet. 2. Identifying when the child usually defecated prior to potty training, then attempting to use the toilet at that time. 3. Create regular times to use the potty, especially 5 to 10 minutes after a meal. 4. Rewarding the child for defecating in a diaper, then trying to retrain using the toilet. ANS: 4 Feedback 1. Incentives for defecation may be the reason some children will have
success because the urge for a treat is important to the child. 2. Timing can help create a known pattern so that the child understands the
expectations at particular times throughout the day. 3. Food that is consumed helps to put pressure on the bowel, which may
lead to defecation if the child does not have to hold it. 4. Reverting back to diapers may make the child refuse to use a toilet
because of the convenience. 53. A child has been prescribed to use Senna to help reduce constipation. The nurse should include which of the following in her teaching? 1. Senna can be used long term. 2. Increasing the amount of fluid intake will be important to help Senna work effectively.
CURRENT DIAGNOSIS AND TREATMENT PEDIATRICS 24TH LATEST 3. Senna should be used after an enema has been given. 4. Senna should also be used with milk of magnesium to improve the outcomes. ANS: 2 Feedback 1. Senna should only be taken short term so that the bowel does not come to depend on the medication for defecation. 2. Fluid is important to help loosen stool and help Senna create the need for defecation. 3. Senna should be used prior to the use of an enema to try to defecate as much as possible in the least invasive manner possible. 4. Senna and Milk of Magnesium should not be used at the same time. 54. A child with issues of constipation should include which of the following foods in his/her diet to facilitate defecation? 1. Skittles 2. Apples 3. White bread 4. Grilled chicken breast ANS: 2 Feedback 1.A candy may increase constipation because of the sodium content. 2.Apples will increase the amount of fiber, thus making the bowel act to defecate. 3.White bread lacks fiber and is not effective for constipation. 4.Chicken breasts lack the fiber that is needed for the defecation. 55. A cardinal sign of Hirschsprungs Disease in neonates is: 1. A thin abdomen. 2. Constipation since birth.
CURRENT DIAGNOSIS AND TREATMENT PEDIATRICS 24TH LATEST 3. Dry, pebble-like stools. 4. Crying every time the neonate is fed. ANS: 2 Feedback 1. The abdomen becomes obtunded because of the collection of stool in the colon. 2. Constipation is present because of the collection of stool in a particular portion of the colon. 3. The stool is thin and ribbon-like in appearance. 4. The neonate tends to vomit after eating because the food cannot move through the digestive tract. 56. The family of a child with suspected Hirschsprungs Disease is asking the nurse about the preparation for confirming the diagnosis. The nurse should explain which of the following procedures to the family? 1. A barium enema 2. A rectal biopsy 3. A transabdominal ultrasound 4. A bronchoscope ANS: 2 Feedback 1. Not advised because there is blockage in the colon 2. Will indicate if Hirschsprungs Disease is present 3. Does not provide a clear view of the diseased tissue 4. The test looks at the lungs, not the intestines. 57. The nurse is assessing a child with a history of Hirschsprungs Disease. The nurse should expect to have assessment findings of: 1. Frequent bloody stools.
CURRENT DIAGNOSIS AND TREATMENT PEDIATRICS 24TH LATEST 2. Abdominal cramping and fecal soiling in the childs underwear. 3. A low hematocrit. 4. Thin, ribbon-like, foul smelling stool and a distended abdomen ANS: 4 Feedback 1. Few stools occur with a child with this disease. 2. Abdominal cramping may occur, but the child is able to use the toilet. 3.There is no bleeding involved in the disease process, so a low hematocrit is rare. 4.The thin, ribbon-like stools occur because of the small passageway for stool to move through the affected area. The abdomen is distended because of the gas and fecal buildup in the affected area. 58. A neonate has been born with a known diagnosis of an omphalocele. The nurse should provide all of the following cares for the neonate except: 1. Swaddling the baby in blankets to keep body temperature stable. 2. Providing nonnutritive sucking. 3. Maintaining IV access for fluids. 4. Keeping the omphalocele moist and on a warmer bed to keep the body temperature stable. ANS: 1 Feedback 1. The swaddling will introduce material onto the omphalocele and dry out the area, increasing the risk for infection. 2. Nonnutritive sucking will soothe the neonate. 3. IV access will be needed to maintain a fluid balance because of the NPO status. 4. Keeping the omphalocele moist and warm will help keep the body stable. 59. A neonate is born with gastroschisis. The nurse should be the most concerned about which of the following when caring for the child?
CURRENT DIAGNOSIS AND TREATMENT PEDIATRICS 24TH LATEST 1. Keeping the organs dry and warm 2. Assessing the organs to make sure that there is no vascular compromise 3. Starting oral feedings so that the stomach and intestines can start working 4. Maintaining an IV ANS: 2 Feedback 1. The organs should be kept warm and as moist as possible to prevent
drying, cracking, and increasing the risk for infection. 2. Vascular compromise is of concern, so positioning will be important.
3.The child will remain NPO until the gut can be further examined. 4.Maintaining IV access is needed, but not the top priority at this time. 60. A home care nurse is visiting a 5-day-old male infant for a scheduled follow-up appointment to ensure that he is responding to home phototherapy for treatment of jaundice. After completing a thorough assessment and obtaining a history from the parents, the nurse recognizes that this infant is in the first phase of encephalopathy when he exhibits: 1. A high-pitched cry when touched. 2. Severe muscle spasms. 3. Fever and seizures. 4. Hypotonia, lethargy, and a poor suck when feeding. ANS: 4 Feedback 1.A high-pitched cry occurs in later phases of the encephalopathy. 2.Muscle spasms are not present with this diagnosis. 3. Fevers are rare in neonates with encephalopathy. The seizures can occur if the levels continue to climb. 4. Hypotonia, lethargy, and poor feedings are the first symptoms to occur in the first phase of encephalopathy caused by jaundice.
CURRENT DIAGNOSIS AND TREATMENT PEDIATRICS 24TH LATEST
Chapter 24. Kidney and Urinary Tract 1. Which statement made by a school-age girl indicates the need for further teaching about the prevention of urinary tract infections? a. I always wear cotton underwear. b. I really enjoy taking a bubble bath. c. I go to the bathroom every 3 to 4 hours. d. I drink four to six glasses of fluid every day. ANS: B Bubble baths should be avoided because they tend to cause urethral irritation, which leads to urinary tract infection. It is desirable to wear cotton rather than nylon underwear. Nylon tends to hold in moisture and promote bacterial growth, whereas cotton absorbs moisture. Children should be encouraged to urinate at least four times a day. An adequate fluid intake prevents the buildup of bacteria in the bladder. 2. The nurse assessing a child with acute poststreptococcal glomerulonephritis should be alert for which finding? a. Increased urine output b. Hypotension c. Tea-colored urine d. Weight gain ANS: C Acute poststreptococcal glomerulonephritis is characterized by hematuria, proteinuria, edema, and renal insufficiency. Tea-colored urine is an indication of hematuria. In acute poststreptococcal glomerulonephritis, the urine output may be decreased and the blood pressure increased. Edema may be noted around the eyelids and ankles in patients with acute
CURRENT DIAGNOSIS AND TREATMENT PEDIATRICS 24TH LATEST poststreptococcal glomerulonephritis; however, weight gain is associated with nephrotic syndrome. 3. The mother of a child who was recently diagnosed with acute glomerulonephritis asks the nurse why the physician keeps talking about casts in the urine. The nurses response is based on the knowledge that the presence of casts in the urine indicates: a. glomerular injury. b. glomerular healing. c. recent streptococcal infection. d. excessive amounts of protein in the urine. ANS: A The presence of red blood cell casts in the urine indicates glomerular injury. Casts in the urine are abnormal findings and are indicative of glomerular injury, not glomerular healing. A urinalysis positive for casts does not confirm a recent streptococcal infection. Casts in the urine are unrelated to proteinuria. 4. What is a clinical finding that warrants further intervention for the child with acute poststreptococcal glomerulonephritis? a. Weight loss to within 1 pound of the preillness weight b. Urine output of 1 milliliter per kilogram per hour c. A normal blood pressure d. Inspiratory crackles ANS: D Children with excess fluid volume may have pulmonary edema. Inspiratory crackles indicate fluid in the lungs. Pulmonary edema can be a life-threatening complication. Weight loss to within 1 pound of the preillness weight is an indication that the child is responding to treatment. A urine output of 1 milliliter per kilogram per hour is an acceptable urine output and indicates
CURRENT DIAGNOSIS AND TREATMENT PEDIATRICS 24TH LATEST that the child is responding to treatment. A normal blood pressure is also an indication that the child is responding to treatment. 5. Which diagnostic finding is assessed by the nurse when a child has primary nephrotic syndrome? a. Hyperalbuminemia b. Positive ASO titer c. Leukocytosis d. Proteinuria ANS: D Large amounts of protein are lost through the urine as a result of an increased permeability of the glomerular basement membrane. Hypoalbuminemia is present because of loss of albumin through the defective glomerulus and the livers inability to synthesize proteins to balance the loss. ASO titer is negative in a child with primary nephrotic syndrome. Leukocytosis is not a diagnostic finding in primary nephrotic syndrome. 6. Which finding indicates that a child receiving prednisone for primary nephrotic syndrome is in remission? a. Urine is negative for casts for 5 days. b. Urine is 0 to trace for protein for 5 to 7 days. c. Urine is negative for protein for 2 weeks. d. Urine is 0 to trace for blood for 1 week. ANS: B The child receiving steroids for the treatment of primary nephrotic syndrome is considered in remission when the urine is 0 to trace for protein for 5 to 7 days. The absence of casts in the urine gives no indication about the childs response to treatment. The child with primary nephrotic syndrome is considered to be in remission when the urine is negative for protein for 5 to 7 consecutive days. The absence of proteinuria for 2 consecutive weeks indicates a continued
CURRENT DIAGNOSIS AND TREATMENT PEDIATRICS 24TH LATEST remission. The presence or absence of hematuria is not used to determine remission in primary nephrotic syndrome. 7. Which of the following statements made by a parent of a child with nephrotic syndrome indicates an understanding of a no-added-salt diet? a. I only give my child sweet pickles. b. My child just puts a little salt on his food. c. I let my child have slightly salted potato chips. d. I do not put any salt in foods when I am cooking. ANS: D A no-added-salt diet means that no salt should be added to foods, either when cooking or before eating. All types of pickles and potato chips are high in sodium and should not be served to the child on a no-added-salt diet. The child should not be allowed to use a salt shaker at meals when on a no-added-salt diet. 8. Which is an appropriate intervention for a child with nephrotic syndrome who is edematous? a. Teach the child to minimize body movements. b. Change the childs position every 2 hours. c. Avoid the use of skin lotions. d. Bathe every other day. ANS: B Frequent position changes decrease pressure on body parts and help relieve edema in dependent areas. The child with edema is at risk for impaired skin integrity. It is important for the child to change position frequently to prevent skin breakdown. Applying lotion to the skin helps to increase circulation. Bathing daily removes irritating body secretions from the skin. 9. A child with secondary enuresis who complains of dysuria or urgency should be evaluated for which condition?
CURRENT DIAGNOSIS AND TREATMENT PEDIATRICS 24TH LATEST a. Hypocalciuria b. Nephrotic syndrome c. Glomerulonephritis d. Urinary tract infection ANS: D Complaints of dysuria or urgency from a child with secondary enuresis suggest the possibility of a urinary tract infection. An excessive loss of calcium in the urine (hypercalciuria) can be associated with complaints of painful urination, urgency, frequency, and wetting. Nephrotic syndrome is not usually associated with complaints of dysuria or urgency. Glomerulonephritis is not a likely cause of dysuria or urgency. 10. What should the nurse include in a teaching plan for the parents of a child with vesicoureteral reflux? a. Screening for urinary tract infection (UTI) if febrile b. Suggestions for how to maintain fluid restrictions c. The use of bubble baths as an incentive to increase bath time d. The need for the child to hold urine for 6 to 8 hours ANS: A A child with vesicoureteral reflux is screened for a UTI if febrile. Fluids are not restricted when a child has vesicoureteral reflux. In fact, fluid intake should be increased as a measure to prevent urinary tract infections. Bubble baths should be avoided to prevent urethral irritation and possible urinary tract infection. To prevent urinary tract infections, the child should be taught to void frequently and never resist the urge to urinate. 11. Which intervention is appropriate when examining a male infant for cryptorchidism? a. Cooling the examiners hands b. Taking a rectal temperature c. Placing the infant on the examination table
CURRENT DIAGNOSIS AND TREATMENT PEDIATRICS 24TH LATEST d. Warming the room ANS: D For the infants comfort, the infant should be examined in a warm room with the examiners hands warmed. Testes can retract into the inguinal canal if the infant is upset or cold. Examining the infant with cold hands is uncomfortable for the infant and is likely to cause the infants testes to retract into the inguinal canal. It may also cause the infant to be uncooperative during the examination. A rectal temperature yields no information about cryptorchidism. When possible, the infant should be examined in the caregivers lap to elicit cooperation and avoid upsetting the infant. 12. Parents ask the nurse, When should our childs hypospadias be corrected? The nurse responds based upon the knowledge that correction of hypospadias should be accomplished by the time the child is: a. 1 month of age. b. 6 to 8 months of age. c. school age. d. sexually mature. ANS: B The correction of hypospadias should ideally be accomplished by the time the child is 6 to 8 months of age and before toilet training. Surgery to correct hypospadias is not performed when the infant is less than 6 months of age. It is preferable for hypospadias to be surgically corrected before the child enters school so that the child has normal toileting behaviors in the presence of his peers. Corrective surgery for hypospadias is done long before sexual maturity. 13. A nurse is teaching a class on acute renal failure. The nurse relates that acute renal failure as a result of hemolytic-uremic syndrome is classified as: a. prerenal. b. intrarenal.
CURRENT DIAGNOSIS AND TREATMENT PEDIATRICS 24TH LATEST c. postrenal. d. chronic. ANS: B Intrarenal acute renal failure is the result of damage to kidney tissue. Possible causes of intrarenal acute renal failure are hemolytic uremic syndrome, glomerulonephritis, and pyelonephritis. Prerenal acute renal failure is the result of decreased perfusion to the kidney. Possible causes include dehydration, septic and hemorrhagic shock, and hypotension. Postrenal acute renal failure results from obstruction of urine outflow. Conditions causing postrenal failure include ureteropelvic obstruction, ureterovesical obstruction, or neurogenic bladder. Renal failure caused by hemolytic-uremic syndrome is of the acute nature. Chronic renal failure is an irreversible loss of kidney function, which occurs over months or years. 14. Which dietary modification is appropriate for a child with chronic renal failure? a. Decreased salt b. Decreased fat c. Increased potassium d. Increased phosphorus ANS: A Salt is restricted to prevent fluid overload and hypertension. A low-fat diet is not relevant to chronic renal failure. Potassium intake may be restricted because of the kidneys inability to remove it. Phosphorus is restricted to help prevent bone disease. 15. Which condition is characterized by a history of bloody diarrhea, fever, abdominal pain, and low hemoglobin and platelet counts? a. Acute viral gastroenteritis b. Acute glomerulonephritis c. Hemolytic-uremic syndrome d. Acute nephrotic syndrome
CURRENT DIAGNOSIS AND TREATMENT PEDIATRICS 24TH LATEST ANS: C Hemolytic-uremic syndrome is an acute disorder characterized by anemia, thrombocytopenia, and acute renal failure. Most affected children have a history of gastrointestinal symptoms, including bloody diarrhea. Anemia and thrombocytopenia are not associated with acute gastroenteritis. The symptoms described are not suggestive of acute glomerulonephritis or nephrotic syndrome. 16. Which is a true statement describing the differences in the pediatric genitourinary system compared with the adult genitourinary system? a. The young infants kidneys can more effectively concentrate urine than can an adults kidneys. b. After 6 years of age, kidney function is nearly like that of an adult. c. Unlike adults, most children do not regain normal kidney function after acute renal failure. d. Young children have shorter urethras, which can predispose them to urinary tract infections. ANS: D Young children have shorter urethras, which can predispose them to urinary tract infections. The young infants kidneys cannot concentrate urine as efficiently as those of older children and adults because the loop of Henle is not yet long enough to reach the inner medulla, where concentration and reabsorption occur. By 6 to 12 months of age, kidney function is nearly like that of an adult. Unlike adults, most children with acute renal failure regain normal function. MULTIPLE RESPONSE 1. A nurse is planning care for a child admitted with nephrotic syndrome. Which interventions should be included in the plan of care? Select all that apply. a. Administration of antihypertensive medications b. Daily weights c. Salt-restricted diet
CURRENT DIAGNOSIS AND TREATMENT PEDIATRICS 24TH LATEST d. Frequent position changes e. Teach parents to expect tea-colored urine ANS: B, C, D A child with nephrotic syndrome will need to be monitored closely for fluid excess so daily weights are important. The diet is salt restricted to prevent further retention of fluid. Because of the fluid excess, frequent position changes are required to prevent skin breakdown. Nephrotic syndrome does not require antihypertensive medications. These are administered for acute glomerulonephritis. Tea-colored urine is expected with acute glomerulonephritis, but not nephrotic syndrome. The urine in nephrotic syndrome is frothy indicating protein is being lost in the urine. 2. A nurse is assessing an infant for urinary tract infection (UTI). Which assessment findings should the nurse expect? Select all that apply. a. Change in urine odor or color b. Enuresis c. Fever or hypothermia d. Voiding urgency e. Poor weight gain ANS: A, C, E The signs of a UTI in an infant include fever or hypothermia, irritability, dysuria as evidenced by crying when voiding, change in urine odor or color, poor weight gain and feeding difficulties. Enuresis and voiding urgency would be assessed in an older child.
Chapter 25. Neurologic and Muscular Disorders 1. The autonomic nervous system is responsible for: 1. Digesting a meal of hotdogs and chips.
CURRENT DIAGNOSIS AND TREATMENT PEDIATRICS 24TH LATEST 2. Monitoring the heart rate while running. 3. Causing the body to perspire in the hot sun. 4. All of the above are part of the autonomic nervous system. ANS: 4 Feedback 1.The ANS helps with the digestion of food. 2.The heart is regulated by the ANS. 3. Perspiration occurs because of the ANS for the purpose of thermoregulation. 4. The ANS helps with the digestion of food. The heart is regulated by the ANS. Perspiration occurs because of the ANS for the purpose of thermoregulation. 2. The responsibilities of the central nervous system include: 1. Deciding to walk instead of run. 2. Helping to understand a math problem. 3. Digesting the food in the stomach. 4. Keeping the hand on a hot stove. ANS: 1 Feedback 1. The brain is part of the CNS, which helps make decisions about body movements. 2. The CNS does not help with cognitive abilities. 3.Food digestion is part of the ANS. 4.The CNS would tell the hand to move. 3. The blood-brain barrier of an infant is: 1. Less permeable than that of an adult. 2. Impermeable for glucose.
CURRENT DIAGNOSIS AND TREATMENT PEDIATRICS 24TH LATEST 3. Permeable for large proteins. 4. Permeable for large molecules. ANS: 4 Feedback 1.There is no difference between the adult and infant blood-brain barrier. 2.Glucose is permeable for the blood-brain barrier. 3. Large proteins are impermeable for the blood-brain barrier. 4.Large molecules are able to cross the blood-brain barrier. 4. The first assessment a child receives to identify neurological development is:
1. APGAR scores. 2. Scoliosis testing. 3. The Denver II study. 4. Kindergarten testing. ANS: 1 Feedback 1. APGAR stands for Appearance, Pulse, Grimace, Activity, Respiration,
indicating responses of the neurological system. This testing is done right after birth. 2. Scoliosis testing does not occur until the child is a preteen. 3.The Denver II test is not used until the infant is older. 4. Kindergarten testing occurs later in the childs life. 5. Questions about neurological function are raised when a child:
1. Snores. 2. Shows aggression when previously none was shown. 3. Wants attention from a parent. 4. Refuses to follow adult instruction.
CURRENT DIAGNOSIS AND TREATMENT PEDIATRICS 24TH LATEST ANS: 2 Feedback 1.Snoring presents a concern for the airway, not neurological functioning. 2.Changes in personality are signs of abnormal behaviors and should be investigated. 3. Attention-seeking behaviors indicate psychosocial need, not a neurological change. 4. This is normal behavior for a child and does not qualify as a neurological issue. 6. A neonate is born with anencephaly. The prognosis for a neonate with this condition is: 1. A normal outcome. 2. A high risk for hydrocephaly. 3. Can be death. 4. Mental handicap. ANS: 3 Feedback 1.The neonate will not have a brain, thus this is not a normal outcome. 2.The child lacks brain tissue, and hydrocephaly is not common. 3.Death is inevitable for a neonate with anencephaly because of the lack of brain structure. 4.A child with anencephaly has a very short life span, and evaluation for mental handicap is not needed. 7. A child with severe mental and physical handicaps is at risk for: 1. Developing neurocutaneous lesions. 2. A dysmorphic nose and ears. 3. Abnormal cranial nerve function. 4. All of the above are correct.
CURRENT DIAGNOSIS AND TREATMENT PEDIATRICS 24TH LATEST ANS: 4 Feedback 1. Neurocutaneous lesions occur because of high risk for lack of physical movement. 2. Bone structure may be dysmorphic because of chronic abnormal muscle movements and contractures. 3. Because of neurological dysfunction, cranial nerve function will be abnormal. 4. Neurocutaneous lesions occur because of high risk for lack of physical movement. Bone structure may be dysmorphic because of chronic abnormal muscle movements and contractures. Because of neurological dysfunction, cranial nerve function will be abnormal. 8. Neural tube defects can be linked to: 1. A mothers drug habit while pregnant. 2. A mothers lack of folic acid while pregnant. 3. A fetuss exposure to environmental toxins. 4. A mothers alcohol consumption while pregnant. ANS: 2 Feedback 1. Drug habits can be linked to neurological damage and growth
retardation. 2. Folic acid is needed for neural tube closure and should be taken as a
prenatal vitamin. 3. Exposure to toxins can cause various cognitive and physical anomalies. 4.Alcohol can cause cognitive and physical anomalies if taken while pregnant. 9. A nurse is attempting to position a newborn with a myelomeningocele in the lower lumbar region. The best position for the newborn would be: 1. Prone. 2. Laying the newborn on his/her side with support provided to the myelomeningocele.
CURRENT DIAGNOSIS AND TREATMENT PEDIATRICS 24TH LATEST 3. Supine. 4. Any position is acceptable for a neonate with a myelomeningocele. ANS: 2 Feedback 1. Prone does not allow for support of the sac. 2. Laying the newborn on his/her side will provide support for the sac and decrease the chance of a rupture. 3. Supine places too much pressure on the sac and increases the risk for a rupture. 4. Laying the newborn on his/her side will provide the most support for the sac and decrease the chance of a rupture. 10. A neonate was born to a 28-year-old mother with an uneventful pregnancy two hours ago. The baby was delivered via cesarean section and taken directly to the neonatal intensive care unit because of an encephalocele. The mother is coming to see the baby. The nurse should: 1. Be prepared to answer questions about the babys care and condition. 2. Leave the room and give the family time with the neonate. 3. Prepare the mother prior to entering the room about the dysmorphic features and discuss the supportive care being provided. 4. Not let the mother see the child at this point. ANS: 3 Feedback 1. The nurse should be ready to answer questions and needs to prepare the mother for the appearance of her neonate. 2. Time with the neonate is important, but support is the priority for parents at this time. 3. Prior information before seeing the child can help reduce the shock and foster more acceptance of the neonate. 4. The mother needs to see the neonate to help create a bond. 11. A child born with Dandy Walker malformation is receiving palliative care in the pediatric unit. A nurse should:
CURRENT DIAGNOSIS AND TREATMENT PEDIATRICS 24TH LATEST 1. Provide the parents, patient, and family members with supportive care during this time. 2. Ask the parents to be part of the plan of care as much as possible. 3. Attempt to provide a primary nurse for this particular patient on each shift. 4. All of the above are correct. ANS: 4 Feedback 1. Family support is important in order to provide a high quality of life in a
limited amount of time. 2. Parental involvement will create a bond with the child and empower the
parents. 3.A primary nurse is able to form a bond with the family and understand the needs of the child because of frequent interactions. 4.Family support is important in order to provide a high quality of life in a limited amount of time. Parental involvement will create a bond with the child and empower the parents. A primary nurse is able to form a bond with the family and understand the needs of the child because of frequent interactions. 12. A head circumference is being measured at a 4 month olds well-baby checkup. It is noted that the head circumference has not grown since the previous assessment. The nurse should: 1. Ask the mother about the childs nutrition. 2. Notify the doctor. 3. Re-measure the head circumference, check developmental milestones, assess the nutritional status, and discuss the findings with the doctor. 4. Document the normal findings. ANS: 3 Feedback 1. Nutritional assessment is important, but not the priority intervention at
this time.
CURRENT DIAGNOSIS AND TREATMENT PEDIATRICS 24TH LATEST 2. The doctor will receive the information after a re-measurement is taken
to validate the findings. 3. Re-measurement is needed to validate findings, and assessing
milestones will indicate the cognitive and physical abilities of the child. Nutritional information will indicate if adequate nutrition is being given. The doctor will be able to prescribe the best course of action after this information is reported. 4. The findings are abnormal, and further investigation is needed. 13. A child with a diagnosis of schizencephaly is assigned to a new nurse on the pediatric floor. The new nurse has not worked with a child with this diagnosis before. A career nurse discusses the plan of care needed for the child with the new nurse. It will be important to: 1. Assess the side of the body that has paralysis for any lesions or sores. 2. Let the patient do as much as possible for activities of daily. 3. Discourage the patient to move the paralyzed side of the body. 4. Provide full care for the patient. ANS: 1 Feedback 1. Skin breakdown can occur because of the lack of mobility for the
affected side of the body. 2. The child may be lower functioning and not be able to understand how
to do ADLs or have the physical ability to do them. 3. Movement is important, but not the priority. 4.Encouragement to do as much as possible is important for independence, but the child will need supervision. 14. A nurse is assessing a 6-month-old boys suture lines. The nurse notes that the baby has craniosynostosis. The nurse should be concerned because: 1. The suture line closure will not allow the brain to grow. 2. This can lead to hydrocephalus. 3. The child will have immediate developmental delays because of the lack of space for the brain to grow.
CURRENT DIAGNOSIS AND TREATMENT PEDIATRICS 24TH LATEST 4. The child will not require surgery. ANS: 1 Feedback 1.Early closure of the sutures will inhibit brain growth. 2.Fluid buildup is not a concern at this time. 3.A progression of developmental delay, rather than immediate delay, will occur. 4.Surgery may be needed to relieve pressure and allow for growth to occur. 15. A child that had a shunt placed four years ago for hydrocephalus is in the emergency room complaining of a rapid onset of vomiting and increased lethargy. The nurse knows that the child will need: 1. Nothing, as this is a normal complication and not an emergency. 2. To be placed on IV fluids to help maintain an electrolyte balance. 3. Small amounts of fluids until the vomiting has subsided. 4. To consider this a neurological medical emergency and check the childs head circumference. ANS: 4 Feedback 1. This should be considered a neurological emergency, and the child
should be checked. 2. Electrolyte imbalances are more apt to occur when fluid is removed.
3.The history of having a shunt needs to be addressed first to prevent any neurological damage. 4.Measuring the head circumference will give an indication as to the amount of fluid not draining with the shunt and should be considered a medical emergency. 16. Night terrors can occur in adolescents because of: 1. Emotional stress. 2. Alcohol use.
CURRENT DIAGNOSIS AND TREATMENT PEDIATRICS 24TH LATEST 3. Bullying. 4. All of the above can trigger night terrors in adolescents. ANS: 4 Feedback 1.Emotional stress can cause increased thoughts and trigger night terrors. 2.Alcohol causes a disturbance to the chemical balance in the brain, causing the night terrors. 3.Bullying can be an emotional stressor, causing the night terrors. 4.Emotional stress can cause increased thoughts and trigger night terrors. Alcohol causes a disturbance to the chemical balance in the brain, causing the night terrors. Bullying can be an emotional stressor, causing the night terrors. 17. When speaking with a family about their 9-year-old daughters nightmares, it is important to ask: 1. If the child has a history of daytime napping. 2. What medications the child takes during the day. 3. How often the child consumes caffeine. 4. All of the above should be part of the assessment. ANS: 4 Feedback 1. Daytime napping can cause a sleep disturbance pattern because the child is not reaching the REM cycle. 2. Medications can have a side effect of nightmares for children. 3.Caffeine causes sleep disturbance because it is a stimulant. 4.Daytime napping can cause a sleep disturbance pattern because the child is not reaching the REM cycle. Medications can have a side effect of nightmares for children. Caffeine causes sleep disturbance because it is a stimulant. 18. A quality of a partial seizure is: 1. Status epilepticus.
CURRENT DIAGNOSIS AND TREATMENT PEDIATRICS 24TH LATEST 2. Tonic movements. 3. Fluttering eyelids. 4. Clonic movements. ANS: 4 Feedback 1. This occurs after a grand mal seizure. 2. Tonic movements occur with a grand mal seizure. 3.Fluttering eyelids are noted in grand mal seizures. 4.Clonic movements occur in partial seizures and can occur in grand mal seizures. 19. A mother is asking the nurse why her daughter continues to have temporal lobe seizures even though she is on medication. The nurse knows this is occurring because: 1. The medication may not be in the therapeutic range. 2. Temporal lobe seizures do not respond well to medications. 3. The daughter may be missing doses of her medication. 4. The food her daughter eats may have a negative reaction with the medication, causing more seizures. ANS: 2 Feedback 1.Medication regulation is difficult with temporal lobe seizures. 2.Temporal lobe seizures have a poor response rate to medications. 3.Missing doses of medication can lead to seizures, but temporal lobe seizures dont respond well to medications. 4.Foods do not have an influence on temporal lobe seizures. 20. Which of the following types of epilepsy are photosensitive? 1. Juvenile myoclonic epilepsy
CURRENT DIAGNOSIS AND TREATMENT PEDIATRICS 24TH LATEST 2. Temporal lobe epilepsy 3. Febrile seizures 4. Childhood absence epilepsy ANS: 1 Feedback 1. Photosensitivity is common with Juvenile myoclonic epilepsy. 2. Photosensitivity does not usually occur in temporal lobe epilepsy. 3. Febrile seizures are triggered by fevers, not photosensitivity. 4. Childhood absence epilepsy is not influenced by photosensitivity. 21. A child who had a seizure one hour ago is exhibiting signs of paralysis on the left side of the body. The nurse understands the child is exhibiting signs of: 1. Lethargy due to previous seizure activity. 2. Postictal paralysis. 3. Permanent paralysis of the left side of the body. 4. Major brain damage that is going to have long-term effects. ANS: 2 Feedback 1. Neurological fatigue can occur after a seizure, but it is not the reason for
the paralysis. 2. Postictal paralysis will resolve within the next few hours. 3. The paralysis caused by the seizure will resolve within the next few
hours. 4. Serial seizures cause brain damage. One seizure will not. 22. A child with a known history of Benign Rolandic Epilepsy is having a seizure during lunch at the middle school. The school nurse is called to the cafeteria. What is the school nurses priority at this time? 1. Prevent a possible choking incident by checking the students mouth for food.
CURRENT DIAGNOSIS AND TREATMENT PEDIATRICS 24TH LATEST 2. Lay the child down on the floor and make sure the area is safe. 3. Call the EMTs for help. 4. Notify the parents that their daughter is having a seizure. ANS: 1 Feedback 1. This is the priority because the child is in the lunch room. The nurse must check for food to decrease the chance of choking. 2. Making the area safe is important, but not the priority at this time. 3.EMTs are not needed for this situation because this is a common occurrence for the diagnosis. 4.The parents should be notified after the child is safe. 23. An 18 month old is having a seizure when the nurse is assessing him. The nurse notes that the child is fluttering his eyes and smacking his lips. The nurse should document this seizure as: 1. An absence seizure. 2. A tonic-clonic seizure. 3. A myoclonic seizure. 4. A febrile seizure. ANS: 1 Feedback 1.Eye fluttering and lip smacking are common characteristics of an absence seizure. 2.A tonic-clonic seizure has stiffening of the muscles. This child is not exhibiting this characteristic. 3. The child is not exhibiting muscle rigidity that is common with myoclonic seizures. 4. The child does not have a fever to cause the seizure. 24. A 9 month old is admitted to the pediatric unit for seizures of unknown origin. The child has an EEG performed for several hours. The EEG notes several seizures occurring at different intervals. The nurse knows this child:
CURRENT DIAGNOSIS AND TREATMENT PEDIATRICS 24TH LATEST 1. Will develop at the same rate as his peers. 2. May have severe mental and physical challenges due to the frequent seizure activity. 3. May exhibit a slight cognitive delay as he grows. 4. Will grow out of having seizures. ANS: 2 Feedback 1. The continual seizure activity can cause hypoxia to the brain. 2. The frequency of the seizures causes hypoxia to the brain, increasing the
chance for mental and physical challenges. 3. The frequency of the seizures will increase the level of cognitive delays. 4.Because of the type of seizures, the child will not grow out of having seizures. 25. A child has been status epileptics for the last 20 minutes. The child has Depakote, Valporic Acid, and Diazepam gel ordered. The nurse should prepare which medication for administration at this time? 1. Depakote 2. Valporic acid 3. Diazepam 4. None of the medications. The child will stop on his own. ANS: 3 Feedback 1. The Depakote is needed on a regular, scheduled basis to help keep the
level adequate in the body. 2. Valporic acid needs to be given on a regular schedule to keep the
adequate levels in the body. 3. Diazepam can be used as needed to help stop the brain activity for seizures. 4. The seizure activity needs to be stopped because of the hypoxia that is occurring to the brain.
CURRENT DIAGNOSIS AND TREATMENT PEDIATRICS 24TH LATEST 26. Care for a child during status epilepticus should include all of the following except: 1. Turn the patient to the right side. 2. Loosen tight clothes. 3. Move toys out of the area to prevent injury. 4. Stay with the patient until the seizure has stopped. ANS: 1 Feedback 1. Turning the patient to the right side increases the risk for aspiration
because of the positioning of the bronchioles. 2. Loosening clothes helps the person move freely and reduces the chance
of injury. 3. Moving objects out of the area decreases the chance for injury as the
patient moves during the seizure. 4. It is important to make sure the patient stays safe.
27. The nurse is identifying the difference between primary headaches to secondary headaches. Secondary headaches can occur: 1. Because of stress. 2. In relation to low blood pressure. 3. Because of concussions. 4. Because of migraines. ANS: 3 Feedback 1. Stress is a primary cause for headaches. 2. Low blood pressure is a primary cause for headaches. 3. Concussions are a cause of secondary headaches because an injury has
previously occurred to the brain tissue. 4. Migraines are a primary cause for headaches.
CURRENT DIAGNOSIS AND TREATMENT PEDIATRICS 24TH LATEST 28. Cyclic vomiting may: 1. Last for days. 2. Require SSRIs to stop hurting. 3. Not be associated with a headache. 4. Requires pain medication and Zofran. ANS: 3 Feedback 1.Usually short lived 2.SSRIs are not an effective method of pain control for the vomiting. 3.The vomiting can occur for random reasons, but a headache is not a symptom. 4.Pain medication is not usually required to stop the vomiting. 29. A child that has rhythmic, repetitive, involuntary movements is exhibiting: 1. Tremors. 2. Dystonia. 3. Contractures. 4. Tics. ANS: 2 Feedback 1.Tremors are involuntary and have random movements. 2.Twisting and repetitive, involuntary movements are common with dystonia. 3. Contractures can be a permanent placement of the body because of muscle and ligament rigidity. 4. Tics are not rhythmic in nature. 30. Identify a therapeutic management technique for a child with a tic disorder.
CURRENT DIAGNOSIS AND TREATMENT PEDIATRICS 24TH LATEST 1. Behavioral modification to suppress the tics 2. Administer anti-psychotic medications to reduce the tics 3. Education and support for the child and the family 4. Genetic counseling for the family ANS: 3 Feedback 1.Behavior modification does not aid in stopping tics form occurring. 2.Tics do not respond to antic-psychotic medications. 3. Support and education are important so that people understand that tics are involuntary. 4. There is little research to prove that tics are genetic in nature. 31. Identify a true statement about Tourettes Syndrome (TS) is that: 1. Manifestations rarely change once developed. 2. Children with TS do not have obsessive compulsive disorders. 3. The tics of TS can lead to mental deterioration. 4. The tics are involuntary, and the person cannot control the behavior. ANS: 4 Feedback 1. Manifestations change related to the stress level and various other
factors. 2. There is a strong correlation between TS and obsessive compulsive
disorders. 3. The tics do not affect the cognitive ability of a child. 4. The tics are involuntary, and public education about this is important so
that the child is not harassed about the behaviors. 32. The assessment a nurse performed on a 12-year-old boy demonstrated a positive Kernigs sign and a Brudzinskis sign. Identify the priority for the nurses next action.
CURRENT DIAGNOSIS AND TREATMENT PEDIATRICS 24TH LATEST 1. Document the findings and note as normal. 2. Further assess the neurological function of the child and call the doctor with a report. 3. Explain to the patient that the assessment was abnormal and there is no a cause for concern. 4. Prepare the child for a lumbar puncture. ANS: 2 Feedback 1.These findings are abnormal and need further neurological testing. 2.Further assessment is needed because these signs should not be present in a 12-year-old child. 3.Explaining the situation to the patient is important, but there is s possible neurological issue that needs to be addressed with further diagnostic testing. 4.A lumbar puncture is invasive, and other tests should be done before the procedure. 33. Results from cerebrospinal fluid that was tested for meningitis have been received by the nurse. The results indicate bacterial meningitis. The nurse knows this because the results show: 1. A low protein count and a low glucose count. 2. A low red blood cell count. 3. An elevated protein count and a low glucose level. 4. A normal protein count and a high glucose count. ANS: 3 Feedback 1. Does not indicate infection 2. Some red blood cells may show in the specimen if the lumbar puncture
was not a clean catch. 3. Results indicate bacterial meningitis. 4.Results indicate viral meningitis. 34. Identify the false statement about bacterial meningitis.
CURRENT DIAGNOSIS AND TREATMENT PEDIATRICS 24TH LATEST 1. Bacterial meningitis can be fatal if not treated. 2. Bacterial meningitis can spread quickly. 3. Bacterial meningitis cannot be effectively treated with antibiotics. 4. Bacterial meningitis can cause hearing loss in children. ANS: 3 Feedback 1. The illness can cause death if not treated. 2. The illness can spread quickly and be fatal without treatment.
3.Antibiotic therapy can stop the progression of the illness. 4.Because of the bacteria, the illness attacks the ear drum, creating hearing loss in children if not treated early. 35. The nurse is assessing a 6-month-old boy. Which of the following would be an abnormal finding, indicating possible cerebral palsy? 1. The infant can pull to a sitting position while holding onto an adults hand. 2. The infant does not exhibit a Moro reflex. 3. The infant does not exhibit a Babinskis reflex. 4. The infant has an obligatory tonic neck flexion. ANS: 4 Feedback 1.A 6-month-old should be able to pull up with aid from another person. 2.A Moro reflex should not be present after the newborn period. 3.The Babinskis reflex should not be present after the first few weeks of life. 4.A tonic neck flexion can indicate neurological damage because this is not a normal position for an infant. 36. A multidisciplinary meeting is being conducted for a 4-year-old boy with cerebral palsy. A goal for managing this childs condition would be:
CURRENT DIAGNOSIS AND TREATMENT PEDIATRICS 24TH LATEST 1. Assistance with motor control of voluntary muscles. 2. Maximizing the childs capabilities. 3. Surgically correcting deformities. 4. Waiting to place the child in school. ANS: 2 Feedback 1. It is important to have the child be as independent as possible to
maintain optimum function. 2. Concentrating on the capabilities can help the child modify other areas
of weakness to have a better quality of life. 3. Surgery is used only in extreme conditions. 4. Planning for school will be important, but the priority is to maximize the
capabilities so the child is as independent in school as possible. 37. A common trait of Beckers Muscular Dystrophy is: 1. Progressive weakness in the trunk and arms over time. 2. A quick rate of deterioration of the body. 3. Cardiomyopathy. 4. Usually diagnosed by the age of 3. ANS: 3 Feedback 1.This type of dystrophy has weakening in the legs and pelvis areas. 2.This particular type of muscular dystrophy has a longer life expectancy than most other types of muscular dystrophy. 3.Cardiomyopathy occurs because of abnormality in the protein dystrophin in the body. 4.First diagnosis usually does not occur until between the ages 5 and 15. 38. Brian, a 4-year-old boy, is demonstrating the Gowers sign, and his mother is wondering why her child is making this movement. The child is doing this because:
CURRENT DIAGNOSIS AND TREATMENT PEDIATRICS 24TH LATEST 1. The weakness of his arms requires his legs to do more work. 2. The weakness in his hips and thighs requires help from his arms to stand. 3. Weakening trunk and back muscles require the legs and arms to help keep an upright position. 4. Weakening of the trunk requires this movement to help breath. ANS: 2 Feedback 1. The weakness is in the hips and thighs, not the arms. 2. The weakness in the hips and thighs makes it difficult to stand, thus
requiring the arms to help provide stability. 3. The trunk and back muscles are weak, but are not the reason for the Gowers sign. 4. The Gowers sign is seen when attempting to stand. 39. When assessing a 10-year-old child with myasthenia gravis, the nurse notes ptosis and drooping facial expressions. The nurse knows this disease will require all of the following except: 1. Supportive care, as there is no cure for the disease. 2. Administering beta blockers to improve the muscle tone. 3. Check the child for a depressive state due to body image issues. 4. Explain procedures to the child as needed and provide emotional support. ANS: 2 Feedback 1. The lack of a cure will require education and support for the family and
patient. 2. Administration of cholinesterase inhibitors is the common drug used to
help keep the acetylcholine receptors from being blocked. 3. Because of the physical changes, an assessment of emotional and psychosocial issues is important. 4. Education and explanations will allow the child to feel involved in his/her care.
CURRENT DIAGNOSIS AND TREATMENT PEDIATRICS 24TH LATEST 40. Infantile spinal muscular atrophy contains all of the following characteristics except: 1. Muscle wasting of voluntary muscles. 2. Type 1 can begin in utero. 3. Inability to suck occurs early in life. 4. It is associated with children who are intellectually slower. ANS: 4 Feedback 1.Muscle wasting is noted in these dystrophies. 2.Type 1 may start with the fetus. 3. Suckling is the strongest at the earliest points of life. As time progresses, the muscle weakens, making feeding difficult. 4. Children with the disease show cognitive delays. 41. What should the nurse anticipate when reviewing laboratory results of a patient with Guillain-Barre syndrome? 1. Elevated CBC 2. High protein in a cerebral spinal fluid tap 3. Creatinine phosphokinase elevated 4. Sensory nerve conduction time increased ANS: 2 Feedback 1. The CBC should be within normal ranges. 2. The high protein in the cerebral spinal fluid is because of the
inflammation to the area. 3. The laboratory results should be within normal limits. 4.The nerve conduction time is decreased. 42. A common treatment for a person with Guillain-Barre syndrome is:
CURRENT DIAGNOSIS AND TREATMENT PEDIATRICS 24TH LATEST 1. Broad spectrum antibiotics. 2. Intravenous gamma globulins. 3. Antihistamines. 4. Acyclovir. ANS: 2 Feedback 1.Broad spectrum antibiotics are not an effective treatment for the disease. 2.IGg is given to help the body naturally fight the syndrome. 3.Antihistamines are not an effective treatment for the disease. 4.Acyclovir will not treat the illness. 43. After a seizure, a 2-year-old boy would exhibit what type of reaction with the Babinskis reflex? 1. Positive 2. Negative 3. Will be positive one time and negative the next 4. Not a reliable test at this age ANS: 4 Feedback 1.A Babinskis reflex is not reliable at this age. It should only be done with infants. 2.A Babinskis reflex is not reliable at this age. It should only be done with infants. 3. There will not be accurate results 4. Because of the childs age, the results will not be accurate. 44. A teenage boy has received a concussion while playing hockey. A cardinal sign of a concussion is:
CURRENT DIAGNOSIS AND TREATMENT PEDIATRICS 24TH LATEST 1. Confusion. 2. Altered level of consciousness. 3. Loss of consciousness. 4. Fainting. ANS: 3 Feedback 1.Confusion is not a cardinal sign, but may be present. 2.A change in the level of consciousness is not considered a cardinal sign. 3.A loss of consciousness is a cardinal sign for a concussion, and the child should be examined by a professional. 4.Fainting rarely occurs with a concussion. 45. Tiagabine must be monitored when given to teens because: 1. It is less effective during puberty. 2. It has a high incidence of suicidal tendencies in teens. 3. It needs to be titrated with the teens growth pattern. 4. It is known to be sold as a street drug. ANS: 2 Feedback 1. The drug can be effective during puberty. 2. The high level of suicidal rates makes monitoring the teens behavior a
priority. 3. Titration to the growth is not a priority at this time.
4.The medication is not a common street drug.
Chapter 26. Orthopedics 1. A mother brings into the emergency department a frail-looking infant with blue sclera and arm and leg deformities. The mom said she was changing her diaper and heard a snap. She put her
CURRENT DIAGNOSIS AND TREATMENT PEDIATRICS 24TH LATEST shirt on and she heard another snap. She says that the baby has been crying, but eating a little. Mom reports a clavicle fracture in the delivery room, but the doctors told her it was nothing and would heal fine. The nurse notes a small lump on her right clavicle. It is not painful to the touch. The infants left forearm and right femur appear swollen and deformed. Based on your clinical knowledge of birth defects, your best answer to the mother would be: 1. We will call the doctor to do a skeletal survey. 2. We will call the Child Protective Services team to rule out abuse. 3. We will call Genetics to see if any genetic birth defects run in your family. 4. All of the above. ANS: 4 Feedback 1. Often, children with osteogenesis imperfect (OI) are misdiagnosed; therefore, all children should undergo steps to rule out abuse, then proceed with genetic counseling, testing to verify a genetic disorder. The blue sclera and fractures without severe trauma are signs of OI. 2. Often, children with osteogenesis imperfect (OI) are misdiagnosed; therefore, all children should undergo steps to rule out abuse. 3. Often, children with osteogenesis imperfect (OI) are misdiagnosed; therefore, all children should undergo steps to rule out abuse, then proceed with genetic counseling, testing to verify a genetic disorder. 4. All of the responses are correct. Often, children with osteogenesis imperfect (OI) are misdiagnosed; therefore, all children should undergo steps to rule out abuse, then proceed with genetic counseling, testing to verify a genetic disorder. The blue sclera and fractures without severe trauma are signs of OI. 2. A 13-year-old boy, who has played baseball for three years, comes in complaining of left elbow pain and swelling. He is a pitcher. He plays baseball all year. X-rays reveal medial epicondyle irritation. There is no avulsion fracture. Based on these findings, you explain to the child that he can: 1. Swim. Its a safer sport. 2. Resume baseball gradually with pain-free motion.
CURRENT DIAGNOSIS AND TREATMENT PEDIATRICS 24TH LATEST 3. Change positions in baseball for the season. 4. Protect, rest, ice, compress, and elevate the injury as well as stop baseball for three weeks. ANS: 4 Feedback 1.The repetitive motion of swimming can cause the same irritation. 2.Sports injuries are common in adolescents, especially overuse injuries. Open growth plates are locations for overuse injuries. Little Leaguers elbow is characterized by pain, swelling, and repetitive motions. 3. The child may not have an interest in other positions. A modification to his pitching may need to be made. 4. Sports injuries are common in adolescents, especially overuse injuries. Open growth plates are locations for overuse injuries. Little Leaguers elbow is characterized by pain, swelling, and repetitive motions. 3. A 16-year-old soccer player comes into the ER with left knee effusion and extreme pain. The child claims that while kicking the ball away from an opponent, she felt a pop and fell to the ground. She had to be assisted off the field. She is in the playoffs and wants to play again. X-rays are normal, but as the nurse, you know which of the following about sports injuries? 1. X-rays can help classify fractures, but cannot classify tissue, tendons, and ligaments. 2. She probably has an ACL tear based on the symptoms of swelling, pain, inability to bear weight, and hearing a pop before she went down. 3. She will need an MRI. 4. She will need crutches and a knee immobilizer as well as to rest, ice, compress, and elevate the injury. ANS: 4 Feedback 1.The X-ray can see some damage to the ligaments and swollen areas. 2.The pop does not classify the injury. 3.A CT scan would help identify the injured area. 4.Care of acute sports-related injuries include PRICE: protect from further injury, rest, ice, compression, and elevation to relieve pain.
CURRENT DIAGNOSIS AND TREATMENT PEDIATRICS 24TH LATEST 4. A 21-one-year old basketball player comes to the emergency department with a swollen, ecchymotic, painful right ankle. He states he went up for a jump shot and twisted it. He reports pain over the medial malleolus. He cannot walk on it and was assisted off the court at the game. He reports that he has done this before, but it is different this time. He wants an air cast splint and crutches, but no x-rays. What statement indicates that this young athlete may not understand the difference between a fracture and a sprain? 1. I will be fine in 3 days. 2. I can play if you give me the splint. 3. I dont need x-rays today. Ive done this before. 4. All of the above. ANS: 4 Feedback 1.Sprains, strains, and fractures can all have the same symptoms of pain, swelling, inability to bear weight, and ecchymosis that may last a few days. 2.An x-ray will assist with fracture detection at this time. 3.An x-ray will assist with fracture detection. 4. Sprains, strains, and fractures can all have the same symptoms of pain, swelling, inability to bear weight, and ecchymosis. An x-ray will assist with fracture detection. 5. A 5-year-old boy presents to the emergency department with a history of a motor vehicle
accident. He was restrained in the backseat, and the car was hit head on. He has a swollen and deformed left thigh. He is in extreme pain. He has frequent muscle spasms. X-rays reveal a midshaft femur fracture. As you prepare the child prior to surgery, you will assist with anticipatory guidance as well as show and discuss a spica cast. What information will be helpful to this young boy? 1. A hard cast from your waist to your toes will protect your leg so it can heal. 2. We can put a cast like yours on your bear.
CURRENT DIAGNOSIS AND TREATMENT PEDIATRICS 24TH LATEST 3. The IV will give you medicine to make you sleep, wake up, and help with the pain. 4. All of the above. ANS: 4 Feedback 1. Clear explanations in easy to understand language for this age group will help this child cope with new and uncomfortable experiences. Using hands-on demonstrations and familiar play things assist with decreasing anxiety about the unknown. 2. Placing a cast on a stuffed animal may help decrease the anxiety of the patient. 3. Using hands-on demonstrations and familiar play things assist with decreasing anxiety about the unknown. 4. Clear explanations in easy to understand language for this age group will help this child cope with new and uncomfortable experiences. Using hands-on demonstrations and familiar play things assist with decreasing anxiety about the unknown. 6. An 11-year-old boy with a tibia shaft fracture presents for his first follow-up visit after an injury. He is placed into a non-weight bearing, long leg cast with instructions about cast care. Your knowledge about cast care reveals that you should include which of the following statements in your instructions? 1. Keep the cast clean and dry, do not insert objects into the cast, and call the doctor if the cast becomes loose, cracked, or ill fitting. 2. You may take a shower in the cast, just dont put weight on it. 3. Its okay to put coins down the cast. 4. Do not call the office for a foul smell or skin irritation. ANS: 1 1. The proper teaching for care of the cast is to keep the cast clean and dry,
follow weight-bearing instructions, do not place objects in the cast or use anything to scratch inside the cast, call the office if problems with the cast, like pain, foul smell, skin irritation, cracking, loosening etc., occur.
CURRENT DIAGNOSIS AND TREATMENT PEDIATRICS 24TH LATEST 2. The proper teaching for care of the cast is to keep the cast clean and dry.
3.Do not place objects in the cast or use anything to scratch inside the cast. 4.Call the office if problems with the cast like pain, foul smell, skin irritation, cracking, loosening etc., occur. 7. A 2-week-old infant with Developmental Dislocated Hips (DDH) is currently being treated in a Pavlick harness. The mom calls and is concerned that the child is not kicking like before and seems to be in pain. She has a history of colic. Although the harness seemed to fit well in clinic last week, you are concerned. Therefore, the proper statement to the mother would be: 1. Try some Tylenol and call her pediatrician. 2. Remove the harness and we will see you in clinic ASAP. 3. Go to the nearest emergency department to have the child seen. 4. Remove the harness and call for an appointment in three months. ANS: 2 Feedback 1.When in doubt, removing the harness is the least invasive intervention. 2.Hip subluxation or dislocation is not painful. The anxiety of a new parent with a new diagnosis is high. Reassure the mother everything will be okay and that more education and reevaluation will be warranted. Sometimes the next step is casting if the child or the family fail harness treatment. Following up with the primary care physician would be okay to rule out other reasons for pain, like her abdominal issues, ear infection, GERD, etc. When in doubt, removing the harness is the least invasive intervention. 3. Hip subluxation or dislocation is not painful. The anxiety of a new parent with a new diagnosis is high. When in doubt, removing the harness is the least invasive intervention. 4. Hip subluxation or dislocation is not painful. The anxiety of a new parent with a new diagnosis is high. When in doubt, removing the harness is the least invasive intervention. 8. A 2-year-old with Downs syndrome presents to the clinic with a chief complaint of unsteady walking. The physical therapist wants to use orthotics to help with ambulation. The main uses of assistive aids in neuromuscular conditions are:
CURRENT DIAGNOSIS AND TREATMENT PEDIATRICS 24TH LATEST 1. To prevent deformity and improve function. 2. To stop mobility and use a stroller. 3. To cover their feet and keep them warm. 4. To relax muscles in a child with ligament laxity. ANS: 1 Feedback 1. Hallmark uses of assistive devices in neuromuscular conditions are to
improve function and prevent further deformity. 2. The goal is to maintain mobility as long as possible. 3. Attempting to maintain adequate mobility is the most important factor.
4.The muscles are too relaxed, and strength needs to be built. 9. A 7-year-old is having on and off knee, ankle, wrist, and hip pain. The family history is positive for rheumatoid arthritis. No injury has taken place, and the child has occasional swelling of joints. Pain resolves with rest, ice, and NSAIDs. The primary care physician ran labs with positive rheumatoid factor ANA and ESR. This child is a soccer player and a violinist. Your differential diagnosis would be: 1. Juvenile Idiopathic Arthritis (JIA), septic arthritis, osteomyelitis, and fracture. 2. Juvenile Idiopathic Arthritis (JIA), toxic synovitis, osteomyelitis, and Developmental Dislocated Hips (DDH). 3. Juvenile Idiopathic Arthritis (JIA), chronic inflammatory arthritis, chronic osteomyelitis, and Developmental Dislocated Hips (DDH). 4. Juvenile Idiopathic Arthritis (JIA) only. ANS: 4 Feedback 1. Juvenile Idiopathic Arthritis (JIA) is characterized by a long-standing
history of pain and swelling to more than one joint. This can be hereditary. Lab oratory tests are normal in chronic osteo, toxic synovitis, and Developmental Dislocated Hips (DDH).
CURRENT DIAGNOSIS AND TREATMENT PEDIATRICS 24TH LATEST 2. All are factors in Juvenile Idiopathic Arthritis. 3. Juvenile Idiopathic Arthritis (JIA) is characterized by a long-standing
history of pain and swelling to more than one joint. This can be hereditary. Laboratory tests are normal in chronic osteo, toxic synovitis, and Developmental Dislocated Hips (DDH). 4. Juvenile Idiopathic Arthritis (JIA) is characterized by a long-standing history of pain and swelling to more than one joint. It can be hereditary. Laboratory tests are normal in chronic osteo, toxic synovitis, and Developmental Dislocated Hips (DDH). 10. If a child is diagnosed with Juvenile Idiopathic Arthritis (JIA), which of the following statements is true? 1. Usually, a child outgrows the disorder. 2. NSAIDs are the treatment of choice. 3. The child should not participate in sports. 4. All of the children of these parents will have the disorder. ANS: 2 Feedback 1. The disorder lasts for the life of the child. 2. Activity helps the joints stay mobile, and anti-inflammatory medications are the treatment of choice for Juvenile Idiopathic Arthritis. 3. Participation in sports will help maintain mobility. 4. Activity helps the joints stay mobile, and anti-inflammatory medications are the treatment of choice for Juvenile Idiopathic Arthritis. 11. General cast care considerations would include which of the following? 1. Keep cast clean and dry, monitor integrity of cast materials, and elevate the involved extremity for 24 to 48 hours as recommended. Monitor neurovascular status per doctors orders and shower as instructed. 2. Allow the cast to get wet and follow up in 4 to 6 weeks for cast removal. 3. Spray the cast with Febreze if a foul smell occurs.
CURRENT DIAGNOSIS AND TREATMENT PEDIATRICS 24TH LATEST 4. All of the above ANS: 1 Feedback 1. Standard care of casts includes: keeping the cast clean and dry at all
times, elevation within 24 to 48 hours to prevent increased swelling and pain, and neurovascular checks are important to alert the provider of problems that need immediate medical attention. Casts should not become cracked, broken, or cause rubbing. A foul smell may be benign or a sign of infection. 2. Standard care of casts include keeping the cast clean and dry at all times. 3.A foul smell may be benign or a sign of infection. 4.Only one answer is correct. 12. A 12-year-old female is seen in the clinic for a chief complaint of in-toeing and being teased at school for the last six months. She had been seen as a toddler who W sat, and the doctor told her parents that she would outgrow this. However, she is still in-toeing a lot. The mom is concerned that she will grow up and not be able to wear high heels or walk down the stairs without tripping and falling. Your knowledge about femoral anteversion leads you to state which of the following? 1. She will outgrow this by the time she is 20 years old. 2. She will not outgrow this now. It usually resolves by the time children are 8 or 9 years of age, so only surgery will correct this now. 3. She will never be able to play running sports. 4. She should try gymnastics since she is so flexible. ANS: 2 Feedback 1.The child should outgrow this by the time they are 8 or 9 years old. 2.Hallmark signs of femoral anteversion are unresolved in-toeing, ligament laxity, history of tripping and falling, and W-sitting. 3.She will be able to run and play sports. 4.She will be able to participate in sports.
CURRENT DIAGNOSIS AND TREATMENT PEDIATRICS 24TH LATEST 13. A 3-year-old male presents to the clinic, post-hospital discharge, for osteomyelitis. The mother states that he just stopped walking on his left leg for no reason, he had a high fever, and was in extreme pain. We went to the emergency room and laboratory tests and an MRI were done. He is on IV antibiotics, but I think we should stop the medicine now. He is much better. Your understanding of the care and treatment of osteomyelitis enables you to respond with which of the following? 1. Typically speaking, depending on the organism being treated, most patients with osteomyelitis are treated with 6 to 8 weeks of IV antibiotics minimum, followed by a course of oral antibiotics directed by the infection control team. 2. Sure, you can stop the antibiotics anytime. I am glad he is much better. 3. If you stop the antibiotics now, your child will die. 4. None of the above responses are correct. ANS: 1 Feedback 1. Osteomyelitis is diagnosed by laboratory values and diagnostic imaging.
It typically will not resolve without acute care, including IV antibiotics, pain medication, and rest/immobilization. 2. It typically will not resolve without acute care, including IV antibiotics, pain medication, and rest/immobilization. 3. The infection can spread and cause the patient to lose a limb. 4.Osteomyelitis is diagnosed by laboratory values and diagnostic imaging. It typically will not resolve without acute care, including IV antibiotics, pain medication, and rest/immobilization. 14. A 1-year-old toddler presents to the outpatient clinic with the parents concerned about their child being pigeon-toed. The grandparents state a positive family history of clubfoot. The family is sure its clubfoot, and that the child is going to be deformed and disabled now. Upon physical examination, the child would have which of the following characteristics? 1. Fixed deformity in equinus, tight heel cords, inability to place foot flat on the ground, or walking on supinated feet 2. Tight hamstrings and walking on tiptoes
CURRENT DIAGNOSIS AND TREATMENT PEDIATRICS 24TH LATEST 3. Forefoot abduction and heel in neutral alignment 4. Inward rotated tibia shafts ANS: 1 Feedback 1. Clubfoot is a rigid, fixed deformity that cannot be corrected by manipulation alone. It needs to be treated in the newborn phase with Ponseti technique. Failed conservative treatment in older infant/toddler results in tight heel cords and supination of the foot. 2. Failed conservative treatment in older infants/toddlers results in tight heelcords and supination of foot. 3. The heel will not have neutral alignment. 4.The tibial shafts will not be rotated inward. 15. A 1-year-old toddler presents to the outpatient clinic with parents concerned about pigeontoes. The grandparents state a positive family history of clubfoot. The family is sure its clubfoot, and that the child is going to be deformed and disabled now. What is the likely diagnosis? 1. Metatarsus Adductus 2. Tibial Torsion 3. Femoral Anteversion 4. Blounts Disease ANS: 1 Feedback 1. Metatarsus Adductus typically presents with the chief complaint of in-
toeing, yet the child can put his/her foot flat on the floor and the forefoot is turned inward. This is often considered a packaging disorder, but it is easily corrected with stretching and placing shoes on the opposite feet. 2. The tibia is not affected. 3.The femur is not involved. 4.Blounts disease is not demonstrated in this patients history.
CURRENT DIAGNOSIS AND TREATMENT PEDIATRICS 24TH LATEST 16. A 5-year-old female falls off the swings at the school playground. Her elbow is swollen and painful. She is seen in the ER and goes to the OR for a supracondylar elbow pinning. The parents are upset and think she will not be able to do gymnastics again. You respond: 1. This is a common type of fracture for this age group. The children recover really well and often have no long-term side effects to the treatment. 2. She will be casted for three weeks, and then the pins will be removed. She should work on range of motion exercises after the cast is removed. She can do gymnastics again in about 4 to 6 weeks. 3. Her doctor will evaluate her progress with X-rays and let you know when she is ready for activities. 4. All the above ANS: 4 Feedback 1. Many children recover fully and can resume all activities without any
complications. 2. Following the plan of care will help the child gain strength and mobility
in the arm. 3. Supracondylar humerus fractures are very common in the pediatric
setting. Many children recover fully and can resume all activities without any complications. 4. All of the responses are correct. Supracondylar humerus fractures are very common in the pediatric setting. Many children recover fully and can resume all activities without any complications. 17. A mother with a 12-month-old male African American child is in the clinic to discuss his bowed legs. She states that the childs father and grandfather both have it, and it was never treated. She doesnt want him to look funny. With your knowledge of infantile Blounts disease, you know that which of the following is true? 1. Infants that are early walkers are more prone to infantile Blounts disease. 2. Typically, hereditary factors are involved.
CURRENT DIAGNOSIS AND TREATMENT PEDIATRICS 24TH LATEST 3. Long-leg, cumbersome braces are well tolerated. 4. Blounts disease should not be treated until after age 4. ANS: 1 Feedback 1. Risk factors for infantile Blounts disease include early walkers and heavier children. 2. Heredity is not a factor. 3. Long-leg braces are rarely used, and other forms of therapy are recommended. 4. Treatment occurs early in life to achieve the best possible outcome. 18. A 14-year-old male hockey player presents to the clinic with a right clavicle fracture. The mother reports that he doesnt want to keep still and wear the splint. Your response should be: 1. We can try a sling/swathe or a figure 8 splint. Splints are mostly for comfort. 2. It takes about 2 to 3 weeks for the bones to become sticky, then the pain decreases. He should avoid contact sports for about 6 to 8 weeks. 3. Diversion activities are good to try, like movies or video games. 4. All of the above are correct. ANS: 4 Feedback 1.A sling provides supportive care. The patient and parents must also be reminded that the child should not do too much while bone is healing. 2.The pain will decrease in this time span, but still must be protected. 3.Diversional activities will help reduce pain. 4.Clavicle fractures are common in contact sports and heal well unless really translated or comminuted. A sling provides supportive care. The patient and parents must also be reminded that the child should not do too much while bone is healing. 19. A 4-year-old female status post left supracondylar elbow pinning presents to the clinic with a chief complaint of increased swelling to the fingers, a change in color from pink to pale, numbness and tingling, and increased pain from 24 hours ago. Her fiberglass cast was applied
CURRENT DIAGNOSIS AND TREATMENT PEDIATRICS 24TH LATEST immediately after surgery. The mother is concerned and anxious. With your knowledge about neurovascular status post surgery and fracture care, you are concerned about which of the following conditions? 1. Compartment syndrome 2. Refracture 3. Nothing. These are normal findings. 4. The mom is crazy and needs a dose of valium. ANS: 1 Feedback 1. Classic signs of compartment syndrome are the 5 Ps: Pain, Pulse, Pallor,
Parasthesia, and Paralysis. 2. Refracture will not need to occur at this time.
3.These are classic signs of compartmental syndrome. 4.The mother is responding appropriately to her childs needs. 20. A 15-year-old basketball player presents to the clinic with a history of colliding with another player at yesterdays game. He was planted, and his right knee went out after the collision. He has pain and swelling to his knee. He is using crutches and an ace wrap. His mother is concerned and thinks he wont play basketball again. The emergency department told the mother that the x-rays were normal. What other treatment modalities may assist this young athlete? 1. Protect, rest, ice, compression, and elevation (PRICE) 2. A knee Immobilizer 3. Non-steroidal, anti-inflammatory drugs (NSAIDs) 4. All of the above ANS: 4 Feedback
CURRENT DIAGNOSIS AND TREATMENT PEDIATRICS 24TH LATEST 1.Sports injuries are treated initially with PRICE. NSAIDs are antiinflammatories, which would decrease the pain and swelling. 2.A knee immobilizer would protect the injured knee and provide easier ambulation to decrease pain. 3. NSAIDs are anti-inflammatories, which would decrease the pain and swelling. 4. Sports injuries are treated initially with PRICE. NSAIDs are antiinflammatories, which would decrease the pain and swelling. A knee immobilizer would protect the injured knee and provide easier ambulation to decrease pain. 21. A 2-year-old male presents to clinic with his caregiver stating that he slipped and fell running on the hardwood floor with the dog. He cried immediately and refused to bear weight on his left side. X-rays were taken in the emergency room of his hip, knee, and ankle. The report was negative. The caregiver states that he is still uncomfortable and is not walking on it. Laboratory tests were done and came back negative for elevated ESR, CRP, and CBC. Upon physical examination, he has pain and immediately withdraws his leg on palpation of his anterior tibia shaft. He says ouchie and no hurt my leg. Your knowledge and the exam would lead you to diagnose the child with which of the following conditions? 1. Juvenile Idiopathic Arthritis 2. Toddlers Fracture 3. Hip Dysplasia 4. Sprained ankle ANS: 2 Feedback 1.A toddlers fracture is not typically visible on initial X-rays, but point tenderness on the tibia shaft and refusal to bear weight would be common in this age group following a witnessed injury. 2.A toddlers fracture is not typically visible on initial X-rays, but point tenderness on the tibia shaft and refusal to bear weight would be common in this age group following a witnessed injury. 3.The hip dysplasia would be notable with one hip higher than the other or a click when examined.
CURRENT DIAGNOSIS AND TREATMENT PEDIATRICS 24TH LATEST 4.A toddlers fracture is not typically visible on initial X-rays, but point tenderness on the tibia shaft and refusal to bear weight would be common in this age group following a witnessed injury. 22. If an immature child presents to the clinic with point tenderness over the distal physis of the wrist or ankle, but negative fracture by X-ray report or observation of films, your knowledge of the Salter-Harris fracture classification would identify which of the following fractures? 1. Salter Harris I 2. Salter Harris II 3. Salter Harris III 4. Salter Harris IV ANS: 1 1. Salter Harris I fracture classification describes injury through the growth
plate in a skeletally immature child. This is not visible via radiographs, but pain and tenderness over the growth plate would be the best clinical exam. 2. Salter Harris I fracture classification describes injury through the growth plate in a skeletally immature child. 3. Salter Harris I fracture classification describes injury through the growth plate in a skeletally immature child. 4. Salter Harris I fracture classification describes injury through the growth plate in a skeletally immature child. 23. A 10-year-old female gymnast fell off the balance beam, landing on her forearm. She has swelling, pain, and deformity. What diagnostic test would be the most helpful for the physician? 1. Rheumatoid laboratory tests 2. Radiographs of her forearm 3. MRI 4. CT Scan
CURRENT DIAGNOSIS AND TREATMENT PEDIATRICS 24TH LATEST ANS: 2 Feedback 1. Rheumatoid laboratory tests are not needed at this time because this was an acute injury. 2. Diagnosis of fractures includes not only a clinical exam, but also a radiographic examination. MRI and CT scans are more helpful in ligament tears or other soft tissue evaluations. 3. MRI scans are more helpful in ligament tears or other soft tissue evaluations. 4. CT scans are more helpful in ligament tears or other soft tissue evaluations. 24. When offering treatment options for a minor fracture (buckle fracture) care to a child, especially an athlete that wants to continue playing, what are the best recommendations for immobilization? 1. Do not immobilize the fracture. 2. Removable splint 3. Fiberglass cast 4. Ace bandage ANS: 3 Feedback 1.Active athletes can potentially continue sports participation if immobilization in a well-padded cast is allowed by game rules/regulations. 2.A removable splint typically has metal stays in it that would injure another player and are not permitted. 3.A fiberglass cast will allow for immobilization and has less chance of harming other players. 4.An ace bandage, ice, and NSAIDs are treatment modalities better served for initial care to reduce pain and swelling, but not for fracture care. 25. John, a 12-year-old with a fracture of the femur, has developed chest pain and shortness of breath. The nurse suspects a pulmonary embolism. The priority nursing action is to: 1. Elevate the affected extremity.
CURRENT DIAGNOSIS AND TREATMENT PEDIATRICS 24TH LATEST 2. Administer oxygen. 3. Administer pain medication. 4. Start an IV infusion of heparin. ANS: 2 Feedback 1. Elevating may make more clots move and increase the risk for more pulmonary emboli. 2. The child needs oxygen to keep the alveoli open within the lungs to prevent respiratory collapse. 3. Pain medication may decrease the respiratory rate and does not aid in decreasing the shortness of breath. 4. The IV heparin needs to be ordered by the doctor. 26. A family has received the diagnosis of Osteogenesis Imperfecta for their 2-month-old daughter. The parents are asking several questions to the nurse. The nurse knows the child is a Type 1 classification because of all but one of the following characteristics. 1. Blue sclera 2. Normal dentition of teeth 3. Type 1 is responsible for two-thirds of all OI diagnosis. 4. Type 1 classification will be lethal within a few weeks. ANS: 4 Feedback 1. The sclera is commonly blue with this diagnosis. 2. Type 2 has the abnormal dentition. 3. The majority of diagnoses are type 1. 4. Type 2 is lethal in a shorter period of time than type 1. 27. A 9-year-old patient is complaining of skin irritation from the edges of a cast that was applied the previous day. The nurse should take which of the following actions?
CURRENT DIAGNOSIS AND TREATMENT PEDIATRICS 24TH LATEST 1. Petal the cast edges with adhesive tape. 2. Massage the skin at the rim of the cast. 3. File the edges until smooth. 4. Apply more casting material to make the edges softer. ANS: 1 Feedback 1. Petaling will make the edges of the cast soft and help decrease the
irritation to the skin. 2. Massaging the rim may stretch or break the casting. 3. Filing the edges will cause dust to lodge inside the cast, creating a
potential for skin breakdown. 4. More casting materials will not stop the roughness of the cast. It will
create more. 28. A nurse in the pediatric orthopedic clinic is giving discharge teaching to the parents of a child with scoliosis on how to use a brace. Which statement by the parent would indicate the need for further education? 1. I will encourage my child to practice the physical therapy exercises. 2. I will have my child wear soft clothing under her brace to prevent skin breakdown. 3. I should apply lotion under the brace to help prevent skin breakdown. 4. I should not use powder under the brace because it can cause a buildup and be irritating. ANS: 3 Feedback 1. Physical therapy will help maintain optimum health for the child. 2. Soft clothing will help reduce irritation with the brace. 3. Lotion causes moisture to build up and create more skin damage. 4. Powder buildup can be irritating to the skin with a brace.
CURRENT DIAGNOSIS AND TREATMENT PEDIATRICS 24TH LATEST 29. The development of the musculoskeletal system occurs during fetal development. Originally, bones begin as: 1. Cartilage. 2. Osteoclasts. 3. Osteoblasts. 4. Tendons. ANS: 1 Feedback 1. Cartilage is the first step in bone development in utero. 2. Osteoclasts are part of the bone formation. 3. Osteoblasts are the cells that form the bone. 4. Tendons are developed toward the end of fetal development. 30. During fetal development, several factors can affect the outcome of bone development. All of the following influence the development except: 1. Fifths disease. 2. Exposure to cat feces. 3. Caffeine consumption. 4. Contaminated water. ANS: 3 Feedback 1. The disease is known to cause bone deformities if the mother has contracted the illness while pregnant. 2. Toxoplasmosis is found in cat feces and can infect the mother, thus causing deformities in the fetus. 3. No research supports bone issues with bone development. 4.Contaminated water can have bacteria, fungi, and viruses that attach to the fetus, causing bone deformities.
CURRENT DIAGNOSIS AND TREATMENT PEDIATRICS 24TH LATEST 31. A student nurse is performing a musculoskeletal assessment on a 2 year old. When doing the assessment, it is important for the student nurse to start with: 1. Palpating the bony prominences of the body. 2. Participating in a range of motion exercises. 3. Participating in weight-bearing exercises. 4. Inspecting the body visually. ANS: 4 Feedback 1. Palpation should be done last because it is the most invasive for the
child. 2. Range of motion should be done after inspection.
3.Participation should occur after inspection. 4.Starting with the least invasive assessment first will allow the nurse to build trust with the child. 32. The school nurse has been called to the playground for an emergency. When she arrives, she notes that a young girl is laying on the ground, holding her arm with a bone fragment protruding from her skin. What is the priority assessment at this time? 1. Neurovascular check 2. Elevation of the extremity 3. Providing ice 4. Pain management ANS: 1 Feedback 1. Assessing blood flow, capillary refill, color, and pain will help identify
the priority for the child. 2. The arm should be immobilized to decrease risk for further damage.
3.Ice can be applied after a neurovascular check is performed.
CURRENT DIAGNOSIS AND TREATMENT PEDIATRICS 24TH LATEST 4.Pain management can be given after a neurovascular assessment is performed. 33. Discharge teaching is being sent home with a girl who had a right radial fracture and has a cast. The nurse is teaching the family about PRICE. The nurse knows that the family understands the teaching when the mother states: 1. We will make sure to rest the arm, apply ice, and keep it elevated. The important part is to keep the cast for protection. 2. We will be able to let her return to her normal activities when we get home. 3. We will get some pencils so she can put them down the cast to scratch her skin. We should also have her elevate her arm at least two times a day with ice. 4. We will allow her friends to write on the cast with a permanent marker. ANS: 1 Feedback 1. The mother stated the concepts of PRICE. 2. Pain and swelling will continue or become worse if the arm is not
rested. 3. No objects should be placed into the cast because of the chance for skin
breakdown and losing the item inside the cast. 4. Writing on the cast can degrade the material and cause cracking.
= 34. Janice, an adult orthopedic nurse, has been asked to float to the pediatric orthopedic floor for the next two weeks. It is important for Janice to remember that children have more complicated issues with fractures because: 1. Fractures can occur along an immature growth plate. 2. Children have a lower pain tolerance. 3. Children do not understand how to use crutches, and it takes more time to do discharge teaching.
CURRENT DIAGNOSIS AND TREATMENT PEDIATRICS 24TH LATEST 4. A childs bone takes many more weeks to heal than an adult. ANS: 1 Feedback 1. Fractures in this area may take longer to heal or can form calcification,
causing incorrect placement of the bone. 2. Pain is individualized. 3. Children can learn how to adapt to the use of crutches and should be
given time to practice prior to discharge. 4. Because of the rapid growth of a child, the healing process usually does
not take as long as an adult. = 35. A new pediatric orthopedic nurse is learning about the most common areas for a child to have a fracture. She knows all are common sites except the: 1. Tibia. 2. Radius. 3. Femur. 4. Clavicle. ANS: 3 Feedback 1. Easily fractures because of childrens activities 2. Easily fractures because of the childrens activities and how a child will
attempt to protect himself/herself when falling. 3. Not easily fractured because of the size of the bone 4. Easily fractures because of the bone size and the activities of children 36. Ella, an 18 month old, is brought to the clinic because the mother states that her daughter has not been putting weight on her left leg for most of the day. The doctor says that the tibia is broken after reading an X-ray. The nurse is asked to finish the discharge teaching for Ella. The nurse knows that she must discuss all of the following except:
CURRENT DIAGNOSIS AND TREATMENT PEDIATRICS 24TH LATEST 1. Care for the cast. 2. Care for the skin. 3. Discuss signs and symptoms if the cast is too tight or there is an increase in swelling. 4. Schedule only one follow appointment in six weeks. ANS: 4 Feedback 1. This will aid in keeping the cast intact and clean. 2. The teaching will encourage the child and family to be aware of areas of
skin breakdown to prevent complications. 3. Tightness and an increase in swelling may be an indication the cast is too tight. 4. Reoccurring appointments will be needed to make sure that the tibia is healing properly. 37. A new pediatric nurse asks why the pediatric floor has had an increase in the number of femur fractures during the month of July. A veteran staff nurse knows that: 1. An increase in femur fractures is common in summer months because of the outdoor activities children participate in. 2. An increase in femur fractures is common after school gets out because children are not watched as closely. 3. This increase is abnormal. Usually the increase is seen in the winter months with sledding accidents. 4. This increase is abnormal because most children do not receive femur fractures. ANS: 1 Feedback 1. The types of summer activities increase the incidence of fractures to the
femur. 2. This is an assumption, not a fact. 3. Femur fractures are less likely to happen in sledding accidents.
CURRENT DIAGNOSIS AND TREATMENT PEDIATRICS 24TH LATEST 4. Children can receive femur fractures with summer activities.
38. A 6 month old is brought to the ER by her babysitter. The babysitter explains that the child has been irritable all morning and will not bear weight on her right leg since she was dropped off at 8 a.m. The babysitter has attempted to contact the mother, but has not been able to reach her. After the examination, the child is diagnosed with spiral femur fracture. The nurse will need to contact Social Services because: 1. Any time a child enters the ER, Social Services needs to be contacted. 2. The mother did not bring the child in, and the babysitter does not have medical power of attorney, so a social worker will determine the needs of the child. 3. A femur fracture is abnormal unless trauma or abuse has occurred. Social Services will help to rule out child abuse. 4. The social worker will be of no help until the mother asks for services. ANS: 3 Feedback 1. Social Services needs to be contacted when suspected child abuse has
occurred. 2. The best interest of the child needs to be accounted for, and the
babysitter can have the child examined. 3. Femur fractures are rare in this age of a child, except in abuse situations.
4.Social Services will act in the best interests of the child in this situation. Chapter 27. Sports Medicine 1. A client is ordered to receive a nonopioid analgesic. The nurse knows that the client is experiencing
pain.
a. acute severe b. visceral (deep) c. acute mild d. superficial moderate to severe
CURRENT DIAGNOSIS AND TREATMENT PEDIATRICS 24TH LATEST ANS: C Nonopioid analgesics are used for mild to moderate pain and may be available over the counter. 2. The client is complaining of severe pain. The nurse anticipates that the client will be ordered treatment with: a. aspirin. b. acetaminophen. c. diflunisal. d. morphine sulfate. ANS: D Morphine sulfate should be used for severe pain. 3. A 5-year-old client has an elevated temperature as a result of a viral respiratory tract infection. What nonopioid drug should be given to decrease the childs body temperature? a. Aspirin b. Acetaminophen c. Diflunisal d. Sodium salicylate ANS: B Medications containing salicylates are not recommended because of the possibility of developing Reyes syndrome. 4. An adolescent client tells the nurse that she takes acetaminophen (Tylenol) a few times every day because of stress headaches. The nurse advises her to see a primary healthcare provider because overuse of the medication may result in: a. nausea and anorexia. b. gastrointestinal irritation.
CURRENT DIAGNOSIS AND TREATMENT PEDIATRICS 24TH LATEST c. hepatotoxicity. d. diaphoresis and fluid loss. ANS: C Tylenol is metabolized by the liver and, with extended use, may be toxic to liver tissue. 5. A client takes aspirin regularly to deal with the pain of arthritis. Which symptom may be indicative of a serious side effect of the medication? a. Intense abdominal pain b. Frequent constipation c. Excessive perspiration d. Excessive fatigue ANS: A ASA may cause gastric irritation and lead to ulceration. Abdominal pain may indicate this and may be a medical emergency. 6. A client is ordered to receive ibuprofen (Motrin) for dysmenorrhea. The highest priority instruction that the nurse should give the client is to take the drug: a. with fluid or food. b. on an empty stomach. c. upon arising. d. nightly before sleep. ANS: A Ibuprofen may cause gastric irritation, so food or fluid will decrease this effect. 7. A client is prescribed morphine sulfate for management of severe pain. The client tells the nurse that he takes several herbal preparations. Which herbal preparation will be of most concern to the nurse?
CURRENT DIAGNOSIS AND TREATMENT PEDIATRICS 24TH LATEST a. Garlic b. Ginger c. St. Johns wort d. Saw palmetto ANS: C Opioids such as morphine sulfate may increase sedation when taken with St. Johns wort. 8. A nurse is planning the care of a client receiving opioid analgesia. What is considered a priority in planning this care? a. Monitoring respiratory rate b. Listening for adventitious breath sounds c. Assessing for speed of pupillary reaction d. Increasing the IV fluid flow rate ANS: A Assessing respiratory rate is a priority with medications that may cause respiratory depression. 9. A client requires an opioid antagonist after receiving an overdose of an opioid agent. The nurse anticipates that the client will be ordered: a. pentazocine. b. ibuprofen. c. naloxone. d. probenecid. ANS: C Narcan is a opioid antagonist used with overdose or oversedation caused by opioids. 10. The client is ordered methadone. The client is most likely experiencing:
CURRENT DIAGNOSIS AND TREATMENT PEDIATRICS 24TH LATEST a. opioid overdose. b. acute or chronic pain. c. opioid addiction. d. sleep apnea. ANS: C Methadone is used to assist in detoxification and monitoring of people with drug addiction. 11. A nurse is assessing a postoperative client who received morphine sulfate for severe pain 1 hour ago. What common side effects are associated with this medication? a. Constipation and pruritus b. Diarrhea and lethargy c. Tachycardia and hypertension d. Coughing and wheezing ANS: A Constipation and pruritus are known to occur with morphine use and should be assessed in patients. 12. A patient receives nalbuphine (Nubain) for intense pain related to a fracture. Which nursing intervention is an important part of the plan of care 1 hour after administration of this medication? a. Strain all urine. b. Elevate the head of the bed. c. Monitor vital signs when getting out of bed. d. Infuse IV fluid at a rapid rate. ANS: C
CURRENT DIAGNOSIS AND TREATMENT PEDIATRICS 24TH LATEST This medication may cause hypotension, so the nurse should assess vital signs with position changes. 13. An adult client with a head injury complains of severe pain. The nurse notes that the dose of opioid is half the normal adult dose. What is the reason for this? a. Head injury patients do not experience severe pain but are disoriented. b. Respiratory depression can lead to cerebral hemorrhage. c. Opioids decrease heart rate such that the brain becomes hypoxic. d. Respiratory depression allows for a buildup of CO2, a vasodilator. ANS: D If respiratory depression occurs, the respiratory rate may decrease, causing hypoventilation. This allows CO2 to build up, causing cerebral vasodilation and increasing intracranial pressure. 14. A client receives hydromorphone (Dilaudid) following an operative procedure. The nurse assesses the clients urine output in order to monitor for which side effect of this medication? a. Urinary tract infections b. Incontinence c. Urinary retention d. Renal failure ANS: C A common side effect of opioid agents is urinary retention. 15. An adult client has just received morphine sulfate for severe pain. What would indicate that the pain medication was effective? a. Client lies very still in bed b. Reduction of the respiratory rate to 8 breaths per minute c. Facial grimacing and verbalization of relief of pain
CURRENT DIAGNOSIS AND TREATMENT PEDIATRICS 24TH LATEST d. Lowering of tachycardia to within normal limits ANS: D A lowering of the heart rate to within normal limits indicates relief of pain. 16. The client is prescribed Imitrex for migraine headaches. She is nauseated and cannot take the medication by mouth. The nurse anticipates that the client will receive the medication via: a. subcutaneous injection. b. intramuscular injection. c. intravenous infusion. d. sublingual route. ANS: A Imitrex can be administered by mouth, by subcutaneous injection, or by intranasal route. 17. The client receiving Imitrex complains of dizziness. The nurses highest priority intervention is to recognize that this is a(n)
and notify the physician.
a. adverse reaction to the medication b. food-drug interaction c. side effect of the medication d. life-threatening reaction to the drug ANS: C Dizziness is a side effect of treatment with Imitrex but is not life threatening. 18. The client is ordered Nubain intravenously for treatment of severe pain. The client anxiously asks when she can expect to have relief from the pain. The nurse anticipates that the client will have relief within a. 2 to 3
minutes.
CURRENT DIAGNOSIS AND TREATMENT PEDIATRICS 24TH LATEST b. 5 to 6 c. 10 to 11 d. 15 to 16 ANS: A Intravenous administration of Nubain should result in relief from pain within 2 to 3 minutes. 19. The client is taking acetaminophen on a regular basis as well as oral contraceptives. The nurse tells the client that this drug interaction will result in a(n)
in the effectiveness of the
. a. decrease; oral contraceptives b. increase; oral contraceptives c. decrease; acetaminophen d. increase; acetaminophen ANS: C When acetaminophen is combined with oral contraceptives, the result is a decrease in the effectiveness of the acetaminophen. MULTIPLE RESPONSE 1. A nurse has administered 8.0 mg morphine sulfate to an adult client in severe pain. What would the nurse evaluate as positive outcomes of this intervention? (Select all that apply.) a. Respiratory rate of 6 breaths/min b. Heart rate of 80 beats/minute c. Blood pressure 180/110 mm Hg d. Restlessness e. Absence of facial grimacing f. Verbalization of pain relief g. Ability to take deep breaths
CURRENT DIAGNOSIS AND TREATMENT PEDIATRICS 24TH LATEST ANS: B, E, F, G These indicate relief of pain and represent normal vital and objective signs.
Chapter 28. Rehabilitation Medicine 1. A patient who has undergone a lengthy surgical procedure under general anesthesia is unable to breathe on his own following the procedure. Which drug will the nurse expect to be administered as an antidote to the neuromuscular-blocking agent? a. Dantrolene (Dantrium) b. Neostigmine methylsulfate (Prostigmin) c. Ether d. Baclofen (Lioresal) ANS: B Neostigmine methylsulfate is an antidote to neuromuscular-blocking agents. Additional measures to implement include artificial respirations with oxygen and atropine sulfate for bradycardia and hypotension. Dantrolene is a direct-acting skeletal muscle relaxant given for the treatment of
CURRENT DIAGNOSIS AND TREATMENT PEDIATRICS 24TH LATEST neuroleptic malignant syndrome associated with the use of antipsychotic agents. Ether is a general anesthetic. Baclofen is a skeletal muscle relaxant. 2. What is the reason for paraplegic patient to receive baclofen (Lioresal)? a. It interrupts reflexes at the level of the spinal cord. b. It acts directly on the skeletal muscles to reduce spasticity. c. It interrupts transmission of impulses from motor nerves to muscles at the skeletal neuromuscular junction. d. It produces generalized mild weakness of skeletal muscles and decreases the force of reflex muscle contractions. ANS: A Baclofen is a gamma aminobutyric acid derivative that interrupts polysynaptic reflexes at the level of the spinal cord. Baclofen does not appear to act directly on the skeletal muscles. Interrupting transmission of impulses from motor nerves to muscles at the skeletal neuromuscular junction is the mechanism of action of neuromuscular-blocking agents. Producing generalized mild weakness of skeletal muscles and decreasing the force of reflex muscle contractions is the mechanism of action of dantrolene. 3. A patient taking antipsychotic medication for schizophrenia is admitted with a temperature of 106 F. The admitting diagnosis is neuroleptic malignant syndrome resulting from antipsychotic medication. Which drug is indicated in treatment of this condition? a. Edrophonium (Tensilon) b. Dantrolene (Dantrium) c. Baclofen (Lioresal) d. Metaxalone (Skelaxin) ANS: B Dantrolene, a direct acting skeletal muscle relaxant, is used to treat neuroleptic malignant syndrome associated with the use of antipsychotic agents. Edrophonium is an antidote for neuromuscular-blocking agents. Baclofen is a skeletal muscle relaxant used for the treatment of
CURRENT DIAGNOSIS AND TREATMENT PEDIATRICS 24TH LATEST muscle spasticity resulting from multiple sclerosis (MS) and cerebral palsy. Metaxalone is used for muscle spasms. 4. In the intensive care unit, the nurse is taking care of a patient who is on a ventilator and is receiving succinylcholine (Anectine). Which is a priority nursing diagnosis for this patient? a. Pain b. Disturbed body image c. Risk for injury d. Self care deficit ANS: A Pain is the highest priority, according to Maslows hierarchy of needs. Succinylcholine, a neuromuscular-blocking agent, is used to paralyze muscles while a patient is on a ventilator. Neuromuscular-blocking agents do not relieve pain. Disturbed body image is not a priority for this situation, although it does apply. Risk for injury and self care deficit are not as great of priorities for this situation. 5. The patients family asks why neuromuscular blocking agents are used before electroconvulsive therapy. Which explanation by the nurse is most accurate? a. They stimulate respirations while therapy is delivered. b. They prevent aspiration of respiratory secretions during the therapy. c. They decrease intracranial pressure resulting from therapy. d. They reduce the risk of injury during therapy. ANS: D Neuromuscular-blocking agents are used before electroconvulsive therapy to paralyze the skeletal muscles and reduce the risk of fractures during therapy. Neuromuscular-blocking agents do not stimulate respiration, prevent aspiration, or affect intracranial pressure.
CURRENT DIAGNOSIS AND TREATMENT PEDIATRICS 24TH LATEST 6. Which laboratory values will the nurse review prior to beginning medication therapy for skeletal and muscle disorders? a. Sodium, magnesium, and chloride b. C-reactive protein (CRP), human leukocyte antigen (HLA), and liver function tests c. Arterial blood gases (ABGs), complete blood count (CBC), and electrolytes d. Glucose, high density lipoproteins (HDL), and prothrombin time (PT) ANS: B Examine laboratory reports associated with the disease process present (e.g., calcium, phosphorus, lupus testing, rheumatoid factor, uric acid level, CRP, HLA, aldolase, aspartate, creatine kinase). Electrolytes, trace minerals, ABGs, CBC, electrolytes, glucose, HDL, and PT are not associated with this disease process. 7. Which medication will be prescribed for a patient complaining of muscle spasms resulting from a back injury? a. Acetaminophen (Tylenol) b. Morphine sulfate c. Bethanechol (Urecholine) d. Cyclobenzaprine (Flexeril) ANS: D Cyclobenzaprine is a centrally acting skeletal muscle relaxant, which is a group of drugs used to relieve acute muscle spasms. Their exact mechanism of action is unknown, except that they depress central nervous system (CNS) function. All the centrally acting skeletal muscle relaxants produce some degree of relaxation, and health care providers maintain that the benefits from the sedative effects may exceed the benefits from actual muscle relaxation. Acetaminophen, morphine, and bethanechol do not relieve skeletal muscle spasms.
CURRENT DIAGNOSIS AND TREATMENT PEDIATRICS 24TH LATEST 8. Which symptoms will be most important for the nurse to assess for early signs of respiratory distress in the patient who has been given a neuromuscular-blocking agent? a. Nasal flaring and retraction of intercostal muscles b. Dyspnea, increased respiratory rate, and cyanosis c. Restlessness, anxiety, and lethargy d. Pallor, stridor, and diaphoresis ANS: C The signs of restlessness, anxiety, lethargy, decreased mental alertness, and headache are early, subtle clues to respiratory distress. Retractions, flaring, dyspnea, hyperventilation, cyanosis, pallor, stridor, and diaphoresis are not early signs of respiratory distress. 9. Which common adverse effects occur with neuromuscular-blocking agents? a. Fever b. Flushing c. Nausea d. Ataxia ANS: B Neuromuscular-blocking agents cause histamine release, which may cause bronchospasm, bronchial and salivary secretions, flushing, edema, and urticaria. Ensure that the airway is patent and that secretions are suctioned regularly to prevent obstruction. Report evidence of bronchospasm, edema, and urticaria immediately. Neuromuscular-blocking agents do not commonly cause fever, nausea, or ataxia. 10. Which assessment is most important for the nurse to obtain when a patient is being treated with a neuromuscular-blocking agent? a. Skin assessment for rash and urticaria b. Blood pressure assessment for orthostatic hypotension
CURRENT DIAGNOSIS AND TREATMENT PEDIATRICS 24TH LATEST c. Respiratory assessment for patent airway d. Assessment for fluid volume overload ANS: C Histamine release caused by these drugs may produce increased salivation. In patients who are paralyzed or who have incomplete return of control over swallowing, coughing, and deep breathing, these secretions may obstruct the airway. Histamines are not likely to produce rash or urticaria related to the administration of neuromuscular-blocking agents. Patients on neuromuscular-blocking agents are generally kept on bed rest and not subject to orthostatic hypotension. Neuromuscular-blocking agents do not cause fluid volume overload. 11. Which drug interaction may occur when an aminoglycoside or tetracycline is given in conjunction with neuromuscular-blocking agents? a. Deep sedation b. Decreased effectiveness of antibiotics c. Increased neuromuscular-blocking activity d. Sensitivity to antibiotics and possible allergic reaction ANS: C Question antibiotic orders that prescribe aminoglycosides or tetracycline when neuromuscular blockers have been used. These drugs may potentiate the neuromuscular blocking activity. This combination of medications would not cause deep sedation. Neuromuscular-blocking agents do not decrease the effectiveness of antibiotics or increase sensitivity to antibiotics. 12. A patient taking a neuromuscular blocking agent is assessed to have a heart rate of 120 and blood pressure of 80/50. The nurse will anticipate the physician writing an order for: a. ABGs. b. blood glucose level. c. CBC. d. liver function tests.
CURRENT DIAGNOSIS AND TREATMENT PEDIATRICS 24TH LATEST ANS: A Patients taking neuromuscular-blocking agents should be monitored closely for clinical signs of hypoxia and hypercapnia (tachycardia, hypotension, cyanosis). ABG levels may be determined to confirm the clinical observations. Blood glucose levels, CBC, and liver function tests would not be indicated with these symptoms. 13. The nurse receives a conscious patient from the postoperative unit after administration of a neuromuscular blocker. Once the patients vital signs are stable, the best position for the nurse to assist the patient into is
position.
a. Sims b. semi-Fowlers c. supine d. prone ANS: B Patients can usually cough better in a semi-Fowlers or high Fowlers position; therefore, depending on the situation and stability of the patients vital signs, elevating the head of the bed may assist coughing and breathing. Sims, supine, and prone are not the best positions for a patient postoperatively. MULTIPLE RESPONSE 14. Patients with which conditions must be carefully assessed to determine whether they would tolerate treatment with a neuromuscular-blocking agent? (Select all that apply.) a. Pregnancy b. Hepatic disease c. Pulmonary disease d. Renal disease e. Neurologic disorders f. Psychiatric disorders
CURRENT DIAGNOSIS AND TREATMENT PEDIATRICS 24TH LATEST ANS: B, C, D, E Patients with hepatic disease, pulmonary disease, renal disease, and neurologic disorders such as myasthenia gravis, spinal cord injury, or MS must be fully evaluated to assess their ability to tolerate neuromuscular-blocking agents. Neuromuscular-blocking agents can usually be used safely with the pregnant patient and the psychiatric patient. 15. The nurse is examining a patient in the emergency department whose chief complaint is a dislocated shoulder. Which assessment data are needed to evaluate the patient? (Select all that apply.) a. Details related to the mechanism of injury b. Degree of impairment c. Pain level d. Inspection of the affected part for swelling, capillary refill, bruising, redness, localized tenderness, deformities, and paresthesia e. Elevation of the affected extremity ANS: A, B, C, D Important nursing assessment of a musculoskeletal injury includes mechanism of injury, degree of impairment, pain level, and obvious deformity and neurovascular assessment. Elevating the affected extremity would exacerbate the pain and deformity of the injury. 16. An employee at a factory has not been to work because of low back muscle spasms. His wife contacts the occupational health nurse to report that her spouse is on a centrally acting skeletal muscle relaxant and is having problems with sleepiness. Based on the medication action, what will the nurse tell her? (Select all that apply.) a. The health care provider should be notified because these drugs are contraindicated in the treatment of low back pain. b. Your husband should avoid activities requiring alertness, such as driving or operating power equipment. c. I need additional and specific information regarding the amount of your husbands sedation.
CURRENT DIAGNOSIS AND TREATMENT PEDIATRICS 24TH LATEST d. I will review baseline laboratory studies, discuss your husbands status with the health care provider, and call you back. e. Sedation is an adverse effect of these medications and tends to resolve with continued therapy. ANS: B, C, D, E Patients should be instructed to avoid activities that require alertness, such as driving or operating heavy machinery, for safety. When obtaining patient information regarding adverse effects, additional information may be needed to fully assess the degree of adverse effects the patient may be experiencing. Review of baseline liver and kidney function and CBC laboratory data is necessary. Centrally acting skeletal muscle relaxants are used to relieve acute muscle spasm. They act on the CNS, and sedation may be an adverse effect associated with usage. This tends to be mild and will resolve with continued use. These drugs are indicated for the treatment of low back pain. 17. Why are neuromuscular-blocking agents used? (Select all that apply.) a. Alleviation of pain b. Reducing the use and adverse effects of general anesthetics c. Easing endotracheal intubation and prevent laryngospasm d. Producing amnesia during painful procedures e. Decreasing muscular activity in electroshock therapy ANS: B, C, E Neuromuscular-blocking agents are used to produce adequate muscle relaxation during anesthesia to reduce the use (and adverse effects) of general anesthesia, ease endotracheal intubation and prevent laryngospasm, and decrease muscular activity in electroshock therapy. Neuromuscular-blocking agents have no effect on the pain threshold, memory, or consciousness. 18. When assessing a patient for signs and symptoms of early respiratory depression immediately after leaving the postoperative area, the nurse will be alert for signs of what? (Select all that apply.)
CURRENT DIAGNOSIS AND TREATMENT PEDIATRICS 24TH LATEST a. Restlessness b. Anxiety c. Lethargy d. Increased mental alertness e. Cyanosis ANS: A, B, C Early signs of diminished ventilation include restlessness, anxiety, lethargy, decreased mental alertness, and headache. Increased mental alertness is not an early sign of respiratory depression. Cyanosis is a late sign of respiratory complications.
Chapter 29. Rheumatic Diseases 1. A 7-year-old is having on and off knee, ankle, wrist, and hip pain. The family history is positive for rheumatoid arthritis. No injury has taken place, and the child has occasional swelling of joints. Pain resolves with rest, ice, and NSAIDs. The primary care physician ran labs with positive rheumatoid factor ANA and ESR. This child is a soccer player and a violinist. Your differential diagnosis would be: 1. Juvenile Idiopathic Arthritis (JIA), septic arthritis, osteomyelitis, and fracture. 2. Juvenile Idiopathic Arthritis (JIA), toxic synovitis, osteomyelitis, and Developmental Dislocated Hips (DDH). 3. Juvenile Idiopathic Arthritis (JIA), chronic inflammatory arthritis, chronic osteomyelitis, and Developmental Dislocated Hips (DDH). 4. Juvenile Idiopathic Arthritis (JIA) only. ANS: 4 Feedback
CURRENT DIAGNOSIS AND TREATMENT PEDIATRICS 24TH LATEST
1. 2.
3.
4.
Juvenile Idiopathic Arthritis (JIA) is characterized by a long-standing history of pain and swelling to more than one joint. This can be hereditary. Lab oratory tests are normal in chronic osteo, toxic synovitis, and Developmental Dislocated Hips (DDH). All are factors in Juvenile Idiopathic Arthritis. Juvenile Idiopathic Arthritis (JIA) is characterized by a long-standing history of pain and swelling to more than one joint. This can be hereditary. Laboratory tests are normal in chronic osteo, toxic synovitis, and Developmental Dislocated Hips (DDH). Juvenile Idiopathic Arthritis (JIA) is characterized by a long-standing history of pain and swelling to more than one joint. It can be hereditary. Laboratory tests are normal in chronic osteo, toxic synovitis, and Developmental Dislocated Hips (DDH).
2. If a child is diagnosed with Juvenile Idiopathic Arthritis (JIA), which of the following statements is true? 1. Usually, a child outgrows the disorder. 2. NSAIDs are the treatment of choice. 3. The child should not participate in sports. 4. All of the children of these parents will have the disorder. ANS: 2 Feedback 1.
The disorder lasts for the life of the child.
2. 3.
Activity helps the joints stay mobile, and anti-inflammatory medications are the treatment of choice for Juvenile Idiopathic Arthritis. Participation in sports will help maintain mobility.
4.
Activity helps the joints stay mobile, and anti-inflammatory medications are the treatment of choice for Juvenile Idiopathic Arthritis.
3. A 2-year-old male presents to clinic with his caregiver stating that he slipped and fell running on the hardwood floor with the dog. He cried immediately and refused to bear weight on his left side. X-rays were taken in the emergency room of his hip, knee, and ankle. The report was negative. The caregiver states that he is still uncomfortable and is not walking on it. Laboratory tests were done and came back negative for elevated ESR, CRP, and CBC. Upon physical examination, he has pain and immediately withdraws his leg on palpation of his anterior tibia shaft. He says ouchie and no hurt my leg. Your knowledge and the exam would lead you to diagnose the child with which of the following conditions? 1. Juvenile Idiopathic Arthritis 2. Toddlers Fracture
CURRENT DIAGNOSIS AND TREATMENT PEDIATRICS 24TH LATEST 3. Hip Dysplasia 4. Sprained ankle ANS: 2
1.
2. 3.
4.
Feedback A toddlers fracture is not typically visible on initial X-rays, but point tenderness on the tibia shaft and refusal to bear weight would be common in this age group following a witnessed injury. A toddlers fracture is not typically visible on initial X-rays, but point tenderness on the tibia shaft and refusal to bear weight would be common in this age group following a witnessed injury. The hip dysplasia would be notable with one hip higher than the other or a click when examined. A toddlers fracture is not typically visible on initial X-rays, but point tenderness on the tibia shaft and refusal to bear weight would be common in this age group following a witnessed injury.
4. A mother with a child with Juvenile Idiopathic Arthritis calls the clinic nurse because the child is having a pain exacerbation. The mother asks the nurse if the child can continue with her range of motion exercises at the time. The appropriate nursing response is: 1. Range of motion exercises should be done every day. 2. Have the child do isometric exercises during this time. 3. Administer additional pain medication before doing the exercises. 4. Avoid exercises during this time. ANS: 3
1. 2. 3. 4.
Feedback Pain control will be important when doing the exercises and should be addressed prior to the activity. Isometrics can increase the pain. Non-weight-bearing exercises would be a better choice for the child. Pain medication should be provided to aid in the child getting the most out of the therapy. Exercising will be important to maintain as high of a function as possible.
5. A 14 year old has been diagnosed with Blounts disease. The nurse knows that the greatest risk factor for this disease process is: 1. Juvenile Idiopathic Arthritis.
CURRENT DIAGNOSIS AND TREATMENT PEDIATRICS 24TH LATEST 2. Obesity. 3. Diabetes. 4. Asthma. ANS: 2 1.
Feedback Not a risk factor for Blounts disease
2. 3.
Increased weight adds pressure to the legs and can increase the amount of bowing. Not a risk factor for Blounts disease
4.
Not a risk factor for Blounts disease
Chapter 30. Hematologic Disorders 1. The nurse is assessing a client admitted with cyanotic heart disease. Which hematology laboratory result does the nurse expect to assess?
a. Anemia b. Polycythemia c. Normal d. Decreased hemoglobin
ANS: B Polycythemia can occur as a result of hypoxia, such as that experienced at high altitude or when oxygen is not sufficiently directed to the tissues, as in cyanotic heart disease. Anemia is a decrease in the number of RBCs, a reduction in their hemoglobin content, or a reduced volume of packed RBCs. Anemia results from one of two problems: either too rapid a loss of RBCs (by covert or overt bleeding or destruction) or too slow a production of RBCs. Anemias are categorized according to the size of the RBC (macrocytic, microcytic, normocytic) and the content of hemoglobin in the RBC (hypochromic, normochromic). The hematology laboratory result would not be normal because of the chronic hypoxia seen with cyanotic heart disease.
CURRENT DIAGNOSIS AND TREATMENT PEDIATRICS 24TH LATEST 2. What is the best response to a parent who asks the nurse whether her 5-month-old infant can have cows milk?
a. You need to wait until she is 8 months old and eating solids well. b. Yes, if you think that she will eat enough meat to get the iron she needs. c. Infants younger than 12 months need iron-rich formula to get the iron they need. d. Try it and see how she tolerates it.
ANS: C Infants younger than 12 months need iron-fortified formula or breast milk. Infants who drink cows milk do not get adequate iron and are at risk for iron deficiency anemia. A 5-month-old infant cannot get adequate iron without drinking an iron-fortified formula or taking an iron supplement. The American Academy of Pediatrics recommends beginning solid foods at 4 to 6 months of age. Meats are typically introduced in late infancy. Counseling a parent to give a 5month-old infant cows milk is inappropriate.
3. An assessment of a 7-month-old infant with a hemoglobin level of 6.5 mg/dL is likely to reveal which of the following?
a. An infant who is lethargic, pale, and irritable b. An infant who is thin, energetic, and sleeps little c. An infant who is anorexic, vomiting, and has watery stools d.
CURRENT DIAGNOSIS AND TREATMENT PEDIATRICS 24TH LATEST An infant who is flushed, fussy, and tired
ANS: A Pallor, lethargy, irritability, and tachycardia are clinical manifestations of iron-deficiency anemia. Infants with iron-deficiency anemia are not typically thin and energetic but do tend to sleep a lot. Gastrointestinal symptoms are not clinical manifestations associated with irondeficiency anemia. Although the infant with iron-deficiency anemia may be tired and fussy, pallor, rather than a flushed appearance, is characteristic of a low hemoglobin level.
4. Which suggestion is not appropriate for a 14-month-old child with iron-deficiency anemia?
a. Decrease the infants daily milk intake to 24 ounces or less. b. Give oral iron supplements between meals with orange juice. c. Include apricots, dark-green leafy vegetables, and egg yolk in the infants diet. d. Allow the infant to drink the iron supplement from a small medicine cup.
ANS: D Iron supplements should be administered through a straw or by a medicine dropper placed at the back of the mouth because iron temporarily stains the teeth. A daily milk intake in toddlers of less than 24 ounces will encourage the consumption of iron-rich solid foods. Because food interferes with the absorption of iron, iron supplements are taken between meals. Administering this medication with foods rich in vitamin C facilitates absorption of iron. Apricots, dark-green leafy vegetables, and egg yolks are rich sources of iron. Other iron-rich foods include liver, dried beans, Cream of Wheat, iron-fortified cereal, and prunes.
5. Which is true about the genetic transmission of sickle cell disease?
CURRENT DIAGNOSIS AND TREATMENT PEDIATRICS 24TH LATEST
a. Both parents must carry the sickle cell trait. b. Both parents must have sickle cell disease. c. One parent must have the sickle cell trait. d. Sickle cell disease has no known pattern of inheritance.
ANS: A Sickle cell disease has an autosomal recessive transmission pattern, which means that both parents must be carriers of the sickle cell trait. The sickle cell trait, not the disease itself, must be present in the parents for the child to have the disease.
6. What are the nursing priorities for a child with sickle cell disease in vaso-occlusive crisis?
a. Administration of antibiotics and nebulizer treatments b. Hydration and pain management c. Blood transfusions and an increased calorie diet d. School work and diversion
ANS: B
CURRENT DIAGNOSIS AND TREATMENT PEDIATRICS 24TH LATEST Hydration and pain management decrease the cells oxygen demands and prevent sickling. Antibiotics may be given prophylactically. Oxygen therapy rather than nebulizer treatments is used to prevent further sickling. Although blood transfusions and increased calories may be indicated, they are not primary considerations for a vaso-occlusive crisis. Schoolwork and diversion are not major considerations when the child is in a vaso-occlusive crisis.
7. A child with sickle cell disease is seen in the emergency department with increasing back and leg pain for the past 2 days. What is this child most likely experiencing?
a. A vaso-occlusive crisis b. Acute splenic sequestration c. Erythroblastopenia d. Acute chest syndrome
ANS: A A vaso-occlusive crisis is the most common type of crisis and is characterized by mild to severe pain. Pain can occur anywhere, but is typically manifested as bone or joint pain. Symptoms of acute splenic sequestration are associated with blood volume pooling, causing splenic enlargement and hypovolemic shock. Symptoms of pallor, lethargy, headache, and upper respiratory infection seen in erythroblastopenia result from decreased blood cell production by the bone marrow. Chest pain, fever, and cough are characteristic of acute chest syndrome. 8. What should the discharge plan for a school-age child with sickle cell disease include?
a. Restricting the childs participation in outside activities b.
CURRENT DIAGNOSIS AND TREATMENT PEDIATRICS 24TH LATEST Administering aspirin for pain or fever c. Limiting the childs interaction with peers d. Administering penicillin daily as ordered
ANS: D Children with sickle cell disease are at a high risk for pneumococcal infections and should receive long-term penicillin therapy and preventive immunizations. Sickle cell disease does not prohibit the child from outdoor play. Active and passive exercises help promote circulation. Aspirin use should be avoided. Acetaminophen or ibuprofen should be administered for fever or pain. The child needs to interact with peers to meet his developmental needs.
9. How should the nurse respond when asked by the mother of a child with beta-thalassemia why the child is receiving deferoxamine?
a. To improve the anemia. b. To decrease hepatosplenomegaly. c. To prevent organ damage. d. To prepare your child for a bone marrow transplant.
ANS: C Multiple transfusions result in hemosiderosis. Deferoxamine is given to chelate iron and prevent organ damage and complications from repeated transfusions. Preparation for a bone marrow transplant would not include administration of deferoxamine.
CURRENT DIAGNOSIS AND TREATMENT PEDIATRICS 24TH LATEST 10. What is the priority nursing intervention for a child hospitalized with hemarthrosis resulting from hemophilia?
a. Immobilization and elevation of the affected joint b. Administration of aspirin for pain relief c. Assessment of the childs response to hospitalization d. Assessment of the impact of hospitalization on the family system
ANS: A Immobilization and elevation of the joint will prevent further injury until bleeding is resolved. Aspirin inhibits platelet function and is not recommended for the treatment of hemarthrosis. Assessment of a childs response to hospitalization is relevant to all hospitalized children; however, in this situation, psychosocial concerns are secondary to physiological concerns. A priority nursing concern for this child is the management of hemarthrosis. Assessing the impact of hospitalization on the family system is relevant to all hospitalized children, but it is not the priority in this situation.
11. The mother of a child with hemophilia asks the nurse how long her child will need to be treated for hemophilia. What is the best response to this question?
a. Hemophilia is a life-long blood disorder. b. There is a 25% chance that your child will have spontaneous remission and treatment will no longer be necessary.
CURRENT DIAGNOSIS AND TREATMENT PEDIATRICS 24TH LATEST c. Treatment is indicated until after your child has progressed through the toddler years. d. It is unlikely that your child will need to be treated for his hemophilia because your first child does not have the disease.
ANS: A Hemophilia is a life-long hereditary blood disorder with no cure. Prevention by avoiding activities that induce bleeding and by treatment is life long. The management of hemophilia is very individual and depends on the severity of the illness. Because hemophilia has an X chromosome-linked recessive inheritance, there is a risk with each pregnancy that a child will either have the disease or be a carrier.
12. What is an expected outcome for the child with von Willebrands disease?
a. Prevention of injury b. Maintaining adequate hydration c. Compliance with chronic transfusion therapy d. Prevention of respiratory infections
ANS: A Hemorrhage as a result of injury is the childs greatest threat to life. Fluid volume status becomes a concern when hemorrhage has occurred. The treatment of von Willebrands disease is desmopressin acetate (DDAVP), which is administered intranasally or intravenously. Respiratory infections do not constitute a major threat to the child with von Willebrands disease.
CURRENT DIAGNOSIS AND TREATMENT PEDIATRICS 24TH LATEST 13. A child has a platelet count of 45,000/mm3 with petechiae and excessive bruising covering his body. The nurse is aware that these signs are clinical manifestations of:
a. erythroblastopenia. b. von Willebrands disease. c. hemophilia. d. immune thrombocytopenia purpura (ITP).
ANS: D Excessive bruising and petechiae, especially involving the mucous membranes and gums in a child who is otherwise healthy, are the clinical manifestations of ITP, resulting from decreased platelets. The etiology of ITP is unknown, but it is considered to be an autoimmune process. The clinical manifestations of erythroblastopenia are pallor, lethargy, headache, fainting, and a history of upper respiratory infection. The clinical manifestations of von Willebrands disease are bleeding from the gums or nose, prolonged bleeding from cuts, and excessive bleeding after surgery or trauma. Bleeding is the clinical manifestation of hemophilia and results from a deficiency of normal factor activity necessary to produce blood clotting.
14. What is the priority in the discharge plan for a child with immune thrombocytopenic purpura (ITP)?
a. Teaching the parents to report excessive fatigue to the physician b. Monitoring the childs hemoglobin level every 2 weeks
CURRENT DIAGNOSIS AND TREATMENT PEDIATRICS 24TH LATEST c. Providing a diet that contains iron-rich foods d. Establishing a safe, age-appropriate home environment
ANS: D Prevention of injury is a priority concern for a child with ITP. Excessive fatigue is not a significant problem for the child with ITP. ITP is associated with low platelet levels. Increasing the childs intake of iron in the diet will not correct ITP. 15. The care plan for a child with disseminated intravascular coagulation (DIC) includes:
a. hospitalization at the first sign of bleeding. b. teaching the child relaxation techniques for pain control. c. management in the intensive care unit. d. provision of adequate hydration to prevent complications.
ANS: C The child with DIC is seriously ill and needs to be monitored in an intensive care unit. Relaxation techniques and pain control are not high priorities for the child with DIC. The child with DIC is seriously ill and needs to be monitored in an intensive care unit. Hydration is not the major concern for the child with DIC.
16. What would be the nurses best response to a parent with questions about her childs blood disorder?
CURRENT DIAGNOSIS AND TREATMENT PEDIATRICS 24TH LATEST
a. That sounds pretty serious. b. What did the physician tell you? c. Blood diseases are transient, so there is no need to worry. d. Your child will be tired for awhile and then be back to her old self.
ANS: B Providing the parent with an opportunity to express what she was told by the physician allows the nurse to assess the parents understanding. Stating that the condition sounds serious may increase their anxiety. Minimizing the parents concern is inappropriate. The nurse needs to assess the parents knowledge before teaching about the disease.
1. The nurse is caring for a child with aplastic anemia. Which of the following nursing diagnoses would be appropriate? Select all that apply.
a. Acute pain related to vaso-occlusion b. Risk for infection related to inadequate secondary defenses or immunosuppression c. Ineffective protection related to thrombocytopenia d. Ineffective tissue perfusion related to anemia
ANS: B, C, D
CURRENT DIAGNOSIS AND TREATMENT PEDIATRICS 24TH LATEST Risk for infection, ineffective protection, and ineffective tissue perfusion are appropriate nursing diagnoses for the nurse planning care for a child with aplastic anemia. Aplastic anemia is a condition in which the bone marrow ceases production of the cells it normally manufactures, resulting in pancytopenia. The child will have varying degrees of the disease depending on how low the values are for absolute neutrophil count (affecting the bodys response to infection), platelet count (putting the child at risk for bleeding), and absolute reticulocyte count (causing the child to have anemia). Acute pain related to vaso-occlusion is an appropriate nursing diagnosis for sickle cell anemia for the child in vaso-occlusive crisis, but it is not applicable to a child with aplastic anemia.
2. A nurse is caring for a neonate requiring phototherapy for a high bilirubin. Which interventions should the nurse implement? Select all that apply.
a. Cover the neonates eyes with an opaque mask. b. Limit repositioning of the neonate. c. Dress the neonate in a T-shirt and diaper. d. Monitor the bilirubin level. e. Monitor temperature ever 2 to 4 hours.
ANS: A, D, E During hospitalization, to maximize the effectiveness of phototherapy, the neonate should be completely undressed or wearing only a diaper. While phototherapy is in use, the neonates eyes are covered with an opaque mask that is usually secured in place with a headband or cloth adhesive. The neonates position is changed frequently to ensure maximal skin exposure to the light source. Monitoring the neonates bilirubin level is performed one to four times daily to assess the effectiveness of phototherapy. The nurse monitors the neonates temperature every 2 to 4 hours to ensure maintenance within normal limits. The additional heat generated by the phototherapy unit places the neonate at risk for hyperthermia. The lack of clothing on the
CURRENT DIAGNOSIS AND TREATMENT PEDIATRICS 24TH LATEST neonate increases the possibility of heat loss through convection, conduction, radiation, and evaporation and may lead to hypothermia. 3. A nurse is teaching home care instructions to parents of a child with sickle cell disease. Which instructions should the nurse include? Select all that apply.
a. Limit fluid intake. b. Administer aspirin for fever. c. Administer penicillin as ordered. d. Avoid cold and extreme heat. e. Provide for adequate rest periods.
ANS: C, D, E Parents should be taught to avoid cold, which can increase sickling, and extreme heat, which can cause dehydration. Adequate rest periods should be provided. Penicillin should be administered daily as ordered. The use of aspirin should be avoided; acetaminophen or ibuprofen should be used as an alternative. Fluids should be encouraged and an increase in fluid intake is encouraged in hot weather or when there are other risks for dehydration. Chapter 31. Neoplastic Disease 1. The nurse notes that a childs gums bleed easily and bruising and petechiae are evident on the extremities. Which laboratory value would be consistent with these symptoms? a. Platelet count of 19,000/mm3 b. Prothrombin time of 11 to 15 seconds c. Hematocrit of 34
CURRENT DIAGNOSIS AND TREATMENT PEDIATRICS 24TH LATEST d. Leukocyte count of 14,000/mm3 ANS: A The normal platelet count is 150,000 to 400,000/mm3. This finding is very low, indicating an increased bleeding potential. The prothrombin time of 11 to 15 seconds is within normal limits. The normal hematocrit is 35 to 45 and, although this finding is low, it would not create the symptoms presented. A leukocyte count of 14,000/mm3 indicates the probable presence of infection but is not a reflection of bleeding tendency. 2. The nurse understands that the type of precautions needed for children receiving chemotherapy is based on which action of chemotherapeutic agents? a. Gastrointestinal upset b. Bone marrow suppression c. Decreased creatinine level d. Alopecia ANS: B Chemotherapy agents cause bone marrow suppression, which creates the need to institute precautions related to reduced white blood cell, red blood cell, and platelet counts. These precautions focus on preventing infection and bleeding. Although gastrointestinal upset may be an adverse effect of chemotherapy, it is not caused by all chemotherapeutic agents. No special precautions are instituted for gastrointestinal upset. A decreased creatinine level is consistent with renal pathological conditions, not chemotherapy. Not all chemotherapeutic agents cause alopecia. No precautions are taken to prevent alopecia. 3. The nurse should base a response to a parents question about the prognosis of acute lymphoblastic leukemia (ALL) on which information? a. Leukemia is a fatal disease although chemotherapy provides increasingly longer periods of remission. b. Research to find a cure for childhood cancers is very active.
CURRENT DIAGNOSIS AND TREATMENT PEDIATRICS 24TH LATEST c. The majority of children go into remission and remain symptom free when treatment is completed. d. It usually takes several months of chemotherapy to achieve a remission. ANS: C Children diagnosed with the most common form of leukemia, ALL, can almost always achieve remission, with a 5-year disease-free survival rate approaching 85%. With the majority of children surviving 5 years or longer, it is inappropriate to refer to leukemia as a fatal disease. Although research to find a cure for childhood cancers is very active, it does not address the parents concern. About 95% of children achieve remission within the first month of chemotherapy. If significant numbers of blast cells are still present in the bone marrow after a month of chemotherapy, a new and stronger regimen is begun. 4. Bone marrow transplantation is the standard treatment for which childhood cancer? a. Acute lymphoblastic leukemia (ALL) b. Non-Hodgkins lymphoma c. Wilms tumor d. Acute myeloblastic leukemia (AML) ANS: D Bone marrow transplantation is currently the standard treatment for children in their first remission with AML. The standard treatment for ALL is combination chemotherapy. The standard treatment for non-Hodgkins lymphoma is chemotherapy. Bone marrow transplantation is used to treat non-Hodgkins lymphoma that is resistant to conventional chemotherapy and radiation. The treatment for Wilms tumor consists of surgery and chemotherapy alone or in combination with radiation therapy. 5. A child with a history of fever of unknown origin, excessive bruising, lymphadenopathy, and fatigue is exhibiting symptoms most suggestive of which condition? a. Ewings sarcoma
CURRENT DIAGNOSIS AND TREATMENT PEDIATRICS 24TH LATEST b. Wilms tumor c. Neuroblastoma d. Leukemia ANS: D Symptoms of a history of fever of unknown origin, excessive bruising, lymphadenopathy, and fatigue reflect bone marrow failure and organ infiltration, which occur in leukemia. Symptoms of Ewings sarcoma involve pain and soft tissue swelling around the affected bone. Wilms tumor usually manifests as an abdominal mass with abdominal pain and may include renal symptoms, such as hematuria, hypertension, and anemia. Neuroblastoma manifests primarily as an abdominal, chest, bone, or joint mass. Symptoms are dependent on the extent and involvement of the tumor. 6. A nurse assesses a whitish reflection instead of the expected red reflex when evaluating an infants eyes. Which action should the nurse take? a. Notify the physician. b. Chart the finding. c. Check the chart to verify that the infant received eye prophylaxis at birth. d. Reassess for a red reflex in 1 month. ANS: A A whitish reflex in the eye, leukocoria, is a common finding of retinoblastoma. It is an overt sign of cancer and the physician needs to be notified. Checking the chart for routine eye prophylaxis at birth would be done if there was an eye infection. The finding is not normal and by charting the finding or reassessing in 1 month treatment will be delayed. 7. Which nursing diagnosis is a priority for the 4-year-old child newly diagnosed with leukemia? a. Ineffective breathing pattern related to mediastinal disease b. Risk for infection related to immunosuppressed state c. Disturbed body image related to alopecia
CURRENT DIAGNOSIS AND TREATMENT PEDIATRICS 24TH LATEST d. Impaired skin integrity related to radiation therapy ANS: B Leukemia is characterized by the proliferation of immature white blood cells, which lack the ability to fight infection. An ineffective breathing pattern related to mediastinal disease would apply to a child with non-Hodgkins lymphoma or any cancer involving the chest area. Disturbed body image related to alopecia is a nursing diagnosis related to chemotherapy, but it is not of the highest priority. Not all children have a body image disturbance as a result of alopecia, especially not preschoolers. This would be of more concern to an adolescent. Radiation therapy is not a treatment for leukemia. 8. A nurse determines that parents understood the teaching from the pediatric oncologist if the parents indicate that which test confirms the diagnosis of leukemia in children? a. Complete blood cell count (CBC) b. Lumbar puncture c. Bone marrow biopsy d. Computed tomography (CT) scan ANS: C The confirming test for leukemia is microscopic examination of bone marrow obtained by bone marrow aspiration and biopsy. A CBC may show blast cells that would raise suspicion of leukemia. It is not a confirming diagnostic study. A lumbar puncture is done to check for central nervous system involvement in the child who has been diagnosed with leukemia. A CT scan may be done to check for bone involvement in the child with leukemia. It does not confirm a diagnosis. 9. Which statement made by a nurse to the parents of a child with leukemia should be included in discharge instructions? a. Your sons blood pressure must be taken daily while he is on chemotherapy. b. Limit your sons fluid intake just in case he has central nervous system (CNS) involvement.
CURRENT DIAGNOSIS AND TREATMENT PEDIATRICS 24TH LATEST c. Your son must receive all of his immunizations in a timely manner. d. Your sons temperature should be taken daily. ANS: D An elevated temperature may be the only sign of an infection in an immunosuppressed child. Parents should be instructed to monitor their childs temperature daily because of the risk for infection, but it is not necessary to take a blood pressure daily. Fluid is never withheld as a precaution against increased intracranial pressure. If a child had confirmed CNS involvement with increased intracranial pressure, limiting fluid intake might be more appropriate. Children who are immunosuppressed should not receive any live virus vaccines. 10. What is the most appropriate nursing action when the nurse notes a reddened area on the forearm of a neutropenic child with leukemia? a. Massage the area. b. Turn the child more frequently. c. Document the finding and continue to observe the area. d. Notify the physician immediately. ANS: D Any signs of infection in a child who is immunosuppressed must be reported immediately because it is considered a medical emergency. When a child is neutropenic, pus may not be produced and the only sign of infection may be redness. In a child with neutropenia, a reddened area may be the only sign of an infection. The area should never be massaged. The forearm is not a typical pressure area; therefore, the likelihood of the redness being related to pressure is very small. The observation should be documented, but because it may be a sign of an infection and immunosuppression, the physician must also be notified. 11. What is the nurses best response to a mother whose child has a diagnosis of acute lymphoblastic leukemia and is expressing guilt about not having responded sooner to her childs symptoms?
CURRENT DIAGNOSIS AND TREATMENT PEDIATRICS 24TH LATEST a. You should always call the physician when your child has a change in what is normal for him. b. It is better to be safe than sorry. c. It is not uncommon for parents not to notice subtle changes in their childrens health. d. I hope this delay does not affect the treatment plan. ANS: C Suggesting that noticing subtle changes in their childrens health is not uncommon minimizes the role the mother played in not seeking early medical attention. It also displays empathy, which helps to build trust, thereby enabling the mother to talk about her feelings. Identifying concerns and clarifying misconceptions will help families cope with the stress of chronic illness. The goal is to relieve the mothers guilt and build trust so that she can talk about her feelings. Telling the mother that she should have called the pediatrician will only reinforce her guilt. Adages such as It is better to be safe than sorry are flippant and reinforce the belief that the mother was negligent, which will only increase her guilt. Telling the mother that you hope the delay does not affect the treatment plan shows a total lack of empathy and would increase the mothers feelings of guilt. 12. Which is an appropriate nursing action before surgery when caring for a child diagnosed with a Wilms tumor? a. Limit fluid intake. b. Do not palpate the abdomen. c. Force oral fluids. d. Palpate the abdomen every 4 hours. ANS: B Excessive manipulation of the tumor area can cause seeding of the tumor and spread of the malignant cells. Fluids are not routinely limited in a child with a Wilms tumor. However, intake and output are important because of the kidney involvement. Fluids are not forced on a child with a Wilms tumor. Normal intake for age is usually maintained. The abdomen of a child with a
CURRENT DIAGNOSIS AND TREATMENT PEDIATRICS 24TH LATEST Wilms tumor should never be palpated because of the danger of seeding the tumor and spreading malignant cells. 13. A child with acute lymphocytic leukemia (ALL) is getting chemotherapy for the first time and is at risk for tumor lysis syndrome (TLS). The nurse monitors for which risk associated with TLS? a. Liver failure b. Central nervous system (CNS) deficit c. Kidney failure d. Respiratory distress ANS: C In TLS, the tumors intracellular contents are dumped into the childs extracellular fluid as the tumor cells are lysed in response to chemotherapy. Because of the large volume of these cells, their intracellular electrolytes overload the kidneys and, if not monitored, can cause kidney failure. TLS is related to intracellular electrolytes overloading the kidney as a response to the rapid lysis of tumor cells. This does not affect the liver, the CNS, or the lungs and cause respiratory distress. 14. Which is a symptom of a brain tumor in an infant? a. Blurred vision b. Increased head circumference c. Vomiting when getting out of bed d. Headache ANS: B Manifestations of brain tumors vary with tumor location and the childs age and development. Infants with brain tumors may be irritable or lethargic, feed poorly, and have increased head circumference with a bulging fontanel. Visual changes such as nystagmus, diplopia, and strabismus are manifestations of a brain tumor but would not be able to be verbalized by an
CURRENT DIAGNOSIS AND TREATMENT PEDIATRICS 24TH LATEST infant. The change in position on awakening causes an increase in intracranial pressure, which is manifested as vomiting. Vomiting on awakening is considered a hallmark symptom of a brain tumor, but infants do not get themselves out of bed in the morning. Increased intracranial pressure resulting from a brain tumor is manifested as a headache but could not be verbalized by an infant. 15. Which is an expected physical assessment finding for an adolescent with a diagnosis of Hodgkins disease? a. Protuberant, firm abdomen b. Enlarged painless, firm cervical lymph nodes c. Soft tissue swelling d. Soft to hard, nontender mass in the pelvic area ANS: B Painless, firm, movable adenopathy (enlarged lymph nodes) palpated in the cervical region is an expected assessment finding in Hodgkins disease. Other systemic symptoms include unexplained fevers, weight loss, and night sweats. A protuberant, firm abdomen is present in many cases of neuroblastoma. Soft tissue swelling around the affected bone is a manifestation of Ewings sarcoma. A soft to hard, nontender mass can be palpated when rhabdomyosarcoma is present. 16. Which nursing intervention should not be included in the postoperative plan of care for a child undergoing surgery for a brain tumor? a. Position with head lower than body. b. Perform neurological assessments. c. Assess dressings for drainage. d. Monitor temperature. ANS: A The child is never placed in the Trendelenburg position because it increases intracranial pressure and the risk of bleeding. Increased intracranial pressure is a risk in the postoperative period. The
CURRENT DIAGNOSIS AND TREATMENT PEDIATRICS 24TH LATEST nurse would assess the childs neurological status frequently. Hemorrhage is a risk in the postoperative period. The childs dressing would be inspected frequently for bleeding. Temperature is monitored closely because the child is at risk for infection in the postoperative period. 17. A child with non-Hodgkins lymphoma will be starting chemotherapy. Which intervention is initiated before chemotherapy to prevent tumor lysis syndrome? a. Insertion of a central venous catheter b. Intravenous (IV) hydration containing sodium bicarbonate c. Placement of an externalized ventriculoperitoneal (VP) shunt d. Administration of pneumococcal and H. influenzae type B vaccines ANS: B Intensive hydration with an IV fluid containing bicarbonate alkalinizes the urine to help prevent the formation of uric acid crystals, which damage the kidney. A central venous catheter is placed to assist in delivering chemotherapy. An externalized VP shunt may be placed to relieve intracranial pressure caused by a brain tumor. If a splenectomy is necessary for a child with Hodgkins disease, the pneumococcal and H. influenzae vaccines are administered before the surgery. 18. The nurse is aware that an abdominal mass found in a 10-month-old infant corresponds with which cancer? a. Osteogenic sarcoma b. Rhabdomyosarcoma c. Neuroblastoma d. Non-Hodgkins lymphoma ANS: C Neuroblastoma is found exclusively in infants and children. In most cases of neuroblastoma, a primary abdominal mass and protuberant, firm abdomen are present. Osteogenic sarcoma is a
CURRENT DIAGNOSIS AND TREATMENT PEDIATRICS 24TH LATEST bone tumor. Bone tumors typically affect older children. Rhabdomyosarcoma is a malignancy of muscle, or striated tissue. It occurs most often in the periorbital area, in the head and neck in younger children, or in the trunk and extremities in older children. Non-Hodgkins lymphoma is a neoplasm of lymphoid cells. Painless, enlarged lymph nodes are found in the cervical or axillary region. Abdominal signs and symptoms do not include a mass. 19. Which is an appropriate intervention for a hospitalized child with an absolute neutrophil count (ANC) of 900/mm3? a. Inspect the childs skin for breaks and redness every shift. b. Administer the measles, mumps, rubella (MMR) vaccine if the child is not fully immunized. c. Check urine and stools for blood. d. Teach the child to blow the nose gently. ANS: A An ANC of 900/mm3 reflects a moderate risk of infection. The skin provides a barrier against infection. All areas should be inspected carefully for signs of infection. A live virus vaccine, such as MMR, could produce infection in an immunocompromised child. An appropriate intervention for a child with a low platelet count would be to check urine and stools for blood; an appropriate precaution would be to teach the child to blow the nose gently. 20. A child with acute myeloblastic leukemia is scheduled to have a bone marrow transplant (BMT). The donor is the childs own umbilical cord blood that had been previously harvested and banked. What type of BMT would this be? a. Autologous b. Allogeneic c. Syngeneic d. Stem cell ANS: A
CURRENT DIAGNOSIS AND TREATMENT PEDIATRICS 24TH LATEST In an autologous transplant, the childs own marrow or previously harvested and banked cord blood is used. In an allogeneic BMT, histocompatibility has been matched with a related or unrelated donor. In a syngeneic transplant, the child receives bone marrow from an identical twin. A stem cell transplantation uses a unique immature cell present in the peripheral circulation. 21. What should the nurse teach parents about oral hygiene for the child receiving chemotherapy? a. Brush the teeth briskly to remove bacteria. b. Use a mouthwash that contains alcohol. c. Inspect the childs mouth daily for ulcers. d. Perform oral hygiene twice a day. ANS: C The childs mouth is inspected regularly for ulcers. At the first sign of ulceration, an antifungal drug is initiated. The teeth should be brushed with a soft-bristled toothbrush. Excessive force with brushing should be avoided because delicate tissue could be broken, causing infection or bleeding. Mouthwashes containing alcohol may be drying to oral mucosa, thus breaking down the protective barrier of the skin. 22. Which is the best choice when a child with mucositis asks for something to drink? a. Hot chocolate b. Lemonade c. Popsicle d. Orange juice ANS: C Cool liquids are soothing and ice pops are usually well tolerated. A hot beverage can be irritating to mouth ulcers. Citrus products may be very painful to an ulcerated mouth.
CURRENT DIAGNOSIS AND TREATMENT PEDIATRICS 24TH LATEST MULTIPLE RESPONSE 1. When an adolescent with a new diagnosis of Ewings sarcoma asks the nurse about treatment, the nurses response should be based on which information? Select all that apply. a. This type of tumor invades the bone. b. Management includes chemotherapy, surgery, and radiation. c. Ewings sarcoma is usually not responsive to either chemotherapy or radiation. d. Affected bones such as ribs and proximal fibula may be removed to excise the tumor. ANS: A, B, D Ewings sarcoma invades the bone and is found most often in the midshaft of long bones, especially the femur, vertebrae, ribs, and pelvic bones. Treatment for Ewings sarcoma begins with chemotherapy to decrease tumor bulk, followed by surgical resection of the primary tumor. Local control of the tumor can be achieved with surgery or radiation. The affected bone may be removed if it will not affect the childs functioning. Ribs and the proximal fibula are considered expendable and may be removed to excise the tumor without affecting function. Ewings sarcoma is responsive to both chemotherapy and radiation. 2. A child with a brain tumor is undergoing radiation therapy. The nurse should include which information in the discharge instructions to the childs parents? Select all that apply. a. Apply over-the-counter creams to the area daily. b. Avoid excessive skin exposure to the sun. c. Use a washcloth when cleaning the area receiving radiation. d. Plan for adequate rest periods for the child. e. A darkening of the skin receiving radiation is expected. ANS: B, D, E Children receiving cranial radiation are particularly affected by fatigue and an increased need for sleep during and shortly after completion of the course of radiation. Skin damage can include
CURRENT DIAGNOSIS AND TREATMENT PEDIATRICS 24TH LATEST changes in pigmentation (darkening), redness, peeling, and increased sensitivity. Extra care must be taken to avoid excessive skin exposure to heat, sunlight, friction (such as rubbing with a towel or washcloth), and creams or moisturizers. Only topical creams and moisturizers prescribed by the radiation oncologist should be applied to the radiated skin. SHORT ANSWER 1. What is the absolute neutrophil count (ANC) for a WBC of total count 3000 with 30% neutrophils and 25% bands? ANS: 1650 The absolute neutrophil count can be easily calculated using the results from the childs CBC. Use the following formula: Add the percent of neutrophils and the percent of bands. Convert the summed percentage into decimal form (e.g., 55% = 0.55). Multiply that figure by the WBC (stated in thousands).
Chapter 32. Pain Management and Palliative Care 1. The nurse is aware when assessing a child for pain that: a. neonates do not feel pain. b. pain is an individualized experience. c. children do not remember pain. d. a child must cry to express pain.
CURRENT DIAGNOSIS AND TREATMENT PEDIATRICS 24TH LATEST ANS: B The manner and intensity of how a child expresses pain are dependent on the individual childs experiences. It is a myth that neonates do not feel pain. Neonates do express a total-body response to pain with a cry that is intense, high pitched, and harsh sounding. It is a myth that children do not remember pain. Children of all ages have been reported to have sleeping and eating disruptions after painful experiences. Not all children will cry to express pain. 2. When pain is assessed in an infant, it would be inappropriate to assess for: a. facial expressions of pain. b. localization of pain. c. crying. d. thrashing of extremities. ANS: B Infants cannot localize pain to any great extent. Frowning, grimacing, and facial flinching in an infant may indicate pain. Infants often exhibit high-pitched, tense, harsh crying to express pain. Infants may exhibit thrashing extremities in response to a painful stimulus. 3. The nurse is aware that physiological changes associated with pain in the infant include which finding(s)? a. Increased blood pressure and decreased arterial saturation b. Decreased blood pressure and increased arterial saturation c. Increased urine output and increased heart rate d. Decreased urine output and increased blood pressure ANS: A Increased blood pressure and heart rate and decreased arterial saturation are physiological responses to pain in the neonate. An increase in blood pressure and a decrease in arterial saturation are documented when the neonate is feeling pain. Although an increase in heart rate is
CURRENT DIAGNOSIS AND TREATMENT PEDIATRICS 24TH LATEST associated with pain and an increase in blood pressure occurs with pain, urine output changes have not been associated with pain. 4. Which statement best reflects a myth that may interfere with the treatment of pain in infants and children? a. Infants may have sleep difficulties after a painful event. b. Children and infants are more susceptible to respiratory depression from narcotics. c. Pain in children is multidimensional and subjective. d. A childs cognitive level does not influence the pain experience. ANS: B No data are available to support the belief that infants and children are at higher risk of respiratory depression when given narcotic analgesics. This is a myth. It is true that infants may have sleep difficulties after a painful event. This is not a myth. It is true that pain in children is multidimensional and subjective. This is not a myth. The childs cognitive level, along with emotional factors and past experiences, does influence the perception of pain in children. This is not a myth. 5. The nurse caring for the child in pain knows that distraction: a. can give total pain relief to the child. b. is effective when the child is in severe pain. c. is the best method for pain relief. d. must be developmentally appropriate to refocus attention. ANS: D Distraction can be very effective in helping to control pain, but it must be appropriate to the childs developmental level. It is rarely able to provide total pain relief and is not the best method for pain relief. Children in severe pain are not distractible. 6. Which medication is the most effective choice for treating pain associated with inflammation?
CURRENT DIAGNOSIS AND TREATMENT PEDIATRICS 24TH LATEST a. Opioids b. Acetaminophen c. Ibuprofen d. Midazolam ANS: C Ibuprofen is a type of nonsteroidal antiinflammatory drug (NSAID) that is used primarily for pain associated with inflammation. Opioids are the preferred drugs for the management of acute, severe pain, including postoperative pain, posttraumatic pain, pain from vaso-occlusive crisis, and chronic cancer pain. Acetaminophen lacks the antiinflammatory effects of NSAIDs and provides only minimal antiinflammatory relief. Midazolam (Versed) is a short-acting drug used for conscious sedation and preoperative sedation, and as an induction agent for general anesthesia. 7. When using the Poker Chip Tool, it is important for the nurse to know which fact? a. Any number of chips can be used. b. Only a specified number of chips can be used. c. The assessment tool is used with adolescents. d. The assessment tool is most effectively used with 2-year-old children. ANS: B In the Poker Chip Tool, four chips are used to represent a hurt. One chip represents a little hurt, and four chips represent the most hurt the child could have. Pain tools are valid only if used as directed; this tool uses four chips. Adolescents are able to think abstractly. They can describe, quantify, and identify intensity and feelings about pain. The Poker Chip Tool is recommended for children ages 4 to 12. Self-report tools are effective in children older than 3 years of age, not 2 years of age. 8. An appropriate tool to assess pain in a 3-year-old child would be the: a. Visual Analogue Scale (VAS).
CURRENT DIAGNOSIS AND TREATMENT PEDIATRICS 24TH LATEST b. Adolescent and Pediatric Pain Tool. c. Oucher Tool. d. Poker Chip Tool. ANS: C The Oucher Tool can be used to assess pain for children 3 to 12 years of age. The VAS is indicated for use with older school-age children and adolescents. It can be used with younger school-age children, although less-abstract tools are more appropriate. The Adolescent and Pediatric Pain Tool is indicated for use with children 8 to 17 years of age. The Poker Chip Tool can be used to assess pain in children 4 to 12 years of age. 9. At which developmental stage is the child first able to localize pain and describe both the amount and the intensity of the pain felt? a. Toddler stage b. Preschool stage c. School-age stage d. Adolescent stage ANS: B The preschool stage is the period when the child is first able to describe the location and intensity of pain, stating, for example, ear hurts bad, when feeling pain. The toddler expresses pain by guarding or touching the painful area, verbalizes words that indicate discomfort, such as ouch and hurt, and demonstrates generalized restlessness when feeling pain. The school-age child describes both the location of the pain and its intensity. The adolescent also describes the location and intensity of pain. 10. Which statement indicates a nurses lack of understanding about the use of patient-controlled analgesia (PCA) therapy? a. Children as young as 3 years old can effectively and successfully use a PCA pump.
CURRENT DIAGNOSIS AND TREATMENT PEDIATRICS 24TH LATEST b. Two registered nurses (RNs) are required to double check the dosage and programmed administration of opioids. c. The child should be carefully monitored for signs and symptoms of overmedication with opioids. d. Naloxone (Narcan) should be readily available. ANS: A Children as young as 5 years old have effectively used PCA therapy. Further data are needed to evaluate the use of PCA therapy in children younger than 5 years of age. Two RNs are needed to check the amount of opioid being administered. Once the opioid infusion is hung and programmed, a second RN must double check the process. Children receiving PCA therapy should be monitored closely to ensure effective pain control and for signs or symptoms of overmedication. Initially, vital signs should be monitored every 15 to 30 minutes and then every 2 to 4 hours. Respiratory rate should be assessed every hour. Narcan should be readily available to reverse opioid overmedication exhibited by respiratory distress. 11. Which assessment indicates to a nurse that a 2-year-old child is in need of pain medication? a. The child is restless and guarding the painful site. b. The childs current vital signs are consistent with vital signs over the past 4 hours. c. The child is quieted when held and cuddled. d. The child has just returned from the recovery room and is crying. ANS: A Behaviors such as generalized restlessness, guarding the site, and touching the painful area are signs of pain in the toddler. Current vital signs that are consistent with earlier vital signs do not suggest that the child is feeling pain. Response to comforting behaviors does not suggest the child is feeling pain. Crying in a child who is returning from the recovery room may not be indicative of pain. The child may just be fearful or having anxiety because of the strange surroundings and having just completed surgery. 12. When assessing pain in any child, the nurse should consider which information?
CURRENT DIAGNOSIS AND TREATMENT PEDIATRICS 24TH LATEST a. Any pain assessment tool can be used to assess pain in children. b. Children as young as age 1 year use words to express pain. c. The childs behavioral, physiological, and verbal responses are valuable when assessing pain. d. Pain assessment tools are minimally effective for communicating about pain. ANS: C Childrens behavioral, physiological, and verbal responses are indicative when assessing pain. The use of pain measurement tools greatly assists in communicating about pain. The childs age is important in determining the appropriate pain assessment tool to use. Developmentally appropriate assessment tools need to be used to effectively identify and determine the level of pain felt by a child. Toddlers may use words such as ouch or hurt to identify pain, but infants and young children may not have the language or cognitive abilities to express pain. Pain assessment tools when used appropriately are successful and efficient in identifying and quantifying pain with children. Behavioral and physiological signs and symptoms in combination with pain assessment tools are most effective in diagnosing pain levels in children. MULTIPLE RESPONSE 1. A nurse is administering an opioid medication to a child. Which side effects should the nurse watch for with this classification of medication? Select all that apply. a. Respiratory depression b. Hepatic damage c. Constipation d. Pruritus e. Gastrointestinal bleeding ANS: A, C, D The nurse should remember opioids can produce sedation and respiratory depression, in addition to analgesia. Other adverse effects can include constipation, pruritus, nausea, vomiting, cough
CURRENT DIAGNOSIS AND TREATMENT PEDIATRICS 24TH LATEST suppression, and urinary retention. Acetaminophen (Tylenol) is associated with hepatic damage and nonsteroidal antiinflammatory drugs (NSAIDs) are associated with gastrointestinal bleeding. 2. A nurse is assessing pain on a 12-year-old child. Which pain assessment tools are developmentally appropriate for the nurse to use? Select all that apply. a. CRIES pain scale b. Numeric Rating Scale c. Visual Analog Scale d. FLACC e. Adolescent and Pediatric Pain Tool ANS: B, C, E Developmentally appropriate pain assessment tools for a 12-year-old child include the Numeric Rating Scale, the Visual Analog Scale, and the Adolescent and Pediatric Pain Tool. The CRIES pain scale is recommended for neonates to 6 months and the FLACC is recommended for children who are preverbal.
Chapter 33. Immunodeficiency 1. A nurse in a well-child clinic is teaching parents about their childs immune system. Which statement, made by the nurse, is correct? a. The immune system distinguishes and actively protects the bodys own cells from foreign substances. b. The immune system is fully developed by 1 year of age. c. The immune system protects the child against communicable diseases in the first 6 years of life. d. The immune system responds to an offending agent by producing antigens. ANS: A
CURRENT DIAGNOSIS AND TREATMENT PEDIATRICS 24TH LATEST The immune system responds to foreign substances, or antigens, by producing antibodies and storing information. Intact skin, mucous membranes, and processes such as coughing, sneezing, and tearing help maintain internal homeostasis. Children up to the age of 6 or 7 years have limited antibodies against common bacteria. The immunoglobins reach adult levels at different ages. Immunization is the basis from which the immune system activates protection against some communicable diseases. Antibodies are produced by the immune system against invading agents, or antigens. 2. A nurse is teaching parents about the importance of immunizations for infants because of immaturity of the immune system. The parents demonstrate that they understand the teaching if they make which statement? a. The spleen reaches full size by 1 year of age. b. IgM, IgE, and IgD levels are high at birth. c. IgG levels in the newborn infant are low at birth. d. Absolute lymphocyte counts reach a peak during the first year. ANS: D Absolute lymphocyte counts reach a peak during the first year. The spleen reaches its full size during adulthood. IgM, IgE, and IgD are normally in low concentration at birth. IgM, IgE, IgA, and IgD do not cross the placenta. The term newborn infant receives an adult level of IgG as a result of transplacental transfer from the mother. 3. Which statement is true regarding how infants acquire immunity? a. The infant acquires humoral and cell-mediated immunity in response to infections and immunizations. b. The infant acquires maternal antibodies that ensure immunity up to 12 months of age. c. Active immunity is acquired from the mother and lasts 6 to 7 months. d. Passive immunity develops in response to immunizations. ANS: A
CURRENT DIAGNOSIS AND TREATMENT PEDIATRICS 24TH LATEST Infants acquire long-term active immunity from exposure to antigens and vaccines. Immunity is acquired actively and passively. The term infants passive immunity is acquired from the mother and begins to dissipate during the first 6 to 8 months of life. Active immunity develops in response to immunizations. 4. A nurse is teaching parents about transmission of human immunodeficiency virus (HIV) in the pediatric population. The nurse should relate that the most common mode of transmission of HIV virus is: a. Perinatal transmission b. Sexual abuse c. Blood transfusions d. Poor hand washing ANS: A Perinatal transmission accounts for the highest percentage of HIV infections in children. Infected women can transmit the virus to their infants across the placenta during pregnancy, at delivery, and through breast-feeding. Cases of HIV infection from sexual abuse have been reported; however, perinatal transmission accounts for most pediatric HIV infections. Although in the past some children became infected with HIV through blood transfusions, improved laboratory screening has significantly reduced the probability of contracting HIV from blood products. Poor hand washing is not an etiology of HIV infection. 5. A nurse is preparing to administer routine immunizations to an infant who is HIV positive. What is the American Academy of Pediatrics recommendation for immunizing infants who are HIV positive? a. Follow the routine immunization schedule. b. Routine immunizations are administered. Assess CD4+ counts before administering the MMR and varicella vaccinations. c. Do not give immunizations because of the infants altered immune status. d. Eliminate the pertussis vaccination because of the risk of convulsions.
CURRENT DIAGNOSIS AND TREATMENT PEDIATRICS 24TH LATEST ANS: B Routine immunizations are appropriate. CD4+ cells are monitored when deciding whether to provide live virus vaccines. If the child is severely immunocompromised, the MMR vaccine is not given. The varicella vaccine can be considered on the basis of the childs CD4+ counts. Only inactivated polio (IPV) should be used for HIV-infected children. Immunizations are given to infants who are HIV positive. The pertussis vaccination is not eliminated for an infant who is HIV positive. 6. Which recommendation by the nurse is appropriate for a mother who has a preschool child who refuses to take the medications for HIV infection? a. Mix medications with chocolate syrup or follow with chocolate candy. b. Mix the medications with milk or an essential food. c. Skip the dose of medication if the child protests too much. d. Mix the medication in a syringe, hold the child down firmly, and administer the medication. ANS: A Liquid forms of HIV medications may be foul tasting or have a gritty texture. Chocolate would help to make these foods more palatable and is liked by most children. Medications should be mixed with nonessential foods. Doses of medication should never be skipped. Fighting with the child or using force should be avoided. A nonessential food that will make the taste of the medication more palatable for the child should be the correct action. The administration of medications for the child with HIV becomes part of the familys everyday routine for years. 7. What is the primary nursing concern for a hospitalized child with HIV infection? a. Maintaining growth and development b. Eating foods that the family brings to the child c. Consideration of parental limitations and weaknesses d. Resting for 2 to 3 hours twice a day
CURRENT DIAGNOSIS AND TREATMENT PEDIATRICS 24TH LATEST ANS: A Maintaining growth and development is a major concern for the child with HIV infection. Frequent monitoring for failure to thrive, neurological deterioration, or developmental delay is important for HIV-infected infants and children. Nutrition, which contributes to a childs growth, is a nursing concern; however, it is unnecessary for family members to bring food to the child. Although an assessment of parental strengths and weaknesses is important, it will be imperative for healthcare providers to focus on the parental strengths not weaknesses. This is not as important as the frequent assessment of the childs growth and development. Rest is a nursing concern, but it is not as high a priority as maintaining growth and development. Rest periods twice a day for 2 to 3 hours may not be appropriate. 8. What should the nurse include in a teaching plan for a mother of a toddler who will be taking prednisone for several months? a. The medication should be taken between meals. b. The medication needs to be discontinued because of the risks associated with long-term usage. c. The medication should not be stopped abruptly. d. The medication may lower blood glucose so the mother needs to observe the child for signs of hypoglycemia. ANS: C The dosage must be tapered before the drug is discontinued to allow the gradual return of function in the pituitary-adrenal axis. Prednisone should be taken with food to minimize or prevent gastrointestinal bleeding. Although there are adverse effects from long-term steroid use, the medication must not be discontinued without consulting a physician. Acute adrenal insufficiency can occur if the medication is withdrawn abruptly. The dosage needs to be tapered. The medication puts the child at risk for hyperglycemia, not hypoglycemia. 9. A nurse assesses a child on long-term systemic corticosteroid therapy for which condition? a. Hypotension b. Dilation of blood vessels in the cheeks
CURRENT DIAGNOSIS AND TREATMENT PEDIATRICS 24TH LATEST c. Growth delays d. Decreased appetite and weight loss ANS: C Growth delay is associated with long-term steroid use related to protein catabolism and decreased growth hormone. Hypertension is a clinical manifestation of long-term systemic steroid administration. Dilation of blood vessels in the cheeks is associated with an excess of topically administered steroids. Increased appetite and weight gain are clinical manifestations of excess systemic corticosteroid therapy. 10. Which statement by the parent of a 5-year-old child with acquired immunodeficiency syndrome (AIDS) regarding prescribed antiretroviral agents indicates that she has a good understanding of disease management? a. When my childs pain increases, I double the recommended dosage of antiretroviral medication. b. Addiction is a risk, so I use the medication only as ordered. c. Doses of the antiretroviral medication are selected on the basis of my childs age and growth. d. By the time my child is an adolescent, she will not need her antiretroviral medications any longer. ANS: C Doses of antiretroviral medication to treat HIV infection for infants and children are based on individualized age and growth considerations. Antiretroviral medications are not administered for pain relief. Doubling the recommended dosage of any medication is not appropriate without an order from the physician. Addiction is not a realistic concern with antiretroviral medications. Antiretroviral medications are still needed during adolescence. Doses for adolescents are based on pubertal status by Tanner staging. 11. A mother of a child in the terminal stages of AIDS tells the nurse that her child wants to celebrate his birthday early because he wont be here on his birthday. Which is the best response the nurse can make to this mother?
CURRENT DIAGNOSIS AND TREATMENT PEDIATRICS 24TH LATEST a. What does your husband think about giving the party for the child? b. How does the family feel about your giving in to the child? c. Ill children can be very manipulative. d. Is this the first time he has spoken about death? ANS: D Dying children know they are dying. Disclosure of awareness of death comes in various ways and needs to be identified by the family and the nurse. The major concern is the childs disclosure of awareness of death, not the husbands reaction. Making statements such as giving in is inappropriate when seeking information. Manipulation is not a major concern during the terminal stage of disease. 12. Which intervention is appropriate for a child receiving high doses of steroids? a. Limit activity and receive home schooling. b. Decrease the amount of potassium in the diet. c. Administer a killed virus vaccine. d. Monitor for seizure activity. ANS: C The child on high doses of steroids should not receive live virus vaccines because of immunosuppression. Limiting activity and home schooling are not routine for a child receiving high doses of steroids. The child receiving steroids is at risk for hypokalemia and needs potassium in the diet. Children on steroids are not typically at risk for seizures. 13. The nurse observes a red rash that spreads across the childs cheeks and nose. This assessment finding is characteristic of which of the following conditions? a. Systemic lupus erythematosus (SLE) b. Rheumatic fever c. Kawasaki disease d. Anaphylactic reaction
CURRENT DIAGNOSIS AND TREATMENT PEDIATRICS 24TH LATEST ANS: A A red, flat, or raised malar butterfly rash over the cheeks and bridge of the nose is a clinical manifestation of SLE. A major manifestation of rheumatic fever is erythema marginatum, which appears as red skin lesions spread peripherally over the trunk. An erythematous rash, induration of the hands and feet, and erythema of the palms and soles are manifestations of Kawasaki disease. Initial symptoms of anaphylaxis include severe itching and rapid development of erythema. 14. What is the major nursing concern for a child having an anaphylactic reaction? a. Identifying the offending allergen b. Ineffective breathing pattern c. Increased cardiac output d. Positioning to facilitate comfort ANS: B Laryngospasms resulting in ineffective breathing patterns are a life-threatening manifestation of anaphylaxis. The primary action is to assess airway patency, respiratory rate and effort, level of consciousness, oxygen saturation, and urine output. Determining the cause of an anaphylactic reaction is important to implement the appropriate treatment, but the primary concern is the airway. During anaphylaxis, the cardiac output is decreased. During the acute period of anaphylaxis, the nurses primary concern is the childs breathing. Positioning for comfort is not a primary concern during a crisis. 15. What is the drug of choice the nurse would administer in the acute treatment of anaphylaxis? a. Diphenhydramine (Benadryl) b. Cimetidine (Tagamet) c. Epinephrine (Adrenaline) d. Albuterol (Ventolin) ANS: C
CURRENT DIAGNOSIS AND TREATMENT PEDIATRICS 24TH LATEST Epinephrine is the first drug of choice in the immediate treatment of anaphylaxis. Treatment must be initiated immediately because it may only be a matter of minutes before shock occurs. Although diphenhydramine and a histamine inhibitor such as cimetidine may be indicated, epinephrine is the first drug of choice in the immediate treatment of anaphylaxis. Albuterol is not usually indicated for the treatment of anaphylaxis. MULTIPLE RESPONSE 1. Which home care instructions should the nurse provide to the parents of a child with acquired immunodeficiency syndrome (AIDS)? Select all that apply. a. Give supplemental vitamins as prescribed. b. Avoid yearly influenza vaccination. c. Administer trimethoprim-sulfamethoxazole (Bactrim) as prescribed. d. Notify the physician if child develops a cough or congestion. e. Missed doses of antiretroviral medication do not need to be recorded. ANS: A, C, D The parents should be taught that supplemental vitamins will be prescribed to aid in nutritional status. Bactrim is administered to prevent the opportunistic infection of Pneumocystis pneumonia. The physician should be notified if the child with AIDS develops a cough and congestion. The yearly influenza vaccination is recommended and any missed doses of antiretroviral medication need to be recorded and reported. OTHER 1. A child is having an anaphylactic response. Place in order the interventions a nurse should perform beginning with the initial (highest priority) intervention and ending with the lowest priority intervention. Use the following format for your answers: A, B, C, D a. Administer steroids and antihistamines as prescribed. b. Ensure an adequate airway.
CURRENT DIAGNOSIS AND TREATMENT PEDIATRICS 24TH LATEST c. Administer epinephrine as prescribed. d. Administer oxygen. e. Determine the cause of the reaction. ANS: B, C, D, A, E The airway should be stabilized first and then the epinephrine administered. Oxygen would be given next and the steroids and antihistamines given after the airway, epinephrine, and oxygen are initiated. Determining the cause of the reaction should be done last.
Chapter 34. Endocrine Disorders 1. A nurse is describing the endocrine system to a nursing student. Which statement best describes the negative feedback system of the pituitary gland? a. The pituitary gland produces five hormones that are interdependent. b. Target organs stimulate the hypothalamus, which in turn produces hormones. c. The anterior pituitary stimulates the production of posterior pituitary hormones. d. The pituitary gland secretes hormones that stimulate target organs to produce specific hormones. As hormonal secretions of the target organs increase, the pituitary gland decreases the secretion of the stimulating hormone. ANS: D The pituitary stimulates target organs to produce specific hormones. When enough hormone is produced, the target organ signals the pituitary to stop secretion of the stimulating hormone.
CURRENT DIAGNOSIS AND TREATMENT PEDIATRICS 24TH LATEST There are six hormones secreted by the anterior pituitary; they are not interdependent. The anterior pituitary gland stimulates target organs to produce specific hormones and does not control production of posterior pituitary hormones.
2. Which is the priority outcome of mandatory newborn screening for hypothyroidism? a. Appropriate community referral for affected families b. Parental education about raising a special needs child c. Early identification of the disorder d. Facilitation of parentchild bonding ANS: C Early identification of hypothyroidism is basic to the prevention of mental retardation in the child. Community referral is appropriate after a diagnosis is made. With early identification and treatment with thyroid hormone replacement, mental retardation is prevented. Facilitation of parentchild bonding is a consideration for all newborn infants. 3. What is a nursing goal for a 12-year-old child with Graves disease? a. Relief of constipation b. Allowing the adolescent to make decisions about taking her medication c. Verbal demonstration of an understanding about the medication regimen d. Development of alternative educational goals ANS: C To adhere to the medication schedule, children need to understand that the medication must be taken two or three times per day. The adolescent with Graves disease is not constipated. Making decisions about her medication is not appropriate for a 12-year-old child. The management of Graves disease does not interfere with school attendance and does not require alternative educational plans.
CURRENT DIAGNOSIS AND TREATMENT PEDIATRICS 24TH LATEST 4. Which symptom should be reported to the healthcare provider when a child is taking propylthiouracil? a. Nausea and vomiting b. Fever and sore throat c. Increased appetite and diarrhea d. Weight gain and mood swings ANS: B Propylthiouracil or methimazole is the treatment of choice for childhood hyperthyroidism. Fever and sore throat are clinical manifestations of neutropenia, a side effect of propylthiouracil. Nausea and vomiting are not common side effects of propylthiouracil. Increased appetite and diarrhea are clinical manifestations of Graves disease. Weight gain and mood swings are not considered side effects of propylthiouracil. 5. What information provided by the nurse would be helpful to a 15-year-old adolescent taking propylthiouracil three times a day? a. Pill dispensers and alarms on her watch can remind her to take the medication as ordered. b. She can take the medication when she is nervous and feels she needs it. c. She can take two pills before school and one pill at dinner, which will be easier for her to remember. d. Her mother can be responsible for reminding her when it is time to take her medication. ANS: A Propylthiouracil is an antithyroid medication that should be taken three times a day. Reminders will facilitate taking medication as ordered. This medication needs to be taken regularly, not on an as-needed basis. The dosage cannot be combined to reduce the frequency of administration. Because of the adolescents school schedule and activities, she, rather than her mother, needs to be responsible for her medication.
CURRENT DIAGNOSIS AND TREATMENT PEDIATRICS 24TH LATEST 6. A nurse is assessing a child with diabetes insipidus. Which sign should the nurse expect to note? a. Weight gain b. Increased urine specific gravity c. Increased urination d. Serum sodium level of 130 mEq/L ANS: C The deficiency of antidiuretic hormone associated with diabetes insipidus causes the body to excrete large volumes of dilute urine. Weight gain results from retention of water when there is an excessive production of antidiuretic hormone; in diabetes insipidus there is a decreased production of antidiuretic hormone. Concentrated urine is a sign of the syndrome of inappropriate antidiuretic hormone (SIADH), in which there is an excessive production of antidiuretic hormone. A deficiency of antidiuretic hormone, as with diabetes insipidus, results in an increased serum sodium concentration (greater than 145 mEq/L). 7. What should the nurse include in the teaching plan for parents of a child with diabetes insipidus who is receiving DDAVP intranasally? a. Increase the dosage of DDAVP as the urine specific gravity (SG) increases. b. Give DDAVP only if the urine output decreases. c. Child should have free access to water and toilet facilities at school. d. Cleanse the skin before administering the transdermal patch. ANS: C The childs teachers should be aware of the diagnosis and the child should have free access to water and toilet facilities at school. DDAVP needs to be given as ordered by the physician. If the parents are monitoring urine SG at home, they would not increase the medication dose for increased SG; the physician may order an increased dosage for very dilute urine with decreased SG. DDAVP needs to be given continuously as ordered by the physician. DDAVP is typically given intranasally or by subcutaneous injection. For nocturnal enuresis, it may be given orally.
CURRENT DIAGNOSIS AND TREATMENT PEDIATRICS 24TH LATEST 8. A nurse is explaining growth hormone deficiency to parents of a child admitted to rule out this problem. Which metabolic alteration should the nurse explain to the parent that is related to growth hormone deficiency? a. Hypocalcemia b. Hypoglycemia c. Diabetes insipidus d. Hyperglycemia ANS: B Growth hormone helps maintain blood sugar at normal levels. Symptoms of hypocalcemia are associated with hypoparathyroidism. Diabetes insipidus is a disorder of the posterior pituitary. Growth hormone is produced by the anterior pituitary. Hyperglycemia results from an insufficiency of insulin, which is produced by the beta cells in the islets of Langerhans in the pancreas. 9. Which is the most appropriate intervention for the parents of a 6-year-old child with precocious puberty? a. Advise the parents to consider birth control for their daughter. b. Explain the importance of having the child foster relationships with same-age peers. c. Assure the childs parents that there is no increased risk for sexual abuse because of her appearance. d. Counsel parents that there is no treatment currently available for this disorder. ANS: B Despite the childs appearance, the child needs to be treated according to her chronological age and to interact with children in the same age group. An expected outcome is that the child will adjust socially by exhibiting age-appropriate behaviors and social interactions. Advising the parents of a 6-year-old to put their daughter on birth control is not appropriate. Parents need to be aware that there is an increased risk of sexual abuse for a child with precocious puberty.
CURRENT DIAGNOSIS AND TREATMENT PEDIATRICS 24TH LATEST Treatment for precocious puberty is the administration of gonadotropin-releasing hormone blocker, which slows or reverses the development of secondary sexual characteristics and slows rapid growth and bone aging. 10. A neonate is displaying tremors, twitching, and irritability. This is suggestive of: a. hypocalcemia. b. hypothyroidism. c. hypoglycemia. d. phenylketonuria (PKU). ANS: A When hypocalcemia is present, neonates may display twitching, tremors, irritability, jitteriness, electrocardiographic changes, and, rarely, seizures. An infant with hypothyroidism may exhibit skin mottling, a large fontanel, a large tongue, hypotonia, slow reflexes, a distended abdomen, prolonged jaundice, lethargy, constipation, feeding problems, and coldness to touch. Hypoglycemia causes the neonate to exhibit jitteriness, poor feeding, lethargy, seizures, and respiratory alterations including apnea, hypotonia, high-pitched cry, bradycardia, cyanosis, and temperature instability. Infants with PKU may initially have digestive problems with vomiting, and they may have a musty or mousy odor to the urine, infantile eczema, hypertonia, and hyperactive behavior. 11. What is the best time for the nurse to assess the peak effectiveness of subcutaneously administered Regular insulin? a. Two hours after administration b. Four hours after administration c. Immediately after administration d. Thirty minutes after administration ANS: A
CURRENT DIAGNOSIS AND TREATMENT PEDIATRICS 24TH LATEST The peak action for Regular (short-acting) insulin is 2 to 3 hours after subcutaneous administration. The duration of Regular (short-acting) insulin is only 3 to 6 hours. Peak action occurs 2 to 3 hours after the insulin is administered. Subcutaneously administered Regular (short-acting) insulin has an onset of action of 30 to 60 minutes after injection. The effectiveness of subcutaneously administered, short-acting insulin cannot be assessed immediately after administration. Thirty minutes corresponds to the onset of action for Regular (short-acting) insulin. 12. Which statement made by a 14-year-old adolescent who is newly diagnosed with insulindependent diabetes mellitus (IDDM) indicates a need for further teaching? a. I should eat meals and snack at the same time every day. b. Exercise will decrease my insulin requirements. c. It is okay to drink chocolate milk with meals. d. I need to check my sugars before meals and at bedtime. ANS: C Chocolate milk is high in carbohydrates. Carbohydrates raise blood glucose levels. A beverage low in carbohydrates is a better choice. Meals and snacks should be eaten at regular times. Exercise decreases insulin requirements. Checking serum glucose before breakfast and dinner is appropriate. 13. What is the primary concern for a 7-year-old child with insulin-dependent diabetes mellitus (IDDM) who asks his mother not to tell anyone at school that he has diabetes? a. The childs safety b. The privacy of the child c. Development of a sense of industry d. Peer group acceptance ANS: A
CURRENT DIAGNOSIS AND TREATMENT PEDIATRICS 24TH LATEST Safety is the primary issue. School personnel need to be aware of the signs and symptoms of hypoglycemia and hyperglycemia and the appropriate interventions. Privacy is not a lifethreatening concern. The treatment of IDDM should not interfere with the school-age childs development of a sense of industry. Peer group acceptance and body image are issues for the early adolescent with IDDM. This is not of greater priority than the childs safety. 14. Which is the best nursing action when a child with insulin-dependent diabetes mellitus is sweating, trembling, and pale? a. Offer the child a glass of water. b. Give the child 5 units of Regular insulin subcutaneously. c. Give the child a glass of orange juice. d. Give the child glucagon subcutaneously. ANS: C Four ounces of orange juice is an appropriate treatment for the conscious child who is exhibiting signs of hypoglycemia. A glass of water is not indicated in this situation. An easily digested carbohydrate is indicated when a child exhibits symptoms of hypoglycemia. Insulin would lower blood glucose and is contraindicated for a child with hypoglycemia. Subcutaneous injection of glucagon is used to treat hypoglycemia when the child is unconscious. 15. Which sign is the nurse most likely to assess in a child with hypoglycemia? a. Urine positive for ketones and serum glucose greater than 300 mg/dL b. Normal sensorium and serum glucose greater than 160 mg/dL c. Irritability and serum glucose less than 70 mg/dL d. Increased urination and serum glucose less than 120 mg/dL ANS: C Irritability and serum glucose less than 70 mg/dL are neuroglycopenic manifestations of hypoglycemia. Serum glucose greater than 300 mg/dL and urine positive for ketones are indicative of diabetic ketoacidosis. Normal sensorium and serum glucose greater than 160 mg/dL
CURRENT DIAGNOSIS AND TREATMENT PEDIATRICS 24TH LATEST are associated with hyperglycemia. Increased urination is an indicator of hyperglycemia. A serum glucose level less than 120 mg/dL is within normal limits. 16. When would a child diagnosed with insulin-dependent diabetes mellitus most likely demonstrate a decreased need for insulin? a. During the honeymoon phase b. During adolescence c. During growth spurts d. During minor illnesses ANS: A During the honeymoon phase, which may last from a few weeks to a year or longer, the child is likely to need less insulin. During adolescence, physical growth and hormonal changes contribute to an increase in insulin requirements. Insulin requirements are typically increased during growth spurts. Stress either from illness or from events in the environment can cause hyperglycemia. Insulin requirements are increased during periods of minor illness. 17. What should a nurse suggest to the parent of a child with insulin-dependent diabetes mellitus (IDDM) who is not eating as a result of a minor illness? a. Give the child half his regular morning dose of insulin. b. Substitute calorie-containing liquids for solid food to maintain normal serum glucose levels. c. Give the child plenty of unsweetened, clear liquids to prevent dehydration. d. Take the child directly to the emergency department. ANS: B Calorie-containing liquids will maintain normal serum glucose levels and decrease the risk of hypoglycemia. The child should receive his regular dose of insulin even if he does not have an appetite. If the child is not eating as usual, he needs calories to prevent hypoglycemia. During periods of minor illness, the child with IDDM can be managed safely at home.
CURRENT DIAGNOSIS AND TREATMENT PEDIATRICS 24TH LATEST 18. Which is the nurses best response to the parents of a 10-year-old child newly diagnosed with insulin-dependent diabetes mellitus (IDDM) who are concerned about the childs continued participation in soccer? a. Consider the swim team as an alternative to soccer. b. Encourage intellectual activity rather than participation in sports. c. It is okay to play sports such as soccer when the weather is moderate. d. Give the child an extra 15 to 30 grams of carbohydrate snack before soccer practice. ANS: D Exercise lowers blood glucose levels. A snack with 15 to 30 grams of carbohydrates before exercise will decrease the risk of hypoglycemia. Soccer is an appropriate sport for a child with IDDM as long as the child prevents hypoglycemia by eating a snack. Participation in sports is not contraindicated for a child with IDDM. The child with IDDM may participate in sports activities regardless of climate. 19. Which comment made by a 12-year-old child with insulin-dependent diabetes mellitus (IDDM) indicates a knowledge deficit? a. I rotate my insulin injection sites every time I give myself an injection. b. I keep records of my glucose levels and insulin sites and amounts. c. Ill be glad when I can take a pill for my diabetes like my uncle does. d. I keep Lifesavers in my school bag in case I have a low-sugar reaction. ANS: C Children with IDDM will require life-long insulin therapy. Rotating injection sites is appropriate because insulin absorption varies at different sites. Keeping records of serum glucose and insulin sites and amounts is appropriate. Prompt treatment of hypoglycemia reduces the possibility of a severe reaction. Keeping hard candy on hand is an appropriate action.
CURRENT DIAGNOSIS AND TREATMENT PEDIATRICS 24TH LATEST 20. Which laboratory findings would confirm that a child with insulin-dependent diabetes mellitus (IDDM) is experiencing diabetic ketoacidosis? a. No urinary ketones b. Low arterial pH c. Elevated serum carbon dioxide d. Elevated serum phosphorus ANS: B Severe insulin deficiency produces metabolic acidosis, which is indicated by a low arterial pH. Urinary ketones, often in large amounts, are present when a child is in diabetic ketoacidosis. Serum carbon dioxide is decreased in diabetic ketoacidosis. Serum phosphorus is decreased in diabetic ketoacidosis. MULTIPLE RESPONSE 1. A child is diagnosed with hypothyroidism. The nurse should expect to assess which symptoms associated with hypothyroidism? Select all that apply. a. Weight loss b. Fatigue c. Diarrhea d. Dry, thick skin e. Cold intolerance ANS: B, D, E A child with hypothyroidism will display fatigue, dry, thick skin, and cold intolerance. Weight loss and diarrhea are signs of hyperthyroidism. OTHER 1. A nurse is preparing to administer 10 units of Regular insulin and 5 units of Lente insulin. Place in order the steps the nurse should follow to administer the total dosage of 15 units of
CURRENT DIAGNOSIS AND TREATMENT PEDIATRICS 24TH LATEST insulin. Place the initial step first and end with the final step. Use the following format for your answers: A, B, C, D a. Inject 5 units of air into the Lente insulin vial. b. Draw up the 5 units of Lente insulin. c. Inject 10 units of air into the Regular insulin vial. d. Cleanse the insulin vials with alcohol wipes. e. Draw up the 10 units of Regular insulin. ANS: D, A, C, E, B Cleanse the insulin vials with alcohol wipes initially. When mixing two different types of insulin, inject the appropriate amount of air into both vials and then withdraw the short-acting (clear) insulin first. So the steps should be to cleanse the insulin vials, inject air into the Lente, inject air into the Regular insulin vial, then draw the Regular (clear) insulin, and lastly draw the Lente (cloudy) insulin.
Chapter 35. Diabetes Mellitus 1. Overproduction of the anterior pituitary hormones can cause all of the following except: 1. Hyperthyroidism. 2. Hypercorticosolism. 3. Precocious puberty. 4. Diabetes. ANS: 4 1.
Feedback Hyperthyroidism occurs because of the
CURRENT DIAGNOSIS AND TREATMENT PEDIATRICS 24TH LATEST oversecretion of thyroid stimulating hormone (TSH). 2.
Hypercorticosolism occurs because of oversecretion in the anterior pituitary gland.
3.
Precocious puberty occurs because of oversecretion in the anterior pituitary gland. The cortex produces the hormones called glucocorticoids, which increase blood sugar, leading to a risk of diabetes.
4.
2. An infant is diagnosed with diabetes insipidus. An electrolyte panel has returned from the laboratory. The following results indicate which condition? NA: 162 mEq/L K+: 4.0 mEq/L CL: 99 CO2: 18 Ca+: 6.4 1. Hypokalemia 2. Hypernatremia 3. Hyperkalemia 4. Hyponatremia ANS: 2 1.
Feedback The laboratory results do not indicate hypokalemia.
2. 3.
The increase in sodium is common in a patient with diabetes insipidus because of the risks of dehydration. The potassium is in a normal range for the child.
4.
The sodium level is elevated.
3. A nurse conducting an assessment of a 3-year-old with diabetes mellitus Type 1 would expect to find: 1. Polyuria. 2. Polydipsia. 3. Polyphagia. 4. All of the above. ANS: 4
CURRENT DIAGNOSIS AND TREATMENT PEDIATRICS 24TH LATEST
1.
Feedback Known as one of the 3 Ps of DM 1.
2. 3.
Known as one of the 3 Ps of DM 1. Known as one of the 3 Ps of DM 1.
4.
These are the 3Ps of DM 1.
4. When a child with diabetes mellitus Type 1 is ill, it is important for the family to provide all of the following except: 1. Blood glucose monitoring every four hours. 2. Adjusting dosing requirements as needed. 3. Discontinuing the insulin until the child is feeling better. 4. Strongly encouraging the intake of fluids. ANS: 3
1. 2.
Feedback The illness causes rapidly changing glucose levels and needs to be frequently monitored. Insulin will need frequent adjustment during the illness.
3.
Insulin continues to be important to help the body heal.
4.
Fluids should be encouraged to help keep an adequate hydration status.
5. Kussmaul breathing will occur in children with diabetes mellitus Type 1 when: 1. The body is near death. 2. The body is attempting to correct the metabolic acidosis state. 3. The child has consumed too much sugar. 4. The body is attempting to balance the metabolic alkalosis state. ANS: 2
2. 3.
Feedback Kussmaul breathing may be seen near death, but the body is attempting to correct the metabolic acidosis. The body is attempting to reach equilibrium and correct the metabolic acidosis, thus creating the slower breathing. The body is producing too much insulin, thus not needing sugar.
4.
The body is trying to balance itself from a metabolic acidosis state.
1.
CURRENT DIAGNOSIS AND TREATMENT PEDIATRICS 24TH LATEST 6. Diabetes mellitus Type 2 has been increasing in children because of: 1. Environmental surroundings. 2. Autoimmune disorders. 3. Obesity. 4. Earlier use of drugs and alcohol in children. ANS: 3 Feedback 1. Not enough evidence supports environmental surroundings being a
cause for DM2 in children. 2. Autoimmune disorders can cause DM2, but they are not the primary
reason for the increase. 3. Obesity has increased in children, placing a higher risk on developing
DM2 at a young age. 4. The use of drugs and alcohol does not increase the risk for DM2 in
children. 7. Home management for a teen diagnosed with diabetes mellitus Type 2 would include: 1. Learning to measure foods and count carbohydrates. 2. Participating in exercise, such as riding a bike. 3. Creating a chart to indicate what foods should be eaten each day. 4. All of the above should be included in home management. ANS: 4 Feedback 1. The measurement and counting of carbohydrates is needed to maintain an adequate balance for a teens blood glucose level. 2. Exercise will help the body burn fat and increase metabolism to decrease the need for insulin. 3.A chart will help a teen identify the correct foods to eat. 4.The measurement and counting of carbohydrates is needed to maintain an adequate balance for a teens blood glucose level. Exercise will help
CURRENT DIAGNOSIS AND TREATMENT PEDIATRICS 24TH LATEST the body burn fat and increase metabolism to decrease the need for insulin. A chart will help a teen identify the correct foods to eat. 8. While assessing a 14 year old with a known history of diabetes mellitus Type 2, the nurse notes darker skin around the teens axilla. The nurse should: 1. Contact the doctor immediately 2. Provide a wash cloth and soap to cleanse the area. 3. Document the findings and note that acanthosis nigricans is present. 4. Ask the patient if he/she has had a recent infection. ANS: 3 Feedback 1. This is a common condition in a teen with DM2 and does not require immediate doctor notification. 2. The area is clean. This is a discoloration in the skin. 3.Acanthosis nigricans occurs in individuals with DM2 and is a discoloration of the skin. 4.DM2 is the reasoning for the darker skin, not an infection. 9. A child drinks a glass of orange juice after experiencing symptoms of hypoglycemia. This rapid-releasing sugar should be followed by which of the following? 1. Protein 2. Fruit juice 3. Several glasses of water 4. Complex carbohydrates and protein ANS: 4 Feedback 1. Once the childs blood sugar has increased, the child should immediately eat something that contains a protein.
CURRENT DIAGNOSIS AND TREATMENT PEDIATRICS 24TH LATEST 2. Foods and beverages with simple sugars will temporarily increase the
blood glucose level and are good for emergency situations. 3. Water lacks the protein and complex carbohydrates needed to help
decrease the blood sugar levels. 4. Foods and beverages with simple sugars will temporarily increase blood glucose and are good for emergency situations. But once the childs blood sugar has increased, the child should immediately eat something that contains a protein and complex carbohydrate serving, such as peanut butter and crackers or cheese and wheat crackers. 10. Manifestations of hypoglycemia include which of the following? 1. Lethargy 2. Confusion 3. Nausea and Vomiting 4. Shakiness and Dizziness ANS: 4 Feedback 1.A child is least likely to be lethargic with hypoglycemia. 2. If the child has reached a state of confusion, the child is very ill and needs medical attention immediately. 3. Nausea and vomiting are usually the cause for hypoglycemia. 4. The most common side effects of children with hypoglycemia include shakiness, dizziness, headache, extreme hunger, and perspiration. 11. The nurse should recognize that when a child develops diabetic ketoacidosis, this diagnosis is which of the following? 1. An expected outcome 2. A life-threatening outcome 3. Best treated at home 4. Best treated at the practitioners office
CURRENT DIAGNOSIS AND TREATMENT PEDIATRICS 24TH LATEST ANS: 2 Feedback 1. It is not a normal, expected outcome. 2. Diabetic ketoacidosis is a life-threatening situation. The child should be taken to the emergency room and/or a practitioners office for intervention. 3. This cant be treated effectively at home, as there is no emergency equipment available that might be needed. 4. This cant be treated effectively at home, as there is no emergency equipment available that might be needed. 12. The nurse is discussing the various sites used for insulin injections with a child and family. Which of the following sties usually has the fastest rate of absorption? 1. Arm 2. Leg 3. Buttock 4. Abdomen ANS: 4 Feedback 1. The arm has a lower level of subcutaneous tissue, so the absorption rate
is slower. 2. The leg has a lower level of subcutaneous tissue, so the absorption rate is slower. 3. The buttock has a lower level of subcutaneous tissue, so the absorption rate is slower and is not easily accessible for self-injections. 4. Because of a larger amount of subcutaneous tissue in the abdomen, the medication is absorbed faster. 13. Children tend to be more active in summer. Which change in the management of the child with diabetes would the nurse teach the patient and family to help treat episodes of hypoglycemia? 1. Increase food intake prior to exercise
CURRENT DIAGNOSIS AND TREATMENT PEDIATRICS 24TH LATEST 2. Decrease food intake prior to exercise 3. Give an additional dose of insulin prior to exercise 4. Avoid foods high in protein ANS: 1 Feedback 1. This measure should be taken because exercise can lower blood sugar levels, so adding the extra carbohydrate and protein snack will help the blood sugar levels maintain a normal level. This means that the child is less likely to have a hypoglycemic episode. 2. Children with diabetes must have extra carbohydrate and protein servings prior to exercise. This is because exercise can lower blood sugar levels. 3. The extra dose does not stabilize the blood glucose level long term. 4.Protein snacks help keep the blood glucose level stable and within normal limits. 14. Which of the following is a characteristic of Type 1 diabetes mellitus? 1. Ketoacidosis is infrequent 2. Onset of the condition is gradual 3. Oral anti-hyperglycemic agents are an effective treatment options 4. Age at onset is usually less than 20 years old ANS: 4 Feedback 1. The onset of Type 1 DM is very rapid and usually results in the
emergent condition known as diabetic ketoacidosis. 2. This condition can be frequent, especially in children or teens who are
poorly managed. 3. Oral anti-hyperglycemics are not indicated for Type 1 diabetes mellitus. 4.Diabetes mellitus Type 1 is a childhood disease and is mostly diagnosed before the age of 20. Type 2 diabetes mellitus usually occurs in middleaged or older adults.
CURRENT DIAGNOSIS AND TREATMENT PEDIATRICS 24TH LATEST 15. Which of the following is considered a cardinal sign of diabetes mellitus? 1. Impaired vision 2. Seizures 3. Frequent urination 4. Nausea ANS: 3 Feedback 1.Not a cardinal sign 2.Not a cardinal sign 3. Frequent urination (Polyuria) excessive thirst (Polydipsia) and excessive hunger (Polyphagia) are the cardinal signs of Type 1 diabetes mellitus. 4. Not a cardinal sign
Chapter 36. Inborn Errors of Metabolism 1. Discharge teaching for a child with Congenital Adrenal Hyperplasia includes the administration of medication for stress dosing. This should be followed when: 1. The child is going to be more active for the day. 2. The child has a high fever. 3. A dose has been missed. 4. The child has not had adequate nutrition for a day. ANS: 2 1.
Feedback The initial dosing should be effective for an active child.
2. 3.
A high fever increases metabolism and can create a need for a higher dose of the medication. Missed doses should be taken as soon as remembered.
4.
An inadequate diet for one day does not change the medication dosing.
CURRENT DIAGNOSIS AND TREATMENT PEDIATRICS 24TH LATEST
Chapter 37. Genetics and Dysmorphology 1. A parent whose child has been diagnosed with a cognitive deficit should be counseled that intellectual impairment: a. is usually due to a genetic defect. b. may be caused by a variety of factors. c. is rarely due to first-trimester events. d. is usually caused by parental intellectual impairment. ANS: B There is a multitude of causes for intellectual impairment. In nearly half of the cases, a specific cause has not been identified. Only 5% of children with intellectual impairment are affected by a genetic defect. One-third of children with intellectual impairment are affected by first-trimester events. Intellectual impairment can be transmitted to a child only if the parent has a genetic disorder. 2. A parent asks the nurse why a developmental assessment is being conducted for a child during a routine well-child visit. The nurse answers based on the knowledge that routine developmental assessments during well-child visits are: a. not necessary unless the parents request them. b. the best method for early detection of cognitive disorders. c. frightening to parents and children and should be avoided. d. valuable in measuring intelligence in children. ANS: B Early detection of cognitive disorders can be facilitated through assessment of development at each well-child examination. Developmental assessment is a component of all well-child
CURRENT DIAGNOSIS AND TREATMENT PEDIATRICS 24TH LATEST examinations. Developmental assessments are not frightening when the parent and child are educated about the purpose of the assessment and are not intended to measure intelligence. 3. The father of a child recently diagnosed with developmental delay is very rude and hostile toward the nurses. This father was cooperative during the childs evaluation a month ago. What is the best explanation for this change in parental behavior? a. The father is exhibiting symptoms of a psychiatric illness. b. The father may be abusing the child. c. The father is resentful of the time he is missing from work for this appointment. d. The father is in the anger stage of the grief process. ANS: D After a child is diagnosed with a developmental delay, the family may feel grief. The grief process begins with a stage of disbelief and denial and then progresses to anger. It is not possible to determine that a parent is exhibiting symptoms of a psychiatric illness on the basis of a single situation. The scenario does not give any information to suggest child abuse. Although the father may have difficulty balancing his work schedule with medical appointments for his child, a more likely explanation for his behavior change is that he is grieving the loss of a normal child. 4. An appropriate nursing diagnosis for a child with a cognitive dysfunction who has a limited ability to anticipate danger is: a. Impaired social interaction. b. Deficient knowledge. c. Risk for injury. d. Ineffective coping. ANS: C The nurse needs to know that limited cognitive abilities to anticipate danger lead to risk for injury. Impaired social interaction is indeed a concern for the child with a cognitive disorder but does not address the limited ability to anticipate danger. Because of the childs cognitive deficit,
CURRENT DIAGNOSIS AND TREATMENT PEDIATRICS 24TH LATEST knowledge will not be retained and will not decrease the risk for injury. Ineffective individual coping does not address the limited ability to anticipate danger. 5. Anticipatory guidance for the family of a preadolescent with a cognitive dysfunction should include information about: a. institutional placement. b. sexual development. c. sterilization. d. clothing. ANS: B Preadolescents who have a cognitive dysfunction may have normal sexual development without the emotional and cognitive abilities to deal with it. It is important to assist the family and child through this developmental stage. Preadolescence does require the child to be institutionalized. Sterilization is not an appropriate intervention when a child has a cognitive dysfunction. By the time a child reaches preadolescence, the family should have received counseling on ageappropriate clothing. 6. The mother of a 9-year-old child with Down syndrome discusses the childs language abilities. The nurse is not surprised to learn which information about the childs language development? a. Can take turns during conversation b. Has good grammar c. Can speak a foreign language d. Has difficulty in carrying on a conversation ANS: A Social language involves maintaining a conversation on a specific topic and taking turns during the conversation. Children with Down syndrome generally have good social language. The language development of children with Down syndrome involves difficulty with grammar but strength in social usage. It would not be expected for children with Down syndrome to be
CURRENT DIAGNOSIS AND TREATMENT PEDIATRICS 24TH LATEST characterized as typically knowing a foreign language. Children with Down syndrome have a general strength in social language such as greeting others and carrying on a conversation in a give-and-take manner and have social skills that exceed expected skills on the basis of intellectual capacity. 7. The infant with Down syndrome is closely monitored during the first year of life for which condition? a. Thyroid complications b. Orthopedic malformations c. Dental malformation d. Cardiac abnormalities ANS: D The high incidence of cardiac defects in children with Down syndrome makes assessment for signs and symptoms of these defects important during the first year. Infants with Down syndrome are not known to have thyroid complications. Orthopedic malformations may be present, but special attention is given to assessment for cardiac and gastrointestinal abnormalities. Dental malformations are not a major concern compared with the life-threatening complications of cardiac defects. 8. Which action is contraindicated when a child with Down syndrome is hospitalized? a. Determine the childs vocabulary for specific body functions. b. Assess the childs hearing and visual capabilities. c. Encourage parents to leave the child alone. d. Have meals served at the childs usual meal times. ANS: C The child with Down syndrome needs routine schedules and consistency. Having familiar people present, especially parents, helps to decrease the childs anxiety. To communicate effectively with the child, it is important to know the childs particular vocabulary for specific body functions.
CURRENT DIAGNOSIS AND TREATMENT PEDIATRICS 24TH LATEST Children with Down syndrome have a high incidence of hearing loss and vision problems and should have hearing and vision assessed whenever they are in a healthcare facility. Routine schedules and consistency are important to children. 9. A nurse is giving a parent information about autism. Which statement made by the parent indicates understanding of the teaching? a. Autism is characterized by periods of remission and exacerbation. b. The onset of autism usually occurs before 2 1/2 years of age. c. Children with autism have imitation and gesturing skills. d. Autism can be treated effectively with medication. ANS: B The onset of autism usually occurs before 30 months of age. Autism does not have periods of remissions and exacerbations. Autistic children lack imitative skills. Medications are of limited use in children with autism. 10. Which should the nurse keep in mind when planning to communicate with a child who is autistic? a. The child has normal verbal communication. b. Expect the child to use sign language. c. The child may exhibit monotone speech and echolalia. d. The child is not listening if she is not looking at the nurse. ANS: C Children with autism have abnormalities in the production of speech such as a monotone voice or echolalia and inappropriate volume, pitch, rate, rhythm, or intonation. The child has impaired verbal communication and abnormalities in the production of speech. Some autistic children may use sign language, but it is not assumed. Children with autism often are reluctant to initiate direct eye contact.
CURRENT DIAGNOSIS AND TREATMENT PEDIATRICS 24TH LATEST 11. What is the best intervention when a child with autism is hospitalized? a. Limit the individuals who enter the childs room. b. Perform all of the childs activities of daily living for her. c. Make sure the nurses know this child may be violent. d. Assign the strongest nurse to control the child. ANS: A The child with autism is often unable to tolerate the slightest change in routine. Limiting who enters the childs room to those knowledgeable about the childs routine will facilitate the childs adaptation to the hospital environment. The most important nursing consideration when planning care for a child with autism is to assign the child to a nurse who is familiar with the childs routine and to follow that routine. The child should be encouraged to perform toileting and selfcare activities as she normally would if she were not in the hospital. There is no indication that the child will be violent. Limiting the number of individuals in contact with the child and maintaining a routine will decrease any chance of violence. Strength should not be a consideration in assignments. 12. Intense stress and isolation as a result of caring for a child with developmental disabilities often lead parents to: a. heightened parental achievement. b. overuse of the healthcare system. c. overindulgence and obesity. d. child abuse. ANS: D Child abuse and developmental disabilities are often associated. Stress and isolation may hinder parents from reaching their potential. Parents may feel isolated from support and healthcare services. They report that professionals have limited understanding of their childrens needs. Although overindulgence and obesity may occur, the best answer is child abuse.
CURRENT DIAGNOSIS AND TREATMENT PEDIATRICS 24TH LATEST 13. A child with Asperger syndrome has also been diagnosed with depression. The nurse understands that two or more disorders in an individual is termed: a. comorbidity. b. congenital syndrome. c. mental retardation. d. developmental impairment. ANS: A Comorbidity by definition means more than one disorder in an individual. Congenital syndrome means the disorder originated before birth. Mental retardation refers to subaverage intellectual functioning. Developmental impairment refers to functional level. 14. Self-injury, fecal smearing, and severe temper tantrums in a preschool child are symptoms of: a. mild intellectual impairment. b. severe intellectual impairment. c. psychosocial deprivation. d. separation anxiety. ANS: B Self-injury, fecal smearing, and severe temper tantrums in a preschool child are symptoms of severe intellectual impairment. Mild intellectual impairment is characterized by social isolation or depression. Psychosocial deprivation may be a cause of intellectual impairment. The symptoms listed are characteristic of severe intellectual impairment. Symptoms of separation anxiety include protest, despair, and detachment. 15. Throughout their life span, cognitively impaired children are less capable of managing environmental challenges and are at risk for which problem? a. Nutritional deficits
CURRENT DIAGNOSIS AND TREATMENT PEDIATRICS 24TH LATEST b. Visual impairments c. Physical injuries d. Psychiatric problems ANS: C Safety is a challenge for cognitively impaired children. Decreased capability to manage environmental challenges may lead to physical injuries. Nutritional deficits are related more to dietary habits and the caregivers understanding of nutrition. Visual impairments are unrelated to cognitive impairment. Psychiatric problems may coexist with cognitive impairment but are not environmental challenges. 16. The parents of a child born at 36 weeks of gestation who had respiratory problems requiring 3 days of oxygen therapy are concerned that the infant may have an intellectual impairment. The best nursing statement to the parents is which of the following? a. A diagnosis of intellectual impairment is not made until the child enters school and experiences academic failure. b. Routine assessment of development during pediatric visits is the best method of early detection. c. The baby is not at risk for an intellectual impairment. d. Tests for intellectual impairments are not reliable for children younger than 3 years. ANS: B Routine assessment of development from birth is the best method for early detection of problems. Intellectual impairment may be detected before school age. The baby may be at risk for an intellectual impairment as a result of poor oxygenation. The Denver Developmental Screening test may be unreliable for children younger than 3 years, but other assessment tools are available. Several neuropsychological tests are available. 17. Parents of a child with fragile X syndrome ask the nurse about genetic transmission of this syndrome. In response, the nurse correctly explains that fragile X syndrome is:
CURRENT DIAGNOSIS AND TREATMENT PEDIATRICS 24TH LATEST a. most commonly seen in girls. b. acquired after birth. c. usually transmitted by the male carrier. d. usually transmitted by the female carrier. ANS: D The gene causing fragile X syndrome is transmitted by the mother. Fragile X syndrome is most common in males, is congenital, and is not transmitted by a male carrier. 18. The best setting for daytime care for a 5-year-old autistic child whose mother works is: a. private day care. b. public school. c. his own home with a sitter. d. a specialized program that facilitates interaction by use of behavioral methods. ANS: D Autistic children can benefit from specialized educational programs that address their special needs. Day care programs generally do not have resources to meet the needs of severely impaired children. To best meet the needs of an autistic child, the public school may refer the child to a specialized program. A sitter might not have the skills to interact with an autistic child. 19. Parents have learned that their 6-year-old child is autistic. The nurse may help the parents to cope by explaining that the child will: a. have abnormal ways of interacting with other children and adults. b. outgrow the condition by early adulthood. c. have average social skills. d. probably have age-appropriate language skills. ANS: A
CURRENT DIAGNOSIS AND TREATMENT PEDIATRICS 24TH LATEST Abnormal interaction with people is one of the several characteristics of autism. No evidence supports the belief that autism is outgrown. Autistic children have abnormal ways of relating to people (social skills). Speech and language skills are usually delayed in autistic children. 20. An autistic child is hospitalized with asthma. The nurse should plan care so that the: a. parents expectations are met. b. childs routine habits and preferences are maintained. c. child is supported through the autistic crisis. d. parents need not be at the hospital. ANS: B Children with autism are often unable to tolerate even slight changes in routine. Focus of care is on the childs needs rather than on the parents desires. Autism is a life-long condition. The presence of the parents is almost always required when an autistic child is hospitalized.
MULTIPLE RESPONSE 1. Which of the following treatment guidelines would be contraindicated when counseling the family of an infant with fragile X syndrome? Select all that apply. a. Advise genetic testing for family members. b. Delay speech therapy until the child is 2 years of age. c. Educate the family that their child will probably have normal intelligence. d. Refer the family to an early intervention program. ANS: B, C Speech therapy should be started in the first year of life and continued on an ongoing basis. Waiting until the child is 2 years old would not be appropriate. Children with fragile X syndrome have a high incidence of intellectual, language, and social dysfunctions. It is the most common inherited cause of mental retardation. Because fragile X syndrome is an X-linked recessive
CURRENT DIAGNOSIS AND TREATMENT PEDIATRICS 24TH LATEST disorder, genetic testing is appropriate. Early intervention programs assess the child and develop a plan of intervention; this is appropriate. 2. A nurse is assessing a newborn for facial feature characteristics associated with fetal alcohol syndrome. Which characteristics should the nurse expect to assess? Select all that apply. a. Short palpebral fissures b. Smooth philtrum c. Low set ears d. Inner epicanthal folds e. Thin upper lip ANS: A, B, E Infants with fetal alcohol syndrome may have characteristic facial features, including short palpebral fissures, a smooth philtrum (the vertical groove in the median portion of the upper lip), and a thin upper lip. Low set ears and inner epicanthal folds are associated with Down syndrome. 3. A nurse should plan to implement which interventions for a child admitted with inorganic failure to thrive? Select all that apply. a. Observation of parentchild interactions b. Assignment of different nurses to care for the child from day to day c. Use of 28 calorie per ounce concentrated formulas d. Administration of daily multivitamin supplements e. Role-modeling appropriate adultchild interactions ANS: A, D, E The nurse should plan to assess parentchild interactions when a child is admitted for nonorganic failure to thrive. The observations should include how the child is held and fed, how eye contact is initiated and maintained, and the facial expressions of both the child and the caregiver during interactions. Role modeling and teaching appropriate adultchild interactions (including holding, touching, and feeding the child) will facilitate appropriate parentchild relationships, enhance
CURRENT DIAGNOSIS AND TREATMENT PEDIATRICS 24TH LATEST parents confidence in caring for their child, and facilitate expression by the parents of realistic expectations based on the childs developmental needs. Daily multivitamin supplements with minerals are often prescribed to ensure that specific nutritional deficiencies do not occur in the course of rapid growth. The nursing staff assigned to care for the child should be consistent. Providing a consistent caregiver from the nursing staff increases trust and provides the child with an adult who anticipates his or her needs and who is able to role model child care to the parent. Caloric enrichment of food is essential, and formula may be concentrated in titrated amounts up to 24 calories per ounce. Greater concentrations can lead to diarrhea and dehydration.
Chapter 38. Allergic Disorders 1. A young mother calls the pediatric triage nurse because she is worried about her 4-month-old daughter. Her daughter had immunizations administered yesterday. The infant is not eating and is more irritable than usual. The nurse verifies that the infant had a DTaP immunization. The nurse knows that these signs and symptoms are: 1. A medical emergency. The mother should bring the infant to an emergency room to make sure that the child is not having an allergic reaction. 2. A normal reaction and will subside in a day or two. 3. Abnormal, and the mother should follow up with the doctor in two days. 4. Only occurring because the child has not been given Tylenol for the pain at the injection site. ANS: 2
CURRENT DIAGNOSIS AND TREATMENT PEDIATRICS 24TH LATEST Feedback 1. This is not a medical emergency. This is one of the side effects of the vaccine 2. This is a normal response to the vaccine. 3. This is normal and is one of the side effects of the vaccine. 4. The question does not assess what pain management has been done. 2. The nurse is performing an assessment on a 16-year-old patient and notices that his nasal mucosa appears pale, gray, and swollen. What would be the most appropriate question to ask the patient? a. Are you aware of having any allergies? b.
Do you have an elevated temperature?
c. Have you had any symptoms of a cold? d. Have you been having frequent nosebleeds? ANS: A With chronic allergies, the mucosa looks swollen, boggy, pale, and gray. Elevated body temperature, colds, and nosebleeds do not cause these mucosal changes. 3. The nurse is palpating the sinus areas. If the findings are normal, then the patient should report which sensation? a. No sensation b.
Firm pressure
c. Pain during palpation d. Pain sensation behind eyes ANS: B The person should feel firm pressure but no pain. Sinus areas are tender to palpation in persons with chronic allergies or an acute infection (sinusitis). 4. During an assessment of a 16 year old at the clinic for a spot on my lip I think is cancer, the nurse notices a group of clear vesicles with an erythematous base around them located at the lipskin border. The patient mentions that she just returned from Hawaii. What would be the most appropriate response by the nurse? a. Tell the patient she needs to see a skin specialist. b.
Discuss the benefits of having a biopsy performed on any unusual lesion.
c.
Tell the patient that these vesicles are indicative of herpes simplex I or cold sores and that they will heal in 4 to 10 days.
Tell the patient that these vesicles are most likely the result of a riboflavin d. deficiency and discuss nutrition. ANS: C Cold sores are groups of clear vesicles with a surrounding erythematous base. These evolve into pustules or crusts and heal in 4 to 10 days. The most likely site is the lip-skin junction. Infection often recurs in the same site. Recurrent herpes infections may be precipitated by sunlight, fever, colds, or allergy.
CURRENT DIAGNOSIS AND TREATMENT PEDIATRICS 24TH LATEST 5. When examining the nares of a 15-year-old patient who has complaints of rhinorrhea, itching of the nose and eyes, and sneezing, the nurse notices the following: pale turbinates, swelling of the turbinates, and clear rhinorrhea. Which of these conditions is most likely the cause? a. Nasal polyps b. Acute sinusitis c. Allergic rhinitis d. Acute rhinitis ANS: C Rhinorrhea, itching of the nose and eyes, and sneezing are present with allergic rhinitis. On physical examination, serous edema is noted, and the turbinates usually appear pale with a smooth, glistening surface. 6. During a checkup, a 17-year-old woman tells the nurse that she uses an over-the-counter nasal spray because of her allergies. She also states that it does not work as well as it used to when shefirst started using it. The best response by the nurse would be: a. You should never use over-the-counter nasal sprays because of the risk of addiction. b. You should try switching to another brand of medication to prevent this problem. c.
Continuing to use this spray is important to keep your allergies under control. Using these nasal medications irritates the lining of the nose and may cause rebound swelling.
d. ANS: D The misuse of over-the-counter nasal medications irritates the mucosa, causing rebound swelling, which is a common problem. 7. During an assessment, a patient mentions that I just cant smell like I used to. I can barely smell the roses in my garden. Why is that? For which possible causes of changes in the sense of smell will the nurse assess? Select all that apply. a. Chronic alcohol use b. Cigarette smoking c. d.
Frequent episodes of strep throat Chronic allergies
e. Aging f. Herpes simplex virus I ANS: B, D, E The sense of smell diminishes with cigarette smoking, chronic allergies, and aging. Chronic alcohol use, a history of strep throat, and herpes simplex virus I are not associated with changes in the sense of smell. Chapter 39. Antimicrobial Therapy 1. A toddler is being sent to the operating room for surgery at 9 AM. As the nurse prepares the child, what is the priority intervention?
CURRENT DIAGNOSIS AND TREATMENT PEDIATRICS 24TH LATEST a. b.
Administering preoperative antibiotic Verifying that the child and procedure are correct
c. Ensuring that the toddler has been NPO since midnight d. Informing the parents where they can wait during the procedure ANS: B The most important intervention is to ensure that the correct child is going to the operating room for the identified procedure. It is the nurses responsibility to verify identification of the child and what procedure is to be done. If an antibiotic is ordered, administering it is important, but correct identification is a priority. Clear liquids can be given up to 2 hours before surgery. If the child was NPO (taking nothing by mouth) since midnight, intravenous fluids should be administered. Parents should be encouraged to accompany the child to the preoperative area. Many institutions allow parents to be present during induction. 2. The practitioner has ordered a liquid oral antibiotic for a toddler with otitis media. The prescription reads 1 1/2 tsp four times per day. What should the nurse consider in teaching the mother how to give the medicine? a. A measuring spoon should be used, and the medication must be given every 6 hours. b. The mother is not able to handle this regimen. Long-acting intramuscular antibiotics should be administered. c. A hollow-handled medication spoon is advisable, and the medication should be equally spaced while the child is awake. d. A household teaspoon should be used and the medicine given when the child wakes up, around lunch time, at dinner time, and before bed. ANS: C A hollow-handled medication spoon allows the mother to measure the correct amount of medication. The order is written for four times a day; every 6 hours dosing is not necessary. There is no indication that the mother is not able to adhere to the medication regimen. She is asking for clarification so she can properly care for her child. Long-acting intramuscular antibiotics are not indicated. Household teaspoons vary greatly and should not be used. 3. Guidelines for intramuscular administration of medication in school-age children include what standard?
CURRENT DIAGNOSIS AND TREATMENT PEDIATRICS 24TH LATEST a. Inject medication as rapidly as possible. b. Insert needle quickly, using a dartlike motion. c. Have the child stand if at all possible and if the child is cooperative. d. Penetrate the skin immediately after cleansing the site while the skin is moist. ANS: B The needle should be inserted quickly in a dartlike motion at a 90-degree angle unless contraindicated. Inject medications slowly. Allow skin preparation to dry completely before the skin is penetrated. Place the child in a lying or sitting position. 4. What is an advantage of the ventrogluteal muscle as an injection site in young children? a. Easily accessible from many directions b. Free of significant nerves and vascular structures c. Can be used until child reaches a weight of 9 kg (20 lb) d. Increased subcutaneous fat, which provides sustained drug absorption ANS: B Being free of significant nerves and vascular structure is one of the advantages of the ventrogluteal site. In addition, it is considered less painful than the vastus lateralis. The major disadvantage is lack of familiarity by health professionals and controversy over whether the site can be used before weight bearing. The vastus lateralis is a more accessible site. The ventrogluteal muscle site has safely been used from newborn through adulthood. Clinical guidelines address the need for the child to be walking. The site has less subcutaneous tissue, which facilitates intramuscular deposition of the drug rather than subcutaneous. 5. When teaching a mother how to administer antibiotic eye drops, where should the nurse tell her to place them? a. At the lacrimal duct b. On the sclera while the child looks to the outside c. In the conjunctival sac when the lower eyelid is pulled down
CURRENT DIAGNOSIS AND TREATMENT PEDIATRICS 24TH LATEST d. Carefully under the eyelid while it is gently pulled upward ANS: C The lower eyelid is pulled down, forming a small conjunctival sac. The solution or ointment is applied to this area. The medication should not be administered directly on the eyeball. The lacrimal duct is not the appropriate placement for the eye medication. It will drain into the nasopharynx, and the child will taste the drug.
Chapter 40. Infections: Viral and Rickettsial 1. An immunosuppressed child is admitted to the hospital with varicella (chickenpox). The nurse should be prepared to administer which prescribed medication? a. Erythromycin (Pediazole) b. Varicella vaccine c. Acetylsalicylic acid (Aspirin) d. Acyclovir (Zovirax) ANS: D Acyclovir is the antiviral medication that would be used in this case. Response to antiviral drugs is good provided that treatment is started early in the illness. Erythromycin is an antibiotic that is prescribed for bacterial infections. The varicella vaccine is a live virus and would be
CURRENT DIAGNOSIS AND TREATMENT PEDIATRICS 24TH LATEST contraindicated for an immunosuppressed child. Aspirin is not given to children with viral illnesses because of the risk of the child developing Reyes disease. 2. The mother of an infant with multiple anomalies tells the nurse that she had a viral infection in the beginning of her pregnancy. Which viral infection is associated with fetal anomalies? a. Measles b. Roseola c. Rubella d. Herpes simplex virus (HSV) ANS: C The rubella virus can cross the placenta and infect the fetus, causing fetal anomalies. Measles is not associated with congenital defects. Most cases of roseola occur in children 6 to 18 months old. HSV can be transmitted to the newborn infant during vaginal delivery, causing multisystem disease. It is not transmitted transplacentally to the fetus during gestation. 3. What is the best response to a parent of a 2-month-old infant who asks when the infant should receive the measles vaccine? a. Your baby can get the measles vaccine now. b. The first dose is given any time after the first birthday. c. She should be vaccinated between 4 and 6 years of age. d. This vaccine is administered when the child is 11 years old. ANS: B The first measles, mumps, rubella (MMR) vaccine is recommended routinely at 1 year of age. Some immunizations are initiated at 2 months of age, but not the measles vaccine. The second dose of MMR is recommended at 4 to 6 years of age. Children should receive their second MMR dose no later than 11 to 12 years of age.
CURRENT DIAGNOSIS AND TREATMENT PEDIATRICS 24TH LATEST 4. What discharge information should the nurse give to the parents of an adolescent who has been diagnosed with the Epstein-Barr virus? a. It is particularly important to protect the adolescents head during physical activities. b. The teen will feel like himself and be back to his usual routines in a week. c. The treatment of the Epstein-Barr virus is prolonged bed rest, usually lasting several months. d. Fatigue may persist, and the adolescent may need to increase school activities gradually. ANS: D The recovery period is often lengthy and fatigue may continue, necessitating a gradual return to school activities. During the acute and recovery phases, activity restrictions, which include no contact sports or roughhousing, are implemented to protect the childs enlarged spleen from rupture. The recovery process is a slow and gradual one and bed rest is indicated during the acute stage of the illness, usually lasting 2 to 4 weeks. 5. Which should the nurse teach parents to expect to observe in the prodromal phase of rubeola? a. Macular rash on the face b. Kopliks spots c. Petechiae on the soft palate d. Crops of vesicles on the trunk ANS: B Kopliks spots appear approximately 2 days before the appearance of a rash. The macular rash with rubeola appears after the prodromal stage. Petechiae on the soft palate occur with rubella. Crops of vesicles on the trunk are characteristic of varicella.
6. Infections that can be transmitted through vertical transmission are:
CURRENT DIAGNOSIS AND TREATMENT PEDIATRICS 24TH LATEST a. influenza, meningitis, and varicella
b.rubella, herpes, and human immunodeficiency virus c. staphylococci, streptococci, and Escherichia coli d.hepatitis B and C and Rickettsia rickettsii ANS: B Feedback AIncorrect. Influenza, meningitis, and varicella are not transmitted through vertical transmission. BCorrect. Vertical transmission can occur through the placenta while the infant is in utero, during delivery, or through breast milk following delivery. Among the infections transmitted in this way are rubella, herpes, hepatitis B and C, and human immunodeficiency virus (HIV). CIncorrect. Staphylococci, streptococci, and Escherichia coli are not transmitted through vertical transmission. DIncorrect. Hepatitis B and C are transmissible by way of vertical transmission, but not Rickettsia rickettsii. 7. Which of the following groups of children are susceptible to infections from staphylococci and
Escherichia coli? a. adolescents c. toddlers b.preschoolersd.newborns ANS: D Feedback AIncorrect. Adolescents are not susceptible to invasive infection by bacterial organisms such as staphylococci and Escherichia coli. BIncorrect. Preschoolers are not susceptible to invasive infection by bacterial organisms such as staphylococci and Escherichia coli. CIncorrect. Toddlers are not susceptible to invasive infection by bacterial organisms such as staphylococci and Escherichia coli. DCorrect. Newborns are susceptible to invasive infection by bacterial organisms such as staphylococci and Escherichia coli. 8. Caregivers could best reduce the number of infections that infants have by:
a. taking the infant to the pediatricians office whenever the infant has a fever, diarrhea, or nasal discharge b.washing hands frequently, not kissing the infants hands, and washing all objects that go into a babys mouth c. limiting the number of people who visit the home and not letting other people pick up and hold the baby d.starting medication sooner whenever the infant has a fever or any other symptoms of infection
CURRENT DIAGNOSIS AND TREATMENT PEDIATRICS 24TH LATEST ANS: B Feedback AIncorrect. Taking the infant to the pediatricians office whenever the infant has a fever, diarrhea, or nasal discharge is in reaction to an infection, not to prevention. BCorrect. Caregivers could best reduce the number of infections that infants have by washing hands frequently, not kissing the infants hands, and washing all objects that go into a babys mouth. CIncorrect. Limiting the number of people who visit the home and not letting other people pick up and hold the baby is not the best means to reduce the number of infections that infants have. DIncorrect. Starting medication sooner whenever the infant has a fever or any other symptoms of infection is in reaction to an infection, not to prevention. 9. You are a nurse working with toddlers and preschoolers. One of the children has a skin
infection, which is diagnosed as tinea corporis. In assessing this condition, you would ask the caregivers which of the following questions? a. What kind of detergent have you been using? b.Has your child been swimming in a public swimming pool? c. Does your child have any known contact allergies? d.Has your child been playing with cats or dogs? ANS: D Feedback AIncorrect. Tinea corporis requires contact with dogs and cats, not detergent. BIncorrect. Tinea corporis requires contact with dogs and cats, not swimming in a public pool. CIncorrect. Tinea corporis requires contact with dogs and cats and is not related to allergies. DCorrect. Playing with infected animals may cause tinea corporis (ringworm of the body or cat scratch fever). 10. You are a nurse working with children of various ages. You would teach the caregivers not to
give acetylsalicylic acid (aspirin) to children: a.
of any age
c.
under 13 years
b.
under 6 years
d.
under 18 years
ANS: C Feedback
CURRENT DIAGNOSIS AND TREATMENT PEDIATRICS 24TH LATEST
A
B
Incorrect. You would teach the caregivers not to give acetylsalicylic acid (aspirin) to children under 13 years, not of any age. Incorrect. You would teach the caregivers not to give acetylsalicylic acid (aspirin) to children under 6 years of age, and not until they reach 13 years.
C
Correct. Caregivers should not use aspirin to treat the pain and fever of viral illnesses such as influenza and varicella (chickenpox) in children under 13 years of age. The use of aspirin has been linked to the subsequent development of Reyes syndrome, which is a disorder combining both encephalopathy and fatty degeneration of the liver.
D
Incorrect. You would teach the caregivers not to give acetylsalicylic acid (aspirin) to children under the age of 13, not 18 years.
11. The school nurse, on discovering any outbreaks of head lice, scabies, and pinforms, will teach
the children to avoid: a.
sitting next to infected children
b.
wearing head scarves or any head covering
c.
keeping their coats and hats on their desks and to instead hang them on the coat rack
d.
sharing and swapping clothing, as well as avoiding sharing personal items
ANS: D Feedback
A
B
C
Incorrect. Sitting next to infected children cannot result in outbreaks of head lice, scabies, pinforms, tinea corporis, and tinea capitis. Incorrect. The school nurse will teach that sharing head scarves is to be avoided, not wearing them. Incorrect. The school nurse will teach that sharing their coats and hats is to be avoided, not keeping them on their desk and hanging them on the coat rack.
CURRENT DIAGNOSIS AND TREATMENT PEDIATRICS 24TH LATEST Correct. Sharing beds and linens and clothes as well as personal items such as combs can result in outbreaks of head lice, scabies, pinforms, tinea corporis, and tinea capitis.
D 12. Adolescents most often lack which of the following types of immunity and need to be immunized because of the danger of acquiring this type of infection through sexual contact? a.
hepatitis A
c.
chlamydia
b.
hepatitis B
d.
gonorrhea
ANS: B Feedback
A B
C
D
Incorrect. The hepatitis A series may be indicated only for adolescents in high-risk categories. Correct. Adolescents most often lack immunity against hepatitis B, a viral infection that could be acquired during this period or early adulthood through sexual contact. Incorrect. Chlamydia is a parasite, categorized as a specialized bacteria, is a sexually transmitted disease, for which no vaccine exists. Incorrect. Gonorrhea is a bacteria, sexually transmitted disease, for which no vaccine exists.
13. What is the bodys main defense against viruses?
a.
T lymphocytes
c.
interferon
b.
B lymphocytes
d.
interleukin
ANS: A Feedback
CURRENT DIAGNOSIS AND TREATMENT PEDIATRICS 24TH LATEST
A
Correct. The bodys main defense against viruses is the T lymphocytes which direct and regulate the immunologic response by secreting lymphokines. Incorrect. The bodys main defense against viruses is not the B lymphocytes.
B Incorrect. The bodys main defense against viruses is not interferons. C Incorrect. The bodys main defense against viruses is not interleukins. D
Chapter 41. Human Immunodeficiency Virus Infection 1. Over 84% of newborn babies prenatally infected with human immunodeficiency virus (HIV) are of which of the following races? a.
Caucasian and American Indian
c.
Asian
b.
African American and Hispanic
d.
Eskimo
ANS: B Feedback
CURRENT DIAGNOSIS AND TREATMENT PEDIATRICS 24TH LATEST
A
B
Incorrect. Over 84% of newborn babies prenatally infected with HIV are not Caucasian and American Indian. Correct. Over 84% of newborn babies prenatally infected with HIV are African American and Hispanic demonstrating this disease disproportionately affects children of color. Incorrect. Over 84% of newborn babies prenatally infected with HIV are not Asian.
C Incorrect. Over 84% of newborn babies prenatally infected with HIV are not Eskimo. D 2. The nurse working with pregnant women and new mothers in developed countries will advise
these women to avoid: a.
breastfeeding
c.
supplementing breast milk with formula
b.
sunlight
d.
exercising in any health clubs or gyms
ANS: A Feedback
A
Correct. The nurse working with pregnant women and new mothers in developed countries will advise these women to avoid breastfeeding. If they have HIV, their infant may become infected through breast milk. Incorrect. Sunlight has no relationship to the etiology of HIV.
B C
Incorrect. The nurse will advise these women to avoid breastfeeding; therefore, supplementing breast milk with formula is not a logical choice.
D
Incorrect. Exercising in any health clubs or gyms has no relationship to the etiology of HIV.
3. Which of the following groups of clients with human immunodeficiency virus (HIV) will the
nurse find have the most severe symptoms?
a.
young adult women
c.
children diagnosed with HIV then young
CURRENT DIAGNOSIS AND TREATMENT PEDIATRICS 24TH LATEST
b.
men diagnosed with HIV as adolescents
d.
middle-aged men
ANS: C Feedback Incorrect. Young adult women will not have the most severe symptoms. A
B C
Incorrect. Men diagnosed with HIV as adolescents will not have the most severe symptoms. Correct. Children diagnosed with HIV when young will have the most severe symptoms. Incorrect. Middle-aged men will not have the most severe symptoms.
D
Chapter 42. Infections: Bacterial and Spirochetal 1. Which statement made by an adolescent girl indicates an understanding about the prevention of sexually transmitted diseases (STDs)? a. I know the only way to prevent STDs is not to be sexually active. b. I practice safe sex because I wash myself right after sex. c. I wont get any kind of STD because I take the pill. d. I have sex only if my boyfriend wears a condom. ANS: A Abstinence is the only foolproof way to prevent an STD. STDs are transmitted through body fluids (semen, vaginal fluids, blood). Perineal hygiene will not prevent an STD. Oral contraceptives do not protect women from contracting STDs. A condom can reduce but not eliminate an individuals chance of acquiring an STD. 2. Which sexually transmitted disease would the nurse suspect when an adolescent girl comes to the clinic because she has a vaginal discharge that is white with a fishy smell?
CURRENT DIAGNOSIS AND TREATMENT PEDIATRICS 24TH LATEST a. Human papillomavirus b. Bacterial vaginosis c. Trichomonas d. Chlamydia ANS: B Bacterial vaginosis is characterized by a profuse, white, malodorous (fishy smelling) vaginal discharge that sticks to the vaginal walls. Manifestations of the human papillomavirus are anogenital warts that begin as small papules and grow into clustered lesions. Infections with Trichomonas are frequently asymptomatic. Symptoms in females may include dysuria, vaginal itching, burning, and a frothy, yellowish-green, foul-smelling discharge. Many people with chlamydial infections have few or no symptoms. Urethritis with dysuria, urinary frequency, or mucopurulent discharge may indicate chlamydial infection. 4. A nurse is conducting a health education class for a group of school-age children. Which statement, made by the nurse, is correct about the bodys first line of defense against infection in the innate immune system? a. Nutritional status b. Skin integrity c. Immunization status d. Proper hygiene practices ANS: B The first lines of defense in the innate immune system are the skin and intact mucous membranes. Nutritional status is an indicator of overall health, but it is not the first line of defense in the innate immune system. Immunizations provide artificial immunity or resistance to harmful diseases. Practicing good hygiene may reduce susceptibility to disease, but it is not a component of the innate immune system. 7. Which statement made by a parent indicates incorrect information about an intervention for a childs fever?
CURRENT DIAGNOSIS AND TREATMENT PEDIATRICS 24TH LATEST a. I should keep her covered lightly when she has a fever. b. Ill give her plenty of liquids to keep her hydrated. c. I can give her acetaminophen (Tylenol) for a temperature higher than 101 F. d. Ill look for over-the-counter preparations that contain salicylates (Aspirin). ANS: D Aspirin products are avoided because of the possibility of developing Reyes syndrome. The parent should check labels on all over-the-counter products to be sure they do not contain aspirin. Dressing the child in light clothing and using lightweight covers will help reduce fever and promote the childs comfort. Adequate hydration will help maintain a normal body temperature. Acetaminophen or ibuprofen should be used as directed for fever control. 8. A 6-month-old infant is due for routine immunizations. The parent reports the infant was exposed to pertussis 2 days ago. The nurse should: a. give the 6 month immunizations as scheduled. b. hold the 6 month immunizations until the next visit. c. hold the 6 month immunizations for 3 weeks. d. notify the physician. ANS: A An infant exposed to pertussis should receive routine immunizations as scheduled. The nurse would not hold scheduled immunizations or notify the physician for an infant who was only exposed to pertussis. 9. Which symptom should be reported to the primary care provider if it occurs when a child with an infectious disease is febrile? a. Anorexia b. Fatigue c. Itching d. Headache
CURRENT DIAGNOSIS AND TREATMENT PEDIATRICS 24TH LATEST ANS: D Parents should be told that headache is a symptom that should be reported promptly to the childs primary care provider when a child has an elevated temperature. It would not be abnormal for a child who is febrile to have a loss of appetite. Fatigue is an expected response when a child is febrile during the course of an infectious disease. Rash and skin lesions occur with many viral illnesses. It would not be unusual for the child to complain of itchiness. The primary care provider may be contacted for interventions to relieve itching. 10. A parent asks the nurse how she would know whether her child has fifth disease. The nurse would advise the parent to be alert for which manifestation? a. Bulls-eye rash at the site of a tick bite b. Lesions in various stages of development on the trunk c. Maculopapular rash on the trunk that lasts for 2 days d. Bright red rash on the cheeks that looks like slapped cheeks ANS: D Fifth disease presents with an intense, fiery red, edematous rash on the cheeks, which gives a slapped cheek appearance. The bulls-eye rash at the site of a tick bite is a manifestation of Lyme disease. Varicella is manifested as lesions in various stages of developmentmacule, papule, then vesicle, first appearing on the trunk and scalp. Roseola manifests as a maculopapular rash on the trunk that can last for hours or up to 2 days. 11. Which intervention is appropriate for a hospitalized child who has crops of lesions on the trunk that appear as a macular rash and vesicles? a. Place the child in strict isolation and on airborne precautions. b. Continue to practice standard precautions. c. Pregnant women should avoid contact with the child. d. Screen visitors for immunity to measles. ANS: A
CURRENT DIAGNOSIS AND TREATMENT PEDIATRICS 24TH LATEST The childs skin lesions are characteristic of varicella. In the hospital setting, children with varicella should be placed in strict isolation and on airborne precautions. The purpose is to prevent transmission of microorganisms by inhalation of small-particle droplet nuclei and to protect other patients and healthcare providers from acquiring this disease. Screening visitors for immunity to measles is irrelevant. It would be important to screen visitors for immunity to varicella. Certain viral illnesses such as rubella and fifth disease are known to affect the fetus if the woman contracts the disease during pregnancy. This child appears to have varicella. Pregnancy is not a contraindication to caring for a child with varicella. 12. How should the nurse respond to a parent who asks, How can I protect my baby from whooping cough? a. Dont worry; your baby will have maternal immunity to pertussis that will last until your baby is approximately 18 months old. b. Make sure your child gets the pertussis vaccine. c. See the doctor when the baby gets a respiratory infection. d. Have your pediatrician prescribe erythromycin. ANS: B Primary prevention of pertussis can be accomplished through administration of the pertussis vaccine. Infants do not receive maternal immunity to pertussis and are susceptible to pertussis. Pertussis is highly contagious and is associated with a high infant mortality rate. Prompt evaluation by the primary care provider for respiratory illness will not prevent pertussis. Erythromycin is used to treat pertussis. It will not prevent the disease. 13. Which statement indicates that a parent understands the treatment for a child who has scarlet fever? a. I can stop the medicine when my daughter feels better. b. I will apply antibiotic cream to her rash twice a day. c. I will give the penicillin for the full 10 days. d. My daughter can go back to school when she has been on the antibiotic for a week.
CURRENT DIAGNOSIS AND TREATMENT PEDIATRICS 24TH LATEST ANS: C It is necessary to give the entire course of antibiotic for 10 to 14 days. Penicillin is the preferred treatment for any streptococcal infection. The bacteria will not be eradicated if a partial course of antibiotics is given. Treatment of scarlet fever does not include topical antibiotic cream. The child is no longer contagious after 24 hours of antibiotic therapy and can return to day care or school. 14. Which should be included in the follow-up care for a neonate who received aminoglycoside therapy for sepsis? a. Hemoglobin level monitoring b. Hearing test c. Serial platelet counts d. Glucose testing ANS: B A long-term side effect of aminoglycoside therapy is hearing impairment. Aminoglycosides do not affect the hemoglobin level, glucose control, or directly affect platelets. 15. Health teaching to prevent Lyme disease would include which action? a. Complete the immunization series in early infancy. b. Wear long sleeves and pants tucked into socks while in wooded areas. c. Give low-dose antibiotics to the child before exposure. d. Restrict activities that might lead to exposure for the child. ANS: B Wearing long sleeves and pants and tucking the pants into socks keeps ticks on the clothing and prevents them from hiding on the body. Currently there is no vaccine available for Lyme disease. The Lyme disease vaccine had been approved for persons aged 15 to 70 years but was withdrawn
CURRENT DIAGNOSIS AND TREATMENT PEDIATRICS 24TH LATEST from the market in 1992. Antibiotics are used to treat, not prevent, Lyme disease. Children should be allowed to maintain normal growth and development with activities such as hiking. 16. Which action is initiated when a child is scratched by a rabid animal? a. No intervention unless the child becomes symptomatic b. Administration of immune globulin around the wound c. Administration of rabies vaccine on days 3, 7, 14, and 28 d. Administration of both immune globulin and vaccine as soon as possible after exposure ANS: D Human rabies immune globulin and the first dose of the rabies vaccine are given after exposure. Transmission of rabies can occur from bites with contaminated saliva, scratches from the claws of infected animals, airborne transmission in bat-infested caves, or in a laboratory setting. Rabies is fatal if no intervention is taken to prevent the disease. Human rabies immune globulin is infiltrated locally around the wound and the other half of the dose is given intramuscularly. This is only part of the treatment after rabies exposure. The rabies vaccine is given within 48 hours of exposure and again on days 3, 7, 14, and 28. MULTIPLE RESPONSE 1. A hospitalized child has developed a methicillin-resistant Staphylococcus aureus (MRSA) infection. The nurse plans which interventions when caring for this child? Select all that apply. a. Airborne isolation b. Administration of vancomycin (Vancocin) c. Contact isolation d. Administration of mupirocin (Bactroban) ointment to the nares e. Administration of cefotaxime (Cefotetan) ANS: B, C, D
CURRENT DIAGNOSIS AND TREATMENT PEDIATRICS 24TH LATEST Vancomycin is used to treat MRSA along with mupirocin ointment to the nares. The client is placed in contact isolation to prevent spread of the infection to other clients. The infection is not transmitted by the airborne route so only contact isolation is required. The infection is resistant to cephalosporins.
Chapter 43. Infections: Parasitic and Mycotic 1. Adolescents most often lack which of the following types of immunity and need to be immunized because of the danger of acquiring this type of infection through sexual contact? a.
hepatitis A
c.
chlamydia
b.
hepatitis B
d.
gonorrhea
ANS: B Feedback
A B
C
D
Incorrect. The hepatitis A series may be indicated only for adolescents in high-risk categories. Correct. Adolescents most often lack immunity against hepatitis B, a viral infection that could be acquired during this period or early adulthood through sexual contact. Incorrect. Chlamydia is a parasite, categorized as a specialized bacteria, is a sexually transmitted disease, for which no vaccine exists. Incorrect. Gonorrhea is a bacteria, sexually transmitted disease, for which no vaccine exists.
2. The nurse should provide which information to parents about the prevention of parasitic infections? Select all that apply. a. Perform good handwashing. b. Diaper a child when swimming. c. Avoid cleaning the bathroom facilities with bleach. d. Shoes should be worn outside.
CURRENT DIAGNOSIS AND TREATMENT PEDIATRICS 24TH LATEST e. Fruits and vegetables should be washed before eating. ANS: A, D, E Children are more commonly infected with parasites than adults, primarily as a result of frequent hand-to-mouth activity and the likelihood of fecal contamination. Good handwashing can prevent the transmission. Shoes should be worn when outside to prevent transmission and fruits and vegetables should be washed before eating. The child should not swim in a pool that allows diapered children. The bathroom facilities should be cleaned with bleach to decrease the chance of transmission.
Chapter 44. Sexually Transmitted Infections 1. Which of the following is a high risk group among children and adolescents for contracting Hepatitis B? 1. Those living in institutions 2. Individuals infected by sexual partners 3. Children who are hemophiliacs or are receiving frequent blood transfusions 4. All of the above ANS: 4 1. 2.
3.
4.
Feedback High risk groups include those living in institutions. High risk groups include those living in institutions, those involved in IV drug use, and those infected by sexual partners. Hepatitis B is transmitted by blood or blood products, or by sexual contact. Children who are hemophiliacs or are receiving frequent blood transfusions are at risk. Individuals who have traveled to Africa or Asia are also at higher risk. Hepatitis B is transmitted by blood or blood products, or by sexual contact. High risk groups include those living in institutions, those involved in IV drug use, those infected by sexual partners, and children who are hemophiliacs or are receiving frequent blood transfusions. Individuals who have traveled to Africa or Asia are also at higher risk
2. A 16-year-old girl comes into the office for a sports physical. The recommendation is made for the girl to receive the Human Papillomavirus vaccine. Her mother is upset about the recommendation because she is sure her daughter is not yet sexually active. Which of the following responses would be most appropriate for the nurse to give?
CURRENT DIAGNOSIS AND TREATMENT PEDIATRICS 24TH LATEST 1. Your daughter should receive the vaccine even though she has a yeast allergy. 2. We need to give her a pregnancy test as well prior to administering the vaccine. 3. She is overdue for the vaccine because she should have received it when she was 9. 4. It is best if the vaccine is given before she becomes sexually active in order to prevent genital warts and cervical cancer in the future. ANS: 4 1.
2. 3.
4.
Feedback The vaccine should not be given if the child is allergic to yeast. The nurse should educate the mother and daughter that the vaccine is best administered prior to becoming sexually active to prevent genital warts and cervical cancer from an HPV infection. The HPV vaccine is generally given to girls ages 11 to 18, but can be given as early as age 9. The HPV vaccine protects girls against HPV infections that may lead to genital warts and cervical cancer. It is generally given to girls ages 11to 18 prior to being sexually active, but can be given as early as age 9. The vaccine should not be administered if the girl is allergic to yeast. Health-care workers should check for pregnancy before giving the vaccine. Boys ages 9 to 18 may also receive the vaccine.
3. Stella, a 16year-old girl, has come to the clinic to receive the HPV vaccine. Prior to the administration of the vaccine, the nurse must assess for: 1. Allergies to latex. 2. Allergies to eggs. 3. The last sexual encounter. 4. Pregnancy. ANS: 4 1.
Feedback Latex is not a concern with this vaccination.
2. 3.
An allergy to eggs would not be a concern. The last sexual encounter will not influence administration of the vaccine.
4.
A woman who is pregnant should not receive the vaccine.
Chapter 45. Fluid, Electrolyte, and Acid-Base Disorders and Therapy
CURRENT DIAGNOSIS AND TREATMENT PEDIATRICS 24TH LATEST 1. The parents of a child with acid-base imbalance ask the nurse about mechanisms that regulate acid-base balance. Which statement by the nurse accurately explains the mechanisms regulating acid-base balance in children? a. The respiratory, renal, and chemical-buffering systems regulate acid-base in the body. b. The kidneys balance acid; the lungs balance base. c. The cardiovascular and integumentary systems work together to control acidbase. d. The skin, kidney, and endocrine systems control the bodys acid-base. ANS: A The acid-base system is regulated by chemical buffering, respiratory control of carbon dioxide, and renal regulation of bicarbonate and secretion of hydrogen ions. Both the kidneys and the lungs, along with the buffering system, contribute to acid-base balance. Neither system regulates acid or base balances exclusively. The cardiovascular and integumentary systems are not part of acid-base regulation in the body. Chemical buffers, the lungs, and the kidneys work together to keep the blood pH within a normal range. 2. A child has a 2-day history of vomiting and diarrhea. He has hypoactive bowel sounds and an irregular pulse. Electrolyte values are sodium, 139 mEq/L; potassium, 3.3 mEq/L; and calcium, 9.5 mg/dL. This child is likely to have which of the following electrolyte imbalances? a. Hyponatremia b. Hypocalcemia c. Hyperkalemia d. Hypokalemia ANS: D A serum potassium level less than 3.5 mEq/L is considered hypokalemia. Clinical manifestations of hypokalemia include muscle weakness, decreased bowel sounds, cardiac irregularities, hypotension, and fatigue. The normal serum sodium level is 135 to 145 mEq/L. A level of 139
CURRENT DIAGNOSIS AND TREATMENT PEDIATRICS 24TH LATEST mEq/L is within normal limits. A serum calcium level less than 8.5 mg/dL is considered hypocalcemia. A serum potassium level greater than 5 mEq/L is considered hyperkalemia. 3. Which statement best describes why infants are at greater risk for dehydration than older children? a. Infants have an increased ability to concentrate urine. b. Infants have a greater volume of intracellular fluid. c. Infants have a smaller body surface area. d. Infants have an increased extracellular fluid volume. ANS: D The larger ratio of extracellular fluid to intracellular fluid predisposes the infant to dehydration. Because the kidneys are immature in early infancy, there is a decreased ability to concentrate the urine. Infants have a larger proportion of fluid in the extracellular space. Infants have a proportionately greater body surface area in relation to body mass, which creates the potential for greater fluid loss through the skin and gastrointestinal tract. 4. Which assessments are most relevant to the care of an infant with dehydration? a. Temperature, heart rate, and blood pressure b. Respiratory rate, oxygen saturation, and lung sounds c. Heart rate, capillary refill, and skin color d. Diet tolerance, bowel function, and abdominal girth ANS: C Changes in heart rate, capillary refill, and skin color are early indicators of impending shock in the child. Children can compensate and maintain an adequate cardiac output when they are hypovolemic. Blood pressure is not as reliable an indicator of shock as are changes in heart rate, sensorium, and skin color. Respiratory assessments will not provide data about impending hypovolemic shock. Diet tolerance, bowel function, and abdominal girth are not as important indicators of shock as heart rate, capillary refill, and skin color.
CURRENT DIAGNOSIS AND TREATMENT PEDIATRICS 24TH LATEST 5. Which factor is the most important in determining the rate of fluid replacement in the dehydrated child? a. The childs weight b. The type of dehydration c. Urine output d. Serum potassium level ANS: B Isotonic and hypotonic dehydration resuscitation involves fluid replacement over 24 hours. Hypertonic dehydration involves a slower replacement rate to prevent a sudden decrease in the sodium level. The childs weight determines the amount of fluid needed, not the rate of fluid replacement. One milliliter of body fluid is equal to 1 gram of body weight; therefore a loss of 1 kilogram (2.2 pounds) is equal to 1 liter of fluid. Urine output is not a consideration for determining the rate of administration of replacement fluids. Potassium level is not as significant in determining the rate of fluid replacement as the type of dehydration.
6. Which is the priority nursing intervention for a 6-month-old infant hospitalized with diarrhea and dehydration? a. Estimating insensible fluid loss b. Collecting urine for culture and sensitivity c. Palpating the posterior fontanel d. Measuring the infants weight ANS: D Weight is a crucial indicator of fluid status. It is an important criterion for assessing hydration status and response to fluid replacement. Infants have a greater total body surface area and therefore a greater potential for fluid loss through the skin. It is not possible to measure insensible fluid loss. Urine for culture and sensitivity is not usually part of the treatment plan for
CURRENT DIAGNOSIS AND TREATMENT PEDIATRICS 24TH LATEST the infant who is dehydrated from diarrhea. The posterior fontanel closes by 2 months of age. The anterior fontanel can be palpated during an assessment of an infant with dehydration. 7. What assessment should the nurse make before initiating an intravenous (IV) infusion of dextrose 5% in 0.9% normal saline solution with 10 mEq of potassium chloride for a child hospitalized with dehydration? a. Fluid intake b. Number of stools c. Urine output d. Capillary refill ANS: C Potassium chloride should never be added to an IV solution in the presence of oliguria or anuria (urine output less than 0.5 mL/kg/hr). Fluid intake does not provide information about renal function. Stool count sheds light on intestinal function. Renal function is the concern before potassium chloride is added to an IV solution. Assessment of capillary refill does not provide data about renal function. 8. A nurse is teaching parents about diarrhea. Which statement by the parents indicates understanding of the teaching? a. Diarrhea results from a fluid deficit in the small intestine. b. Organisms destroy intestinal mucosal cells, resulting in an increased intestinal surface area. c. Malabsorption results in metabolic alkalosis. d. Increased motility results in impaired absorption of fluid and nutrients. ANS: D Increased motility and rapid emptying of the intestines result in impaired absorption of nutrients and water. Electrolytes are drawn from the extracellular space into stool, and dehydration results. Diarrhea results from fluid excess in the small intestine. Destroyed intestinal mucosal cells result
CURRENT DIAGNOSIS AND TREATMENT PEDIATRICS 24TH LATEST in decreased intestinal surface area. Loss of electrolytes in the stool from diarrhea results in metabolic acidosis. 9. Which is the best nursing response to a parent asking about antidiarrheal medication for her 18-month-old child? a. It is okay to give antidiarrheal medication to a young child as long as you follow the directions on the box for correct dosage. b. Antidiarrheal medication is not recommended for young children because it slows the bodys attempt to rid itself of the pathogen. c. Im sure your child wont like the taste so give her extra fluids when you give the medication. d. Antidiarrheal medication will decrease the frequency of stools, but give your child Gatorade to maintain electrolyte balance. ANS: B Antidiarrheal medications may actually prolong diarrhea because the body will retain the organism causing the diarrhea, further increasing fluid and electrolyte losses. The use of these medications is not recommended for children aged younger than 2 years because of their binding nature and potential for toxicity. It is not appropriate to advise a parent to use antidiarrheal medication for a child aged younger than 2 years. Education about appropriate oral replacement fluids includes avoidance of sugary drinks, apple juice, sports beverages, and colas. 10. Which is the primary intervention in treating a child with persistent vomiting? a. Detecting the cause of vomiting b. Preventing metabolic acidosis c. Positioning the child supine d. Recording intake and output ANS: A The primary focus of managing vomiting is detection of the cause and then treatment of the cause. Metabolic alkalosis results from persistent vomiting. Prevention of complications is the
CURRENT DIAGNOSIS AND TREATMENT PEDIATRICS 24TH LATEST secondary focus of treatment. The child with persistent vomiting should be positioned upright or side-lying to prevent aspiration. Recording intake and output is a nursing intervention but it is not the primary focus of treatment. 11. Which is the best nursing response to give a parent about contacting the physician regarding an infant with diarrhea? a. Call your pediatrician if the infant has not had a wet diaper for 6 hours. b. The pediatrician should be contacted if the infant has two loose stools in an 8hour period. c. Call the doctor immediately if the infant has a temperature greater than 100 F. d. Notify the pediatrician if the infant naps more than 2 hours. ANS: A No urine output in 6 hours needs to be reported because it indicates dehydration. Two loose stools in 8 hours is not a serious concern. If blood is obvious in the stool or the frequency increases to one bowel movement every hour for more than 8 hours, the physician should be notified. A fever greater than 101 F should be reported to the infants physician. It is normal for the infant who is not ill to nap for 2 hours. The infant who is ill may nap longer than the typical amount. 12. Which diet would the nurse recommend to the mother of a child who is having mild diarrhea? a. Rice, potatoes, bread, and cereal b. Bananas, rice, applesauce, and toast c. Apple juice, hamburger, and salad d. Whatever the child would like to eat ANS: A Bland but nutritional foods including complex carbohydrates (rice, potatoes, bread, and cereals) are recommended for mild diarrhea. Bananas, rice, applesauce, and toast used to be
CURRENT DIAGNOSIS AND TREATMENT PEDIATRICS 24TH LATEST recommended for diarrhea (BRAT diet). These foods are easily tolerated but the BRAT diet is low in energy, density, fat, and protein. Fatty foods, spicy foods, and foods high in simple sugars should be avoided. The child should be offered foods he or she likes but should not be encouraged to eat fatty foods, spicy foods, and foods high in simple sugars. MULTIPLE RESPONSE 1. Which assessment findings indicate to the nurse that a child has fluid volume excess? Select all that apply. a. Weight gain b. Decreased blood pressure c. Moist breath sounds d. Poor skin turgor e. Rapid bounding pulse ANS: A, C, E A child with fluid volume excess will have weight gain, moist breath sounds due to the excess fluid in the pulmonary system, and a rapid bounding pulse. Other signs seen with fluid volume excess are increased blood pressure, edema, and fatigue. Decreased blood pressure and poor skin turgor are signs of fluid volume deficit. SHORT ANSWER 1. A nurse is totaling an 8 hour output for a child admitted with vomiting and diarrhea. The child had an emesis at 0800 of 50 mL and an emesis at 1200 of 35 mL. The child had 3 diapers weighed and calculated during the 8 hour shift: 1 diaper (urine and stool) = 30 mL; 1 diaper (urine only) = 25 mL; and 1 diaper (stool only) = 20 mL. What is the total 8 hour output for this child? ANS: 160
CURRENT DIAGNOSIS AND TREATMENT PEDIATRICS 24TH LATEST 160 mL Total emesis output = 85 mL and total urine and stool output = 75 mL. These are added together to get the total output for the 8 hours = 160. Emesis, urine, and stool are all counted when totaling output.
Chapter 46. Pediatric Laboratory Medicine and Reference Ranges show less 1. The nurse is preparing to administer a medication to a 6-month-old infant. The nurse will monitor closely for signs of drug toxicity based on the knowledge that, compared to adults, infants have a. an increased percentage of total body fat. b. immature hepatic and renal function. c. more protein receptor sites. d. more rapid gastrointestinal transit time. ANS: B The liver and kidneys are the primary organs for metabolism and excretion and are immature in infants. This allows drugs to accumulate and increases the risk for drug toxicity. Infants have a lower proportion of body fat than adults and fewer protein receptors. They do have more rapid gastrointestinal transit time, but this decreases the amount of drug absorbed. 2. The nurse reviews information about a drug and learns that it is best absorbed in an acidic environment. When giving this drug to a 1-year-old patient, the nurse will expect to administer a dose that will be a. equal to an adult dose. b. less than an adult dose. c. more than an adult dose. d. twice the usual adult dose. ANS: C Because the childs gastric pH is more alkaline than the adults, less drug will be absorbed. Therefore, the dose should be increased. 3. The nurse assumes care for an infant who is showing signs of drug toxicity to a drug given several hours prior. The nurse checks the dose and confirms that the dose is consistent with standard dosing guidelines. Which characteristic of the drug will likely explain this response in this patient? a. It is acidic. b. It is highly protein-bound. c. It is not fat-soluble. d. It is water-soluble. ANS: B With fewer protein-binding sites, there is more active drug available. This requires a reduction in the dose for infants. Drugs that are acidic are not as readily absorbed in infants, since their
CURRENT DIAGNOSIS AND TREATMENT PEDIATRICS 24TH LATEST gastric pH tends to be more alkaline. Infants have a lower proportion of body fat; fat-soluble drugs would need to be decreased to prevent toxicity. Until about age 2 years of age, pediatric patients require larger than usual doses of water-soluble drugs to achieve therapeutic effects. 4. The parent is concerned about giving a child medication because of the lack of knowledge about the effects of drugs on children. The nurse discusses legislation passed in 2002 and 2003 about pediatric pharmacology. Which is true about these laws? a. They forbid providers from prescribing medications unless they have been FDA- approved for use in children. b. They mandate consistent, evidence-based dosing guidelines for use in children. c. They provide federal grants to fund pediatric pharmaceutical research. d. They require drug manufacturers to study pediatric medication use. ANS: D In 2003, a law known as the Pediatric Research Equity Act joined the Best Pharmaceuticals Act of 2002 to require drug manufacturers to study pediatric medication use and offer incentives for pediatric pharmacology research. Providers are not forbidden to prescribe drugs in children that are not FDA-approved. The laws do not mandate the use of evidence-based guidelines and do not provide grants to fund research. 5. The nurse will administer an intravenous medication to an adolescent patient. When preparing the adolescent for the IV insertion, which is an appropriate action by the nurse? a. Allowing the patient to verbalize concerns about the procedure b. Covering the insertion site with a bandage after the procedure is completed c. Explaining any possible adverse drug reactions d. Reassuring the patient that only one body part will be used ANS: A Allowing the adolescent to verbalize concerns about the medication and its regimen may offer opportunities to clarify misconceptions and teach new information. Preschool-age children may have concerns about harm to their body and need to have sites covered. Adolescents still have a present focus, so discussing future adverse reactions is not especially helpful. Preschool and school-age children fear bodily harm and require reassurance that only one body part will be affected. 6. An infant will receive a topical medication. What instruction will the nurse include when teaching the parents how to administer the medication? a. Apply a thin layer to the affected area. b. Apply liberally to the skin on and around the area. c. Use the medication less frequently than what is recommended for adults. d. Use the medication more frequently than what is recommended for adults. ANS: A Topical medications may be altered by skin tissue condition. Children have thinner, more porous skin and have a proportionately higher skin surface area than adults and thus absorb topical medications more readily. Caregivers should be advised to use only a thin layer on the affected body part. This difference in skin does not affect the frequency of administering topical medications. 7. The provider has ordered that vitamin D drops be given to a newborn. Based on the knowledge of drug distribution in infants, the nurse understands that the infant may need
CURRENT DIAGNOSIS AND TREATMENT PEDIATRICS 24TH LATEST a. a higher dose. b. a lower dose. c. less frequent dosing. d. more frequent dosing. ANS: B Neonates and young infants tend to have less body fat than older children, meaning that they need less of fat-soluble medications since these medications wont be bound in fat tissue. Higher doses would lead to drug toxicity. Body fat does not affect the frequency of dosing. 8. The nurse is caring for a 5-year-old child. The child is taking a drug that has a known therapeutic range in adults, and the nurse checks that the ordered dose is correct and notes that the childs serum drug level is within normal limits. The child complains of a headache, which is a common sign of toxicity for this drug. Which action will the nurse take? a. Administer the drug since the drug levels are normal. b. Attribute the headache to non-drug causes. c. Hold the next dose and contact the provider. d. Request an order for an analgesic medication. ANS: C The therapeutic ranges established for many drug levels are based on adult studies, so it is important for the nurse to assess pediatric patients in conjunction with monitoring drug levels. The nurse should notify the provider of the reaction. Because headaches are a symptom of toxicity for this drug, the nurse should not ignore the symptom. 9. The nurse is preparing to give a 7-year-old child a bitter-tasting oral medication. The child asks the nurse if the medicine tastes bad. To help the child take this medication, which action will the nurse take? a. Allow the child to delay taking the medication until the parent arrives. b. Enlist the assistance of other staff to help restrain the child. c. Tell the child that it doesnt taste bad if it is swallowed quickly. d. Tell the child that it tastes bad and offer a choice of beverages to drink afterwards. ANS: D School-age children should be permitted more control, involvement in the process, and honest information. The nurse should tell the child the truth and offer the child a choice about what to drink to wash down the medicine. Medications must be given on schedule, so allowing the child a choice about when to take a medication is not acceptable. Restraining a child should not be used unless other methods have failed. Telling the child the medication doesnt taste bad is not honest and will reduce the childs trust in the nurse. 10. The nurse is preparing to administer an oral liquid medication to an 11-month-old child who is fussy and uncooperative. Which action will the nurse take to facilitate giving this medication? a. Adding honey to the medication to improve the taste b. Putting the medication in the infants formula c. Requesting an injectable form of the medication d. Using a syringe and allowing the parent to give the medication ANS: D When possible, family members or caregivers should be solicited to assist in medication administration. Infants should not receive honey because of the risk of botulism. A syringe allows more control over the amount of medication in the infants mouth and should be used.
CURRENT DIAGNOSIS AND TREATMENT PEDIATRICS 24TH LATEST Mixing the medication in a bottle requires ensuring that the infant takes the entire bottle in order to get the medication dose. Using an injectable form of medication is more traumatic and should be used only when an oral route is not possible or is contraindicated. 11. A 2-year-old child will receive several doses of an intramuscular medication. The nurse caring for this child will use which intervention to help the child cope with this regimen? a. Allowing the child to give pretend shots to a doll with an empty syringe b. Allowing the child to select a Band-Aid to wear after each medication is given c. Ensuring privacy while giving the medication d. Explaining that the medicine will help the child to feel better ANS: A Simple explanations, a firm approach, and enlisting the imagination of a toddler through play may enhance cooperation. Allowing the child to practice on a doll may help the toddler tolerate the injections. Preschool and school-age children fear bodily injury, and Band-Aids are important with those age groups. Adolescents need privacy, and school-age children and adolescents can understand the use of a medication in relation to future outcomes. 12. A preschool-age child has moderate dehydration and needs a rapid bolus of fluids. To provide atraumatic care and administer fluids most effectively, what action will the nurse take? a. Apply a eutectic mixture of local anesthetic (EMLA) just before inserting an intravenous line. b. Ask the childs parents to restrain the child during venipuncture so fluids may be administered. c. Request an order for nasogastric (NG) fluids to avoid the trauma of venipuncture. d. Use a powdered lidocaine preparation prior to insertion of the intravenous needle. ANS: D One method to ensure atraumatic care is through the use of topical analgesics before IV injections. Powdered lidocaine preparations are effective in reducing the pain and fear associated with invasive procedures, such as venipuncture. EMLA is useful only if applied 1 to 2.5 hours prior to IV insertion. Asking parents to restrain the child for a painful procedure can cause stress and anxiety for both the child and the parents. NG fluids are traumatic and are uncomfortable long past the insertion of the NG tube. 13. The nurse is preparing to administer an intramuscular medication to a 4-year-old child who starts to cry and screams, I dont want a shot! What is the nurses next action? a. Acknowledge that shots hurt and tell the child to be brave. b. Engage the child in a conversation about preschool and favorite activities. c. Enlist the assistance of another nurse to help restrain the child. d. Explain to the child that it will only hurt for a few seconds. ANS: B Distraction may be used for pain and anxiety control in this age group. Engaging the child in a conversation may distract the child from the anxiety of the imminent injection. It is not correct to tell the child to be brave since this belittles the feelings expressed by the child. Preschool children have a limited sense of time, so telling the child that the pain will only last a few seconds may not be effective. Restraining the child with other staff should be used last after other methods have failed. 14. A 14-year-old female who has type 1 diabetes mellitus that has been well-controlled for several years is admitted to the hospital for treatment of severe hyperglycemia. The patients lab values indicate poor glycemic control for the past 3 months. The nurse caring for this patient will
CURRENT DIAGNOSIS AND TREATMENT PEDIATRICS 24TH LATEST suspect which cause for the change in diabetic control? a. Adolescent rebellion and noncompliance b. Changes in cognitive function c. Hormonal fluctuations d. Possible experimentation with drugs or alcohol ANS: C In adolescence, hormonal changes and growth spurts may necessitate changes in medication dosages; many children with chronic illness require dosage adjustments in the early teen years. 15. The nurse is teaching a 15-year-old female patient and her parents about an antibiotic the adolescent will begin taking. The drug is known to decrease the effectiveness of oral contraceptive pills (OCPs). The nurse will a. ask the adolescent and her parents whether she is taking OCPs. b. tell her parents privately that pregnancy may occur if she is taking OCPs. c. tell her privately that the medication may decrease the effectiveness of OCPs. d. warn her and her parents that she may get pregnant if she is relying on OCPs. ANS: C When soliciting adolescent health histories, the nurse should consider issues related to sexual practices and should provide privacy when asking sensitive questions or giving sensitive information. The other actions do not allow for patient privacy.